Download as pdf or txt
Download as pdf or txt
You are on page 1of 188

02

03

Table of Content
SYLLABUS 04
SOURCES TO BE REFERRED TO 04
DISTRIBUTION OF THE PREVIOUS 7 YEARS’ QUESTIONS 05
QUESTION SOURCE-TYPE ANALYSIS 07
ANALYSIS OF SOURCES 08
DIFFICULTY LEVEL DISTRIBUTION 09
DIFFICULTY LEVEL ANALYSIS 09
OVERALL OBSERVATIONS 10
ADVICE FOR STUDENTS 11
MACROECONOMICS
EĂƟŽŶĂů/ŶĐŽŵĞĐĐŽƵŶƟŶŐ 12
DŽŶĞLJĂŶĚDŽŶĞLJ^ƵƉƉůLJ 16
DŽŶĞƚĂƌLJWŽůŝĐLJ 23
ĂŶŬŝŶŐĂŶĚĂŶŬŝŶŐZĞŐƵůĂƟŽŶƐ 30
WĂLJŵĞŶƚĐŽƐLJƐƚĞŵ 46
/ŶŇĂƟŽŶĂŶĚZĞĐĞƐƐŝŽŶ 52
&ŝƐĐĂůWŽůŝĐLJĂŶĚ'ŽǀĞƌŶŵĞŶƚƵĚŐĞƟŶŐ 58
džƚĞƌŶĂů^ĞĐƚŽƌ 77
&ŝŶĂŶĐŝĂůDĂƌŬĞƚ 104
INDIAN ECONOMY
>ĂŶĚZĞĨŽƌŵƐ 118
ŐƌŝĐƵůƚƵƌĞ 119
/ŶĚƵƐƚƌŝĞƐ 140
/ŶĨƌĂƐƚƌƵĐƚƵƌĞ 143
ŶĞƌŐLJ^ĞĐƚŽƌ 146
WůĂŶŶŝŶŐ 150
ϭϵϵϭZĞĨŽƌŵƐ 152
WŽǀĞƌƚLJ 155
^ŬŝůůĐƋƵŝƐŝƟŽŶĂŶĚ,ƵŵĂŶĞǀĞůŽƉŵĞŶƚ 157
>ĂďŽƵƌĂŶĚŵƉůŽLJŵĞŶƚ 161
^ŽĐŝĂů^ĞĐƵƌŝƚLJ 165
YƵĂůŝƚLJĂŶĚ^ƚĂŶĚĂƌĚŝnjĂƟŽŶ 169
ŝŐŝƟnjĂƟŽŶ 171
MICROECONOMICS
KƉƉŽƌƚƵŶŝƚLJŽƐƚ 174
>ĂǁŽĨĞŵĂŶĚĂŶĚ^ƵƉƉůLJ 175
2 QUESTIONS DROPPED BY THE UPSC 177
04

SYLLABUS
Economic and Social Development-Sustainable Development, Poverty, Inclusion, Demographics,
Social Sector Initiatives, etc.

b¨˜ΑÞü½½êÞΑ«ÞΑ«ÄõÚ«½üΑØÄæ«ËʔΑ«ÄΑ樘ΑÄËæ«ĈŽæ«ËÄΑ樁æΑfT[Αژ½˜Þ˜ÞΑ˜õ˜ÚüΑü˜ÚΑæΑ樘Α˜£«ÄÄ«Ä£ΑË¢Α
樘ΑŽüŽ½˜Π
b¨˜Α Þü½½êÞΑ¢ËÚΑ ŽËÄËëŽÞΑö¨«Ž¨Α «ÞΑ ØÄæ«ËʔΑ Ëõ˜Α ×טÚÞΑ ژÞæÚ«Žæ˜”ΠΑ +ÄΑ ژ½«æüΡΑ樘ޘΑ ½˜ÞÞΑ樁ÄΑ ͎͊Α
öËڔÞΑ¨«”˜Αö«æ¨«ÄΑ£Ú˜æΑÁÄüΑæË׫ŽÞΠΑA«Ä”Α樘ΑϏ˜æŽΠϐΑ[æÚæ«Ä£Α¢ÚËÃΑ樘ΑāæêژΑË¢ΑÃËĘüΡΑæËΑö¨æΑ樘ΑÚ˽˜ÞΑË¢Α
樘ΑΑΑΑ”«ą˜Ú˜ÄæΑ×½ü˜ÚÞΑ«ÄΑÄΑ˜ŽËÄËÃüΑÚ˜ΡΑæËΑ¨ËöΑÃꎨΑË¢Αö¨æΑ«ÞΑ×Ú˔ꎘ”Α«ÄΑ樘ΑŽËêÄæÚüΡΑŽËØΑêĔ˜ÚΑ樘Α
Þü½½êÞΠΑ

SOURCES TO BE REFERRED TO

Essential Material (EM)


͊ΠΑAŽÚ˘ŽËÄËëŽÞΑλBWbΑ½ÞÞΑ͊͋μ
͋ΠΑ+Ĕ«ÄΑŽËÄËëŽΑ˜õ˜½Ë×ØÄæΑλBWbΑ½ÞÞΑ͊͊μ
͌ΠΑA«ŽÚ˘ŽËÄËëŽÞΑλBWbΑ½ÞÞΑ͊͋μ
͍ΠΑ êޫĘÞÞΑ[æꔫ˜ÞΑλBWbΑ½ÞÞΑ͊͋μ
͎ΠΑfÄ«ËÄΑ ꔣ˜æ
͏ΠΑŽËÄËëŽΑ[êÚõ˜ü
*Business Studies NCERT (for Financial Market only)

Essential Newspapers (EN)


͊ΠΑb¨˜Α(«Ä”ê
͋ΠΑb¨˜Α+Ĕ«ÄΑû×ژÞÞ
*You can selectively refer to The Hindu Business Line or the Live Mint

Reference Material (RM)


͊ΠΑT+ ΑλÚ櫎½˜Þμ
͋ΠΑÚ樁ט”«Αλö˜Þ«æ˜μ
͌ΠΑq«ºÞט”«Αλö˜Þ«æ˜μ
͍ΠΑ+Äõ˜ÞæËט”«Αλö˜Þ«æ˜μ
*You can selectively refer to the websites of the RBI, the WTO, Britannica etc.

Random Resources (RR)


õ˜ÚüΑ ü˜ÚΑ ÞËØΑ Ùê˜Þæ«ËÄÞΑ ¢˜æêژΑ «ÄΑ 樘Α fT[Α Tژ½«ÃÞΑ ö¨«Ž¨Α Ú˜Α Þ˜”Α ËÄΑ 樘ØÞΑ 樁æΑ Ú˜Α ÄËæΑ
ŽËõ˜Ú˜”Α«ÄΑ樘ΑÞæÄ”Ú”ΑÞËêڎ˜ÞΑØÄæ«ËʔΑËõ˜ΑλAΡΑBΑËÚΑWAμΠ

Π£ΠΑ樘ΑÙê˜Þæ«ËÄΑËÄΑϏræ˜ÚΑژ”«æϐΑ«ÄΑ͉͋͋͊ΠΑΑ[ꎨΑ樘ØÞΑÚ˜Α½˜ÞޘÚαºÄËöÄΑÄ”Α¨˜ÄŽ˜ΑÞꎨΑÞËêڎ˜ÞΑ
λ¢ÚËÃΑö¨˜Ú˜Α樘Α樘ØÞΑÚ˜ΑÞËêڎ˜”μΑ¨õ˜Α˜˜ÄΑŽ½ÞÞ«Ĉ˜”ΑÞΑêĎËÄõ˜Äæ«Ëā½ΑËÚΑWÄ”ËÃΑW˜ÞËêڎ˜ÞΑ
λWWμ
05

DISTRIBUTION OF THE PREVIOUS 7 YEARS’ QUESTIONS


ACROSS DIFFERENT TOPICS

TOTAL ECONOMICS QUESTIONS - 135

1 Question
Law of Demand and Supply
TOTAL
1 Question
Opportunity Cost 135 Questions
2 Questions
Digitization Please Note:
2 Questions ͋ΑÙê˜Þæ«ËÄÞΑö˜Ú˜Α”ÚË×ט”ΑüΑfT[ΣΑËĘΑ
Quality and Standardization
«ÄΑ ͉͉͋͋Α Ä”Α 樘Α Ë樘ÚΑ «ÄΑ ͉͋͋͋ΠΑ b¨˜Α ͋Α
3 Questions
Social Security
Ùê˜Þæ«ËÄÞΑ¨õ˜Α˜˜ÄΑ˜ûŽ½ê”˜”ΑæËΑÚÚ«õ˜Α
æΑ樘ΑæË恽Π
4 Questions
Labour and Employment

3 Questions
Skill Acquisition and
Human Development
1 Question 3 Questions
Poverty National Income Accounting

2 Questions 6 Questions
1991 Reforms Money and Money Supply

1 Question 6 Questions
Planning Monetary Policy
3 Questions
Energy Sector 13 Questions
2 Questions Banking and Banking Regulations
Infrastructure
2 Questions
Industries

6 Questions
Payment Ecosystem

17 Questions 4 Questions
Agriculture Inflation and Recession

13 Questions
1 Question Fiscal Policy and
Land Reforms Government Budgeting

11 Questions
Financial Market

MACROECONOMICS

INDIAN ECONOMY 28 Questions


External Sector
MICROECONOMICS
06
3 Questions
National Income Accounting

6 Questions
3 Questions Money and Money Supply
National Income Accounting

6 Questions
Monetary Policy

13 Questions
Banking and Banking
Regulations

28 Questions
External Sector

6 Questions
Payment Ecosystem

4 Questions
Inflation and Recession

MACROECONOMICS
13 Questions
Fiscal Policy and Government Budgeting

2 Questions 3 Questions
Digitization National Income Accounting
2 Questions
Quality and Standardization

3 Questions
Social Security

4 Questions
Labour and Employment 17 Questions
Agriculture

3 Questions
Skill Acquisition and
Human Development

1 Questions
Poverty
2 Questions
1991 Reforms
1 Questions
2 Questions
Planning
Industries
3 Questions 2 Questions
Energy Sector Infrastructure
INDIAN ECONOMY
07

1 Question 1 Question
Opportunity Cost Law of Demand and Supply

MICROECONOMICS

QUESTION SOURCE-TYPE ANALYSIS

54 Questions
40 Questions Essential Newspaper
60 Essential Material
26 Questions
50 Reference Material

40
15 Questions
30 Random Resources

20
10
0
08

ANALYSIS OF SOURCES
b¨˜Α Ëõ˜Α ”«õ«Þ«ËÄΑ «ÞΑ Þ½«£¨æ½üΑ ”«ĆŽê½æΑ æËΑ Áº˜Α ÞΑ some questions have both static and dynamic
angles Ä”Α¨˜ÄŽ˜ΑŽÄÄËæΑ˜ΑÄÞö˜Ú˜”ΑËĽüΑüΑژ¢˜ÚÚ«Ä£ΑæËΑBWbΑË˺ÞΠΑ"ËÚΑ˜ûÃ×½˜ΑÙê˜Þæ«ËÄÞΑژ½æ˜”Α
æËΑ˜ûæ˜Úā½ΑޘŽæËÚΠ

b¨˜Ú˜Α«ÞΑÄËΑ”ËêæΑ«ÄΑ樘Α¢ŽæΑ樁æΑnewspaper reading is indispensableΑæËΑ×ژׁځæ«ËÄΑ¢ËÚΑ樘Α”üāëŽΑ


ÞטŽæÞΑ«ÄΑŽËÄËëŽÞΠ

Please note:Α"ËÚΑÃËÞæΑË¢Α樘ΑÙê˜Þæ«ËÄÞΡΑö¨˜Ú˜ΑĘöÞׁטÚΑ½«ÄºÞΑÚ˜ΑÞ¨Ú˜”ΡΑÄÞö˜ÚÞΑÄ”Α«Ä¢ËÚÁæ«ËÄΑ
ŽË꽔Α½ÞËΑ˜Α¢ËêĔΑ«ÄΑŽËÄËëŽΑ[êÚõ˜üÞΡΑêæΑ樘ΑށØΑŽË꽔ΑÄËæΑ˜Α¢ËêĔΑæΑËĘΑ×½Ž˜ΠΑb¨æΑ«æΑö¨˜Ú˜Α
樘Αژ”«Ä£ΑË¢ΑĘöÞׁטÚΑ«ÞΑ«Ã×ËÚæÄæΠ

A few questions were based on very unconventional or lesser-known themesΑÄ”ΑÞËØΑÙê˜Þæ«ËÄÞΑ


¨”ΑÞææ˜Ã˜ÄæÞΑö¨«Ž¨ΑÚ˜Α”«ĆŽê½æΑæËΑĈĔΑ«ÄΑŽËÄõ˜Äæ«Ëā½ΑÞËêڎ˜ÞΠΑBËöΑ樁æΑü˜ÚÞΑ¨õ˜ΑׁÞޘ”ΡΑÞËØΑ
Ë¢Α樘Α樘ØÞΑ×טÚΑõ˜ÚüΑŽËÃÃËÄΡΑêæΑæΑ樘Αæ«Ã˜Αö¨˜ÄΑ樘üΑÚ˜ΑÞº˜”ΡΑŽËÄõ˜Äæ«Ëā½ΑÞËêڎ˜ÞΑö˜Ú˜Α
”«ĆŽê½æΑæËΑĈĔΠ

ΑΑü˜Úαö«Þ˜ΑÄ½üÞ«ÞΑ«ÞΑÞ¨ËöÄΑ˜½Ëö

2017:Αb¨˜ΑÙê˜Þæ«ËÄÞΑËÄΑVꁽ«æüΑËêĎ«½ΑË¢Α+Ĕ«ΑÄ”ΑT(f

2018: b¨˜ΑÙê˜Þæ«ËÄÞΑËÄΑB[[HΑ£Ú«Žê½æêځ½Α[êÚõ˜üΑÄ”Α樘Α«æ˜ÃÞΑËÄΑö¨«Ž¨Α#[bΑ«ÞΑ××½«˜”ΑλÄËæΑ˜Þüμ

2019:ΑÃËÞæΑË¢Α樘Α¢ŽæÞΑŽË꽔Α˜Αˍ恫ʔΑ¢ÚËÃΑŽËÄõ˜Äæ«Ëā½ΑÙê˜Þæ«ËÄÞ

2020:Αb¨˜ΑÙê˜Þæ«ËÄÞΑژ½æ˜”ΑæËΑϏÚ«Ž˜Α×Ú«Ž˜ϐΡΑϏ×ꍽ«ŽΑ«Äõ˜ÞæØÄæΑ«ÄΑ£Ú«Žê½æêژϐΡΑϏÄËÄαĈāĎ«½Α”˜æÞϐ

2021:Αb¨˜ΑÙê˜Þæ«ËÄΑËÄΑϏræ˜ÚΑژ”«æϐ

2022: b¨˜ΑÙê˜Þæ«ËÄΑËÄΑϏ˜αŽËÃØڎ˜ϐ

2023:Αb¨˜ΑÙê˜Þæ«ËÄÞΑËÄΑ樘Α×ËÚæÞΑ먁õ«Ä£ΑÄΑÄ£½˜ΑË¢Α˜ŽËÄËëŽΑ£˜Ë£Ú×¨üμΡΑ[͌«ΡΑϏÞ恍«½«æüΑÄ”Α£ÚËöæ¨Α
ׁŽæΠ

Vikaspedia and Arthapedia are hidden gemsΑ Ä”Α ŽËÃטÄށæ˜Α ¢ËÚΑ 樘Α ëÞÞ«Ä£Α «Ä¢ËÚÁæ«ËÄΑ «ÄΑ 樘Α
BWbΑË˺ÞΠΑ(Ëö˜õ˜Úю˜Ú恫ÄΑÄêØګŽ½Α¢ŽæÞΑÞ¨Ë꽔Α˜ΑŽÚËÞÞᎨ˜Žº˜”Α«ÄΑŽÞ˜Αæ¨ËޘΑÚ˜Α¢ËêĔΑæËΑ˜Α
Ë¢Α˽”˜ÚΑ”æ˜ÞΠ
09

DIFFICULTY LEVEL ANALYSIS


õ˜ÚüΑ ü˜ÚΡΑ «ÚژÞטŽæ«õ˜Α Ë¢Α ¨ËöΑ ÁÄüΑ êĎËÄõ˜Äæ«Ëā½Α Ä”Α ”«ĆŽê½æüΑ 樘ØÞΑ ¢˜æêژΡΑ ËÄΑ ÄΑ õ˜Ú£˜Α ͍͉Α
טڎ˜ÄæΑË¢Α樘ΑÙê˜Þæ«ËÄÞΑ¨õ˜Α½öüÞΑ˜˜ÄΑ[xΠΑÄË樘ÚΑ͉͌α͍͉Αטڎ˜ÄæΑÙê˜Þæ«ËÄÞΑÚ˜ΑË¢ΑAHWbΑHWΑ
A+fAΑ½˜õ˜½ΑÄ”ΑËĽüΑ͌α͍ΑÙê˜Þæ«ËÄÞΑÚ˜Α+""+f;bΠ

DIFFICULTY LEVEL DISTRIBUTION

YEAR NO. OF QUESTIONS EASY MEDIUM DIFFICULT


͉͋͊͐ ͎͋ 9 ͍͊ ͋

͉͋͊͑ ͉͋ ͑ 9 3

͉͋͊͒ ͋͌ ͑ 7 ͑

͉͉͋͋ ͉͋ ͑ ͑ 4

͉͋͋͊ ͎͊ 7 6 ͋

͉͋͋͋ ͍͊ ͎ 6 3

͉͋͋͌ ͊͑ ͑ 7 3
10

OVERALL OBSERVATIONS
õ˜ÚüΑü˜ÚΡΑËÄΑÄΑõ˜Ú£˜ΡΑ͊͑α͉͋ΑÙê˜Þæ«ËÄÞΑÚ˜ΑÞº˜”Α¢ÚËÃΑ樘ΑŽËÄËëŽÞΑޘŽæ«ËÄΠΑr˜ΑŽ½ÞÞ«¢üΑ
ÞËØΑË¢Α樘ΑÙê˜Þæ«ËÄÞΑêĔ˜ÚΑêÚژÄæΑą«ÚÞΑλ¨˜ÄŽ˜Α樘ΑÄê͘ÚΑ×טÚÞΑÞ½«£¨æ½üΑ½ËöμΠ

60 percent of the questions asked in the 7 YEARS (2017-2023) have been from the Macro-
economics sectionsΑ λ£Ë«Ä£Α üΑ 樘Α BWbΑ Ž½ÞÞ«ĈŽæ«ËÄμΑ ŽËÃ×Ú«Þ«Ä£Α 樘Α ûæ˜Úā½Α [˜ŽæËÚΡΑ
AËĘüΑ[ê××½üΑÄ”Α Äº«Ä£ΑޘŽæ«ËÄÞΠΑ

˜Ú恫ÄΑ樘ØÞΑ¨õ˜Α˜˜ÄΑ¨«£¨½üΑژŽêÚÚ«Ä£Α½«º˜ΑTüØÄæΑŽËÞüÞæ˜ÃÞΑλËÚΑ«£«æ½Α"«ÄÄŽ˜μΠΑ+ÄΑ
͉͋͊͐ΡΑ ͋Α Ùê˜Þæ«ËÄÞΑ ö˜Ú˜Α Þº˜”Α ¢ÚËÃΑ 樁æΑ ޘŽæ«ËÄΡΑ ͌Α ö˜Ú˜Α Þº˜”Α «ÄΑ ͉͋͊͑Α Ä”Α ͊Α «ÄΑ ͉͋͊͒ΠΑ b¨˜Α
ژÞËÄΑöÞΑŽ½˜ÚΡΑ樘Αdemonetisation-led growth in the sector of Digital Finance made it a
×Ë×꽁ÚΑ樘ØΠ

b¨˜ΑÙê˜Þæ«ËÄÞΑ¢ÚËÃΑ+Ĕ«ÄΑŽËÄËÃüΑλ£Ë«Ä£ΑüΑ樘ΑBWbΑŽ½ÞÞ«ĈŽæ«ËÄμΑ¨õ˜Α˜˜ÄΑ˜Ùꁽ½üΑ
ژÞËā½˜Α«ÄΑÄê͘ÚΠΑSo far, Indian Agriculture has been the dominating section. ͉͋͋͊Α
Ä”Α͉͋͋͋Α¨õ˜Α˜˜ÄΑ˜Úځæ«ËÄÞΑæ¨Ë꣨ΑÞΑ«ÄΑæ¨ËޘΑæöËΑü˜ÚÞΑ樘ژΑö˜Ú˜ΑÄËΑŽËÄËëŽÞΑÙê˜Þα
æ«ËÄÞΑÞ˜”ΑËÄΑ+Ĕ«ÄΑ£Ú«Žê½æêژΠ

+ÄΑ£Ú«Žê½æêÚ˜Ρ Minimum Support Price (MSP) ¨ÞΑ˜˜ÄΑΑõ˜ÚüΑژŽêÚÚ«Ä£Α樘ØΠΑ+ÄΑ͉͉͋͋Α«æޘ½¢ΡΑ


ÞΑ ÁÄüΑ ͌Α Ùê˜Þæ«ËÄÞΑ λژ½æ˜”ΑæËΑ A[TÞΑ «ÄΑõÚü«Ä£Α ”˜£Ú˜˜ÞμΑö˜Ú˜Α Þº˜”ΠΑb¨˜Α ĘûæΑ «Ã×ËÚæÄæΑ
ÞêαޘŽæ«ËÄΑ¨ÞΑ˜˜ÄΑ£Ú«Žê½æêځ½Αbځ”˜ΠΑ

b¨˜Ú˜Α½ÞËΑ¨õ˜Α˜˜ÄΑژטæ«æ«ËÄΑË¢ΑÙê˜Þæ«ËÄÞΑö«æ¨ËêæΑÄüΑ«ËæΑË¢ΑŽ¨Ä£˜ΠΑ+ÄΑ͉͋͊͒ΑÄ”Α͉͋͋͊ΡΑ
樘ΑށØΑÙê˜Þæ«ËÄΑöÞΑÞº˜”Π

½ÞËΡΑ ÄºΑ ˁڔΑ êژêΑ¨ÞΑ˜˜ÄΑΑژŽêÚÚ«Ä£Α樘ØΠ

A«ŽÚ˘ŽËÄËëŽÞΑޘŽæ«ËÄΑ¨ÞΑ˜˜ÄΑ«£ÄËژ”ΡΑêæΑ樁æΑ”˘ÞΑÄËæΑ؁ÄΡΑfT[Α¨ÞΑÄ”ËʔΑ«æΑ
¢Ëژõ˜ÚΠΑ+ÄΑ樘Α͐Αü˜ÚÞΡΑËĽüΑ͋ΑÙê˜Þæ«ËÄÞΑ¨õ˜Α¢˜æêژ”Π

fT[Α «ÞΑ ½ÞËΑ £Ë«Ä£Α ˜üËĔΑ 樘Α ŽËÄõ˜Äæ«Ëā½Α ”ËÁ«ÄΠΑ Α ¢˜öΑ Ùê˜Þæ«ËÄÞΑ ¢ÚËÃΑ Accountancy
¨õ˜Α½ÞËΑ¢˜æêژ”ΣΑ˜Π£ΠΑژ½æ˜”ΑæËΑŽ×«æ½έژõ˜Äê˜ΑË¢ΑĈÚÃÞΡΑÄ” ‘cyber insurance.’

˜æö˜˜ÄΑ ͉͋͊͐Α Ä”Α ͉͋͋͌ΡΑ ÞËΑ¢ÚΑ ͋Α Ùê˜Þæ«ËÄÞΑ¢ÚËÃΑ ŽËÄËëŽÞΑ ¨õ˜Α ˜˜ÄΑ ”ÚË×ט”ΠΑb¨˜ΑĈÚÞæΑ
Ùê˜Þæ«ËÄΑöÞΑ¢ÚËÃΑ͉͉͋͋ΑλÞ˜”ΑËÄΑ樘Α+Äæ˜Úāæ«Ëā½Αbځ”˜ΑË¢Α+Ĕ«μΑÄ”Α樘ΑޘŽËĔΑËĘΑöÞΑ
¢ÚËÃΑ͉͋͋͋Αλ×«æ½Α"½«£¨æμΠ
11

ADVICE FOR STUDENTS

fĔËêæ˜”½üΡΑŽËÄËëŽÞΑ«ÞΑËĘΑË¢Α樘ΑÃËÞæΑ«Ã×ËÚæÄæΑÞꍶ˜ŽæÞΑ¢ÚËÃΑ樘ΑΑfT[Α[Α
Tژ½«ÃÞΑ×Ë«ÄæΑË¢Αõ«˜öΠ

+æΑژÙê«Ú˜ÞΑΑæ¨ËÚË꣨Α×ژׁځæ«ËÄΠΑÞΑ樘Αځģ˜ΑË¢Α樘ØÞΑŽËõ˜Ú˜”Α«ÞΑõ˜ÚüΑö«”˜ΑÄ”Α
”«õ˜ÚޘΡΑ ÞæꔘÄæÞΑ Ú˜Α ”õ«Þ˜”Α æËΑ ÄËæΑ £˜æΑ Ëõ˜Úö¨˜½Ã˜”Α Ä”Α ½öüÞΑ 恺˜Α Þ昁”üΑ
üαÞæ˜×ÞΠΑ

½öüÞΑÞæÚæΑö«æ¨Α樘ΑBWbΑË˺ÞΑ뽁ÞÞΑ͊͊ΑÄ”Α͊͋μΠΑ+¢ΑËĘΑöÄæÞΑæËΑÞæÚæΑ¢ÚËÃΑÄΑ
˜õ˜ÄΑÃËژΑÞ«ŽΑ½˜õ˜½ΡΑ樘ÄΑ½ÞÞΑ͒ΑË˺ÞΑÚ˜ΑΑ£Ë˔ΑË×æ«ËÄΠ

#ځ”ꁽ½üΑĘöÞׁטÚΑژ”«Ä£ΑŽÄΑ˜ΑÞæÚ昔ΠΑHĽüΑÞ¢æ˜ÚΑژõ«Þ«Ä£ΑÄ”ΑêĔ˜ÚÞæÄ”«Ä£Α
樘ΑÞ«ŽΑŽËϘ×æÞΑË¢ΑÁŽÚ˘ŽËÄËëŽÞΡΑËĘΑŽÄΑŽËÃ×ژ¨˜Ä”Α樘Αæځ¶˜ŽæËÚüΑË¢Α樘Α
+Ĕ«ÄΑŽËÄËÃüΠ

bÚüΑæËΑÞ櫎ºΑæËΑËĘΑÞæÄ”Ú”ΑĘöÞׁטÚΑλ樘ΑŽ¨Ë«Ž˜Α«ÞΑüËêÚÞμ

½öüÞΑژ”Α樘ΑfÄ«ËÄΑ ꔣ˜æΑæځÄގګ×æΑÄ”Α樘ΑŽËÄËëŽΑ[êÚõ˜üΑژ½˜Þ˜”Α樁æΑü˜ÚΠ

ĔΑÃËÞæΑ«Ã×ËÚæÄæ½üΑÞê××½˜Ã˜ÄæΑüËêÚΑ×ژׁځæ«ËÄΑüΑ恺«Ä£Αژ£ê½ÚΑÃˎºΑæ˜ÞæÞΠ

Have Faith in Yourself and Study Well!


12

MACROECONOMICS
NATIONAL INCOME ACCOUNTING
(3 Questions)

Year 2018 GROSS NATIONAL PRODUCT (GNP)

Q1. Increase in absolute and per capital real GNP do NOT connote a higher level of economic
development, if
λμΑ«Ä”êÞæÚ«½ΑËêæ×êæΑ¢«½ÞΑæËΑº˜˜×ΑׁŽ˜Αö«æ¨Α£Ú«Žê½æêځ½ΑËêæ×êæΠΑ
λμΑ£Ú«Žê½æêځ½ΑËêæ×êæΑ¢«½ÞΑæËΑº˜˜×ΑׁŽ˜Αö«æ¨Α«Ä”êÞæÚ«½ΑËêæ×êæΠΑ
λŽμΑ×Ëõ˜ÚæüΑÄ”ΑêĘÃ×½ËüØÄæΑ«ÄŽÚ˜Þ˜ΠΑ
λ”μΑ«Ã×ËÚæÞΑ£ÚËöΑ¢Þæ˜ÚΑ樁ÄΑ˜û×ËÚæÞΠ

ANSWERS C

EXPLANATION
An essential aspect of development is to enable the maximum number to experience the fruits of
development. ËϘ×æÞΑË¢ΑטÚΑŽ×«æΑ«ÄŽËØΑλטÚΑŽ×«æΑ#TΑËÚΑטÚΑŽ×«æΑB[TμΑÚ˜ΑÄËæΑ½˜ΑæËΑŽ×æêژΑ
樫ÞΑ ÞטŽæΑ Ë¢Α ”˜õ˜½Ë×ØÄæΠΑb¨˜Ú˜Α ÁüΑ ˜Α Α ŽÞ˜Αö¨˜Ú˜«ÄΑ «ÄŽÚ˜Þ˜Α «ÄΑ Þ˽êæ˜Α Ä”Α טÚΑ Ž×«æΑ #BTΑ «ÞΑ
ژĉ˜Žæ«õ˜ΑË¢Α£ÚËöæ¨Α«ÄΑ«ÄŽËØΑË¢ΑΑÞÁ½½ΑޘŽæ«ËÄΑË¢ΑÞˎ«˜æüΑÄ”Α樁æΑÁ¶ËÚ«æüΑË¢Α樘Α×Ë×꽁æ«ËÄΑ«ÞΑ×Ëõ˜ÚæüΑ
ÞæÚ«Žº˜ÄΑÄ”ΑêĘÃ×½Ëü˜”ΠΑAê½æ«Α”«Ã˜ÄÞ«Ëā½ΑÄËÄαÃËĘæÚüΑÞˎ«½Α«Ä”«ŽæËÚÞΑ(like poverty or prosperity
and employment or unemployment)Α Ú˜Α ˜ææ˜ÚΑ ژĉ˜ŽæËÚÞΑ Ë¢Α Ëõ˜Ú½½Α ˜ŽËÄËëŽΑ ”˜õ˜½Ë×ØÄæΑ «ÄΑ 樘Α
Þˎ«˜æüΠΑHence option (c) is the correct answer.

ANALYSIS:
ANALYSIS:

Scope for elimination


ÞΑ ŽÄΑ ˜Α ޘ˜ÄΡΑ Ë×æ«ËÄΑ λŽμΑ «ÞΑ 樘Α ËĽüΑ Ë×æ«ËÄΑ ö¨«Ž¨Α 恽ºÞΑ
ËêæΑ Ùꁽ«ææ«õ˜Α £ÚËöæ¨Α λ×Ë×꽁æ«ËÄΑ ˜Ä˜Ĉææ«Ä£Α ”ê˜Α æËΑ
£ÚËöæ¨μΠΑ½½ΑË樘ÚΑË×æ«ËÄÞΑÚ˜Αژ½æ˜”ΑæËΑÙêÄæ«ææ«õ˜Α£ÚËöæ¨ΠΑ

b¨«ÞΑ Ùê˜Þæ«ËÄΑ ŽË꽔Α ¨õ˜Α ˜˜ÄΑ ˜Þ«½üΑ ÄÞö˜Ú˜”Α êÞ«Ä£Α 樘Α


ºÄËö½˜”£˜Α¢ÚËÃΑ樘ΑAŽÚ˘ŽËÄËëŽÞΑλBWbΑ½ÞÞΑ͊͋μΑË˺Α
ûŽ˜Ú×æΑ¢ÚËÃΑBWb

Ä”Α樘Α+Ĕ«ÄΑŽËÄËëŽΑ˜õ˜½Ë×ØÄæΑλBWbΑ½ÞÞΑ͊͊μΠΑb¨˜Α
(êÁÄΑ#˜Ë£Ú×¨üΑλBWbΑ½ÞÞΑ͊͋μΑ½ÞËΑ”«ą˜Ú˜Ä櫁æ˜ÞΑö˜½½Α
˜æö˜˜ÄΑϏ˜õ˜Ë½×ØÄæϐΑÄ”ΑϏ#ÚËöæ¨ϐ
13

SOURCE : Indian Economic Development (NCERT Class 11), Human Geography (NCERT Class 12)

Additional Information
Gross National Product (GNP)Α«ÞΑΑ؁ÞêژΑË¢Α樘ΑæË恽Α˜ŽËÄËëŽΑËêæ×êæΑ×Ú˔ꎘ”ΑüΑ樘Α
ژޫ”˜ÄæÞΑË¢ΑΑŽËêÄæÚüΡΑËæ¨Αö«æ¨«ÄΑ«æÞΑËڔ˜ÚÞΑÄ”ΑÚˁ”ΡΑ”êÚ«Ä£ΑΑÞטŽ«ĈŽΑæ«Ã˜Αטګ˔Π
Gross Domestic Product: +æΑ«ÞΑ樘ΑÃËĘæÚüΑõ½ê˜ΑË¢Α½½ΑĈā½Α£Ë˔ÞΑϞΑޘÚõ«Ž˜ÞΑ×Ú˔ꎘ”Α
«ÄÞ«”˜Α樘Α”ËØÞ櫎Αæ˜ÚÚ«æËÚüΑË¢ΑΑŽËêÄæÚüΑ”êÚ«Ä£ΑËĘΑĈāĎ«½Αü˜ÚΑËÚΑΑÙêÚæ˜ÚΠ
Net Domestic Product (NDP) «ÞΑÄΑÄÄꁽΑ؁ÞêژΑË¢Α樘Α˜ŽËÄËëŽΑËêæ×êæΑË¢ΑΑāæ«ËÄΑ
樁æΑ«ÞΑ”¶êÞ昔ΑæËΑŽŽËêÄæΑ¢ËÚΑ”˜×ژŽ«æ«ËÄΡΑŽ½Žê½æ˜”ΑüΑÞêæځŽæ«Ä£Α”˜×ژŽ«æ«ËÄΑ¢ÚËÃΑ樘Α£ÚËÞÞΑ”ËØÞα
櫎Α×Ú˔êŽæΑλ#TμΠΑBTϽΑ#Tα˜×ژŽ«æ«ËÄ
Net National Product at Market Price (NNP)Α«ÞΑ樘ΑæË恽ΑÃËĘüΑõ½ê˜ΑË¢Α½½Α樘ΑĈā½Α£Ë˔ÞΑÄ”ΑޘÚõ«Ž˜ÞΑ
˜ûŽ½ê”«Ä£Α”˜×ژŽ«æ«ËÄΡΑö¨«Ž¨Α«ÞΑ樘ΑŽËÄÞêÃ×æ«ËÄΑË¢ΑĈû˜”ΑŽ×«æ½ΠΑb¨êÞΑBBTΑϽΑ#BTΑγΑ˜×ژŽ«æ«ËÄ
National income = B˜æΑBæ«Ëā½ΑTÚ˔êŽæΑæΑ¢ŽæËÚΑŽËÞæΑϽΑB˜æΑBæ«Ëā½ΑTÚ˔êŽæΑæΑÁں˜æΑ×Ú«Ž˜ΑγΑ+Ĕ«Ú˜ŽæΑ
æû˜ÞΑϹΑ[êÞ«”«˜ÞΠ

Year 2018 CAPITAL FORMATION

V͋ΠΑ˜Þ׫æ˜Α˜«Ä£ΑΑ¨«£¨Αށõ«Ä£Α˜ŽËÄËÃüΡΑŽ×«æ½Α¢ËÚÁæ«ËÄΑÁüΑÄËæΑژÞê½æΑ«ÄΑÞ«£Ä«ĈŽÄæΑ«ÄŽÚ˜Þ˜Α«ÄΑ
output due to
λμΑö˜ºΑ”ëīÞæځæ«õ˜ΑÁŽ¨«Ä˜ÚüΑ
λμΑ«½½«æ˜ÚŽüΑ
λŽμΑ¨«£¨Α×Ë×꽁æ«ËÄΑ”˜ÄÞ«æü
Αλ”μΑ¨«£¨ΑŽ×«æ½αËêæ×êæΑځæ«Ë

ANSWERS D

EXPLANATION

×«æ½Α¢ËÚÁæ«ËÄΑ؁ÄÞΑ«ÄŽÚ˜Þ«Ä£Α樘ΑÞæˎºΑË¢Αژ½ΑŽ×«æ½Α«ÄΑΑŽËêÄæÚüΠΑ+ÄΑË樘ÚΑöËڔÞΡΑŽ×«æ½Α¢ËÚÁæ«ËÄΑ
«Äõ˽õ˜ÞΑÁº«Ä£ΑË¢ΑÃËژΑŽ×«æ½Α£Ë˔ÞΑÞꎨΑÞΑÁŽ¨«Ä˜ÞΡΑæË˽ÞΡΑ¢ŽæËÚ«˜ÞΡΑæځÄÞ×ËÚæΑ˜Ùê«×ØÄæΡΑÁæ˜Ú«½ÞΡΑ
˜½˜ŽæÚ«Ž«æüΡΑ˜æŽΠΡΑö¨«Ž¨ΑÚ˜Α½½Αêޘ”Α¢ËÚΑ¢êæêژΑ×Ú˔êŽæ«ËÄΑË¢Α£Ë˔ÞΠΑ"ËÚΑÁº«Ä£Α””«æ«ËÄÞΑæËΑ樘ΑÞæˎºΑË¢Α
×«æ½ΡΑ ށõ«Ä£Α Ä”Α «Äõ˜ÞæØÄæΑ Ú˜Α ˜ÞޘÄ櫁½ΠΑ ×«æ½Α Ëêæ×êæΑ ځæ«ËΑ «ÞΑ 樘Α ÃËêÄæΑ Ë¢Α Ž×«æ½Α ʘ”˜”Α æËΑ
×Ú˔ꎘΑËĘΑêÄ«æΑË¢ΑËêæ×êæΠΑ
"ËÚΑ ˜ûÃ×½˜ΡΑ Þê××ËޘΑ樁æΑ «Äõ˜ÞæØÄæΑ «ÄΑ ÄΑ ˜ŽËÄËÃüΡΑ «Äõ˜ÞæØÄæΑ «ÞΑ ͌͋АΑ λË¢Α #TμΡΑ Ä”Α樘Α ˜ŽËÄËëŽΑ
£ÚËöæ¨ΑŽËÚژÞ×ËĔ«Ä£ΑæËΑ樫ÞΑ½˜õ˜½ΑË¢Α«Äõ˜ÞæØÄæΑ«ÞΑ͑АΠΑ(˜Ú˜ΡΑΑWÞΑ͌͋Α«Äõ˜ÞæØÄæΑ×Ú˔ꎘÞΑÄΑËêæ×êæΑË¢Α
WÞΑ͑ΠΑ×«æ½ΑËêæ×êæΑځæ«ËΑ«ÞΑ͌͋έ͑ΑËÚΑ͍ΠΑ+ÄΑË樘ÚΑöËڔÞΡΑæËΑ×Ú˔ꎘΑËĘΑêÄ«æΑË¢ΑËêæ×êæΡΑ͍ΑêÄ«æΑË¢ΑŽ×«æ½Α«ÞΑ
ʘ”˜”ΠΑ (˜ÄŽ˜ΡΑ «¢Α 樘Α Ž×«æ½αËêæ×êæΑ ځæ«ËΑ «ÞΑ ¨«£¨ΡΑ 樘ژΑ ö«½½Α ÄËæΑ ˜Α Þ«£Ä«ĈŽÄæΑ «ÄŽÚ˜Þ˜Α «ÄΑ Ëêæ×êæΑ
despite high savings and investment.
14

ANALYSIS:
ANALYSIS:
Scope for Elimination: H¢Α½½Α樘Α¢ËêÚΑË×æ«ËÄÞΑ«ÄΑ樘ΑÙê˜Þæ«ËÄΡ only option (d) as a proper mathematical
relationship with capital formation. b¨˜ΑË樘ÚΑË×æ«ËÄÞΑÁüΑ¨õ˜ΑÄΑ˜ą˜ŽæΡΑêæΑ樘ΑށØΑŽÄÄËæΑ˜ΑÙêÄα
æ«Ĉ˜”Αށæ«Þ¢ŽæËÚ«½üΡΑö¨˜Ú˜ÞΑŽ×«æ½αËêæ×êæΑځæ«ËÄΑŽÄΑ˜Α˜û×ژÞޘ”ΑÁ樘Á櫎½½üΑÞΑ×«æ½έHêæ×êæΠΑ+¢Α
¢ËÚΑ樘ΑށØΑŽ×«æ½ΑλÄêØځæËÚμΡΑËêæ×êæΑλ”˜ÄËëāæËÚμΑ«ÞΑ¨«£¨˜ÚΡΑ樘ÄΑŽ×«æ½αËêæ×êæΑö«½½Α˜Α½ËöΠΑb¨êÞΡΑΑ
½ËöΑŽ×«æ½αËêæ×êæΑځæ«ËΑ«ÞΑ”˜Þ«Ú½˜Π

SOURCE : : Business Studies (NCERT Class 12)

Please Note:
õ˜ÄΑ æ¨Ë꣨Α Α Þ«ŽΑ ŽËϘ×æΡΑ «æΑ «ÞΑ ÄËæΑ ”«Ú˜Žæ½üΑ ŽËõ˜Ú˜”Α «ÄΑ 樘Α BWbΑ AŽÚ˘ŽËÄËëŽÞΑ Ë˺Α
뽁ÞÞΑ͊͋μΡΑ樘ΑŽËϘ×æΑ«ÞΑ˜û×½«Ä˜”Α«ÄΑΑŽ¨×æ˜ÚΑ«ÄΑ樘ΑBWbΑ êޫĘÞÞΑ[æꔫ˜ÞΑË˺Α뽁ÞÞΑ͊͋μΠΑ+¢Α
æ«Ã˜Α טÚëæÞΑõ˜ÚüΑ ޘ½˜Žæ˜”Α ×ËÚæ«ËÄÞΑ Ë¢Α樘Α BWbΑ ½ÞÞΑ ͊͋Α êޫĘÞÞΑ [æꔫ˜ÞΑ Ž¨×æ˜ÚÞΑ ŽÄΑ ˜Α
ژ”ΠΑE.g. Chapter 13 of the Class 12 Business StudiesΑ˜û×½«ÄÞΑ樘ΑŽËϘ×æΑË¢Α×«æ½ΑHêæ×êæΑ
Wæ«ËΑÄ”Α+HWΑλ+ĎژØÄ恽Α×«æ½ΑHêæ×êæΑWæ«Ëμ

Additional Information
Incremental Capital Output Ratio (ICOR) «ÞΑ Α ØæÚ«ŽΑ 樁æΑ ÞޘÞޘÞΑ 樘Α Áڣ«Ä½Α
ÃËêÄæΑË¢Α«Äõ˜ÞæØÄæΑŽ×«æ½Αʎ˜ÞށÚüΑ¢ËÚΑΑŽËêÄæÚüΑËÚΑË樘ÚΑ˜Äæ«æüΑæËΑ£˜Ä˜Úæ˜Α樘Α
ĘûæΑêÄ«æΑË¢Α×Ú˔êŽæ«ËÄΠΑ
Α½Ëö˜ÚΑ+HWΑ«ÞΑ×ژ¢˜Úژ”ΑÞΑ«æΑ«Ä”«Žæ˜ÞΑΑŽËêÄæÚüϖÞΑ×Ú˔êŽæ«ËÄΑ«ÞΑÃËژΑ˜ĆŽ«˜ÄæΠΑHõ˜Ú½½ΡΑ
Α¨«£¨˜ÚΑ+HWΑõ½ê˜Α«ÞΑÄËæΑ×ژ¢˜Úژ”Α˜ŽêޘΑ«æΑ«Ä”«Žæ˜ÞΑ樁æΑ樘Α˜Äæ«æüϖÞΑ×Ú˔êŽæ«ËÄΑ«ÞΑ
«Ä˜ĆŽ«˜ÄæΠΑ b¨˜Α ؁ÞêژΑ «ÞΑ êޘ”Α ×ژ”ËëāÄæ½üΑ «ÄΑ ”˜æ˜ÚëīģΑ Α ŽËêÄæÚüϖÞΑ ½˜õ˜½Α Ë¢Α
×Ú˔êŽæ«ËÄΑ˜ĆŽ«˜ÄŽüΠ

Year 2022 REAL SECTOR

Q3. Which of the following activities constitute real sector in the economy?
͊ΠΑ"ÚØÚÞΑ¨Úõ˜Þæ«Ä£Α樘«ÚΑŽÚË×Þ
͋ΠΑb˜û櫽˜Αë½½ÞΑŽËÄõ˜Úæ«Ä£ΑځöΑŽËææËÄΑ«ÄæËΑ¢Ú«ŽÞ
͌ΠΑΑŽËÃØڎ«½ΑÄºΑ½˜Ä”«Ä£ΑÃËĘüΑæËΑΑæځ”«Ä£ΑŽËÃׁÄü
͍ΠΑΑŽËÚ×Ëځæ˜ΑË”üΑ«ÞÞê«Ä£ΑWêט˜Α˜ÄËëā昔Α ËĔÞΑËõ˜ÚޘÞ
[˜½˜ŽæΑ樘ΑŽËÚژŽæΑÄÞö˜ÚΑêÞ«Ä£Α樘ΑŽË”˜Α£«õ˜ÄΑ˜½Ëö΢
λμΑ͊ΑÄ”Α͋ΑËĽü
λμΑ͋ΡΑ͌ΑÄ”Α͍ΑËĽü
λŽμΑ͊ΡΑ͌ΑÄ”Α͍ΑËĽü
λ”μΑ͊ΡΑ͋ΡΑ͌ΑÄ”Α͍

ANSWERS A
15

EXPLANATION
b¨˜Αژ½ΑޘŽæËÚΑË¢ΑÄΑ˜ŽËÄËÃüΑ«ÞΑ樘Αº˜üΑޘŽæ«ËÄΑÞΑŽæ«õ«æ«˜ÞΑË¢Α樫ÞΑޘŽæËÚΑטÚÞꁔ˜Α˜ŽËÄËëŽΑËêæ×êæΑÄ”Α
«ÞΑژ×ژޘÄ昔ΑüΑæ¨ËޘΑ˜ŽËÄËëŽΑޘ£Ã˜ÄæÞΑ樁æΑÚ˜Α˜ÞޘÄ櫁½Α¢ËÚΑ樘Α×ÚˣژÞÞΑË¢Α#TΑË¢Α樘Α˜ŽËÄËÃüΠΑ
"ËÚΑ «ÄÞæÄŽ˜ΡΑ¢ÚØÚÞΑ ¨Úõ˜ÚÞæ«Ä£Α樘«ÚΑ ŽÚË×ÞΑ ËÚΑæ˜û櫽˜Α ë½½ÞΑ ŽËÄõ˜Úæ«Ä£Α ځöΑ ŽËææËÄΑ «ÄæËΑ¢Ú«ŽÞΑ ˜ÄÞêژΑ
«ÄŽÚ˜Þ˜ΑË¢Α˜ŽËÄËëŽΑËêæ×êæΑÄ”Α«ÄΑæêÚÄΑ×ÚˣژÞÞΑË¢Α#TΠΑb¨˜ΑޘŽæËÚΑ«ÞΑŽÚꎫ½Α¢ËÚΑ樘ΑÞêÞ恫ā«½«æüΑË¢Α樘Α
˜ŽËÄËÃüΑ˜ŽêޘΑË¢Α«æÞΑ×Ú˔êŽæ«õ˜ΑŽ×«½«æüΑæËΑؘæΑāæ«ËÄÞϐΑ££Ú˜£æ˜Α”˜ÃÄ”Π Hence options 1 and
2 are correct.
HÄΑ Ë樘ÚΑ ¨Ä”ΡΑ樘ΑĈāĎ«½Α ޘŽæËÚΑ «ÞΑ Α ޘŽæ«ËÄΑ Ë¢Α樘Α ˜ŽËÄËÃüΑ Á”˜Α ê×Α Ë¢ΑĈÚÃÞΑ Ä”Α «ÄÞæ«æêæ«ËÄÞΑ樁æΑ
×ÚËõ«”˜Α ĈāĎ«½Α ޘÚõ«Ž˜ÞΑ æËΑ ŽËÃØڎ«½Α Ä”Α ژ恫½Α ŽêÞæËØÚÞΠΑ b¨˜Α ĈāĎ«½Α ޘŽæËÚΑ £˜Ä˜Úæ˜ÞΑ Α £Ë˔Α
×ËÚæ«ËÄΑË¢Α«æÞΑژõ˜Äê˜Α¢ÚËÃΑ½ËÄÞΑÄ”ΑÃËÚ棁£˜ÞΠΑb¨«ÞΑޘŽæËÚΑŽËÃ×ګޘÞΑΑÚˁ”Αځģ˜ΑË¢Α«Ä”êÞæÚ«˜ÞΑ«ÄŽ½ê”α
«Ä£ΑÄºÞΡΑ«Äõ˜ÞæØÄæΑŽËÃׁī˜ÞΡΑ«ÄÞêځϘΑŽËÃׁī˜ÞΡΑÄ”Αژ½Α˜Þææ˜ΑĈÚÃÞΠΑb¨êÞΡΑΑŽËÃØڎ«½ΑÄºΑ
½˜Ä”«Ä£Α ÃËĘüΑ æËΑ Α æځ”«Ä£Α ŽËÃׁÄüΑ ËÚΑ Α ŽËÚ×Ëځæ˜Α Ë”üΑ «ÞÞê«Ä£Α Úêט˜Α ”˜ÄËëā昔Α ËĔÞΑ Ëõ˜ÚޘÞΑ
ŽËÄÞæ«æêæ˜Α ĈāĎ«½Α ޘŽæËÚΑ Žæ«õ«æ«˜ÞΑ Ä”Α ÄËæΑ ژ½Α ޘŽæËÚΑ Žæ«õ«æ«˜ÞΠΑ Hence options 3 and 4 are not
correct.

ANALYSIS:

Scope for elimination


ΑŽËÃØڎ«½ΑÄºΑ½˜Ä”«Ä£ΑÃËĘüΑæËΑΑæځ”«Ä£ΑŽËÃׁÄüΑÄ”ΑΑŽËÚ×Ëځæ˜ΑË”üΑ«ÞÞê«Ä£ΑWêט˜Α˜ÄËëāæα
˜”Α ËĔÞΑËõ˜ÚޘÞΡΑΑ樘ޘΑÚ˜ΑĈāĎ«½ΑŽæ«õ«æ«˜ÞΑÄ”ΑÄËæΑژ½ΡΑÞËΑ«¢Αö˜Α˜½«Ã«Äæ˜Α͌ΑÄ”Α͍ΡΑö˜ΑÚ˜Α½˜¢æΑö«æ¨Α
Ë×æ«ËÄÞΑ͊ΑÄ”Α͋ΠΑ+ÄΑÞꎨΑÙê˜Þæ«ËÄÞΡΑËĘΑÞ¨Ë꽔ΑæÚüΑæËΑĈĔΑޫ뽁ÚΑË×æ«ËÄÞΑλ×êæΑ樘ÃΑæË£˜æ¨˜ÚμΑÄ”Α樘ÄΑޘׁα
ځæ˜Α¢ÚËÃΑ”«Þޫ뽁ÚΑË×æ«ËÄÞΠΑ;Ë£«Ž½½üΡΑ½½Α樘Α¢ËêÚΑË×æ«ËÄÞΑŽÄÄËæΑ˜ΑŽËÚژŽæΑæË£˜æ¨˜ÚΠ

SOURCE : ¨ææ×Þ΢έέöööΠ«Äõ˜ÞæËט”«ΠŽËÃέæ˜ÚÃÞέ¢έĈāĎ«½εޘŽæËÚЁÞ×
https://www.imf.org/external/region/tlm/rr/pdf/Jan1.pdf
16

MONEY AND MONEY SUPPLY


(6 Questions)

Year 2022 TYPES OF MONEY

Q1. Which one of the following statements correctly describes the meaning of legal tender money?
(a) The money which is tendered in courts of law to defray the fee of legal cases
(b) The money which a creditor is under compulsion to accept in settlement of his claims
(c) The bank money in the form of cheques, drafts, bills of exchange etc.
(d) The metallic money in circulation in a country

ANSWERS B

EXPLANATION

;˜£½Αæ˜Ä”˜ÚΑ«ÞΑÄüΑËƎ«½Αؔ«êÃΑË¢ΑׁüØÄæΑژŽË£Ä«ā˜”ΑüΑ½öΑ樁æΑŽÄΑ˜Αêޘ”ΑæËΑ˜ûæ«Ä£ê«Þ¨ΑΑ×ꍽ«ŽΑ
ËÚΑ×Ú«õæ˜Α”˜æΡΑËÚΑؘæΑΑĈāĎ«½Αˍ½«£æ«ËÄΠΑΑŽÚ˜”«æËÚΑ«ÞΑˍ½«£æ˜”ΑæËΑŽŽ˜×æΑ½˜£½Αæ˜Ä”˜ÚΑæËöÚ”Αژׁüα
ØÄæΑË¢ΑΑ”˜æΠΑ;˜£½Αæ˜Ä”˜ÚΑŽÄΑËĽüΑ˜Α«ÞÞꘔΑüΑ樘Αāæ«Ëā½ΑË”üΑ樁æΑ«ÞΑêæ¨ËÚ«ā˜”ΑæËΑ”ËΑÞËΠΑLegal
tenders cannot be refused by any citizen of the country for settlement of any kind of transaction.
¨˜Ùê˜ÞΑ ”ځöÄΑ ËÄΑ ށõ«Ä£ÞΑ ËÚΑ ŽêÚژÄæΑ ŽŽËêÄæÞΡΑ ¨Ëö˜õ˜ÚΡΑ ŽÄΑ ˜Α ژ¢êޘ”Α üΑ ÄüËĘΑ ÞΑ Α Ã˔˜Α Ë¢Α
ׁüØÄæΠΑ(˜ÄŽ˜ΡΑ”˜ÃÄ”Α”˜×ËÞ«æÞΑÚ˜ΑÄËæΑ½˜£½Αæ˜Ä”˜ÚÞΠ Hence option (b) is the correct answer.

ANALYSIS:

SOURCE : Macroeconomics (NCERT: Class 12)

ûŽ˜Ú×æΑ¢ÚËÃΑBWb
17

Year
Year2019
2019 MONEY MULTIPLIER

Q2. The money multiplier in an economy increases with which one of the following?
λμΑ+ĎژÞ˜Α«ÄΑ樘ΑŽÞ¨ΑژޘÚõ˜Αځæ«Ë
λμΑ+ĎژÞ˜Α«ÄΑ樘ΑÄº«Ä£Α¨«æΑË¢Α樘Α×Ë×꽁æ«ËÄ
λŽμΑ+ĎژÞ˜Α«ÄΑ樘ΑÞææêæËÚüΑ½«Ù꫔«æüΑځæ«Ë
λ”μΑ+ĎژÞ˜Α«ÄΑ樘Α×Ë×꽁æ«ËÄΑË¢Α樘ΑŽËêÄæÚü

ANSWERS B

EXPLANATION
C

AËĘüΑAê½æ«×½«˜ÚΑ«ÞΑ樘Αځæ«ËΑË¢Α樘ΑÞæˎºΑË¢ΑÃËĘüΑæËΑ樘ΑÞæˎºΑË¢Α¨«£¨Α×Ëö˜Ú˜”ΑÃËĘüΠΑ+æΑ«ÞΑ樘Αژ½æ«ËÄÞ¨«×Α
˜æö˜˜ÄΑ樘ΑÃËĘæÚüΑÞ˜ΑÄ”ΑÃËĘüΑÞê××½üΑË¢ΑÄΑ˜ŽËÄËÃüΠΑ+æΑ˜û×½«ÄÞΑ樘Α«ÄŽÚ˜Þ˜Α«ÄΑ樘ΑÃËêÄæΑË¢Α
ŽÞ¨Α«ÄΑŽ«ÚŽê½æ«ËÄΑ£˜Ä˜Úæ˜”ΑüΑ樘ΑÄºÞϖΑ«½«æüΑæËΑ½˜Ä”ΑÃËĘüΑËêæΑË¢Α樘«ÚΑ”˜×ËÞ«æËÚÞϖΑ¢êĔÞΠΑb¨˜Ú¢ËژΡΑ«æΑ
ژ¢˜ÚÞΑæËΑ¨ËöΑÄΑ«Ä«æ«½Α”˜×ËÞ«æΑŽÄΑ½˜”ΑæËΑΑ«££˜ÚΑĈā½Α«ÄŽÚ˜Þ˜Α«ÄΑ樘ΑæË恽ΑÃËĘüΑÞê××½üΠ

"ËÚΑ˜ûÃ×½˜ΡΑ«¢Α樘ΑŽËÃØڎ«½ΑÄºÞΑ£«ÄΑ”˜×ËÞ«æÞΑË¢ΑWÞ͊Α;º¨ΑÄ”Α樫ÞΑ½˜”ÞΑæËΑΑĈā½ΑÃËĘüΑÞê××½üΑË¢ΑWÞΑ
͉͊Α½º¨ΠΑb¨˜ΑÃËĘüΑÃê½æ«×½«˜ÚΑ«ÞΑ͉͊ΠΑb¨˜Ú˜¢ËژΡΑ«ÄŽÚ˜Þ˜Α«ÄΑÄº«Ä£Α¨«æΑË¢Α樘Α×Ë×꽁æ«ËÄΑöË꽔Α½˜”ΑæËΑ
ÃËژΑ”˜×ËÞ«æÞΑÄ”Α¨˜ÄŽ˜Α«ÄŽÚ˜Þ˜Α«ÄΑAËĘüΑAê½æ«×½«˜ÚΠΑHence (b) is the correct answer.

ANALYSIS:
ANALYSIS:
+æΑ«ÞΑΑژŽêÚÚ«Ä£Α樘ØΑÄ”Α樫ÞΑ«ÞΑΑÞ«ŽΑŽËϘ×æΑÞæꔫ˜”Α«ÄΑÁŽÚ˘ŽËÄËëŽÞΑÄ”ΑÄüËĘΑ×טÚ«Ä£Α¢ËÚΑ
˜ûÃÞΑÞ¨Ë꽔Α˜ΑŽËõ˜Ú«Ä£Α樫ÞΑæË׫ŽΠ

Scope for elimination

+ĎژÞ«Ä£ΑWWΑÄ”Α[;WΑö«½½Αˍõ«ËêÞ½üΑژ”ꎘΑ樘ΑÃËĘüΑĉËöΑÄ”Α«ÄŽÚ˜Þ«Ä£Α×Ë×꽁æ«ËÄΑ¨ÞΑÄËΑ«ÃׁŽæΑËÄΑ
樘ΑÃËĘüΑÃê½æ«×½«˜ÚΠ By this we can eliminate three options (a,c,d).

SOURCE : Macroeconomics (NCERT Class 12)


18

ûŽ˜Ú×æΑ¢ÚËÃΑBWb
b¨˜ΑËõ˜Α”˜ÞŽÚ«×æ«ËÄΑ¨ÞΑ˜˜ÄΑ恺˜ÄΑ¢ÚËÃΑÄΑÚ櫎½˜Α¢ÚËÃΑ樘ΑΑb«Ã˜ÞΑË¢Α+Ĕ«
[Ëêڎ˜΢Α¨ææ×Þ΢έέæ«Ã˜ÞËĈĔ«Π«Ä”«æ«Ã˜ÞЎËÃݍ½Ë£Þέ樘αÁŽÚËα¢«Ú˜έö¨æα”Ú«õ˜ÞαÄᘎËÄËÃüαÃËĘüέ

Year
Year2020
2020 MONEY SUPPLY

Q3. If you withdraw Rs. 1,00,000 in cash from your Demand Deposit Account at your bank, the
«Ãؔ«æ˜Α˜ą˜ŽæΑËÄΑ££Ú˜£æ˜ΑÃËĘüΑÞê××½üΑ«ÄΑ樘Α˜ŽËÄËÃüΑö«½½Α˜Α
λμΑæËΑژ”ꎘΑ«æΑüΑWÞΠΑ͊Ρ͉͉Ρ͉͉͉Α
λμΑæËΑ«ÄŽÚ˜Þ˜Α«æΑüΑWÞΠΑ͊Ρ͉͉Ρ͉͉͉Α
λŽμΑæËΑ«ÄŽÚ˜Þ˜Α«æΑüΑÃËژΑ樁ÄΑWÞΠΑ͊Ρ͉͉Ρ͉͉͉Α
λ”μΑæËΑ½˜õ˜Α«æΑêϨÄ£˜”

ANSWERS D

EXPLANATION
b¨˜Ú˜ΑÚ˜Α͍ΑŽËϘ×æÞΑË¢ΑÃËĘüΑÞê××½ü΢ΑA͊ΡΑA͋ΡΑA͌ΑÄ”ΑA͍Α
A͊ΑϽΑΑϹΑΑϹΑHΑ
C Is the currency held by the public. (Public money means that money which is held by everybody other than the
government and the banks. It includes companies, general organisations, households. It does not include inter-bank
or government deposits in banks)
DD means net demand deposits with banks. ‘Net’ here indicates the deposits of only the public in banks.
OD means other deposits. These are the deposits with the RBI, held by certain individuals and institutions • Individu-
als - like the former governors of the RBI • institutions- like IMF deposits

A͌ΑϽΑA͊ΑϹΑbΑϽΑΑϹΑΑϹΑHΑϹΑbΑλ Úˁ”ΑÃËĘüμΑϖbϖΑ؁ÄÞΑæ«Ã˜Α”˜×ËÞ«æÞΑ
A͌ΑÞ¨ËöÞΑ樘ΑæË恽Α×êڎ¨Þ«Ä£Α×Ëö˜ÚΑ«ÄΑ樘Α˜ŽËÄËÃüΠΑTherefore, when we say money supply in general,
it means M3.
[ËΡΑÄËÚÁ½½üΡΑ«ÄΑĘöÞׁטÚÞΑ˜æŽΠΑö¨˜ÄΑ樘ΑöËڔΑÃËĘüΑÞê××½üΑ«ÞΑêޘ”ΡΑ«æΑ؁ÄÞΑA͌Αλ[ËØæ«Ã˜ÞΡΑA͊Α«ÞΑ½ÞËΑ
êޘ”ΑαΑ؁īģΑ͉͉͊АΑ½«Ù꫔ΑÃËĘüΑαΑêæΑËõ˜Ú½½ΑTTΑ«ÞΑÞ¨ËöÄΑüΑA͌μΑ
BËöΡΑ«ÄΑ樘Α£«õ˜ÄΑŽÞ˜ΡΑö¨«½˜Α樘ΑϖϖΑŽËÃ×ËĘÄæΑö«½½Α¢½½ΑüΑWÞΠΑ͊Ρ͉͉Ρ͉͉͉ΡΑ樘ΑϖϖΑŽËÃ×ËĘÄæΑö«½½Α«ÄŽÚ˜Þ˜Α
üΑWÞΠΑ͊Ρ͉͉Ρ͉͉͉ΡΑ樘ژüΡΑ½˜õ«Ä£Α樘ΑÃËĘüΑÞê××½üΑêϨÄ£˜”ΠΑHence option (d) is the correct answer.
19

ANALYSIS:
ANALYSIS:
Scope for elimination
½æ¨Ë꣨Α樘ژΑ «ÞΑ ÄËΑ ”«Ú˜ŽæΑ ގËטΑ¢ËÚΑ ˜½«Ã«Äæ«ËÄΡΑ «¢Αö˜Α ʘ”ΑæËΑ ºÄËöΑ樁æΑ恺«Ä£Α ÃËĘüΑ¢ÚËÃΑ ”˜ÃÄ”Α
”˜×ËÞ«æΑ ŽŽËêÄæΑ λ¢ËÚΑ ˜ûÃ×½˜Α γΑ ށõ«Ä£ÞΑ ŽŽËêÄæμΑö«½½Α ÄËæΑ ˜ÚÞ˜Α樘Α ÃËĘüΑ ¢ÚËÃΑ樘Α ££Ú˜£æ˜Α ÃËĘüΑ
Þê××½üΑ«ÄΑ樘Α˜ŽËÄËÃüΑÞΑ«æΑ¨ÞΑËĽüΑŽ¨Ä£˜”Α¨Ä”ÞΑλ¢ÚËÃΑº˜˜×«Ä£Α«æΑö«æ¨Α樘ΑÄºΑæËΑ樘Α”˜×ËÞ«æËÚμΡΑ樘Α
ÃËĘüΑ«ÞΑÞ櫽½Α樘ژΑ«ÄΑ樘Α˜ŽËÄËÃüΑλ«Ãؔ«æ˜Α˜ą˜ŽæμΠΑ(˜ÄŽ˜ΡΑΑ£˜Ä˜Ú½Α«”˜ΑŽË꽔Α¨õ˜Α¨˜½×˜”ΑæËËΑæËΑ
ÄÞö˜ÚΑ樫ÞΑÙê˜Þæ«ËÄΠΑ

SOURCE : Macroeconomics (NCERT Class 12)

Year 2021 BLACK MONEY

r¨«Ž¨ΑËĘΑË¢Α樘Α¢Ë½½Ëö«Ä£Α˜ą˜ŽæÞΑË¢ΑŽÚ˜æ«ËÄΑË¢Α½ŽºΑÃËĘüΑ«ÄΑ+Ĕ«Α¨ÞΑ˜˜ÄΑ樘ΑÁ«ÄΑŽêޘΑË¢Α
worry to the Government of India?
λμΑ«õ˜ÚÞ«ËÄΑË¢ΑژÞËêڎ˜ÞΑæËΑ樘Α×êڎ¨Þ˜ΑË¢Αژ½Α˜Þææ˜ΑÄ”Α«Äõ˜ÞæØÄæΑ«ÄΑ½êûêÚüΑ¨ËêÞ«Ä£Α
λμΑ+Äõ˜ÞæØÄæΑ«ÄΑêÄ×Ú˔êŽæ«õ˜ΑŽæ«õ«æ«˜ÞΑÄ”Α×êڎ¨Þ˜ΑË¢Α×ژõ«ËêÞΑÞæËĘÞΡΑ¶˜ö˜½½˜ÚüΡΑ£Ë½”ΡΑ˜æŽΠΑ
λŽμΑ;Ú£˜Α”Ëāæ«ËÄÞΑæËΑ×˽«æ«Ž½ΑׁÚ櫘ÞΑÄ”Α£ÚËöæ¨ΑË¢Αژ£«Ëā½«ÞÃΑ
λ”μΑ;ËÞÞΑË¢Αژõ˜Äê˜ΑæËΑ樘Α[ææ˜ΑûŽ¨˜Ùê˜ÚΑ”ê˜ΑæËΑæûΑ˜õÞ«ËÄ

ANSWERS D

EXPLANATION

b¨˜Ú˜Α«ÞΑÄËΑËƎ«½Α”˜ĈÄ«æ«ËÄΑË¢Α½ŽºΑÃËĘüΑ«ÄΑ˜ŽËÄËëŽΑ樘ËÚüΡΑö«æ¨Αޘõ˜Ú½Α”«ą˜Ú˜ÄæΑæ˜ÚÃÞΑÞꎨΑÞΑׁځ½α
½˜½Α˜ŽËÄËÃüΡΑ½ŽºΑÃËĘüΡΑ½ŽºΑ«ÄŽËØÞΡΑêāŽŽËêÄ昔Α˜ŽËÄËÃüΡΑ«½½˜£½Α˜ŽËÄËÃüΑÄ”Α«Úژ£ê½ÚΑ˜ŽËÄËÃüΑ
½½Α˜«Ä£Αêޘ”ΑÃËژΑËÚΑ½˜ÞÞΑÞüÄËÄüÃËêÞ½üΠΑb¨˜ΑÞ«Ã×½˜ÞæΑ”˜ĈÄ«æ«ËÄΑË¢Α½ŽºΑÃËĘüΑŽË꽔Α×ËÞÞ«½üΑ˜ΑÃËĘüΑ
樁æΑ«ÞΑ¨«””˜ÄΑ¢ÚËÃΑæûΑêæ¨Ëګ櫘ÞΠΑb¨æΑ«ÞΡΑ½ŽºΑÃËĘüΑŽÄΑŽËØΑ¢ÚËÃΑæöËΑÚˁ”ΑŽæ˜£ËÚ«˜Þ΢Α«½½˜£½ΑŽæ«õ«α
æüΑÄ”Α½˜£½ΑêæΑêÄژ×ËÚ昔ΑŽæ«õ«æüΠΑb¨˜ΑĈÚÞæΑŽæ˜£ËÚüΑ«ÞΑ樘ΑÃËژΑˍõ«ËêÞΑË¢Α樘ΑæöËΠΑAËĘüΑ樁æΑ«ÞΑ˜ÚʔΑ
æ¨ÚË꣨Α«½½˜£½ΑŽæ«õ«æüΑ«ÞΑˍõ«ËêÞ½üΑÄËæΑژ×ËÚ昔ΑæËΑ樘ΑæûΑêæ¨Ëګ櫘ÞΡΑÄ”ΑÞËΑ«ÞΑ½ŽºΠΑb¨˜ΑޘŽËĔΑŽæ˜£Ëα
ÚüΑŽËÃ×ګޘÞΑ«ÄŽËØΑ¢ÚËÃΑ½˜£½ΑŽæ«õ«æüΑ樁æΑ«ÞΑÄËæΑژ×ËÚ昔ΑæËΑ樘ΑæûΑêæ¨Ëګ櫘ÞΠΑ
"ËÚΑ˜ûÃ×½˜ΡΑ½˜æΑêÞΑÞÞêØΑ樁æΑΑ׫˜Ž˜ΑË¢Α½Ä”Α«ÞΑÞ˽”ΡΑö«æ¨Α樘ΑׁüØÄæΑÁ”˜Α«ÄΑ樘Α×ÚË×ËÚæ«ËÄΑË¢Α͏͉АΑüΑ
Ž¨˜Ùê˜ΑËÚΑ˜½˜ŽæÚËÄ«ŽΑæځÄÞ¢˜ÚΡΑÄ”Α͍͉АΑ«ÄΑŽÞ¨ΠΑ+¢Α樁æΑ͍͉АΑŽÞ¨ΑŽËÃ×ËĘÄæΑ«ÞΑÄËæΑژ×ËÚ昔ΑæËΑ樘Α+ĎËØΑ
bûΑ˜×ÚæØÄæΡΑ樘ÄΑ«æΑ«ÞΑ½ŽºΑÃËĘüΠΑΑ½Ú£˜ΑÄê͘ÚΑË¢ΑÞÁ½½ΑÞ¨Ë×ÞΑÚËêĔΑ樘ΑŽËêÄæÚüΑ½ÃËÞæΑ˜ûŽ½êα
Þ«õ˜½üΑ”ËΑêޫĘÞÞΑ«ÄΑŽÞ¨Αö«æ¨ËêæΑژŽ˜«×æÞΠΑ½½ΑË¢Α樫ÞΑŽË꽔Α×Ëæ˜Ä櫁½½üΑ˜Α½ŽºΑÃËĘüΠΑÄË樘ÚΑÁ¶ËÚΑ
ÞËêڎ˜ΑË¢Α½ŽºΑÃËĘüΑ«ÞΑ«ÄŽËØΑ˜ÚʔΑüΑŽËÃׁī˜ÞΑ樁æΑ«ÞΑÚËê昔Αæ¨ÚË꣨ΑÞ¨˜½½ΑŽËÃׁī˜ÞΑÚˁ”ΡΑ
樘ژüΑ˜õ”«Ä£ΑæûΑêæ¨Ëګ櫘ÞΠ
b¨˜Ú˜ΑÚ˜Αޘõ˜Ú½ΑöüÞΑæËΑŽêڍΑ½ŽºΑÃËĘüΑÄ”Α樘ΑĈÚÞæΑ«ÞΑæ¨ÚË꣨Α½˜£«Þ½æ«õ˜ΑŽæ«ËÄΠΑb¨˜Α£Ëõ˜ÚÄØÄæΑ¨ÞΑ
½Ú˜”üΑ˜ÄŽæ˜”Αޘõ˜Ú½Α½öÞΑ樁æΑޘ˜ºΑæËΑ¢ËÚÁ½«Þ˜Α樘Α˜ŽËÄËÃüΑÄ”ΑÁº˜Α«æΑʎ˜ÞށÚüΑæËΑژ×ËÚæΑ˜ŽËÄËÃα
«ŽΑæځÄށŽæ«ËÄÞΠΑb¨˜Þ˜Α«ÄŽ½ê”˜Α樘Α˜Äæځ½Α#Ë˔ÞΑÄ”Α[˜Úõ«Ž˜ÞΑbûΑŽæΡΑ樘ΑõÚ«ËêÞΑ#[bΑŽæÞΑæΑ樘Α[ææ˜Α
½˜õ˜½ÞΡΑ 樘Α ½ŽºΑ AËĘüΑ λfĔ«ÞŽ½Ëޘ”Α "Ëژ«£ÄΑ +ĎËØΑ Ä”ΑÞޘæÞμΑ Ä”Α +Ã×ËÞ«æ«ËÄΑ Ë¢ΑbûΑŽæΡΑ ͉͎͋͊ΡΑ 樘Α
˜ÄÃ«ΑbځÄށŽæ«ËÄÞΑλTÚ˨««æ«ËÄμΑØĔØÄæΑŽæΡΑÄ”Α樘Α"꣫æ«õ˜ΑŽËÄËëŽΑHą˜Ä”˜ÚÞΑŽæΑæËΑāØΑΑ
¢˜öΠΑÄË樘ÚΑØæ¨Ë”Α˜Ã×½Ëü˜”ΑüΑ樘Α£Ëõ˜ÚÄØÄæΑæËΑÁº˜Α«æΑ¨Ú”˜ÚΑ¢ËÚΑæځÄށŽæ«ËÄÞΑæËΑ˜Α¨«””˜ÄΑ«ÞΑæËΑ
ÁĔæ˜Α樘Αژ×ËÚæ«Ä£ΑË¢ΑTBΑ¢ËÚΑæځÄށŽæ«ËÄÞΑË¢ΑÃËژΑ樁ÄΑϲ͋Π͎Α½º¨ΡΑÄ”Α樘Α×Ú˨««æ«ËÄΑË¢ΑŽÞ¨ΑژŽ˜«×æÞΑ
Ë¢Αϲ͋Α½º¨ΑËÚΑÃËژΑÄ”ΑΑטā½æüΑ˜ÙꁽΑæËΑ樘ΑÃËêÄæΑË¢ΑÞꎨΑژŽ˜«×æÞΑ«¢ΑΑטÚÞËÄΑŽËÄæځõ˜Ä˜ÞΑ樘Α×ÚËõ«α
Þ«ËÄΠΑÞææ˜Ã˜ÄæÞΑ£«õ˜ÄΑ«ÄΑË×æ«ËÄΑΡΑΡΑÄ”ΑŽΑÚ˜ΑöüÞΑË¢ΑŽÚ˜æ«ËÄΑË¢Α½ŽºΑÃËĘüΠΑHence option (d) is the
correct answer.
20

ANALYSIS:

W˜Ž¨«Ä£Α樘ΑŽËÚژŽæΑÄÞö˜ÚΑæËΑ樫ÞΑÙê˜Þæ«ËÄΑ«ÞΑ”˜×˜Ä”˜ÄæΑËÄΑ樘ΑŽÄ”«”æ˜ϐÞΑŽËϘ×æꁽΑŽ½Ú«æüΠΑ(Ëö˜õ˜ÚΡΑ«æΑ
«ÞΑÞ˜”ΑËÄΑ××½«˜”έ×ځŽæ«Ž½ΑË×æ«ËÄÞΠΑõ˜ÚΑޫϘΑ樘Α”˜ÃËĘæ«Þæ«ËÄΡΑϏ½ŽºΑÃËĘüϐΑ¨ÞΑ½öüÞΑ˜˜ÄΑ«ÄΑ樘Α
ĘöÞΠ

Scope for elimination


Scope for Elimination:Α½Ú˜”üΑ£«õ˜ÄΑ«ÄΑ樘Α˜û×½Äæ«ËÄΠΑ

SOURCE : https://www.thehindu.com/business/Economy/what-is-black-mon-
˜üαÄ”αö¨üα«Þα«æαæËαÞËα”«ĆŽê½æαæËαÙêÄæ«¢üα«æ݁Ú櫎½˜͎͋͑͋͋͑͑͌ИŽ˜

Year 2021 MONEY MULTIPLIER

ΑV͎ΠΑb¨˜ΑÃËĘüΑÃê½æ«×½«˜ÚΑ«ÄΑÄΑ˜ŽËÄËÃüΑ«ÄŽÚ˜Þ˜ÞΑö«æ¨Αö¨«Ž¨ΑËĘΑË¢Α樘Α¢Ë½½Ëö«Ä£ΧΑ
λμΑ+ĎژÞ˜Α«ÄΑ樘ΑÞ¨ΑW˜Þ˜Úõ˜ΑWæ«ËΑ«ÄΑ樘ΑÄºÞΑ
λμΑ+ĎژÞ˜Α«ÄΑ樘Α[ææêæËÚüΑ;«Ù꫔«æüΑWæ«ËΑ«ÄΑ樘ΑÄºÞΑ
ANALYSIS:
λŽμΑ+ĎژÞ˜Α«ÄΑ樘ΑÄº«Ä£Α¨«æΑË¢Α樘ΑטË×½˜Α
λ”μΑ+ĎژÞ˜Α«ÄΑ樘Α×Ë×꽁æ«ËÄΑË¢Α樘ΑŽËêÄæÚü

ANSWERS C

EXPLANATION
½Ú˜”üΑ˜û×½«Ä˜”Α«ÄΑ樘ΑÙê˜Þæ«ËÄΑ樁æΑ¢˜æêژ”Α«ÄΑ樘ΑΑ[ΑTژ½«ÃÞΑ͉͋͊͒
Formula: Money Multiplier = Money Supply/Base Money
Hence option (c) is the correct answer.

ANALYSIS:

Refer to the question in 2019. The same question was asked, only two options were interchanged.
That directly shows the importance of practicing previous year questions.

SOURCE : Macroeconomics (NCERT Class 12)


21

Year 2023 CENTRAL BANK DIGITAL CURRENCIES (CBDC)

Q6. With reference to Central Bank Digital Currencies, consider the following statements :
1. It is possible to make payments in a digital currency without using US dollar or SWIFT system.
2. A digital currency can be distributed with a condition programmed into it such as a time-fame for spending it.
Which of the statements given above is/are correct?
(a) 1 Only
(b) 2 Only
(c) Both 1 and 2
(d) Neither 1 nor 2

ANSWERS C

EXPLANATION

½æ˜Úāæ«õ˜ÞΑæËΑ [r+"bΑ Ä”Α樘Α fΠ[ΠΑ ”˽½ÚΑ Ú˜Α ŽËëģΑ ¢ÚËÃΑæöËΑ ”«Ú˜Žæ«ËÄÞ΢Α ŽÚü×æˎêÚژĎ«˜ÞΑ Ä”Α Ž˜Äæځ½Α
ÄºΑ”«£«æ½ΑŽêÚژĎ«˜ÞΠΑËêÄæÚ«˜ÞΑö«½½Α˜Α½˜ΑæËΑ”«Ú˜Žæ½üΑ˜ûŽ¨Ä£˜Α”«£«æ½ΑŽêÚژĎ«˜ÞΑ«ÄΑΑ«½æ˜Ú½ΑöüΑÄ”Α
ö«æ¨ËêæΑ£Ë«Ä£Αæ¨ÚË꣨Α[r+"bΑËÚΑޫ뽁ÚΑޘæ潘ØÄæΑÞüÞæ˜ÃÞΠΑHence statement 1 is correct.

r¨˜ÄΑ樘Α明¨Ä˽ˣüΑ½½ËöÞΑޘÃ½˜ÞÞΑÄ”Α«ÄÞæÄæÄ˜ËêÞΑŽËÄõ˜Ú櫍«½«æüΑ¢ÚËÃΑËĘΑÞËõ˜Ú˜«£ÄΑŽêÚژĎüΑ«ÄæËΑ
ÄË樘ÚΡΑ«æΑŽ¨Ä£˜ÞΑ樘Α×ځŽæ«Ž½Αʘ”Α¢ËÚΑΑ”ËëāÄæΑ£½Ë½ΑژޘÚõ˜ΑŽêÚژĎüΠΑ ΑŽË꽔Α˜Α˜Ã×½Ëü˜”Α¢ËÚΑ
Ĉގ½ΑæځÄÞ¢˜ÚÞΑæËΑ¨Ëêޘ¨Ë½”ÞΑËÚΑĈÚÃÞΡΑÞꎨΑÞΑژ½«˜¢ΑËÚΑÞæ«Ãê½êÞΑׁüØÄæÞΠΑ[ꎨΑ¨˜½«ŽË×æ˜ÚΑ”ÚË×ÞΑËÚΑÞêÞ«α
”«˜ÞΑöË꽔Α×Ëæ˜Ä櫁½½üΑ˜ŽËØΑ˜Þ«˜ÚΑö¨˜ÄΑ樘ژΑ«ÞΑö«”˜Þ×ژ”Α”Ë×æ«ËÄΑË¢Α ΑŽŽËêÄæÞΠΑb¨˜ΑæځÄÞ¢˜ÚΑ
ׁüØÄæÞΑ ŽË꽔Α ½ÞËΑ ˜Α ύ×ÚˣځÃÁ½˜ώΡΑ ö«æ¨Α ŽËĔ«æ«ËÄÞΑ ÞꎨΑ ÞΑ ˜û׫ځæ«ËÄΑ ê×ËÄΑ Α Ž˜Ú恫ÄΑ ”æ˜Α ËÚΑ Α
ژÙê«Ú˜Ã˜ÄæΑæËΑÞטĔΑ樘Α¢êĔÞΑæΑŽ˜Ú恫ÄΑõ˜Ä”ËÚÞΠΑHence statement 2 is correct.

ANALYSIS:

b¨«ÞΑÙê˜Þæ«ËÄΑ«ÞΑژ½æ˜”ΑæËΑ Α«ÄΑ£˜Ä˜Ú½ΡΑêæΑ«ÞΑˍõ«ËêÞ½üΑ”˜Ú«õ˜”Α¢ÚËÃΑ樘ΑŽêÚژÄæΑą«ÚÞΠΑb¨˜ΑW˜Þ˜Úõ˜Α
ÄºΑ Ë¢Α +Ĕ«Α «ÞΑ «Ã«Ä£Α æΑ ˜ûׁĔ«Ä£Α樘Α ׫½ËæΑ  Α ×Ú˶˜ŽæΠΑ AÄüΑ ŽËêÄæÚ«˜ÞΑ «ÄΑ樘ΑöËÚ½”Α ¨õ˜Α ÞæÚ昔Α
 ΠΑׁÚæΑ¢ÚËÃΑ樁æΡΑ樘ژΑ«ÞΑ½ÞËΑΑ£ÚËö«Ä£Αæ˜Ä”˜ÄŽüΑÃËÄ£ΑÁÄüΑŽËêÄæÚ«˜ÞΑæËΑŽËêÄæ˜ÚΑ樘Α”˽½ÚΑ¨˜£˜α
ÃËÄüΠΑ[r+"bΑöÞΑ½ÞËΑ«ÄΑ樘ΑĘöÞΑÞΑWêÞÞ«ΑöÞΑژÃËõ˜”Α¢ÚËÃΑ樘Α[r+"bΑޘæ潘ØÄæΑÞüÞæ˜ÃΠΑb¨«ÞΑÙê˜Þα
æ«ËÄΑ«ÞΑæ¨êÞΑ«ÄÞ׫ژ”ΑüΑÁÄüΑŽêÚژÄæΑ˜õ˜ÄæÞΠΑfT[Α¨ÞΑ˜˜ÄΑÞº«Ä£ΑÙê˜Þæ«ËÄÞΑژ½æ˜”ΑæËΑׁüØÄæΑ«Äæ˜Ú¢Žα
˜ÞΡΑ”«£«æ½ΑÄº«Ä£ΑÞüÞæ˜ÃÞΑ¢ÚËÃΑæ«Ã˜ΑæËΑæ«Ã˜Π More such questions can be expected in the future, not
¶êÞæΑژ½æ˜”ΑæËΑ樘ΑÞ«ŽΑ¢˜æêژÞΑËÚΑ”˜ĈÄ«æ«ËÄÞΑêæΑ½ÞËΑژ½æ˜”ΑæËΑ樘Α××½«Žæ«ËÄÞΑË¢Α樘ΑĘöΑ明¨Ä˽α
Ë£«˜ÞΑêޘ”Α«ÄΑÄº«Ä£ΑÄ”ΑĈāϘΠ
Only through the daily newspaper reading.

SOURCE : Daily Newspapers


22

Digital Currency Cryptocurrency


Normally backed by the Central Not backed by the central bank
Bank (The RBI has already intro-
duced it in India) (e.g. the RBI does

Centralised Decentralised

Transparent; not as transparent Transparent due to the distributed


as only the sender, receiver and ledger system; Every detail
the banking authorities will be regarding cryptocurrency trans-
aware of the transactions actions is in the public domain.

receives the SWIFT


To Transfer message of
incoming payment
the money

Goes with Account


number & SWIFT Code

Person 1 Person 2
23

MONETARY POLICY
(6 Questions)

Year 2017 MONETARY POLICY COMMITTEE

Q1. Which of the following statements is/are correct regarding the ‘Monetary Policy Committee
(MPC)?
1. It decides the RBI’s benchmark interest rates.
2. It is a 12-member body including the Governor of RBI and is reconstituted every year.
3. It functions under the chairmanship of the Union Finance Minister.
Select the correct answer using the code given below:
(a) 1 only
(b) 1 and 2 only
(c) 3 only
(d) 2 and 3 only

ANSWERS A

EXPLANATION

b¨˜ΑW˜Þ˜Úõ˜Α ÄºΑË¢Α+Ĕ«ΑŽæΡΑ͍͊͒͌ΑλW +ΑŽæμΑ¨ÞΑ˜˜ÄΑÃ˜Ä”˜”ΑüΑ樘Α"«ÄÄŽ˜ΑŽæΡΑ͉͋͊͏ΡΑæËΑ×ÚËõ«”˜Α¢ËÚΑΑ


ÞææêæËÚüΑÄ”Α«ÄÞæ«æêæ«Ëā½«Þ˜”Α¢ÚÃ˜öËÚºΑ¢ËÚΑΑAËĘæÚüΑT˽«ŽüΑËÃëæ昘Α«æΑöË꽔Α˜Α˜ÄæÚêÞ昔Αö«æ¨Α樘Α
æÞºΑË¢ΑĈû«Ä£Α樘Α˜ÄŽ¨ÃÚºΑ×˽«ŽüΑځæ˜Αλژ×ËΑځæ˜μΑژÙê«Ú˜”ΑæËΑŽËÄ恫ÄΑ«Äĉæ«ËÄΑö«æ¨«ÄΑ樘ΑÞטŽ«Ĉ˜”ΑæÚ£˜æΑ
½˜õ˜½ΠΑHence Statement 1 is correct.
ÞΑטÚΑ樘Α×ÚËõ«Þ«ËÄÞΑË¢Α樘ΑW +ΑŽæΡΑËêæΑË¢Α樘ΑÞ«ûΑA˜Ã˜ÚÞΑË¢ΑAËĘæÚüΑT˽«ŽüΑËÃëæ昘ΡΑæ¨Ú˜˜ΑA˜Ã˜ÚÞΑ
ö«½½Α˜Α¢ÚËÃΑ樘ΑW +ΑÄ”Α樘ΑË樘ÚΑæ¨Ú˜˜ΑA˜Ã˜ÚÞΑË¢ΑATΑö«½½Α˜Α××Ë«Ä昔ΑüΑ樘Α˜Äæځ½Α#Ëõ˜ÚÄØÄæΠΑ
Hence statement 2 is not correct. b¨˜Α #Ëõ˜ÚÄËÚΑ Ë¢Α 樘Α ÄºδΑ ˜ûΑ ËƎ«ËΑ ¨«ÚטÚÞËÄΑ Ë¢Α ATΠΑ Hence
statement 3 is not correct.

ANALYSIS:

Scope for elimination


+æΑ«ÞΑö˜½½αºÄËöÄΑ樁æΑĈގ½Α×˽«ŽüΑ«ÞΑ樘ΑژÞ×Ëīލ«½«æüΑË¢Α樘ΑfÄ«ËÄΑ"«ÄÄŽ˜ΑA«Ä«ÞæÚüΑÄ”ΑAËĘæÚüΑT˽«ŽüΑ«ÞΑ
樁æΑË¢Α樘ΑW˜Þ˜Úõ˜Α ÄºΑË¢Α+Ĕ«ΠΑ(˜ÄŽ˜Α½Ë£«Ž½½üΑATΑÞ¨Ë꽔Α˜ΑŽ¨«Ú˜”ΑüΑ樘ΑW +Α#Ëõ˜ÚÄËÚΠΑfÞ«Ä£Α樁æΑ
Þ«Ã×½˜Α½Ë£«ŽΡΑÞææ˜Ã˜ÄæΑ͌ΑŽÄΑ˜Α˜½«Ã«Äæ˜”Π
+Ã×ËÚæÄæΑ+ÄÞæ«æêæ«ËÄÞΑö¨«Ž¨Αö«½½Α””Α½ËæΑË¢Αõ½ê˜ΑÄ”ΑæځÄÞׁژĎüΑæËΑÃËĘæÚüΑ×˽«ŽüΑ”˜Ž«Þ«ËÄÞΑ«ÄΑ+Ĕ«ΠΑBËæΑ
ŽËõ˜Ú˜”Α «ÄΑ ÄüΑ Ë¢Α樘Α ÞޘÄ櫁½Α Aæ˜Ú«½ΡΑ êæΑ ATΑ ¨”Α ˜˜ÄΑ «ÄΑ ĘöÞΑ ޫϘΑ ޫϘΑ樘Α ”Ë×æ«ËÄΑ Ë¢Α +Äĉæ«ËÄΑ
bÚ£˜æ«Ä£ΑW˜£«Ã˜ΠΑ

SOURCE : ¨ææ×΢έέ׫ΠÄ«ŽΠ«ÄέĘöÞ«æ˜έTÚ«ÄæW˜½˜Þ˜ΠÞ×ûΧژ½«”Ͻ͎͊͊͋͏͍
24

REMEMBER:
+æΑ«ÞΑ樘Α˜Äæځ½Α#Ëõ˜ÚÄØÄæΑö¨«Ž¨ΑޘæÞΑ樘Α«Äĉæ«ËÄΑæÚ£˜æΑÄ”ΑÄËæΑ樘ΑW +ΠΑb¨˜Α
W +ΑËĽüΑ恺˜ÞΑËÚΑÞ꣣˜ÞæÞΑ؁ÞêژÞΑæËΑŽËÄæÚ˽Α«Äĉæ«ËÄΠ
ÞΑטÚΑ[˜Žæ«ËÄΑ͍͎Α}ΑË¢Α樘ΑW +ΑŽæΡΑ͍͊͒͌Ρ
+Äĉæ«ËÄΑ æÚ£˜æΠδλ͊μΑ b¨˜Α ˜Äæځ½Α #Ëõ˜ÚÄØÄæΑ Þ¨½½ΡΑ «ÄΑ ŽËÄÞê½ææ«ËÄΑ ö«æ¨Α 樘Α
ÄºΡΑ”˜æ˜ÚëĘΑ樘Α«Äĉæ«ËÄΑæÚ£˜æΑ«ÄΑæ˜ÚÃÞΑË¢Α樘ΑËÄÞêØÚΑTÚ«Ž˜Α+Ĕ˜ûΡΑËϘΑ
«ÄΑ˜õ˜ÚüΑĈõ˜Αü˜ÚÞΠΑ

Year 2019 MONETARY POLICY

Q2. Which one of the following is not the most likely measure the Government/RBI takes to stop
the slide of Indian rupee?
(a) Curbing imports of non-essential goods-and promoting exports
(b) Encouraging Indian borrowers to issue rupee denominated Masala Bonds
(c) Easing conditions relating to external commercial borrowing
(d) Following an expansionary monetary policy

ANSWERS B

EXPLANATION

b¨˜ΑË×æ«ËÄÞΑλμΡΑλμΑÄ”ΑλŽμΑÚ˜Α½½Α؁ÞêژÞΑö¨«Ž¨Α«Äæ˜Ä”ΑæËΑÞ¨«˜½”Α樘Α+Ĕ«ÄΑŽêÚژĎüΑ£«ÄÞæΑ”˜×ژŽ«α
æ«ËÄΠΑ H×æ«ËÄΑ λ”μΑ ö¨«Ž¨Α «ÞΑ Ϗ”Ë×æ«Ä£Α ÄΑ ˜ûׁÄÞ«ËāÚüΑ ÃËĘæÚüΑ ×˽«ŽüϐΑ öË꽔Α ÄËæΑ ˜Α Α ½«º˜½üΑ ؁ÞêژΑ ÞΑ
˜ûׁÄÞ«ËāÚüΑÃËĘæÚüΑ×˽«ŽüΑ«ÞΑ樘ΑÃËĘæÚüΑ×˽«ŽüΑ”Ë×昔ΑüΑ樘ΑW +ΑλW˜Þ˜Úõ˜Α ÄºΑË¢Α+Ĕ«μΑö¨˜ÄΑ樘Α
W +Α «ÃÞΑ æËΑ Þæ«Ã꽁æ˜Α 樘Α ˜ŽËÄËÃüΑ Ä”Α 樁æΑ ŽËØÞΑ ö«æ¨Α Α ŽËÞæΑ λ«Äĉæ«ËÄΡΑ ö¨«Ž¨Α ½Ëö˜ÚÞΑ ×êڎ¨Þ«Ä£Α
power and causes money to lose value).Αb¨æΑ «ÄŽÚ˜Þ˜ÞΑ樘Α ÃËĘüΑ Þê××½üΡΑ ½Ëö˜ÚÞΑ «Äæ˜Ú˜ÞæΑ ځæ˜ÞΡΑ Ä”Α
«ÄŽÚ˜Þ˜ÞΑ££Ú˜£æ˜Α”˜ÃÄ”ΠΑHence option (b) is the correct answer.
;Ëö˜ÚΑ «Äæ˜Ú˜ÞæΑ ځæ˜ÞΑ ö«½½Α ½ÞËΑ æ˜Ä”Α æËΑ ژ”ꎘΑ 樘Α õ½ê˜Α Ë¢Α 樘Α ŽêÚژĎüΠΑ +¢Α ”ËØÞ櫎Α «Äæ˜Ú˜ÞæΑ ځæ˜ÞΑ ¢½½Α
ژ½æ«õ˜ΑæËΑ˜½Þ˜ö¨˜Ú˜ΡΑ«æΑ˜ŽËØÞΑ½˜ÞÞΑææځŽæ«õ˜ΑæËΑށõ˜ΑÃËĘüΑ«ÄΑ”ËØÞ櫎ΑÄºÞΠΑb¨˜Ú˜¢ËژΡΑ«æΑö«½½Α½˜”ΑæËΑ
ÄΑËêæĉËöΑË¢Α¢Ëژ«£ÄΑŽêÚژĎüΑÄ”Α樘ژ¢ËژΡΑÞ½«”˜ΑË¢Α+Ĕ«ÄΑWêט˜Π

ANALYSIS:

Þ«ŽΑ êĔ˜ÚÞæÄ”«Ä£Α «ÞΑ ˜ÄË꣨ΑæËΑ Þ˽õ˜Α樫ÞΑ Ùê˜Þæ«ËÄΠΑ +¢Α ˜ûׁÄÞ«ËāÚüΑ ÃËĘæÚüΑ ×˽«ŽüΑ «ÞΑ¢Ë½½Ëö˜”Α樁æΑ
؁ÄÞΑ×˽«ŽüΑځæ˜ÞΑö«½½Α¨õ˜ΑæËΑ˜Αº˜×æΑ½ËöΑö¨«Ž¨Αö«½½ΑÄËæΑژט½ΑÄ”ΑÄËæΑææځŽæΑ¢Ëژ«£ÄΑ«Äõ˜ÞæËÚÞΑæ¨êÞΑژ”êŽα
«Ä£Α樘Α”˜ÃÄ”ΑË¢Α+Ĕ«ÄΑŽêÚژĎüΑ¢êÚ樘ÚΑæ¨êÞΑ””«Ä£ΑæËΑ«æÞΑ”˜×ژŽ«æ«ËÄΠ

SOURCE : Macroeconomics (NCERT Class 12)


25

Year 2020 EXPANSIONIST MONETARY POLICY

Q3. If the RBI decides to adopt an expansionist monetary policy, which of the following would it
not do?
͊ΠΑêæΑÄ”ΑË×æ«Ã«Þ˜Α樘Α[ææêæËÚüΑ;«Ù꫔«æüΑWæ«ËΑ
͋ΠΑ+ĎژÞ˜Α樘ΑAÚ£«Ä½Α[æÄ”«Ä£Α"Ž«½«æüΑWæ˜Α
͌ΠΑêæΑ樘Α ÄºΑWæ˜ΑÄ”ΑW˜×ËΑWæ˜Α
[˜½˜ŽæΑ樘ΑŽËÚژŽæΑÄÞö˜ÚΑêÞ«Ä£Α樘ΑŽË”˜Α£«õ˜ÄΑ˜½Ëö΢Α
(a) 1 and 2 only
(b) 2 only
(c) 1 and 3 only
(d) 1, 2 and 3

ANSWERS B

EXPLANATION

ûׁÄÞ«ËÄ«ÞæݘûׁÄÞ«ËāÚüΑÃËĘæÚüΑ×˽«ŽüΑ«ÞΑö¨˜ÄΑ樘ΑŽ˜Äæځ½ΑÄºΑË¢ΑΑŽËêÄæÚüΑ«ÄŽÚ˜Þ˜ÞΑÃËĘüΑÞê××½üΑ
æËΑÞæ«Ã꽁æ˜Α樘Α˜ŽËÄËÃüΠΑb¨˜ΑæË˽ÞΑêޘ”ΑüΑ樘ΑW +ΑæËΑŽËÄæÚ˽ΑÃËĘüΑÞê××½üΑ«ÄΑ樘Α˜ŽËÄËÃüΑŽÄΑ˜ΑÙêÄæ«α
ææ«õ˜ΑËÚΑÙꁽ«ææ«õ˜ΠΑVêÄæ«ææ«õ˜ΑæË˽ÞΑŽËÄæÚ˽Α樘Α˜ûæ˜ÄæΑË¢ΑÃËĘüΑÞê××½üΑüΑŽ¨Ä£«Ä£Α樘ΑÞ¨ΑW˜Þ˜Úõ˜Α
Wæ«ËΑ λWWμΡΑ ËÚΑ [ææêæËÚüΑ ;«Ù꫔«æüΑ Wæ«ËΑ λ[;WμΡΑ ËÚΑ ÄºΑ ځæ˜Α ËÚΑ ;«Ù꫔«æüΑ ”¶êÞæØÄæΑ "Ž«½«æüΑ λ;"μΑ 樁æΑ
«ÄŽ½ê”˜ÞΑ AÚ£«Ä½Α [æÄ”«Ä£Α "Ž«½«æüΑ λA["μΠΑ +¢Α W +Α Ž¨Ä£˜ÞΑ ژޘÚõ˜Α ځæ«ËÞΡΑ 樫ÞΑ öË꽔Α ½˜”Α æËΑ Ž¨Ä£˜ÞΑ «ÄΑ
½˜Ä”«Ä£ΑüΑ樘ΑÄºÞΑö¨«Ž¨ΡΑ«ÄΑæêÚÄΡΑöË꽔Α«ÃׁŽæΑ樘Α”˜×ËÞ«æÞΑÄ”Α¨˜ÄŽ˜ΡΑ樘ΑÃËĘüΑÞê××½üΠΑ
SLR is the ratio of liquid assets to the demand and time liabilities (NDTL). W +Α«ÄŽÚ˜Þ˜ÞΑ[;WΑæËΑژ”ꎘΑ
ÄºΑŽÚ˜”«æΑ”êÚ«Ä£Α樘Αæ«Ã˜ΑË¢Α«Äĉæ«ËÄΠΑ[«Ã«½Ú½üΡΑ«æΑژ”ꎘÞΑ[;WΑ”êÚ«Ä£Α樘Αæ«Ã˜ΑË¢ΑژŽ˜ÞÞ«ËÄΑæËΑ«ÄŽÚ˜Þ˜Α
ÄºΑŽÚ˜”«æΠΑb¨˜Ú˜¢ËژΡΑŽêææ«Ä£ΑÄ”ΑË×æ«Ã«Þ«Ä£Α[;WΑ«ÞΑÞËØ樫ģΑ樘ΑW +ΑöË꽔Α”ËΑêĔ˜ÚΑ«æÞΑ˜ûׁÄÞ«ËÄ«ÞæΑ
ÃËĘæÚüΑ×˽«ŽüΠ Hence option 1 is not correct.
MSF is an emergency window available to scheduled banks to borrow from RBIΑËÄΑÄΑËõ˜ÚÄ«£¨æΑÞ«ÞΑ
üΑ×½˜”£«Ä£Α£Ëõ˜ÚÄØÄæΑޘŽêګ櫘ÞΠΑ+ĎژÞ˜Α樘ΑA["Αځæ˜ΑË¢Α«Äæ˜Ú˜ÞæΑö«½½ΑÁº˜ΑËÚÚËö«Ä£ΑŽËÞæ½üΡΑÄ”Αæ¨êÞΑ
«ÞΑ ÞËØ樫ģΑ樁æΑ樘Α W +ΑöË꽔Α ÄËæΑ ”ËΑ ÞΑ ׁÚæΑ Ë¢Α «æÞΑ ˜ûׁÄÞ«ËÄ«ÞæΑ ÃËĘæÚüΑ ×˽«ŽüΠΑ Hence option 2 is
correct.
The rate of interest charged by RBI on loans extended to commercial banks is called Bank Rate.ΑW˜×ËΑ
ځæ˜Α«ÞΑ樘Αځæ˜ΑæΑö¨«Ž¨Α樘ΑW +Α½˜Ä”ÞΑæËΑŽËÃØڎ«½ΑÄºÞΑüΑ×êڎ¨Þ«Ä£ΑޘŽêګ櫘ÞΠΑ;Ëö˜ÚΑÄºΑځæ˜ÞΑÄ”Α
ژ×ËΑځæ˜ÞΑöË꽔Αژ”ꎘΑ樘ΑŽËÞæΑË¢ΑËÚÚËö«Ä£ΡΑÄ”Αæ¨êÞΑö«½½Α«ÄŽÚ˜Þ˜Α½«Ù꫔«æüΑ«ÄΑ樘Α˜ŽËÄËÃüΠΑêææ«Ä£Α
ÄºΑ ځæ˜Α Ä”Α ژ×ËΑ ځæ˜Α «ÞΑ ÞËØ樫ģΑ樘Α W +ΑöË꽔Α ”ËΑ êĔ˜ÚΑ «æÞΑ ˜ûׁÄÞ«ËÄ«ÞæΑ ÃËĘæÚüΑ ×˽«ŽüΠ Hence
option 3 is not correct.

ANALYSIS:
b¨˜Α¢êĎæ«ËÄÞΑË¢Α樘Α˜Äæځ½Α ÄºΑ«ÞΑÄΑ«Ã×ËÚæÄæΑæË׫ŽΑ«ÄΑ樘ΑÞü½½êÞΠΑ(˜ÄŽ˜ΡΑΑÞæꔘÄæΑÃêÞæΑ¨õ˜Α”˜˜×Α
ºÄËö½˜”£˜ΑËêæΑõÚ«ËêÞΑ¢êĎæ«ËÄÞΑË¢Α樘Α˜Äæځ½Α ÄºΠΑ

Scope for elimination


+¢Αö˜ΑژÃËõ˜ΑÞææ˜Ã˜ÄæΑ͊ΡΑö˜Αö«½½Α˜Α½˜¢æΑËĽüΑö«æ¨Α樘ΑŽËÚژŽæΑÄÞö˜ÚΡΑ樁æΑ«ÞΑË×æ«ËÄΑλμΠΑ
26

SOURCE : Macroeconomics (NCERT Class 12)

ûŽ˜Ú×æΑ¢ÚËÃΑBWb
Please Note:
The NCERT book does not directly discuss such a situation, but it does discuss the various
monetary tools.

Year 2022 MONETARY TOOLS

V͍ΠΑr«æ¨Αژ¢˜Ú˜ÄŽ˜ΑæËΑ樘Α+Ĕ«ÄΑ˜ŽËÄËÃüΡΑŽËÄÞ«”˜ÚΑ樘Α¢Ë½½Ëö«Ä£ΑÞææ˜Ã˜ÄæÞ΢
1. If the inflation is too high, Reserve Bank of India (RBI) is likely to buy government securities.
2. If the rupee is rapidly depreciating, RBI is likely to sell dollars in the market.
3. If interest rates in the USA or European Union were to fall, that is likely to induce RBI to buy dollars.
r¨«Ž¨ΑË¢Α樘ΑÞææ˜Ã˜ÄæÞΑ£«õ˜ÄΑËõ˜ΑÚ˜ΑŽËÚژŽæΧ
(a) 1 and 2 only
(b) 2 and 3 only
(c) 1 and 3 only
(d) 1, 2 and 3

ANSWERS B

EXPLANATION

ΑŽ˜Äæځ½ΑÄºΑÞꎨΑÞΑW˜Þ˜Úõ˜Α ÄºΑË¢Α+Ĕ«ΑλW +μΡΑטګ˔«Ž½½üΑ«Äæ˜Úõ˜Ä˜ÞΑ«ÄΑ樘Α”˜æΑÁں˜æΑæËΑ«Äĉê˜ÄŽ˜Α


樘Α«Äæ˜Ú˜ÞæΑځæ˜ÞΑÄ”Αځæ˜ΑË¢Α«Äĉæ«ËÄΑ«ÄΑ樘Α˜ŽËÄËÃüΠΑ+¢ΑW +Α¢˜˜½ÞΑ«Äĉæ«ËÄΑ«ÞΑæËËΑ¨«£¨ΡΑ«æΑö«½½Αޘ½½Α£Ëõ˜ÚÄα
ØÄæΑޘŽêګ櫘ÞΡΑÄ”ΑÞꎺΑÃËĘüΑËêæΑË¢Α樘ΑÞüÞæ˜ÃΠΑb¨«ÞΑŽæΑö«½½Α×êÞ¨Αê×Α«Äæ˜Ú˜ÞæΑځæ˜ÞΑ«ÄΑ樘Α˜ŽËÄËÃüΡΑÄ”Α
êޫĘÞÞΑö«½½ΑŽêæΑŽºΑËÄΑŽ×«æ½Α˜ûטĔ«æêژΑĈāŽ˜”ΑüΑ½ËÄÞΡΑژ”ꎫģΑ樘Α”˜ÃÄ”Α¢ËÚΑÃËĘüΠΑHence
statement 1 is not correct.
˜Äæځ½ΑÄºÞΑ½ÞËΑ«Äæ˜Úõ˜Ä˜Αטګ˔«Ž½½üΑ«ÄΑ¢Ëژ«£ÄΑ˜ûŽ¨Ä£˜ΑÁں˜æÞΠΑ+¢Α樘ΑÚêט˜Α«ÞΑځ׫”½üΑ”˜×ژŽ«æ«Ä£ΡΑ
W +Αö«½½Αޘ½½Α”˽½ÚÞΑ«ÄΑ樘ΑÁں˜æΠΑb¨«ÞΑö«½½Α«ÄŽÚ˜Þ˜Α樘ΑÞê××½üΑË¢Α”˽½ÚÞΑÄ”Α樘Α”˜ÃÄ”Α¢ËÚΑÚêט˜ÞΡΑŽêÞα
«Ä£Α樘ΑÚêט˜Α×Ú«Ž˜ΑË¢Α樘Α”˽½ÚΑæËΑŽËØΑ”ËöÄΠΑHÄΑ樘ΑŽËÄæځÚüΡΑ«¢Α樘ΑÚêט˜Α«ÞΑځ׫”½üΑ××ژŽ«æ«Ä£ΡΑW +Αö«½½Α
êüΑ”˽½ÚÞΑÄ”Α«Ä¶˜ŽæΑÚêט˜ÞΑ«ÄæËΑ樘Α˜ŽËÄËÃüΠΑb¨«ÞΑö«½½Α«ÄŽÚ˜Þ˜Α樘Α”˜ÃÄ”Α¢ËÚΑ”˽½ÚÞΑÄ”Α樘ΑÞê××½üΑË¢Α
Úêט˜ÞΡΑ樘ژüΑ½˜”«Ä£ΑæËΑÄΑ«ÄŽÚ˜Þ˜Α«ÄΑ樘ΑÚêט˜Α×Ú«Ž˜ΑË¢Α樘Α”˽½ÚΠΑHence statement 2 is correct.
27
+Äæ˜Ú˜ÞæΑځæ˜ΑÃËõ˜Ã˜ÄæÞΑ«ÄΑΑ¢Ëژ«£ÄΑ˜ŽËÄËÃüΑŽÄΑÞæ«Ã꽁æ˜ΑŽæ«ËÄΑËÄΑ樘ΑׁÚæΑË¢ΑW +ΠΑ+¢Α«Äæ˜Ú˜ÞæΑځæ˜ÞΑ«ÄΑ
樘Αf[ΑËÚΑ樘ΑfΑö˜Ú˜ΑæËΑ¢½½ΡΑ"++ÞΑλ"Ëژ«£ÄΑ+ÄÞæ«æêæ«Ëā½Α«Äõ˜ÞæËÚÞμΑö«½½ΑځÃ×Αê×Α«Äõ˜ÞæØÄæÞΑ«ÄΑ+Ĕ«ΠΑb¨˜Α
ژÞê½æÄæΑ”˜ÃÄ”Α¢ËÚΑÚêט˜ÞΑö«½½ΑŽêޘΑ樘ΑÚêט˜ΑæËΑ××ژŽ«æ˜ΠΑ+ÄΑژÞ×ËÄޘΡΑW +Αö«½½ΑêüΑ”˽½ÚÞΑÄ”Α«Ä¶˜ŽæΑ
Úêט˜ÞΑ«ÄΑæËΑ樘ΑÞüÞæ˜ÃΠΑHence statement 3 is correct.

ANALYSIS:

Motivation
b¨˜ΑÙê˜Þæ«ËÄΑöÞΑ«ÄÞ׫ژ”Α”ê˜ΑæËΑ¢½½έ”˜×Ú«Ž«æ«ËÄΑ«ÄΑÚêט˜Αõ«Þααõ«ÞΑ”˽½ÚΑÄ”ΑÚ«Þ«Ä£Α«Äĉæ«ËÄΑ«ÄΑf[ΑÄ”Α«ÄΑ
+Ĕ«Α ö«æ¨Α ژŽ˜ÄæΑ «Äæ˜Ú˜ÞæΑ ځæ˜ÞΑ ¨«º˜Α «ÄΑ ÁüΑ ͉͋͋͋Α ؘæ«Ä£Α Ë¢Α W +Α ÃËĘæځüΑ ×˽«ŽüΑ ŽËÃë昘Α æËΑ ŽêڍΑ
«Äĉæ«ËÄΠ

Scope for elimination


+¢ΑW +Αö«½½ΑêüΑ£ËõæΑޘŽêګ櫘ÞΡΑ«æΑö«½½Α«ÄŽÚ˜Þ˜ΑÃËĘüΑĉËöΑ«ÄΑÁں˜æΑö¨«Ž¨Αö«½½Α«ÄŽÚ˜Þ˜Α×êڎ¨Þ«Ä£Α×Ëö˜ÚΡΑ
ö¨«Ž¨Αö«½½Α«ÄŽÚ˜Þ˜Α”˜ÃÄ”ΑÄ”Αö«½½ΑژÞê½æΑ«ÄΑ«ÄŽÚ˜Þ˜Α«ÄΑ«Äĉæ«ËÄΡΑ¨˜ÄŽ˜Α͊ΑÞææ˜Ã˜ÄæΑŽÄΑ˜Α˜½«Ã«Äæ˜”ΑÄ”Α
樘ΑÄÞö˜ÚΑ«ÞΑË×æ«ËÄΑλμ

SOURCE : ¨ææ×Þ΢έέöööΠĈāĎ«½˜û×ژÞÞЎËÃέÃËĘüέüËêÚαÃËĘü᎘Äα
æځ½αÄºÞαÚ˽˜α«ÄαÄα«Äæ˜ÚŽËÄĘŽæ˜”αöËÚ½”έ͍͉͋͋͑͋͋έ
https://www.livemint.com/opinion/online-views/the-curi-
ËêÞαŽÞ˜αË¢α«Ä”«αÞαÚ«Þ«Ä£α¢ËژûαژޘÚõ˜ÞαÄ”α¢½½«Ä£αÚêט˜α͊͊͏͍͉͍͌͒͌͋͑͐͑͊Π¨æý
https://www.moneycontrol.com/news/business/r-
«αÁüᨁõ˜αæËαæ˽˜Úæ˜α¨«£¨αü«˜½”Þα×Ú«ËÚ«æ«Þ˜α«Äĉæ«ËÄα˜ûטÚæÞαށüα͎͉͑͋͒͋͊Π¨æý

Year 2023 STERILIZATION / OPEN MARKET OPERATIONS

V͎ΠΑΑr¨«Ž¨ΑËĘΑË¢Α樘Α¢Ë½½Ëö«Ä£ΑŽæ«õ«æ«˜ÞΑË¢Α樘ΑW˜Þ˜Úõ˜Α ÄºΑË¢Α+Ĕ«Α«ÞΑŽËÄÞ«”˜Ú˜”ΑæËΑ˜ΑׁÚæΑË¢ΑΑΑΑΑΑ
'sterilization' ?
(a) Conducting 'Open Market Operations'
(b) Oversight of settlement and payment systems
(c) Debt and cash management for the Central and State Governments
(d) Regulating the functions of Non-banking Financial Institutions

ANSWERS A

EXPLANATION
bËΑ˜Þ˜Α樘Αæ¨Ú˜æΑË¢ΑŽêÚژĎüΑ××ژŽ«æ«ËÄΑËÚΑ«Äĉæ«ËÄΡΑŽ˜Äæځ½ΑÄºÞΑË¢æ˜ÄΑææ˜Ã×æΑö¨æΑ«ÞΑºÄËöÄΑÞΑ樘Α
ϕÞæ˜Ú«½«āæ«ËÄϕΑË¢ΑŽ×«æ½ΑĉËöÞΠΑ+ÄΑΑÞꎎ˜ÞÞ¢ê½ΑÞæ˜Ú«½«āæ«ËÄΑËטځæ«ËÄΡΑ樘Α”ËØÞ櫎ΑŽËÃ×ËĘÄæΑË¢Α樘ΑÃËĘα
æÚüΑ Þ˜Α λÄºΑ ژޘÚõ˜ÞΑ ×½êÞΑ ŽêÚژĎüμΑ «ÞΑ ژ”ꎘ”ΑæËΑ ËąÞ˜æΑ樘Α ژޘÚõ˜Α «ÄĉËöΡΑ æΑ ½˜ÞæΑæ˜Ã×Ëځګ½üΠΑ +ÄΑ
樘ËÚüΡΑ樫ÞΑŽÄΑ˜ΑŽ¨«˜õ˜”Α«ÄΑޘõ˜Ú½ΑöüÞΡΑÞꎨΑÞΑüΑ˜ÄŽËêځ£«Ä£Α×Ú«õæ˜Α«Äõ˜ÞæØÄæΑËõ˜ÚޘÞΡΑËÚΑ½½Ëöα
«Ä£Α ¢Ëژ«£Ä˜ÚÞΑ æËΑ ËÚÚËöΑ ¢ÚËÃΑ 樘Α ½ËŽ½Α Áں˜æΠΑ b¨˜Α Ž½ÞÞ«Ž½Α ¢ËÚÃΑ Ë¢Α Þæ˜Ú«½«āæ«ËÄΡΑ ¨Ëö˜õ˜ÚΡΑ ¨ÞΑ ˜˜ÄΑ
æ¨ÚË꣨Α樘ΑêޘΑË¢ΑËטÄΑÁں˜æΑËטځæ«ËÄÞΡΑ樁æΑ«ÞΡΑޘ½½«Ä£ΑbژÞêÚüΑ«½½ÞΑÄ”ΑË樘ÚΑ«ÄÞæÚêØÄæÞΑæËΑژ”ꎘΑ
樘Α”ËØÞ櫎ΑŽËÃ×ËĘÄæΑË¢Α樘ΑÃËĘæÚüΑÞ˜Π Hence option (a) is the correct answer.
28

SOURCE : Sterlisation is well-explained in the Chapter 3 of the Introductory Macroeconomics book.


T½˜Þ˜ΑĈĔΑ˜½ËöΑ樘Αގژ˜ÄÞ¨ËæΠ

ûŽ˜Ú×æΑ¢ÚËÃΑBWb
Year 2023 CENTRAL BANKS

Q6. Correct the following statements:


Statement-I: In the post-pandemic recent past, many Central Banks worldwide had carried out interest rate hikes.
Statement-II: Central Banks generally assume that they have the ability to counteract the rising consumer prices
via monetary policy means.
Which one of the following is correct in respect of the above statements?
(a) Both Statement-I and Statement-II are correct and Statement-II is the correct explanation for Statement-I
(b) Both Statement-I and Statement-II are correct and Statement-II is not the correct explanation for Statement-I
(c) Statement-I is correct but Statement-II is incorrect
(d) Statement-I is incorrect but Statement-II is correct

ANSWERS A
29

EXPLANATION

ΑĘöΑÞæê”üΑüΑ樘ΑrËÚ½”Α ÄºΑ¨ÞΑژõ˜½˜”Α樁æΑö«æ¨Α樘ΑŽ˜Äæځ½ΑÄºÞΑŽÚËÞÞΑ樘ΑöËÚ½”ΑÞ«Ãê½æÄ˜ËêÞ½üΑ
¨«º«Ä£Α«Äæ˜Ú˜ÞæΑځæ˜ÞΑ«ÄΑژÞ×ËÄޘΑæËΑ«Äĉæ«ËÄΡΑ樘ΑöËÚ½”ΑÁüΑ˜Α˜”£«Ä£ΑæËöÚ”ΑΑ£½Ë½ΑژŽ˜ÞÞ«ËÄΑ«ÄΑ͉͋͋͌Α
Ä”ΑΑÞæÚ«Ä£ΑË¢ΑĈāĎ«½ΑŽÚ«Þ˜ÞΑ«ÄΑ˜Ã˜Ú£«Ä£ΑÁں˜æÞΑÄ”Α”˜õ˜½Ë׫ģΑ˜ŽËÄËë˜ÞΑ樁æΑöË꽔Α”ËΑ樘ÃΑ½Þæ«Ä£Α
¨ÚÃΠΑ Central banks around the world have been raising interest rates this year with a degree of
ÞüϨÚËÄ«Ž«æüΑÄËæΑޘ˜ÄΑËõ˜ÚΑ樘ΑׁÞæΑĈõ˜Α”˜Ž”˜ÞδΑæژĔΑ樁æΑ«ÞΑ½«º˜½üΑæËΑŽËÄæ«Äê˜Αö˜½½Α«ÄæËΑĘûæΑ
year.
ΑŽ˜Äæځ½ΑÄºΑö«æ¨ΑΑ¨«£¨Α”˜£Ú˜˜ΑË¢ΑŽÚ˜”««½«æüΑĈÚýüΑÄŽ¨ËÚÞΑ˜ûטŽææ«ËÄÞΑË¢Α×Ú«Ž˜ΑÞ恍«½«æüΠΑb¨˜ΑÃËĘæÚüΑ
×˽«ŽüΑ æځÄÞëÞÞ«ËÄΑ ؎¨Ä«ÞÃΑ «ÞΑ Ž¨ÚŽæ˜Ú«Þ˜”Α üΑ ½ËÄ£ΡΑ õÚ«½˜Α Ä”Α êϘÚ恫ÄΑ æ«Ã˜Α ½£ÞΠΑ b¨êÞΑ «æΑ «ÞΑ
”«ĆŽê½æΑæËΑ×ژ”«ŽæΑ樘Α×ژŽ«Þ˜Α˜ą˜ŽæΑË¢ΑÃËĘæÚüΑ×˽«ŽüΑŽæ«ËÄÞΑËÄΑ樘Α˜ŽËÄËÃüΑÄ”Α×Ú«Ž˜Α½˜õ˜½ΠΑHence
Central Banks generally assume that they have the ability to counteract the rising consumer prices
via increasing interest rates. Hence both statements are correct and statement II is the correct
explanation of statement I.

ANALYSIS:

Motivation/Why asked?

˜ŽêޘΑ樘ΑŽæ«ËÄÞΑË¢Α樘ΑŽ˜Äæځ½ΑÄºÞΑ¨õ˜Α˜˜ÄΑö«”˜½üΑŽËõ˜Ú˜”ΑλËæ¨ΑüΑ樘ΑW +ΑÄ”Α樘Α"˜”˜Ú½Α
W˜Þ˜Úõ˜μ
ĔΑÞæÄ”Ú”ΑĘöÞׁטÚÞΑλ¨ææ×Þ΢έέöööΠ樘¨«Ä”êЎËÃݍêޫĘÞÞέŽËÄËÃüέÚα
«αޘæα¢ËÚα¢ËêÚæ¨αÞæځ«£¨æαځæ˜α¨«º˜αæËαÙ꘽½α«Äĉæ«ËÄαށüα˜ûטÚæÞ݁Ú櫎½˜͏͎͍͍͍͒͌͑ИŽ˜μ

SOURCE : macroeconomics (NCERT Class 12; Chapter 3: Money and Banking; the role of the RBI)

ûŽ˜Ú×æΑ¢ÚËÃΑBWb
30

BANKING AND BANKING REGULATIONS


(13 Questions)

Year 2017 RESTRUCTURING OF ASSETS

Q1. Which of the following statements best describes the term ‘Scheme for Sustainable
[æÚêŽæêÚ«Ä£ΑË¢Α[æژÞޘ”ΑÞޘæÞΑλ[͍μϐΡΑژŽ˜Äæ½üΑޘ˜ÄΑ«ÄΑ樘ΑĘöÞΧΑ
(a) It is a procedure for considering ecological costs of developmental schemes formulated by the Government.
(b) It is a scheme of RBI for reworking the financial structure of big corporate entities facing genuine difficulties.
(c) It is a disinvestment plan of the Government regarding Central Public Sector Undertakings.
(d) It is an important provision in ‘The Insolvency and Bankruptcy Code’ recently implemented by the Government.

ANSWERS B

EXPLANATION

The Reserve Bank of India introduced the Scheme for Sustainable Structuring of Stressed Assets (S4A
Scheme) on June 13, 2016.
b¨˜Α[͍ΑüΑW +Α˜Äõ«Þ£˜ÞΑ”˜æ˜Úëāæ«ËÄΑË¢Α樘ΑÞêÞ恫ā½˜Α”˜æΑ½˜õ˜½Α¢ËÚΑΑÞæژÞޘ”ΑËÚÚËö˜ÚΡΑÄ”Α«¢êڎα
æ«ËÄΑË¢Α樘ΑËêæÞæÄ”«Ä£Α”˜æΑ«ÄæËΑÞêÞ恫ā½˜Α”˜æΑÄ”Α˜Ùê«æüέÙêÞ«α˜Ùê«æüΑ«ÄÞæÚêØÄæÞΑö¨«Ž¨ΑÚ˜Α˜ûטŽæ˜”Α
æËΑ×ÚËõ«”˜Αê×Þ«”˜ΑæËΑ樘Α½˜Ä”˜ÚÞΑö¨˜ÄΑ樘ΑËÚÚËö˜ÚΑæêÚÄÞΑÚËêĔΠ
Hence option (b) is the correct answer.

ANALYSIS:

Motivation
[æژÞޘ”ΑÞޘæÞΑÄ”ΑBTÞΑ¨”ΑŽËÄÞæÄæ½üΑژÁ«Ä˜”Α«ÄΑ樘ΑĘöÞΑ«ÄΑ͉͋͊͐ΠΑW +ΑŽÃ˜Αê×Αö«æ¨ΑÁÄüΑގ¨˜Ã˜ÞΑ
æËΑ””Ú˜ÞÞΑ樘ޘΑ«ÞÞê˜ÞΠΑûΑαΑ[͍ΡΑ͎΢͎͋Α˜æŽΠΑBTΑŽÚ«Þ«ÞΑöÞΑËĘΑË¢Α樘Α¨ËæΑæË׫ŽÞΑ«ÄΑ͉͋͊͐ΑÞΑW +Α¨”Α恺˜ÄΑ
ޘõ˜Ú½ΑÞæ˜×ÞΡΑŽËÄËëŽΑ[êÚõ˜üΑ¨”Α”«ÞŽêÞޘ”Α«æΑ«ÄΑ”˜æ«½ΑÄ”ΑÄº«Ä£Α¨˜½æ¨ΑöÞΑæ¨ËÚË꣨½üΑ”«ÞŽêÞޘ”Α«ÄΑ
ĘöÞׁטÚΠΑ

Scope for elimination


HĘΑÞ¨Ë꽔ΑׁüΑŽ½ËޘΑææ˜Äæ«ËÄΑæËΑ樘Αæ˜ÚÃΑϏÞæژÞޘ”ΑÞޘæÞϐΠΑb¨˜ΑÞޘæÞΑ¨˜Ú˜Αژ¢˜ÚÞΑæËΑ樁æΑË¢ΑÄºÞΑÄ”Α
W +Α«ÞΑ樘Αژ£ê½æËÚΑË¢Α½½ΑÄºÞΠΑ"ÚËÃΑ樘Αæ˜ÚÃΑϏÞæژÞޘ”ΑÞޘæÞϐΡΑÞæꔘÄæΑŽÄΑ”˜Ž«×¨˜ÚΑ樁æΑ樘Αގ¨˜Ã˜Α«ÞΑ
ŽËÄĘŽæ˜”ΑæËΑÄº«Ä£ΑÄ”Α”˜æΑژÞæÚêŽæêÚ«Ä£ΠΑThus, options (a) and option (c) get automatically elimi-
nated. Applying that logic, one can reach the correct answer.
31

¨ææ×΢έέöööΠ樘¨«Ä”êЎËÃݍêޫĘÞÞέ+ĔêÞæÚüέ"«ÚÞæα[͍α”˜æαژα
SOURCE :
ŽÞæᣘæÞα××ÚËõ½έÚ櫎½˜͊͏͍͌͐͏͉͎ИŽ˜
¨ææ×΢έέ«Ä”«Ä˜û×ژÞÞЎËÃ݁Ú櫎½˜έêޫĘÞÞݍÄº«Ä£αÄ”αĈα
āϘέڍ«αêÄõ˜«½ÞαĘöαގ¨˜Ã˜αæËα恎º½˜α”α½ËÄÞαË¢α«£αĈÚÃÞα͎͋͑͊͊͑͏έ
¨ææ×Þ΢έέöööΠ樘¨«Ä”êЎËÃݍêޫĘÞÞέ+ĔêÞæÚüέ"«ÚÞæα[͍α”˜æαژα
ŽÞæᣘæÞα××ÚËõ½έÚ櫎½˜͊͏͍͌͐͏͉͎ИŽ˜

ÄΑ˜ΑŽËõ˜Ú˜”ΑÞΑׁÚæΑË¢Α樘ΑŽËÄËëŽΑ[êÚõ˜üΑ[êÃÁÚüΠ

ΑBËÄΑT˜Ú¢ËÚëģΑÞޘæΑλBTμΑ«ÞΑΑ½ËÄΑËÚΑ”õÄŽ˜Α¢ËÚΑö¨«Ž¨Α樘Α×ګϫׁ½ΑËÚΑ«Äæ˜Ú˜ÞæΑׁüØÄæΑژÁ«Ä˜”Α
Ëõ˜Ú”ê˜Α¢ËÚΑΑטګ˔ΑË¢Α͉͒Α”üÞΠΑ
ÄºÞΑÚ˜Α¢êÚ樘ÚΑژÙê«Ú˜”ΑæËΑŽ½ÞÞ«¢üΑBTÞΑ¢êÚ樘ÚΑ«ÄæËΑ
[êÞæÄ”Ú”ΡΑ
Ëêæ¢ê½ΑÄ”Α
;ËÞÞΑÞޘæÞΠΑ
Substandard assets:ΑÞޘæÞΑö¨«Ž¨Α¨ÞΑژÁ«Ä˜”ΑBTΑ¢ËÚΑΑטګ˔Α½˜ÞÞΑ樁ÄΑËÚΑ˜ÙꁽΑæËΑ͊͋ΑÃËÄæ¨ÞΠΑ
Doubtful assets: ÄΑÞޘæΑöË꽔Α˜ΑŽ½ÞÞ«Ĉ˜”ΑÞΑ”Ëêæ¢ê½Α«¢Α«æΑ¨ÞΑژÁ«Ä˜”Α«ÄΑ樘ΑÞêÞæÄ”Ú”ΑŽæ˜£ËÚüΑ
¢ËÚΑΑטګ˔ΑË¢Α͊͋ΑÃËÄæ¨ÞΠΑ
Loss assets:ΑÞΑטÚΑW +ΡΑύ;ËÞÞΑÞޘæΑ«ÞΑŽËÄÞ«”˜Ú˜”ΑêĎ˽½˜Žæ«½˜ΑÄ”ΑË¢ΑÞꎨΑ½«æ潘Αõ½ê˜Α樁æΑ«æÞΑŽËÄæ«Äêα
ÄŽ˜ΑÞΑΑÄº½˜ΑÞޘæΑ«ÞΑÄËæΑöÚځÄ昔ΡΑ½æ¨Ë꣨Α樘ژΑÁüΑ˜ΑÞËØΑށ½õ£˜ΑËÚΑژŽËõ˜ÚüΑõ½ê˜ΠώΑ
+æΑ«ÞΑÁĔæËÚüΑ˜¢ËژΑΑ½ËÄΑŽŽËêÄæΑæêÚÄÞΑ«ÄæËΑΑBTΑ樁æΑÄºÞΑÚ˜ΑژÙê«Ú˜”ΑæËΑ«”˜Äæ«¢üΑ«ÄŽ«×«˜ÄæΑÞæژÞÞΑ
«ÄΑ樘ΑŽŽËêÄæΑüΑŽÚ˜æ«Ä£Αæ¨Ú˜˜ΑÞêαŽæ˜£ËÚ«˜ÞΑêĔ˜ÚΑ樘Α[pecial Mention Account category as given
in the below:
[Aα͉΢ΑTګϫׁ½ΑËÚΑ«Äæ˜Ú˜ÞæΑׁüØÄæΑÄËæΑËõ˜Ú”ê˜Α¢ËÚΑÃËژΑ樁ÄΑ͉͌Α”üÞΑêæΑŽŽËêÄæΑÞ¨Ëö«Ä£ΑÞ«£ÄÞΑË¢Α
«ÄŽ«×«˜ÄæΑÞæژÞÞΑ
[Aα͊΢ΑTګϫׁ½ΑËÚΑ«Äæ˜Ú˜ÞæΑׁüØÄæΑËõ˜Ú”ê˜Α˜æö˜˜ÄΑ͌͊α͏͉Α”üÞΑ
[Aα͋΢ΑTګϫׁ½ΑËÚΑ«Äæ˜Ú˜ÞæΑׁüØÄæΑËõ˜Ú”ê˜Α˜æö˜˜ÄΑ͏͊α͉͒Α”üÞΑ
The parameters for identifying both NPA and Special Mention Accounts are decided by Reserve Bank
of India.
32

Year 2017 SMALL FINANCE BANKS

Q2. What is the purpose of setting up of Small Finance Banks (SFBs) in India?
1. To supply credit to small business units
2. To supply credit to small and marginal farmers
3. To encourage young entrepreneurs to set up business particularly in rural areas.
Select the correct answer using the code given below:
(a) 1 and 2 only
(b) 2 and 3 only
(c) 1 and 3 only
(d) 1, 2 and 3

ANSWERS A

ANSWERS
EXPLANATION

b¨˜Αˍ¶˜Žæ«õ˜ÞΑË¢Αޘææ«Ä£Αê×ΑË¢ΑÞÁ½½ΑĈāϘΑÄºÞΑö«½½Α˜ΑæËΑ¢êÚ樘ÚΑĈāĎ«½Α«ÄŽ½êÞ«ËÄΑü
×ÚËõ«Þ«ËÄΑË¢Αށõ«Ä£ÞΑõ˜¨«Ž½˜ÞΡΑÄ”Α
[ê××½üΑ Ë¢Α ŽÚ˜”«æΑæËΑ ÞÁ½½Α êޫĘÞÞΑ êÄ«æÞΣΑ ÞÁ½½Α Ä”Α Áڣ«Ä½Α¢ÚØÚÞΣΑ ëŽÚËΑ Ä”Α ÞÁ½½Α «Ä”êÞæÚ«˜ÞΣΑ Ä”Α
Ë樘ÚΑêÄËÚ£Ä«Þ˜”ΑޘŽæËÚΑ˜Äæ«æ«˜ÞΡΑæ¨ÚË꣨Α¨«£¨Α明¨Ä˽ˣüα½ËöΑŽËÞæΑËטځæ«ËÄÞΠ
About Small Finance Banks
[Á½½Α"«ÄÄŽ˜Α ÄºÞΑλ[" μΑژ£«Þæ˜Ú˜”ΑÞΑ×ꍽ«ŽΑ½«Ã«æ˜”ΑŽËÃׁī˜ÞΑêĔ˜ÚΑ樘ΑËÃׁī˜ÞΑŽæΡΑ͉͋͊͌Α«Äæ˜Ä”ΑæËΑ
×ÚËõ«”˜ΑĈāĎ«½ΑޘÚõ«Ž˜ÞΑæËΑ樘ΑêÄޘÚõ˜”ΑÄ”ΑêčÄº˜”Αژ£«ËÄΑË¢Α樘ΑŽËêÄæÚüΠΑ
b¨˜ΑW +Α«ÞÞꘔΑ£ê«”˜½«Ä˜ÞΑ¢ËÚΑ樘Αޘææ«Ä£Αê×ΑË¢ΑËæ¨ΑTüØÄæÞΑ ÄºÞΑλT ÞμΑÄ”Α[Á½½Α"«ÄÄŽ˜Α ÄºÞΑλ[" ÞμΡΑ
«ÄΑ͉͍͋͊ΑژÞטŽæ«õ˜½üΠΑ
[" ÞΑ×ګÁګ½üΑêĔ˜Ú恺˜ΑÞ«ŽΑÄº«Ä£ΑŽæ«õ«æ«˜ÞΑË¢ΑŽŽ˜×æÄŽ˜ΑË¢Α”˜×ËÞ«æÞΑλËæ¨Α”˜ÃÄ”ΑÄ”Αæ«Ã˜Α”˜×ËÞα
«æÞμΑÄ”Α½˜Ä”«Ä£Αλö«æ¨Α½«Ã«æÞμΑæËΑêÄαޘÚõ˜”ΑÄ”ΑêĔ˜ÚޘÚõ˜”ΑޘŽæ«ËÄÞΑ«ÄŽ½ê”«Ä£ΑÞÁ½½ΑêޫĘÞÞΑêÄ«æÞΡΑÞÁ½½Α
Ä”ΑÁڣ«Ä½Α¢ÚØÚÞΡΑëŽÚËΑÄ”ΑÞÁ½½Α«Ä”êÞæÚ«˜ÞΡΑÄ”ΑêÄËÚ£Ä«ā˜”ΑޘŽæËÚΑ˜Äæ«æ«˜ÞΡΑêæΑö«æ¨ËêæΑÄüΑژÞæÚ«Žα
æ«ËÄΑ«ÄΑ樘ΑÚ˜ΑË¢ΑËטځæ«ËÄÞΡΑêĽ«º˜ΑW˜£«Ëā½ΑWêځ½Α ÄºÞΑËÚΑ;ˎ½ΑژΑ ÄºÞΠΑ
b¨˜üΑÚ˜ΑÞꍶ˜ŽæΑæËΑ½½Α×ÚꔘÄ櫁½ΑÄËÚÃÞΑÄ”Αژ£ê½æ«ËÄÞΑË¢ΑW +ΑÞΑ××½«Ž½˜ΑæËΑ˜û«Þæ«Ä£ΑŽËÃØڎ«½ΑÄºÞΑ
½«º˜ΑÁ«Äæ˜ÄÄŽ˜ΑË¢ΑÞ¨ΑW˜Þ˜Úõ˜ΑWæ«ËΑλWWμΑÄ”Α[ææêæËÚüΑ;«Ù꫔«æüΑWæ«ËΑλ[;WμΠΑ
b¨˜üΑŽÄΑ«ÞÞê˜Α”˜«æΑŽÚ”ÞΠΑĔΑ樘ΑëīÃêÃΑׁ«”αê×ΑŽ×«æ½ΑژÙê«Ú˜”Α¢ËÚΑÞÁ½½ΑĈāϘΑÄºÞΑ«ÞΑWÞΑ
200 crores. Hence option (a) is the correct answer.

ANALYSIS:

SOURCE : ¨ææ×Þ΢έέڍ«ΠËÚ£Π«Äέގګ×æÞέ [εTژÞÞW˜½˜Þ˜«Þ×½üЁÞ×ûΧ×Ú«”Ͻ͌͋͏͍͊Α

Motivation/Why asked?

[Á½½Α"«ÄÄŽ˜Α ÄºÞΑλ[" ÞμΑ¨õ˜Α˜˜ÄΑ«ÄΑĘöÞΠ


BËæΑ ŽËõ˜Ú˜”Α «ÄΑ ÄüΑ Ë¢Α 樘Α ÞޘÄ櫁½Α Aæ˜Ú«½ΡΑ êæΑ ÞꎨΑ æË׫ŽÞΑ Ë¢æ˜ÄΑ ×טÚΑ «ÄΑ 樘Α ĘöÞׁטÚÞΑ Ä”Α ÁÄüΑΑ
ÞÞˎ«æ˜”Αژ£ê½æ«ËÄÞΑ¢ËÚΑ樘ΑށØΑ¨”ΑŽËØΑλÚΠΑBŽ¨«º˜æΑAËÚΑŽËÃëæ昘ΑژŽËÃØĔæ«ËÄÞμ
33

Year 2018 CAPITAL ADEQUACY RATIO

Q3. Consider the following statements:


1. Capital Adequacy Ratio (CAR) is the amount that banks have to maintain in the form of their own funds to offset
any loss that banks incur if the account-holders fail to repay dues.
2. CAR is decided by each individual bank.
Which of the statements given above is/are correct?
(a) 1 only
(b) 2 only
(c) Both 1 and 2
(d) Neither 1 nor 2

ANSWERS A

EXPLANATION
ANSWERS
Statement 1 is correct:Α×«æ½Α”˜ÙꁎüΑWæ«ËΑλWμΡΑö¨«Ž¨Α«ÞΑ½ÞËΑºÄËöÄΑÞΑCapital to Risk (Weighted)
Assets RatioΑ«ÞΑ樘Αځæ«ËΑË¢ΑΑÄºϖÞΑŽ×«æ½Α«ÄΑژ½æ«ËÄΑæËΑ«æÞΑÚ«ÞºΑö˜«£¨æ˜”ΑÞޘæÞΑÄ”ΑŽêÚژÄæΑ½««½«æ«˜ÞΠΑ+æΑ
«ÞΑ”˜Ž«”˜”ΑüΑŽ˜Äæځ½ΑÄºÞΑÄ”ΑÄºΑژ£ê½æËÚÞΑæËΑ×ژõ˜ÄæΑŽËÃØڎ«½ΑÄºÞΑ¢ÚËÃΑ恺«Ä£Α˜ûŽ˜ÞÞΑ½˜õ˜Ú£˜Α
Ä”Α ˜ŽËëģΑ «ÄÞ˽õ˜ÄæΑ «ÄΑ樘Α ×Úˎ˜ÞÞΠΑ Simply put, it is the amount of money that a bank has in its
reserve to cover any losses in its loans.Α+æΑ«ÞΑŽ½Žê½æ˜”ΑüΑ”«õ«”«Ä£ΑΑÄºϖÞΑŽ×«æ½ΑüΑ«æÞΑÚ«Þºαö˜«£¨æ˜”Α
ÞޘæÞΠ
Statement 2 is not correct:Α+æΑ«ÞΑ樘ΑW˜Þ˜Úõ˜Α ÄºΑË¢Α+Ĕ«ΑλW +μΑ樁æΑޘæÞΑ樘ΑÄËÚÃÞΑ¢ËÚΑ樘ΑÄºÞΑæËΑÁ«Äα
恫ÄΑWΠΑÞΑË¢ΑHŽæˍ˜ÚΑ͉͋͋͌ΡΑ樘ΑW +Α¨ÞΑށ«”Α樁æΑ½½Α樘Αގ¨˜”꽘”ΑŽËÃØڎ«½ΑÄºÞΑÚ˜ΑژÙê«Ú˜”ΑæËΑ
Á«Ä恫ÄΑΑWΑË¢Α͒АΑö¨«½˜Α樘Α+Ĕ«ÄΑ×ꍽ«ŽΑޘŽæËÚΑÄºÞΑÚ˜ΑژÙê«Ú˜”ΑæËΑÁ«Ä恫ÄΑΑWΑË¢Α͊͋АΠ

ANALYSIS:

Motivation/Why asked?
+æΑΑÞ«ŽΑŽËϘ×æΑ«ÄΑÁŽÚ˘ŽËÄËëŽÞΡΑÄ”Α¨ÞΑ½öüÞΑ˜˜ÄΑ«ÄΑ樘ΑĘöÞΑËö«Ä£ΑæËΑ [;ΑBHWA[Α樁æΑ£ê«”˜Α
樘ΑŽ˜Äæځ½ΑÄºÞΑŽÚËÞÞΑ樘ΑöËÚ½”ΑæËΑÁ«Ä恫ÄΑ樘ΑĈāĎ«½ΑÞ恍«½«æüΑË¢Α樘ΑÄºÞΑÄ”Α˜ŽËÄËë˜ÞΠΑWΑŽÄΑ
˜Α Α ژŽêÚÚ«Ä£Α 樘ØΑ ÞΑ «æÞΑ «Ã×ËÚæÄŽ˜Α ژÁ«ÄÞΑ 樘Α ށØΑ ˜ÞטŽ«½½üΑ ˜ŽêޘΑ ÁÄüΑ £½Ë½½üΑ «Ã×ËÚæÄΑ
ÄºÞΑ¨õ˜Α¢«½˜”Α«ÄΑ樘ΑژŽ˜ÄæΑׁÞæΠ

Scope for elimination


ÞΑ«æΑ«ÞΑΑ͋αÞææ˜Ã˜ÄæΑÙê˜Þæ«ËÄΡΑ樘ژΑ«ÞΑ½öüÞΑ½˜ÞÞΑގËטΑ¢ËÚΑ˜½«Ã«Äæ«ËÄΡΑêæΑÞææ˜Ã˜ÄæΑ͋ΑŽÄΑ˜ΑË¢ΑÞËØΑ
¨˜½×Α«ÄΑ˜½«Ã«Äæ«Ä£ΑæöËΑË¢Α樘Α¢ËêÚΑË×æ«ËÄÞΠ If we assume that statement 1 is correct, then logically state-
ment 2 should not be correctΑÞΑÁ«Ä恫īģΑĈގ½ΑÞ恍«½«æüΑŽÄÄËæΑ˜Α«ÄΑ樘Α¨Ä”ÞΑË¢Α樘ΑÄºÞΑö«æ¨ËêæΑ
樘ΑW +Α×½ü«Ä£ΑÄüΑÚ˽˜Π
How to cover? b¨˜Þ˜Αæ˜ÚÃÞΑÚ˜ΑË¢æ˜ÄΑØÄæ«ËʔΑλÄ”ΑÞËØæ«Ã˜ÞΑ˜û×½«Ä˜”μΑ«ÄΑ樘ΑÞæÄ”Ú”ΑĘöÞׁטÚÞΑ
«ÄΑ樘ΑŽËÄæ˜ûæΑË¢Αژ£ê½æËÚüΑŽ¨Ä£˜ÞΑÁ”˜ΑüΑ樘ΑW +Αژ£Ú”«Ä£ΑÄºΑށ¢˜æüΠΑ(˜ÄŽ˜Α樘ΑϏ˜ŽËÄËÃüݍêÞ«α
ĘÞÞϐΑׁ£˜ÞΑË¢Α樘ΑÞæÄ”Ú”ΑĘöÞׁטÚÞΑÃêÞæΑ˜Αژ£ê½Ú½üΑژ”Π
b¨˜Α¢Ë½½Ëö«Ä£Α½«ÄºΑö«½½Α½˜”ΑüËêΑæËΑÚ櫎½˜ΑËÄΑb¨˜Α(«Ä”êΑ×ꍽ«Þ¨˜”Α«ÄΑê£êÞæΡΑ͉͋͊͐
””«æ«Ëā½Α+Ä¢ËÚÁæ«ËÄ
Provisioning Coverage Ratio (PCR) ؁ÞêژÞΑΑÄºϖÞΑ«½«æüΑæËΑŽËõ˜ÚΑ«æÞΑ”Α½ËÄÞΑËÚΑÄËÄαטڢËÚëģΑ
ÞޘæÞΑλBTÞμΑö«æ¨Α樘ΑÃËĘüΑ«æΑ¨ÞΑޘæΑÞ«”˜Α¢ËÚΑ樁æΑõ˜ÚüΑ×êÚ×ËޘΠΑ+æΑ«ÞΑŽ½Žê½æ˜”ΑüΑ”«õ«”«Ä£Α樘ΑæË恽Α
õ½ê˜ΑË¢Α樘ΑÄºϖÞΑ×ÚËõ«Þ«ËÄÞΑ¢ËÚΑ”Α½ËÄÞΑüΑ樘ΑæË恽Αõ½ê˜ΑË¢Α«æÞΑÄËÄαטڢËÚëģΑÞޘæÞΠ
34

Loans to Deposits ratio (LDR) ؁ÞêژÞΑΑÄºϖÞΑ½«Ù꫔«æüΑÄ”Α«æÞΑ«½«æüΑæËΑ¢êĔΑ«æÞΑ½ËÄΑ×ËÚæ¢Ë½«ËΑÄ”Α«ÞΑ


Ž½Žê½æ˜”ΑüΑ”«õ«”«Ä£ΑΑÄºϖÞΑæË恽Α½ËÄÞΑüΑ«æÞΑæË恽Α”˜×ËÞ«æÞΠΑ
Cost to Income ratio (CIR)Α؁ÞêژÞΑ¨ËöΑ˜ĆŽ«˜ÄæΑΑÄºΑ«ÞΑ«ÄΑÁā£«Ä£Α«æÞΑŽËÞæÞΑژ½æ«õ˜ΑæËΑ«æÞΑ«ÄŽËØΠΑ
b¨˜Αځæ«ËΑ«ÞΑŽ½Žê½æ˜”ΑüΑ”«õ«”«Ä£ΑΑÄºϖÞΑËטځæ«Ä£Α˜ûטÄޘÞΑüΑ«æÞΑËטځæ«Ä£Α«ÄŽËØΠΑ

SOURCE : ¨ææ×Þ΢έέöööΠ樘¨«Ä”êЎËÃέË׫īËÄέË×α˜”έö¨æα«ÞαŽ×«æ½α”˜ÙêΑŽüαځæ«Ëα«ÄαĈāĎ˜έÚæ«α
Ž½˜͍͊͒͌͒͒͐͑ИŽ˜

Year 2018 CAPITAL INFUSION IN BANKS

V͍ΠΑr«æ¨Αژ¢˜Ú˜ÄŽ˜ΑæËΑ樘Α£Ëõ˜ÚāϘΑË¢Α×ꍽ«ŽΑޘŽæËÚΑÄº«Ä£Α«ÄΑ+Ĕ«ΡΑŽËÄÞ«”˜ÚΑ樘Α¢Ë½½Ëö«Ä£Α
statements:
1. Capital infusion into public sector banks by the Government of India has steadily increased in the last decade.
2. To put the public sector banks in order, the merger of associate banks with the parent State Bank of India has
been affected.
Which of the statements given above is/are correct?
(a) 1 only
(b) 2 only
(c) Both 1 and 2
(d) Neither 1 nor 2

ANSWERS B

EXPLANATION

Statement 1 is not correct asΑ×«æ½Α«Ä¢êÞ«ËÄΑ«ÄæËΑ×ꍽ«ŽΑޘŽæËÚΑÄºÞΑüΑ樘Α#Ëõ˜ÚÄØÄæΑË¢Α+Ĕ«Α¨ÞΑ


ÄËæΑÞ昁”«½üΑ«ÄŽÚ˜Þ˜”Α«ÄΑ樘Α½ÞæΑ”˜Ž”˜ΠΑ

Statement 2 is correct:Αb¨˜ΑØڣ˜ÚΑË¢Α[ +ΑÞÞˎ«æ˜”ΑÄºÞΑêĔ˜ÚΑ[˜Žæ«ËÄΑ͎͌ΑË¢Α樘Α[ææ˜Α ÄºΑË¢Α+Ĕ«Α


ŽæΡΑ ͎͎͊͒Α ö«½½Α ژÞê½æΑ «ÄΑ 樘Α ŽÚ˜æ«ËÄΑ Ë¢Α Α ÞæÚËÄ£˜ÚΑ Øڣ˜”Α ˜Äæ«æüΠΑb¨«ÞΑ ö«½½Α ëīëā˜Αõê½Ä˜Ú«½«æüΑ æËΑ ÄüΑ
£˜Ë£Ú×¨«ŽΑŽËϘÄæځæ«ËÄΑÚ«ÞºÞΑ¢Ž˜”ΑüΑÞêÞ«”«ÚüΑÄºÞΠΑ+æΑö«½½ΑŽÚ˜æ˜Α«Ã×ÚËõ˜”ΑËטځæ«Ëā½Α˜ĆŽ«˜ÄŽüΑ
Ä”Α˜ŽËÄËë˜ÞΑË¢Αގ½˜ΠΑ+æΑö«½½Α½ÞËΑژÞê½æΑ«ÄΑ«Ã×ÚËõ˜”ΑÚ«ÞºΑÁā£˜Ã˜ÄæΑÄ”ΑêÄ«Ĉ˜”ΑæژÞêÚüΑËטځæ«ËÄÞΠΑ

ANALYSIS:

ÞΑ樫ÞΑÙê˜Þæ«ËÄΑ¢˜æêژ”Α«ÄΑ͉͋͊͑ΡΑϏ½ÞæΑ͉͊Αü˜ÚÞϐΑ؁ÄÞΑ˜æö˜˜ÄΑ͉͉͋͑ΑæËΑ͉͋͊͑ΠΑ+ÄΑ͉͋͊͑ΡΑޘõ˜Ú½ΑĘöÞׁטÚΑ
Ú櫎½˜ÞΑ”«ÞŽêÞޘ”Α樘ΑÄÄËêĎ˜Ã˜ÄæΑÁ”˜Α«ÄΑ樘Α ꔣ˜æΑ͉͋͊͑α͊͒Α¢æ˜ÚΑ樘ΑŽËÄËëŽΑ[êÚõ˜üΑ͉͎͋͊α͊͏ΑΑ
˜ÃרÞ«ā˜”Α樁æΑ””Ú˜ÞÞ«Ä£Α樘ΑÞæژÞޘ”ΑÞޘæÞΑ×Úˍ½˜ÃΑöË꽔ΑژÙê«Ú˜Α͍ΑWϐÞ΢ΑW˜¢ËÚÃΡΑW˜ŽË£Ä«æ«ËÄΡΑW˜Ž×«α
恽«āæ«ËÄΡΑÄ”ΑW˜Þ˽êæ«ËÄΠ
35

FINANCIAL WOES
¨ææ×Þ΢έέöööΠ½«õ˜Ã«ÄæЎËÃέ+ĔêÞæÚüέæ;Þ½Äα 90,000

¶ê½͌͒ q £º6ê9fŽα Public sector banks have 80,000


£9έb¨˜αÞæËÚüαË¢αʘ”Þ˜”αŽ×«æ½α«Ä¢êÞ«ËÄ
80,000 been posting heavy losses
in recent years
Π¨æý 70,000 65,000
¨ææ×Þ΢έέæ«Ã˜ÞËĈĔ«Π«Ä”«æ«Ã˜ÞЎËÃέ«Äα 50,582 37,540
60,000
”«έŽ˜Äα
æÚ˜α¨Þα×êÃט”αÚÞα͋α͏ὁº¨αŽÚËژα«ÄæËα£Ëõ 49,514 37,019
50,000
æαÚêÄαÄºÞαËõ˜Úα͊͊αü˜ÚÞ݁Ú櫎½˜Þ¨Ëöέ͏͋͒͐
͍͑͌͏ЎÃÞ 40,000 44,901
25,000
[¨Ú˜”ΑËÄΑ樘ΑÚ«£¨æΑ«ÞΑÄΑ«Ã£˜Α¢ÚËÃΑΑb«Ã˜ÞΑ 30,000 14,000 24,997
Ë¢Α+Ĕ«ΑÚ櫎½˜
20,000 18,617 12,517
12,000
10,000 6,990
0

-10,000
Capital From Govt
1HW3URǓW
-20,000 -17,993

-30,000 -21,395 NA NA
2010- 2011- 2012- 2013- 2014- 2015- 2016- 2017- 2018-
2011 2012 2013 2014 2015 2016 2017 2018 2019
Source: Budget documents & RBI

Year 2019 RESTRUCTURING OF ASSETS

V͎ΠΑr¨æΑöÞΑ樘Α×êÚ×ËޘΑË¢Α+Äæ˜Úαژ”«æËÚΑ£Ú˜˜Ã˜ÄæΑÞ«£Ä˜”ΑüΑ+Ĕ«ÄΑÄºÞΑÄ”ΑĈāĎ«½Α«ÄÞæ«α
tutions recently?
(a) The lessen the Government of India’s perennial burden of fiscal deficit and current account deficit.
(b) To support the infrastructure projects of Central and State Governments
(c) To act as independent regulator in case of application for loans of Rs. 50 core or more
(d) To aim at faster resolution of stressed assets of Rs. 50 crore or more which are under consortium lending

ANSWERS D

EXPLANATION

b¨˜Α +Äæ˜Úαژ”«æËÚΑ£Ú˜˜Ã˜ÄæΑ ËÚΑ +Α «ÞΑ ÄΑ £Ú˜˜Ã˜ÄæΑ ÃËÄ£Α ÄºÞΑ樁æΑ ¨õ˜Α ”ê˜ÞΑ¢ÚËÃΑ Α ËÚÚËö˜ÚΑ «ÄΑ
ÞæژÞÞΠΑb¨˜Α ׁŽæΑ ÁĔæ˜ÞΑ樘Α ½˜”Α ÄºΑæËΑ¢ËÚÃ꽁æ˜Α Α ژÞ˽êæ«ËÄΑ ×½ÄΑ樁æΑö«½½Α ˜Α ˜û˜Žê昔Α «ÄΑ Αæ«Ã˜Α
ËêĔΑÁÄĘÚΠ
+ÄΑ Α «”ΑæËΑ ˜ÄÞêژΑ Þö«¢æΑ ژÞ˽êæ«ËÄΑ Ë¢Α ÄËÄαטڢËÚëģΑ ÞޘæÞΡΑ ÚËêĔΑ ͋͋Α ×ꍽ«ŽΑ ޘŽæËÚΑ ÄºÞΑ λ«ÄŽ½ê”«Ä£Α
+Ĕ«ΑTËÞæΑTüØÄæÞΑ ÄºμΡΑ͊͒Α×Ú«õæ˜Α½˜Ä”˜ÚÞΑÄ”Α͌͋Α¢Ëژ«£ÄΑÄºÞΑËÄΑAËĔüΑÞ«£Ä˜”Α樘Α«Äæ˜ÚαŽÚ˜”«æËÚΑ
£Ú˜˜Ã˜ÄæΑλ+μΑæËΑ¢ÞæΑæځŽºΑ樘ΑژÞ˽êæ«ËÄΑË¢ΑÞæژÞޘ”ΑÞޘæÞΠΑb¨«ÞΑ¨×טʔΑ«ÄΑ͉͋͊͑ΠΑb¨˜Α樘ÄΑ"«ÄÄŽ˜Α
A«Ä«Þæ˜ÚΑ”˜ÞŽÚ«˜”Α樘ΑÃËõ˜ΑÞΑΑϖ¨ê£˜ΑÞæ˜×Α¢ËÚöÚ”ϖΑ«ÄΑ””Ú˜ÞÞ«Ä£Α樘ΑÄº«Ä£ΑޘŽæËÚϖÞΑ”Α½ËÄÞΠΑb¨˜Α
£Ú˜˜Ã˜ÄæΑöÞΑ½ÞËΑÞ«£Ä˜”ΑüΑ͊͋ΑÁ¶ËÚΑĈāĎ«½Α«ÄÞæ«æêæ«ËÄÞΑ½«º˜Α;+ΡΑ(fHΡΑÃËÄ£ΑË樘ÚÞΠΑ
36

ÞΑטÚΑ樘Α+ΡΑϕb¨˜Α½˜”Α½˜Ä”˜ÚΡΑ樁æΑ«ÞΑ樘Α½˜Ä”˜ÚΑö«æ¨Α樘Α¨«£¨˜ÞæΑ˜û×ËÞêژΡΑÞ¨½½Α˜Αêæ¨Ëګޘ”ΑæËΑ¢ËÚÃêα
½æ˜Α樘ΑژÞ˽êæ«ËÄΑ×½ÄΡΑö¨«Ž¨ΑÞ¨½½Α˜Α×ژޘÄ昔ΑæËΑ樘Α½˜Ä”˜ÚÞΑ¢ËÚΑ樘«ÚΑ××ÚËõ½ΠϕΑb¨˜Α”˜Ž«Þ«ËÄαÁº«Ä£Α
ö«½½Α˜ΑüΑöüΑË¢Α××ÚËõ½ΑË¢ΑϖÁ¶ËÚ«æüΑ½˜Ä”˜ÚÞϖΡΑæ¨ËޘΑö«æ¨Α͏͏ΑטÚΑŽ˜ÄæΑÞ¨Ú˜Α«ÄΑ樘Α££Ú˜£æ˜Α˜û×ËÞêژΠΑThe
ICA agreement covers cases of stressed assets worth at least Rs 50 crore under consortium lending.
Hence option (d) is the correct answer.

ANALYSIS:

Motivation/Why asked?
(”Α˜˜ÄΑ«ÄΑĘöÞΑ¢Ú˜Ùê˜Äæ½üΑޫϘΑ͉͋͊͑ΠΑÄΑ˜ΑŽËõ˜Ú˜”Αö«æ¨Α樘ΑBTΑ×Úˍ½˜ÃΡΑö¨«Ž¨Α¨ÞΑ˜˜ÄΑö«”˜½üΑ×ژõα
½˜ÄæΑ¢ËÚΑ½ÃËÞæΑ͏α͐Αü˜ÚÞΠΑ

SOURCE : ¨ææ×Þ΢έݘŽËÄËëŽæ«Ã˜ÞΠ«Ä”«æ«Ã˜ÞЎËÃέ«Ä”êÞæÚüݍÄº«Ä£έĈα
nance/banking/guidelines-on-inter-creditor-agreement-next-week/articles
¨Ëöέ͏͎͍͉͒͒͏͋ЎÃÞ

BËæΑŽËõ˜Ú˜”Α«ÄΑÄüΑË¢Α樘ΑÞޘÄ櫁½ΑAæ˜Ú«½ΡΑêæΑæË׫ŽÞΑö˜Ú˜Α«ÄΑĘöÞΑ«ÄΑ樘ΑŽËÄæ˜ûæΑË¢ΑژÞæÚêŽæêÚ«Ä£Α˜ąËÚæÞΠΑ

Year 2019 BANK BOARD BUREAU

Q6. The Chairman of public sector banks are selected by the


(a) Banks Board Bureau
(b) Reserve Bank of India
(c) Union Ministry of Finance
(d) Management of the concerned bank

ANSWERS A

EXPLANATION

ÄºÞΑ ˁڔΑ êژêΑ λËÚΑ μΑ ژŽËÃØĔÞΑ¢ËÚΑ ޘ½˜Žæ«ËÄΑ Ë¢Α ¨˜”ÞΑ Ë¢Α ×ꍽ«ŽΑ [˜ŽæËÚΑ ÄºÞΑ Ä”Α "«ÄÄŽ«½Α
+ÄÞæ«æêæ«ËÄÞΑÄ”Α¨˜½×ÞΑÄºÞΑ«ÄΑ”˜õ˜½Ë׫ģΑÞæځ昣«˜ÞΑÄ”ΑŽ×«æ½Αځ«Þ«Ä£Α×½ÄÞΠΑ+æΑöÞΑ«ÄæÚ˔ꎘ”ΑêĔ˜ÚΑ樘Α
Mission IndradhanushΑ¢ËÚΑÄº«Ä£ΑޘŽæËÚΑ½êϨ˜”ΑüΑ樘Α#Ëõ˜ÚÄØÄæΑË¢Α+Ĕ«Α«ÄΑ͉͎͋͊ΠΑHence option (a)
is the correct answer.

ANALYSIS:

Scope for elimination


«æ¨˜ÚΑüËêΑºÄËöΑ樘ΑÄÞö˜ÚΑËÚΑüËêΑ”ËÄϐæΑºÄËöΠΑBËΑގËטΑêæΑ ΑΑ¨ÞΑ½öüÞΑ˜˜ÄΑ«ÄΑ樘ΑĘöÞΠ
b¨˜Α ÄºÞΑ ˁڔΑ êژêΑλ μΑrecently (in 2019)Α«”˜Äæ«Ĉ˜”Α͎͐ΑޘīËÚΑÁā£˜Ã˜ÄæΑטÚÞËÄĘ½ΑË¢Α×ꍽ«ŽΑ
ޘŽæËÚΑ½˜Ä”˜ÚÞΑæËΑ恺˜ΑËõ˜ÚΑ½˜”˜ÚÞ¨«×ΑÚ˽˜ÞΑ«ÄΑ樘Α¢êæêژΠ
37

SOURCE : Standard newspapers


Link: https://www.thehindubusinessline.com/money-and-bank-
«Ä£έα«”˜Äæ«Ĉ˜Þα͎͐αޘīËÚαËƎ˜ÚÞα¢ËÚ὘”˜ÚÞ¨«×αÚ˽˜Þα«Äα×ލÞ݁Ú櫎½˜͋͏͍͑͑͒͏͒ИŽ˜

Please Note:
b¨˜Α#Ëõ˜ÚÄØÄæΑ¨ÞΑژŽÞ昔Α樘Α ÄºΑ ˁڔΑ êژêΑλ μΑ«ÄæËΑ"«ÄÄŽ«½Α[˜Úõ«Ž˜ÞΑ+ÄÞæ«æêæ«ËÄÞΑ
êژêΑλ"[+ μΠΑ+æΑ¨×טʔΑ«ÄΑ͉͋͋͋ΑÞΑ樘Α˜ą˜Žæ«õ˜Ä˜ÞΑË¢ΑΑ ΑöÞΑ«ÄΑÙê˜Þæ«ËÄΠΑ9«Ä”½üΑژ¢˜ÚΑæËΑ
樘Α½«ÄºΑæËΑºÄËöΑÃËژΑËêæΑ樘ΑށØΠ
https://www.thehindu.com/business/government-re-
ŽÞæÞαÄºÞαËÚ”αêژêα«ÄæËα¢Þ«έÚ櫎½˜͏͎͎͑͒͑͏͉ИŽ˜

Year 2019 SERVICE AREA APPROACH

V͐ΠΑb¨˜Α[˜Úõ«Ž˜ΑژΑ××ÚˁŽ¨ΑöÞΑ«Ã×½˜Ã˜Ä昔ΑêĔ˜ÚΑ樘Α×êÚõ«˜öΑË¢
(a) the Integrated Rural Development Programme
(b) Lead Bank Scheme
(c) Mahatma Gandhi National Rural Employment Guarantee Scheme
(d) National Skill Development Mission

ANSWERS B

EXPLANATION

b¨˜Α [˜Úõ«Ž˜ΑژΑ××ÚˁŽ¨Α λ[μΑ «ÄæÚ˔ꎘ”Α «ÄΑ×Ú«½Α ͊͒͑͒ΡΑ «ÄΑ Ëڔ˜ÚΑæËΑ Ú«Ä£Α ËêæΑ Ëڔ˜Ú½üΑ Ä”Α ×½ÄʔΑ
”˜õ˜½Ë×ØÄæΑË¢ΑÚêځ½ΑÄ”ΑޘëαêڍÄΑÚ˜ÞΑË¢Α樘ΑŽËêÄæÚüΡΑöÞΑ˜ûæ˜Ä”˜”ΑæËΑ½½Α+Ĕ«ÄΑގ¨˜”꽘”ΑŽËÃØÚα
Ž«½ΑÄºÞΑ«ÄŽ½ê”«Ä£ΑW˜£«Ëā½ΑWêځ½Α ÄºÞΑλWW ÞμΠ
b¨˜ΑޘÚõ«Ž˜ΑÚ˜Α××ÚˁŽ¨Α«ÞΑÄΑ½æ˜Úāæ«õ˜ΑÄ”Α«Ã×ÚËõ˜”ΑØæ¨Ë”ΑÄ”ΑêĔ˜ÚΑ樘Α樘Α;˜”Α ÄºΑ[Ž¨˜Ã˜Α
¢ËÚΑ樘Α”˜×½ËüØÄæΑË¢ΑÄºΑŽÚ˜”«æΑ¢ËÚΑÚêځ½Α”˜õ˜½Ë×ØÄæΠΑfĔ˜ÚΑ[ΡΑ˜Ž¨ΑÄºΑÚÄŽ¨Α«ÄΑÚêځ½ΑÄ”ΑޘëαêÚα
ÄΑÚ˜ÞΑöÞΑ”˜Þ«£Äæ˜”ΑæËΑޘÚõ˜ΑÄΑÚ˜ΑË¢Α͎͊ΑæËΑ͎͋Αõ«½½£˜ÞΑÄ”Α樘ΑÚÄŽ¨ΑöÞΑژÞ×ËÄÞ«½˜Α¢ËÚΑؘæ«Ä£Α
樘Αʘ”ÞΑË¢ΑÄºΑŽÚ˜”«æΑË¢Α«æÞΑޘÚõ«Ž˜ΑÚ˜Π Hence option (b) is the correct answer.
b¨˜Α ×ګÁÚüΑ ˍ¶˜Žæ«õ˜Α Ë¢Α [ΑöÞΑæËΑ «ÄŽÚ˜Þ˜Α ×Ú˔êŽæ«õ˜Α ½˜Ä”«Ä£Α Ä”Α¢ËÚ£˜Α ˜ą˜Žæ«õ˜Α ½«Äº£˜ÞΑ ˜æö˜˜ÄΑ
ÄºΑŽÚ˜”«æΡΑ×Ú˔êŽæ«ËÄΡΑ×Ú˔êŽæ«õ«æüΑÄ”Α«ÄŽÚ˜Þ˜Α«ÄΑ«ÄŽËØΑ½˜õ˜½Þ
38

ANALYSIS:

Motivation

fT[Α º˜˜×ÞΑ Þº«Ä£Α Ùê˜Þæ«ËÄÞΑ ژ½æ˜”Α æËΑ ˽”Α êæΑ «Ã×ËÚæÄæΑ ގ¨˜Ã˜ÞΠΑ b¨˜Ã˜Α ژטæ«æ«ËÄΑ λ͉͋͊͋Α Ùê˜Þæ«ËÄΑ
Þº˜”ΑËÄΑ½˜”ΑÄºμ

Source/ How to cover? -Α¨ææ×Þ΢έέڍ«ΠËÚ£Π«Äέގګ×æÞέ [εq«˜öAÞ«ÚŽê½Ú”˜æ«½ÞЁÞ×ûΧ«”Ͻ͎͒͑͌

Year 2019 ASSETS OF A BANK

V͑ΠΑr¨«Ž¨ΑË¢Α樘Α¢Ë½½Ëö«Ä£Α«ÞΑÄËæΑ«ÄŽ½ê”˜”Α«ÄΑ樘ΑÞޘæÞΑË¢ΑΑŽËÃØڎ«½ΑÄºΑ«ÄΑ+Ĕ«Χ
(a) Advances
(b) Deposits
(c) Investments
(d) Money at call and short notice

ANSWERS B

EXPLANATION

+ÄΑÞ«Ã×½˜ΑöËڔÞΡΑÄºΑÞޘæÞΑÚ˜ΑŽËÄÞ«”˜Ú˜”Αanything that the bank owns,Αö¨˜Ú˜ÞΑÄºΑ½««½«æ«˜ÞΑÚ˜Α


Äü樫ģΑ樁æΑ樘ΑÄºΑËö˜ÞΑæËΑÞËØËĘΑ˜½Þ˜Π
ΑÄºΑ×½Ž˜ÞΑ«æÞΑ¢êĔÞΑ«ÄΑÞޘæÞΑæËΑ˜ÚÄΑ×ÚËĈæÞΠΑb¨˜ΑÞޘæÞΑ«ÄŽ½ê”˜Α«Äõ˜ÞæØÄæÞΡΑ½ËÄÞΑÄ”Α”õÄŽ˜ÞΡΑ
ÃËĘüΑæΑŽ½½ΑÄ”ΑÞ¨ËÚæΑÄË櫎˜ΡΑ«½½ÞΑ”«ÞŽËêÄ昔ΑÄ”Α×êڎ¨Þ˜”ΠΑ+æΑ½ÞËΑ«ÄŽ½ê”˜ÞΑ樘ΑŽÞ¨Α«ÄΑ¨Ä”Αö«æ¨Α樘Α
ÄºÞΑ Ä”Α ½ÞËΑ樘Α ŽÞ¨Α ¨˜½”Αö«æ¨Α樘Α W +ΠΑb¨˜Α ½««½«æ«˜ÞΑ «ÄŽ½ê”˜Α ”˜×ËÞ«æÞλËæ¨Αæ«Ã˜Α Ä”Α ”˜ÃÄ”μΑ Ä”Α
ËÚÚËö«Ä£ÞΠ Hence (b) is the correct answer.

ANALYSIS:

Scope for elimination

½æ¨Ë꣨Α«æΑ«ÞΑÄΑ ΑæüטΑÙê˜Þæ«ËÄΡΑ樘ΑÄÞö˜ÚΑ
öÞΑÄËæΑ”«ĆŽê½æΑæËΑ£˜æΠ

SOURCE : Macroeconomics (NCERT Class 12)


39

Year 2022 INTEREST COVERAGE RATIO

V͒ΠΑr¨æΑ«ÞΑ樘Α«Ã×ËÚæÄŽ˜ΑË¢Α樘Αæ˜ÚÃΑϕ+Äæ˜Ú˜ÞæΑËõ˜Ú£˜ΑWæ«ËϕΑË¢ΑΑĈÚÃΑ«ÄΑ+Ĕ«ΧΑ
1. It helps in understanding the present risk of a firm that a bank is going to give loan to.
2. It helps in evaluating the emerging risk of a firm that a bank is going to give loan to.
3. The higher a borrowing firm's level of Interest Coverage Ratio, the worse is its ability to service its debt.
[˜½˜ŽæΑ樘ΑŽËÚژŽæΑÄÞö˜ÚΑêÞ«Ä£Α樘ΑŽË”˜Α£«õ˜ÄΑ˜½ËöΠΑ
(a) 1 and 2 only
(b) 2 only
(c) 1 and 3 only
(d) 1, 2 and 3

ANSWERS A

EXPLANATION

b¨˜Α«Äæ˜Ú˜ÞæΑŽËõ˜Ú£˜Αځæ«ËΑ«ÞΑΑ”˜æΑځæ«ËΑÄ”Α×ÚËĈ恍«½«æüΑځæ«ËΑêޘ”ΑæËΑ”˜æ˜ÚëĘΑ¨ËöΑ˜Þ«½üΑΑŽËÃׁÄüΑ
ŽÄΑׁüΑ«Äæ˜Ú˜ÞæΑËÄΑ«æÞΑËêæÞæÄ”«Ä£Α”˜æΠΑΑ¨«£¨˜ÚΑŽËõ˜Ú£˜Αځæ«ËΑ«ÞΑ˜ææ˜ÚΡΑ½æ¨Ë꣨Α樘Α«”˜½Αځæ«ËΑÁüΑ
õÚüΑüΑ«Ä”êÞæÚüΠ Hence statement 3 is not correct. b¨˜Α+Äæ˜Ú˜ÞæΑŽËõ˜Ú£˜Αځæ«ËΑ«ÞΑ½ÞËΑŽ½½˜”Αύæ«Ã˜ÞΑ«Äæ˜Úα
˜ÞæΑ˜ÚʔΠώΑ;˜Ä”˜ÚÞΡΑ«Äõ˜ÞæËÚÞΡΑÄ”ΑŽÚ˜”«æËÚÞΑË¢æ˜ÄΑêޘΑ樫ÞΑ¢ËÚÃ꽁ΑæËΑ”˜æ˜ÚëĘΑΑŽËÃׁÄüϖÞΑÚ«Þº«Ä˜ÞÞΑ
ژ½æ«õ˜ΑæËΑ«æÞΑŽêÚژÄæΑ”˜æΑËÚΑ¢ËÚΑ¢êæêژΑËÚÚËö«Ä£ΠΑ
b¨˜Α«Äæ˜Ú˜ÞæΑŽËõ˜Ú£˜Αځæ«ËΑ«ÞΑêޘ”ΑæËΑޘ˜Α¨ËöΑö˜½½ΑΑĈÚÃΑŽÄΑׁüΑ樘Α«Äæ˜Ú˜ÞæΑËÄΑËêæÞæÄ”«Ä£Α”˜æΠΑ½ÞËΑ
Ž½½˜”Α樘Αæ«Ã˜ÞΑα«Äæ˜Ú˜ÞæΑαΑ˜ÚʔΑځæ«ËΡΑ樫ÞΑځæ«ËΑ«ÞΑêޘ”ΑüΑŽÚ˜”«æËÚÞΑÄ”Α×ÚËÞטŽæ«õ˜Α½˜Ä”˜ÚÞΑæËΑÞޘÞÞΑ
樘ΑÚ«ÞºΑË¢Α½˜Ä”«Ä£ΑŽ×«æ½ΑæËΑΑĈÚÃΠΑHence statements 1 and 2 are correct.

ANALYSIS:

Motivation
+æΑ«ÞΑËĘΑË¢Α樘ΑÚ«ÞºΑÞޘÞÞØÄæΑׁځØæ˜ÚΑË¢ΑΑŽËÃׁÄüΠΑêÚ«Ä£Α樫ÞΑæ«Ã˜ΡΑBTΑÞ«æêæ«ËÄΑ«ÄΑÁÄüΑÄºÞΑ
öÞΑ×ژõ½˜ÄæΠΑ½ÞËΡΑŽËÃׁī˜ÞΑ”˜Ž½Ú˜”ΑÄºÚê×æŽüΑÄ”Α+ ΑŽË”˜ΑöÞΑ˜«Ä£Αêޘ”ΠΑb¨«ÞΑÞ«æêæ«ËÄΑÞæü˜”Α
«ÄΑĘöÞΑ¢ËÚΑΑ£Ë˔Α͌α͍Αü˜ÚÞΑÄ”Α¨˜ÄŽ˜Α«æΑ˜ŽÃ˜Α«Ã×ËÚæÄæΠΑ

Scope for elimination

Statement 3 can be eliminated as higher interest coverage ratio (as the term suggests) mean higher
interest (to be paid)ΑŽÄΑ˜ΑŽËõ˜Ú˜”ΑüΑ樘ΑĈÚÃΑÄ”Α樘ژ¢ËژΡΑ«æÞΑ«½«æüΑæËΑޘÚõ«Ž˜Α«æÞΑ”˜æΑ«ÞΑ˜ææ˜ÚΠΑb¨˜Α
ŽêÚژÄæΑÚ«ÞºΑ×ÚËĈ½˜ΑË¢ΑΑŽËÃׁÄüΑö«½½Α½ÞËΑ¨˜½×Α«ÄΑ˜õ½êæ«Ä£Α¢êæêژΑÚ«ÞºΑ×ÚËĈ½˜ΑË¢Α樘ΑށØΑŽËÃׁÄüΠΑ½ÞËΡΑ
ÄËæ˜Α樁æΑ«ÄΑËæ¨Α樘ΑÞææ˜Ã˜ÄæÞΡΑ£˜Ä˜Ú½ΑöËڔÞΑ½«º˜Α¨˜½×ÞΑ«ÄΑêĔ˜ÚÞæÄ”«Ä£ΑÄ”Α¨˜½×ÞΑ«ÄΑ˜õ½êæ«Ä£ΑÚ˜Α
êޘ”ΠΑ(˜ÄŽ˜ΡΑÄËΑÞטŽ«ĈŽΑº˜üöËڔΑ«ÞΑêޘ”ΡΑ樘ژ¢ËژΡΑÞææ˜Ã˜ÄæΑ͊ΑÄ”Α͋ΑÚ˜ΑÃËÞæ½üΑŽËÚژŽæ

SOURCE : This concept is not covered in the Macroeconomics (NCERT Class 12), while it can be found in
the NCERT Business Studies book
https://ncert.nic.in/ncerts/l/lebs209.pdf
40

Year 2021 THE FUNCTIONING OF THE RBI

Q10. Consider the following statements:


1. The Governor of the Reserve bank of India (RBI) is a appointed by the Central Government.
2. Certain provisions in the Constitution of India give the Central Government the right to issue directions to the
RBI in public interest.
3. The Governor of the RBI draws his power from the RBI Act.
r¨«Ž¨ΑË¢Α樘ΑËõ˜ΑÞææ˜Ã˜ÄæÞΑÚ˜ΑŽËÚژŽæΧΑ
(a) 1 and 2 only
(b) 2 and 3 only
(c) 1 and 3 only
(d) 1, 2 and 3

ANSWERS C

EXPLANATION

ŽŽËڔ«Ä£ΑæËΑ[˜Žæ«ËÄΑ͑ΑË¢ΑW +ΑbΑ͍͊͒͌αΑΑ#Ëõ˜ÚÄËÚΑÄ”ΑλÄËæΑÃËژΑ樁ÄΑ¢ËêÚμΑ˜×êæüΑ#Ëõ˜ÚÄËÚÞΑæËΑ˜Α××Ë«Äæα
˜”ΑüΑ樘Α˜Äæځ½Α#Ëõ˜ÚÄØÄæΠΑHence statement 1 is correct.
ŽŽËڔ«Ä£ΑæËΑ[˜Žæ«ËÄΑ͐ΑË¢ΑW +ΑbΑ͍͊͒͌αΑb¨˜Α˜Äæځ½Α#Ëõ˜ÚÄØÄæΑÁüΑ¢ÚËÃΑæ«Ã˜ΑæËΑæ«Ã˜Α£«õ˜ΑÞꎨΑ”«Ú˜Žæ«ËÄÞΑ
æËΑ樘Α ÄºΑÞΑ«æΑÁüΡΑ¢æ˜ÚΑŽËÄÞê½ææ«ËÄΑö«æ¨Α樘Α#Ëõ˜ÚÄËÚΑË¢Α樘Α ÄºΡΑŽËÄÞ«”˜ÚΑʎ˜ÞށÚüΑ«ÄΑ樘Α×ꍽ«ŽΑ«Äæ˜Úα
˜ÞæΠΑb¨˜Ú˜Α«ÞΑÄËΑÞꎨΑ×ÚËõ«Þ«ËÄΑ«ÄΑ樘ΑŽËÄÞæ«æêæ«ËÄΑË¢Α+Ĕ«ΠΑHence statement 2 is not correct.
ŽŽËڔ«Ä£ΑæËΑ[˜Žæ«ËÄΑ͐ΑË¢ΑW +ΑŽæΑ͍͊͒͌αΑb¨˜Α#Ëõ˜ÚÄËÚΑÄ”Α«ÄΑ¨«ÞΑÞ˜ÄŽ˜Α樘Α˜×êæüΑ#Ëõ˜ÚÄËÚΑÄËëā昔ΑüΑ
¨«ÃΑ«ÄΑ樫ÞΑ˜¨½¢ΡΑÞ¨½½Α½ÞËΑ¨õ˜Α×Ëö˜ÚÞΑË¢Α£˜Ä˜Ú½ΑÞêטګÄæ˜Ä”˜ÄŽ˜ΑÄ”Α”«Ú˜Žæ«ËÄΑË¢Α樘Αą«ÚÞΑÄ”Α樘Α
êޫĘÞÞΑË¢Α樘Α ÄºΡΑÄ”ΑÁüΑ˜û˜ÚŽ«Þ˜Α½½Α×Ëö˜ÚÞΑÄ”Α”ËΑ½½ΑŽæÞΑÄ”Α樫ģÞΑö¨«Ž¨ΑÁüΑ˜Α˜û˜ÚŽ«Þ˜”ΑËÚΑ”ËĘΑ
üΑ樘Α ÄºΠΑHence statement 3 is correct.

ANALYSIS:

Motivation
Þ«ŽΑ«Ä¢ËÚÁæ«ËÄΑËêæΑ樘ΑW +ΣΑ¶êÞæΑ½«º˜ΑΑÙê˜Þæ«ËÄΑËÄΑ樘ΑAËĘæÚüΑT˽«ŽüΑËÃëæ昘Α¢˜æêژ”Π

Scope for elimination

b¨«ÞΑ«ÞΑΑõ˜ÚüΑ˜ÞüΑÙê˜Þæ«ËÄΑ˜ŽêޘΑ樘ΑËÄÞæ«æêæ«ËÄΑË¢Α+Ĕ«Α”˘ÞΑÄËæΑ£«õ˜Α×Ëö˜ÚΑæËΑ樘ΑW˜Þ˜Úõ˜Α ÄºΑË¢Α
+Ĕ«ΠΑ(˜ÄŽ˜ΑêÞ«Ä£Α樁æΡΑö˜ΑŽÄΑ˜½«Ã«Äæ˜ΑË×æ«ËÄÞΑλμΡΑλμΑÄ”Αλ”μΠ

SOURCE : ¨ææ×Þ΢έέڍ«”ËŽÞΠڍ«ΠËÚ£Π«ÄέڔˎÞέTꍽ«Žæ«ËÄÞέT"ÞέW +Aε͉͋͌͏͉͒Πה¢Α

SOURCE : Arthapedia
41

Smart and Effective


UPSC Prelims
Preparation Strategy
b¨˜ΑfT[ΑTژ½«ÃÞΑ«ÞΑ樘ΑĈÚÞæΑÄ”Α¨«£¨½üΑŽËÃטæ«æ«õ˜ΑÞ恣˜ΑË¢Α樘Α«õ«½Α[˜Úõ«Ž˜ÞΑûÃ«Äæ«ËÄΠΑ+æΑŽËÃ×ګޘÞΑæöËΑ
ˍ¶˜Žæ«õ˜αæüטΑׁטÚÞΑλ#˜Ä˜Ú½Α[æꔫ˜ÞΑÄ”Α[bμΑ”˜Þ«£Ä˜”ΑæËΑæ˜ÞæΑΑŽÄ”«”æ˜ϖÞΑºÄËö½˜”£˜ΡΑêĔ˜ÚÞæÄ”«Ä£ΡΑÄ”Α
×æ«æꔘΠΑ
b¨«ÞΑÞ恣˜ΑŽ¨½½˜Ä£˜ÞΑÞ׫ځÄæÞΑæËΑÄËæΑËĽüΑÁÞæ˜ÚΑΑÚˁ”ΑÞü½½êÞΑÄ”Α”×æΑæËΑŽ¨Ä£«Ä£Αׁææ˜ÚÄÞΑêæΑ½ÞËΑ
˜ûŽ˜½Α«ÄΑæ«Ã˜ΑÁā£˜Ã˜ÄæΡΑ«Ä¢ËÚÁæ«ËÄΑژæ˜Äæ«ËÄΡΑÄ”Αāõ«£æ«Ä£Α樘ΑTژ½«ÃÞΑêÄ×ژ”«Žæ«½«æüΠΑ
Ž¨«˜õ«Ä£Α Þꎎ˜ÞÞΑ «ÄΑ樫ÞΑ ˜ûÃΑæځÄގ˜Ä”ÞΑ ØژΑ ¨Ú”ΑöËÚºΣΑ «æΑ ʎ˜ÞÞ«ææ˜ÞΑ Α ¨Ë½«Þ櫎Α Ä”Α
”×æ«õ˜Α××ÚˁŽ¨ΑæËΑ×ژׁځæ«ËÄΠ
[ŽÄΑVWΑ˔˜Α¢ËÚ

Key Strategies for Prelims Preparation


Instant Personalized
Mentoring

Strategized Preparation Plan: ½½ËŽæ˜Α [üÞæ˜Ãæ«ŽΑ êÚژÄæΑ ą«ÚÞΑ Tژׁځæ«ËÄ΢Α


üËêÚΑ Þæê”üΑ æ«Ã˜Α ö«Þ˜½üΑ ŽÚËÞÞΑ Þꍶ˜ŽæÞΡΑΑ 9˜˜×Α Ú˜ÞæΑ ö«æ¨Α êÚژÄæΑ ą«ÚÞΑ æ¨ÚË꣨Α
˜ÄÞêÚ«Ä£ΑüËêΑ ¨õ˜Α ˜ÄË꣨Αæ«Ã˜Α¢ËÚΑ ژõ«Þ«ËÄΑ ĘöÞׁטÚÞΡΑ Á£ā«Ä˜ÞΑ Ä”Α «Ä昣ځæ«Ä£Α 樫ÞΑ
Ä”Α ×ځŽæ«Ž˜Α ÃˎºÞΠΑ TüΑ ææ˜Äæ«ËÄΑ æËΑ üËêÚΑ ºÄËö½˜”£˜Α ö«æ¨Α Þææ«ŽΑ Þꍶ˜ŽæÞΑ æËΑ «Ã×ÚËõ˜Α
ö˜ºΑÚ˜ÞΠ êĔ˜ÚÞæÄ”«Ä£ΑÄ”Αژæ˜Äæ«ËÄΠΑ
Resource Optimization: ¨ËËޘΑ Þæê”üΑ Smart Learning: "ˎêÞΑ ËÄΑ Α êĔ˜ÚÞæÄ”«Ä£Α
Áæ˜Ú«½ÞΑ樁æΑÚ˜ΑËæ¨Αæ¨ËÚË꣨ΑÄ”ΑæËΑ樘Α ŽËϘ×æÞΑ ځ樘ÚΑ 樁ÄΑ ÚËæ˜Α ½˜ÚÄ«Ä£ΡΑ êޘΑ
×Ë«ÄæΠΑ "ˎêÞΑ ËÄΑ Ùꁽ«æüΑ Ëõ˜ÚΑ ÙêÄæ«æüΑ æËΑ ÃĘÃËÄ«ŽÞΡΑ«Ä¢Ë£Ú×¨«ŽÞΡΑÄ”ΑË樘ÚΑ˜ą˜Žæ«õ˜Α
×ژõ˜ÄæΑ£˜ææ«Ä£ΑËõ˜Úö¨˜½Ã˜”Π ½˜ÚÄ«Ä£ΑæË˽ÞΑ¢ËÚΑ˜ææ˜ÚΑژæ˜Äæ«ËÄΠ

Strategic Use of PYQs and Mock Tests: Seeking Personalized Guidance:Α Ä££˜Α
fޘΑ ׁÞæΑ ü˜ÚΑ ׁטÚÞΑ æËΑ êĔ˜ÚÞæÄ”Α 樘Α ö«æ¨ΑØÄæËÚÞΑ¢ËÚΑŽêÞæËëā˜”ΑÞæځ昣«˜ÞΡΑÚ˜ÞΑ
˜ûÃΑׁææ˜ÚÄΡΑ«Ã×ËÚæÄæΑæË׫ŽÞΡΑÄ”ΑÙê˜Þæ«ËÄΑ æËΑ «Ã×ÚËõ˜ΡΑ Ä”Α ÃËæ«õæ«ËÄΠΑ b¨«ÞΑ ØÄæËÚÞ¨«×Α
æژĔÞΠΑ W˜£ê½Ú½üΑ ×ځŽæ«Ž«Ä£Α Ä”Α ÞޘÞÞ«Ä£Α ½ÞËΑ¨˜½×ÞΑ«ÄΑÞæژÞÞΑÁā£˜Ã˜ÄæΡΑ£ê«”«Ä£ΑüËêΑ
×ÚˣژÞÞΑö«æ¨ΑÃˎºΑæ˜ÞæÞΑ«Ã×ÚËõ˜ÞΑ×ژׁځα æ¨ÚË꣨Α ˜ą˜Žæ«õ˜Α ×ځŽæ«Ž˜ÞΑ æËΑ Á«Ä恫ÄΑ
æ«ËÄΑÄ”Αæ«Ã˜ΑÁā£˜Ã˜ÄæΠ Ã˜Ä恽Αö˜½½α˜«Ä£Π

9˜˜×«Ä£Α樘Αʘ”ÞΑË¢Α樘ΑfT[ΑTژ½«ÃÞΑ˜ûÃ«Äæ«ËÄΡΑ樘Αq«Þ«ËÄ+[Α¨ÞΑÚË꣨æΑ«æÞΑ
ÃêŽ¨αŽ˜½˜Úæ˜”ΑAll India GS Prelims Test Series and Mentoring ProgramΑö¨«Ž¨Α
˜ÄŽËÃׁÞÞΑ樘Α˜Äæ«Ú˜ΑfT[ΑÞü½½êÞΑÞ˜”ΑËÄΑ樘Α½æ˜ÞæΑæژĔÞΠ
[ŽÄΑQR code æËΑöæŽ¨ ύfT[ΑTژ½«ÃÞΑ͉͍͋͋΢Α
ą˜Žæ«õ˜Α[æځ昣üΑ¢ËÚΑ͌Π͎ΑAËÄæ¨ÞώΑ
Its key Features include:
ËÃ×ژ¨˜ÄÞ«õ˜ΑŽËõ˜Ú£˜ΑË¢Α樘ΑfT[ΑÞü½½êÞ +Ĕ«õ«”ꁽ«ā˜”ΑT˜ÚÞËā½ΑA˜ÄæËÚ«Ä£
"½˜û«½˜Αæ˜ÞæΑޘګ˜ÞΑގ¨˜”꽘 +ÄÄËõæ«õ˜ΑÞޘÞÞØÄæΑ[üÞæ˜ÃΑÄ”ΑT˜Ú¢ËÚÁϘΑ
ā½üÞ«ÞΑö«æ¨Α½½Α+Ĕ«ΑWÄº«Ä£Þ
;«õ˜Α ËĽ«Ä˜έËć«Ä˜Α æ˜ÞæΑ ”«ÞŽêÞÞ«ËÄΑ Ä”Α ×ËÞæαæ˜ÞæΑ
Ä½üÞ«ÞΑ V꫎ºΑW˜õ«Þ«ËÄΑA˔꽘ΑλVWAμΑ
ÄÞö˜ÚΑº˜üÞΑΑÄ”ΑŽËÃ×ژ¨˜ÄÞ«õ˜Α˜û×½Äæ«ËÄΑ¢ËÚΑ
˜õ˜ÚüΑæ˜ÞæΑׁטÚ

f½æ«Ãæ˜½üΡΑ Α Þæځ昣«ŽΡΑ ö˜½½αÚËêĔ˜”Α ×ژׁځæ«ËÄΑ ŽËÍ«Ä«Ä£Α


[ŽÄΑQR code to register and download
ÞÁÚæΑÞæê”üΑ×½ÄÞΡΑ×ځŽæ«Ž˜ΡΑ¢ËŽêޘ”ΑژÞËêڎ˜ÞΡΑÄ”ΑטÚÞËā½«ā˜”Α brochure “All India GS Prelims Test
£ê«”ÄŽ˜Α«ÞΑº˜üΑæËΑāõ«£æ«Ä£Α樘ΑfT[ΑTژ½«ÃÞΑÞꎎ˜Þޢ꽽üΠ Series and Mentoring Program”
42

Year 2021 URBAN COOPERATIVE BANKS

Q11. With reference to 'Urban Cooperative Banks' in India consider the following statements:
1. They are supervised and regulated by local boards set up by the State Governments.
2. They can issue equity shares and preference shares.
3. They were brought under the purview of the Banking Regulation Act, 1949 through an Amendment in 1966.
Which of the statements given above is/are correct?
(a) 1 only
(b) 2 and 3 only
(c) 1 and 3 only
(d) 1, 2 and 3

ANSWERS B

EXPLANATION
Large cooperative banks with paid-up share capital and reserves of Rs.1 lakh were brought under the
טÚõ«˜öΑË¢Α樘Α Äº«Ä£ΑW˜£ê½æ«ËÄΑŽæΑ͍͊͒͒Αö«æ¨Α˜ą˜ŽæΑ¢ÚËÃΑ͊ÞæΑAÚŽ¨ΡΑ͊͒͏͏ΑÄ”Αö«æ¨«ÄΑ樘ΑÃ«æΑË¢Α樘Α
W˜Þ˜Úõ˜Α ÄºϐÞΑÞêטÚõ«Þ«ËÄΠΑb¨«ÞΑÁں˜”Α樘Α˜£«ÄÄ«Ä£ΑË¢ΑÄΑ˜ÚΑË¢Α”ꁽ«æüΑË¢ΑŽËÄæÚ˽ΑËõ˜ÚΑ樘ޘΑÄºÞΠΑ
Äº«Ä£Αژ½æ˜”Α¢êĎæ«ËÄÞΑλõ«āΠΑ½«Ž˜ÄÞ«Ä£ΡΑÚ˜ΑË¢ΑËטځæ«ËÄÞΡΑ«Äæ˜Ú˜ÞæΑځæ˜ÞΑ˜æŽΠμΑö˜Ú˜ΑæËΑ˜Α£Ëõ˜ÚʔΑüΑ
W +ΑÄ”Αژ£«Þæځæ«ËÄΡΑÁā£˜Ã˜ÄæΡΑê”«æΑÄ”Α½«Ù꫔æ«ËÄΡΑ˜æŽΠΑ£Ëõ˜ÚʔΑüΑ[ææ˜Α#Ëõ˜ÚÄØÄæÞΑÞΑטÚΑ樘Α
×ÚËõ«Þ«ËÄÞΑË¢ΑژÞטŽæ«õ˜Α[ææ˜ΑŽæÞΠΑ[ËΑ樘ΑŽËÄæÚ˽ΑË¢Α樘ΑW +ΑöÞΑׁÚ櫁½ΑÄ”Α«æΑÞ¨Ú˜”Α樘ΑŽËÄæÚ˽Αö«æ¨Α樘Α
ژ£«ÞæځÚΑË¢ΑŽËËטځæ«õ˜ΑÞˎ«˜æ«˜ÞΑË¢Α[ææ˜ÞΡΑ£«õ«Ä£ΑګޘΑæËΑ樘ΑÃêŽ¨α”«ÞŽêÞޘ”Α”ꁽΑŽËÄæÚ˽ΑÄ”Α樘Α”«ĆŽê½α
ties it posed to the central bank. Hence statement 3 is correct.
b¨˜ΑژŽ˜ÄæΑ Äº«Ä£ΑW˜£ê½æ«ËÄΑλØĔØÄæμΑŽæΑ͉͉͋͋Α˜Ä½˜ÞΑ樘ΑW +ΑæËΑ£˜æΑ½½Α樘Α×Ëö˜ÚÞΡΑ«ÄŽ½ê”«Ä£Α
æ¨ËޘΑ¨«æ¨˜ÚæËΑ˜ûŽ½êÞ«õ˜½üΑö«æ¨Α樘Αژ£«ÞæځÚΑË¢ΑŽËËטځæ«õ˜ΑÞˎ«˜æ«˜ÞΠΑ(Ëö˜õ˜ÚΡΑ×Ëö˜ÚÞΑË¢Αژ£«ÞæځÚΑŽËÄæ«Äα
ê˜ΑæËΑ˜Αö«æ¨Α¨«ÃΑêæΑ樘Α×Ëö˜ÚÞΑË¢ΑW +ΑËõ˜ÚÚ«”˜Αæ¨ËޘΑË¢Αژ£«ÞæځÚΠΑHence statement 1 is not correct.
r¨«½˜Α樘ΑÃ˜Ä”ØÄæΑ£«õ˜ÞΑ樘ΑژÙê«Ú˜”Α×Ëö˜ÚÞΑæËΑ樘ΑW +ΑæËΑ恺˜Αæ«Ã˜½üΑŽæ«ËÄΑÄ”ΑÞæ˜×ÞΑæËΑ×ژõ˜ÄæΑf ÞΑ
¢ÚËÃΑ¢«½«Ä£ΑÞËΑ樁æΑ”˜×ËÞ«æËÚÞϐΑÃËÄ«˜ÞΑÚ˜Α×ÚË明昔ΡΑö¨«Ž¨ΑöÞΑ樘ΑÁ«ÄΑ×êÚ×ËޘΑË¢Α樘ΑÃ˜Ä”ØÄæΡΑ«æΑ
½ÞËΑ˜Ä¶Ë«ÄÞΑê×ËÄΑ樘ΑŽ˜Äæځ½ΑÄºΑæËΑÁº˜Αژ£ê½æ«ËÄÞΑêĔ˜ÚΑ WΑŽæΑö«æ¨ËêæΑŽËÃ×ÚËëޫģΑËÄΑ樘ΑŽËËטÚα
ative nature and cooperative principles of the banks. The Reserve Bank has given new guidelines allowing
×ګÁÚüΑêڍÄΑŽËËטځæ«õ˜ΑÄºÞΑλf ÞμΑæËΑê£Ã˜ÄæΑŽ×«æ½Αæ¨ÚË꣨Α«ÞÞêÄŽ˜ΑË¢Α˜Ùê«æüΑÞ¨Ú˜ÞΡΑ×ژ¢˜Úα
˜ÄŽ˜ΑÞ¨Ú˜ÞΑÄ”Α”˜æΑ«ÄÞæÚêØÄæÞΠΑb¨˜Αf ÞΡΑ«æΑށ«”ΡΑŽË꽔Αځ«Þ˜ΑÞ¨Ú˜ΑŽ×«æ½ΑüΑ«ÞÞê˜ΑË¢Α˜Ùê«æüΑæËΑטÚÞËÄÞΑ
ö«æ¨«ÄΑ樘«ÚΑÚ˜ΑË¢ΑËטځæ«ËÄΑ˜ÄÚ˽½˜”ΑÞΑØ͘ÚÞΑÄ”Α½ÞËΑæ¨ÚË꣨Α””«æ«Ëā½Α˜Ùê«æüΑÞ¨Ú˜ÞΑæËΑ樘Α˜û«Þæα
«Ä£ΑØ͘ÚÞΠΑHence statement 2 is correct.

ANALYSIS:

Motivation
fڍÄΑËËטځæ«õ˜ΑÄºÞΑ¨õ˜Α˜˜ÄΑ«ÄΑĘöÞΑ”ê˜ΑæËΑ¢«½êژÞΑË¢ΑõÚ«ËêÞΑÄºÞΑ½«º˜ΑTêĶΑÄ”ΑA¨ÚÞ¨æځΑ
ËËטځæ«õ˜Α ÄºΠΑb¨˜Αژ£ê½æ«ËÄΑŽ¨Ä£˜ÞΑö˜Ú˜ΑÁ”˜ΑژŽ˜Äæ½üΑ«ÄΑ樫ÞΑŽËÄæ˜ûæΠ

SOURCE : https://www.business-standard.com/article/pti-stories/rbi-is-
sues-draft-circular-on-allowing-ucbs-to-augment-capital-121071400994_1.html
43

Year 2021 ROLE OF THE RBI

Q12. In India, the central bank's function as the 'lender of last resort' usually refers to which of the
following?
1. Lending to trade and industry bodies when they fail to borrow from other sources
2. Providing liquidity to the banks having a temporary crisis
3. Lending to governments to finance budgetary deficits
Select the correct answer using the code given below.
(a) 1 and 2
(b) 2 only
(c) 2 and 3
(d) 3 only

ANSWERS B

EXPLANATION

b¨˜Α W˜Þ˜Úõ˜Α ÄºΑ Ë¢Α +Ĕ«Α öÞΑ ˜Þ恍½«Þ¨˜”Α ËÄΑ ×Ú«½Α ͊ΡΑ ͎͊͒͌Α «ÄΑ ŽŽËڔÄŽ˜Α ö«æ¨Α 樘Α ×ÚËõ«Þ«ËÄÞΑ Ë¢Α 樘Α
W˜Þ˜Úõ˜Α ÄºΑ Ë¢Α +Ĕ«Α ŽæΡΑ ͍͊͒͌ΠΑ b¨Ë꣨Α ËÚ«£«Ä½½üΑ ×Ú«õæ˜½üΑ ËöʔΡΑ ޫϘΑ āæ«Ëā½«Þæ«ËÄΑ «ÄΑ ͍͊͒͒ΡΑ 樘Α
W˜Þ˜Úõ˜Α ÄºΑ«ÞΑ¢ê½½üΑËöʔΑüΑ樘Α#Ëõ˜ÚÄØÄæΑË¢Α+Ĕ«ΠΑb¨˜Α¢êĎæ«ËÄÞΑË¢Α樘ΑW˜Þ˜Úõ˜Α ÄºΑŽÄΑ˜ΑŽæ˜£Ëα
rised as follows:
AËĘæÚüΑ×˽«ŽüΑ
W˜£ê½æ«ËÄΑ Ä”Α ÞêטÚõ«Þ«ËÄΑ Ë¢Α 樘Α Äº«Ä£Α Ä”Α ÄËÄαÄº«Ä£Α ĈāĎ«½Α «ÄÞæ«æêæ«ËÄÞΡΑ «ÄŽ½ê”«Ä£Α ŽÚ˜”«æΑΑ
«Ä¢ËÚÁæ«ËÄΑŽËÃׁī˜ÞΑ
W˜£ê½æ«ËÄΑË¢ΑÃËĘüΡΑ¢ËژûΑÄ”Α£Ëõ˜ÚÄØÄæΑޘŽêګ櫘ÞΑÁں˜æÞΑÞΑ½ÞËΑŽ˜Ú恫ÄΑĈāĎ«½Α”˜Ú«õæ«õ˜ÞΑ
˜æΑÄ”ΑŽÞ¨ΑÁā£˜Ã˜ÄæΑ¢ËÚΑ˜Äæځ½ΑÄ”Α[ææ˜Α#Ëõ˜ÚÄØÄæÞΑ
AÄ£˜Ã˜ÄæΑË¢Α¢Ëژ«£ÄΑ˜ûŽ¨Ä£˜ΑژޘÚõ˜ÞΑ
"Ëژ«£ÄΑ˜ûŽ¨Ä£˜ΑÁā£˜Ã˜ÄæΑδΑŽêÚژÄæΑÄ”ΑŽ×«æ½ΑŽŽËêÄæΑÁā£˜Ã˜ÄæΑ
Banker to banks
Äº˜ÚΑæËΑ樘Α˜Äæځ½ΑÄ”Α[ææ˜Α#Ëõ˜ÚÄØÄæÞΑ
Hõ˜ÚÞ«£¨æΑË¢Α樘ΑׁüØÄæΑÄ”Αޘæ潘ØÄæΑÞüÞæ˜ÃÞΑ
êÚژĎüΑÁā£˜Ã˜ÄæΑ
˜õ˜½Ë×ØÄ恽ΑÚ˽˜Α
Research and statistics
ÞΑΑ Äº˜ÚΑæËΑ ÄºÞΡΑ樘ΑW˜Þ˜Úõ˜Α ÄºΑ½ÞËΑacts as the ‘lender of the last resort’. It can come to the
rescue of a bank that is solvent but faces temporary liquidity problems üΑ Þê××½ü«Ä£Α «æΑ ö«æ¨Α ÃꎨΑ
ʘ”˜”Α½«Ù꫔«æüΑö¨˜ÄΑÄËΑËĘΑ˜½Þ˜Α«ÞΑö«½½«Ä£ΑæËΑ˜ûæ˜Ä”ΑŽÚ˜”«æΑæËΑ樁æΑÄºΠΑb¨˜ΑW˜Þ˜Úõ˜Α ÄºΑ˜ûæ˜Ä”ÞΑ樫ÞΑ
¢Ž«½«æüΑæËΑ×ÚË明æΑ樘Α«Äæ˜Ú˜ÞæΑË¢Α樘Α”˜×ËÞ«æËÚÞΑË¢Α樘ΑÄºΑÄ”ΑæËΑ×ژõ˜ÄæΑ×ËÞÞ«½˜Α¢«½êژΑË¢Α樘ΑÄºΡΑ
ö¨«Ž¨Α«ÄΑæêÚÄΑÁüΑ½ÞËΑą˜ŽæΑË樘ÚΑÄºÞΑÄ”Α«ÄÞæ«æêæ«ËÄÞΑÄ”ΑŽÄΑ¨õ˜ΑÄΑ”õ˜ÚޘΑ«ÃׁŽæΑËÄΑĈāĎ«½Α
Þ恍«½«æüΑÄ”Αæ¨êÞΑËÄΑ樘Α˜ŽËÄËÃüΠΑHence option (b) is the correct answer.
44

ANALYSIS:

Þ«ŽΑ«Ä¢ËÚÁæ«ËÄΑËêæΑ樘ΑW˜Þ˜Úõ˜Α ÄºΑË¢Α+Ĕ«ΡΑ恺˜ÄΑ”«Ú˜Žæ½üΑ¢ÚËÃΑ樘ΑÞü½½êÞ

SOURCE : Macroeconomics (NCERT Class 12)


ûŽ˜Ú×æΑ¢ÚËÃΑBWb

Year 2022 BANK BOARD BUREAU

Q13. With reference to the 'Banks Board Bureau (BBB)', which of the following statements are
correct?
1. The Governor of RBI is the Chairman of BBB.
2. BBB recommends for the selection of heads of Public Sector Banks.
3. BBB helps the Public Sector Banks in developing strategies and capital raising plans.
Select the correct answer using the code given below:
(a) 1 and 2 only
(b) 2 and 3 only
(c) 1 and 3 only
(d) 1, 2 and 3

ANSWERS B

EXPLANATION
˜Äæځ½Α#Ëõ˜ÚÄØÄæΑÄËæ«Ĉ˜”Α樘ΑÃ˜Ä”ØÄæΑæËΑ樘ΑBæ«Ëā½«Þ˜”Α ÄºÞΑλAÄ£˜Ã˜ÄæΑÄ”ΑA«ÞŽ˜½½Ä˜ËêÞΑ
TÚËõ«Þ«ËÄÞμΑ[Ž¨˜Ã˜ΡΑ͉͊͒͑Α×ÚËõ«”«Ä£Α樘Α½˜£½Α¢ÚÃ˜öËÚºΑ¢ËÚΑŽËÃ×ËÞ«æ«ËÄΑÄ”Α¢êĎæ«ËÄÞΑË¢Α樘Α ÄºÞΑ ˁڔΑ
êژêΑËÄΑAÚŽ¨Α͋͌ΡΑ͉͋͊͏ΠΑb¨˜Α êژêΑŽŽËڔ«Ä£½üΑÞæÚ昔Α¢êĎæ«ËÄ«Ä£Α¢ÚËÃΑ×Ú«½Α͉͊ΡΑ͉͋͊͏ΑÞΑÄΑêæËÄËα
ÃËêÞΑژŽËÃØĔæËÚüΑË”üΠ
b¨˜ΑËÚ”ΑŽËÃ×ګޘÞΑ樘Α¨«ÚÁÄΡΑæ¨Ú˜˜Α˜ûαËƎ«ËΑØ͘ÚÞΑ«Π˜Α[˜ŽÚ˜æÚüΡΑ˜×ÚæØÄæΑË¢ΑTꍽ«ŽΑÄæ˜Ú×Ú«Þα
˜ÞΡΑ[˜ŽÚ˜æÚüΑË¢Α樘Α˜×ÚæØÄæΑË¢Α"«ÄÄŽ«½Α[˜Úõ«Ž˜ÞΑÄ”Α˜×êæüΑ#Ëõ˜ÚÄËÚΑË¢Α樘ΑW˜Þ˜Úõ˜Α ÄºΑË¢Α+Ĕ«Α
Ä”Αæ¨Ú˜˜Α˜ûטÚæΑØ͘ÚÞΠΑb¨˜Α#Ëõ˜ÚÄËÚΑË¢ΑW +Α«ÞΑÄËæΑ«æÞΑŽ¨«ÚÁÄΠ Hence statement 1 is not correct.
45

b¨˜Α ÄºÞΑ ˁڔΑ êژêΑ «ÞΑ ÁĔæ˜”Α æËΑ ޘ½˜ŽæΑ Ä”Α ××Ë«ÄæØÄæΑ ˁڔΑ Ø͘ÚÞΑ ¢ËÚΑ õÚ«ËêÞΑ ĈāĎ«½Α
«ÄÞæ«æêæ«ËÄÞΑ«ÄΑ×ꍽ«ŽΑޘŽæËÚΑ½«º˜ΑTꍽ«ŽΑޘŽæËÚΑÄºÞΡΑ«ÄÞêځϘΑŽËÃׁī˜ÞΠΑ+æΑ«ÞΑ½ÞËΑژÙê«Ú˜”ΑæËΑêĔ˜Ú恺˜Α
activities in the sphere of governance in these institutions. The functions of the Bureau outlined in the
[˜Žæ«ËÄΑ͐λμΑË¢Α樘Α[Ž¨˜Ã˜ΑÄ”Α«ÄΑÞêÞ˜Ùê˜ÄæΑÃ˜Ä”ØÄæÞΑÞΑטÚΑËڔ˜ÚÞΑË¢Α××Ë«ÄæØÄæΑËÃëæ昘ΑË¢Α
樘Α«Ä˜æΑÚ˜΢
bËΑژŽËÃØĔΑ樘Αޘ½˜Žæ«ËÄΑÄ”Α××Ë«ÄæØÄæΑË¢Α ˁڔΑË¢Α«Ú˜ŽæËÚÞΑ«ÄΑAÄ”æ˜”Α+ÄÞæ«æêæ«ËÄÞΑλr¨Ë½˜Α
b«Ã˜Α«Ú˜ŽæËÚÞΑÄ”Α¨«ÚÁÄμΠΑHence statement 2 is correct.
bËΑ”õ«Þ˜Α樘Α˜Äæځ½Α#Ëõ˜ÚÄØÄæΑËÄΑÁææ˜ÚÞΑژ½æ«Ä£ΑæËΑ××Ë«ÄæØÄæÞΡΑŽËÄĈÚÁæ«ËÄΑËÚΑ˜ûæ˜ÄÞ«ËÄΑË¢Α
æ˜ÄêژΑÄ”Αæ˜Úëāæ«ËÄΑË¢ΑޘÚõ«Ž˜ÞΑË¢Α樘Α«Ú˜ŽæËÚÞΑË¢ΑÁĔæ˜”Α«ÄÞæ«æêæ«ËÄÞΠ
bËΑ”õ«Þ˜Α樘Α˜Äæځ½Α#Ëõ˜ÚÄØÄæΑËÄΑ樘Α”˜Þ«Ú˜”ΑÁā£˜Ã˜ÄæΑÞæÚêŽæêژΑË¢ΑÁĔæ˜”Α«ÄÞæ«æêæ«ËÄÞΡΑæΑ
樘Α½˜õ˜½ΑË¢Α ˁڔΑË¢Α«Ú˜ŽæËÚÞΑÄ”ΑޘīËÚΑÁā£˜Ã˜ÄæΠ
bËΑ”õ«Þ˜Α樘Α˜Äæځ½Α#Ëõ˜ÚÄØÄæΑËÄΑΑÞê«æ½˜ΑטڢËÚÁϘΑ××ځ«Þ½ΑÞüÞæ˜ÃΑ¢ËÚΑÁĔæ˜”Α«ÄÞæ«æêα
tions.
bËΑ ê«½”Α Α ”æΑ ÄºΑ ŽËÄ恫īģΑ ”æΑ ژ½æ«Ä£Α æËΑ 樘Α טڢËÚÁϘΑ Ë¢Α ÁĔæ˜”Α «ÄÞæ«æêæ«ËÄÞΑ Ä”Α «æÞΑ
ËƎ˜ÚÞΠ
bËΑ”õ«Þ˜Α樘Α˜Äæځ½Α#Ëõ˜ÚÄØÄæΑËÄΑ樘Α¢ËÚÃ꽁æ«ËÄΑÄ”Α˜Ä¢Ëڎ˜Ã˜ÄæΑË¢ΑΑŽË”˜ΑË¢ΑŽËĔêŽæΑÄ”Α˜æ¨«ŽÞΑ
¢ËÚΑÁā£˜Ú«½ΑטÚÞËÄĘ½Α«ÄΑÁĔæ˜”Α«ÄÞæ«æêæ«ËÄÞΠ
bËΑ”õ«Þ˜Α樘Α˜Äæځ½Α#Ëõ˜ÚÄØÄæΑËÄΑ˜õ˽õ«Ä£ΑÞê«æ½˜Αæځ«Ä«Ä£ΑÄ”Α”˜õ˜½Ë×ØÄæΑ×ÚˣځÃÞΑ¢ËÚΑÁā£˜α
Ú«½ΑטÚÞËÄĘ½Α«ÄΑÁĔæ˜”Α«ÄÞæ«æêæ«ËÄÞΠ
bËΑ¨˜½×Α樘ΑÄºÞΑ«ÄΑæ˜ÚÃÞΑË¢Α”˜õ˜½Ë׫ģΑêޫĘÞÞΑÞæځ昣«˜ÞΑÄ”ΑŽ×«æ½Αځ«Þ«Ä£Α×½ÄΠ Hence statement
3 is correct.
ÄüΑË樘ÚΑöËÚºΑÞÞ«£Ä˜”ΑüΑ樘Α#Ëõ˜ÚÄØÄæΑ«ÄΑŽËÄÞê½ææ«ËÄΑö«æ¨ΑW˜Þ˜Úõ˜Α ÄºΑË¢Α+Ĕ«Π

ANALYSIS:

W˜ÞæÚêŽæêÚ«Ä£ΑË¢Α ÄºÞΑ ˁڔΑ êژêΑöÞΑ«ÄΑĘöÞΑÞΑ«æÞΑØ͘ÚÞΑ˜ûæ˜Ä”˜”ΑæöËαü˜ÚΑæ˜ÚÃΑŽËĎ½ê”˜”ΑËÄΑ


×Ú«½Α͉͊ΠΑλW˜×˜æ˜”Α樘ØΑαΑ͉͋͊͒μ

Scope for elimination


Scope for elimination:Α+¢Α樘ΑÞæꔘÄæΑºÄËöÞΑ樁æΑW +Α£Ëõ˜ÚÄËÚΑ«ÞΑÄËæΑŽ¨«ÚÁÄΑË¢Α Α樘ÄΑË×æ«ËÄΑΡΑΑÄ”Α
ΑŽÄΑ˜Α˜Þ«½üΑ˜½«Ã«Äæ˜”Π

SOURCE : ¨ææ×Þ΢έݍÄºÞËÚ”êژêΠËÚ£Π«Äݍêژêα×ÚËĈ½˜έ
https://economictimes.indiatimes.com/news/economy/policy/gov-
ernment-to-soon-approve-recast-of-banks-board-bureau/articleshow/91574167.cms
https://economictimes.indiatimes.com/news/company/corpo-
rate-trends/banks-board-bureau-recommends-names-for-chiefs-of-psu-banks/articleshow
/90275752.cms

Please Note:
In the explanation to the 2019 question, it has been mentioned that the BBB has been
replaced by Financial Services Institutions Bureau (FSIB)
46

PAYMENT ECOSYSTEM
(06 Questions)

Year 2017 NPCI

Q1. Consider the following statements:


1. National Payment Corporation of India (NPCI) helps in promoting the financial inclusion in the country.
2. NPCI has launched RuPay, a card payment scheme.
Which of the statements given above is/are correct?
(a) 1 only
(b) 2 only
(c) Both 1 and 2
(d) Neither 1 nor 2

ANSWERS C

EXPLANATION

λBT+μΑ«ÞΑÄΑêÍژ½½ΑËÚ£Ä«āæ«ËÄΑ¢ËÚΑ½½Αژ恫½ΑׁüØÄæÞΑÞüÞæ˜ÃΑ«ÄΑ+Ĕ«ΠΑ+æΑöÞΑޘæΑê×Αö«æ¨Α樘Α£ê«”ÄŽ˜Α
Ä”ΑÞê××ËÚæΑË¢Α樘ΑW˜Þ˜Úõ˜Α ÄºΑË¢Α+Ĕ«ΑλW +μΑÄ”Α+Ĕ«ÄΑ ÄºÞϐΑÞÞˎ«æ«ËÄΑλ+ μΠ
b¨˜ΑŽËژΑˍ¶˜Žæ«õ˜ΑöÞΑæËΑŽËÄÞ˽«”æ˜ΑÄ”Α«Ä昣ځæ˜Α樘ΑÃê½æ«×½˜ΑÞüÞæ˜ÃÞΑö«æ¨ΑõÚü«Ä£ΑޘÚõ«Ž˜Α½˜õ˜½ÞΑ«ÄæËΑ
āæ«ËÄαö«”˜ΑêÄ«¢ËÚÃΑÄ”ΑÞæÄ”Ú”ΑêޫĘÞÞΑ×Úˎ˜ÞÞΑ¢ËÚΑ½½Αژ恫½ΑׁüØÄæΑÞüÞæ˜ÃÞΠΑb¨˜ΑË樘ÚΑˍ¶˜Žæ«õ˜Α
öÞΑæËΑ¢Ž«½«ææ˜ΑÄΑąËڔ½˜ΑׁüØÄæΑ؎¨Ä«ÞÃΑæËΑ˜Ä˜ĈæΑ樘ΑŽËÃÃËÄΑÁÄΑŽÚËÞÞΑ樘ΑŽËêÄæÚüΑÄ”Α
¨˜½×ΑĈāĎ«½Α«ÄŽ½êÞ«ËÄΠΑHence statement 1 is correct.
+æΑ Ëą˜ÚÞΑ ÁÄüΑ ×Ú˔êŽæÞΑ Ä”Α ޘÚõ«Ž˜ÞΑ ö¨«Ž¨Α «ÄŽ½ê”˜Α WêTüΑ ŽÚ”ΡΑ (+AΑ ××ΡΑ fÄ«Ĉ˜”Α ׁüØÄæΑ «Äæ˜Ú¢Ž˜Α
ÃËÄ£ΑË樘ÚÞΠΑHence statement 2 is correct.

ANALYSIS:

BËæΑŽËõ˜Ú˜”Α«ÄΑÄüΑË¢Α樘ΑÞޘÄ櫁½ΑAæ˜Ú«½ΡΑêæΑÞꎨΑæË׫ŽÞΑË¢æ˜ÄΑ×טÚΑ«ÄΑ樘ΑĘöÞׁטÚÞΠΑËõ˜ÚΑ«æΑÞΑ
ׁÚæΑË¢ΑB˜öÞׁטÚΑÄ”ΑêÚژÄæΑą«ÚÞΑÞΑBT+Α«ÞΑژ£ê½Ú½üΑ«ÄΑĘöÞΑ¢ËÚΑÞËØΑËÚΑ樘ΑË樘ÚΑ«Ä«æ«æ«õ˜ΠΑb¨˜Α
«£«æ½ΑׁüØÄæΑ¨ÞΑŽËÄæ«ÄêËêÞ½üΑژÁ«Ä˜”Α«ÄΑĘöÞΑÄ”ΑBT+Α«ÞΑËĘΑË¢Α樘ΑÃËÞæΑ«Ã×ËÚæÄæΑ«ÄÞæ«æêæ«ËÄÞΑ
associated with it.
47

SOURCE : http://pib.nic.in/news- NPCI operated payment platforms


ite/PrintRelease.aspx-
?relid=160339 WêTüΑŽÚ”Þ
fÄ«Ĉ˜”ΑTüØÄæÞΑ+Äæ˜Ú¢Ž˜
http://www.thehin- ¨ÚæΑ «½½ΑTüØÄæÞ
du.com/business/Econo-
”¨ÚΑ˜Ä½˜”ΑTüØÄæÞ
my/smoothen-bhim-re-
"[b£ΑËÚΑBæ«Ëā½Α½˜ŽæÚËÄ«ŽΑb˽½Α˽½˜Žæ«ËÄ
fund-process-it-minister-t
Bæ«Ëā½ΑêæËÁ昔Α½˜Ú«Ä£Α(Ëêޘ
o-npci/article18185655.ece
+Ãؔ«æ˜ΑTüØÄæΑ[üÞæ˜Ã
Ϋ͒͒ά΢ΑΑf[[αÞ˜”ΑׁüØÄæÞΑÞüÞæ˜Ã
Bæ«Ëā½Α"«ÄÄŽ«½Α[ö«æŽ¨
¨˜Ùê˜ΑbÚêρæ«ËÄΑ[üÞæ˜Ã

Year 2017 8QLɩHG3D\PHQW,QWHUIDFH

V͋ΠΑr¨«Ž¨ΑË¢Α樘Α¢Ë½½Ëö«Ä£Α«ÞΑΑÃËÞæΑ½«º˜½üΑŽËÄޘÙê˜ÄŽ˜ΑË¢Α«Ã×½˜Ã˜Äæ«Ä£Α樘ΑϏfÄ«Ĉ˜”ΑTüØÄæÞΑΑΑΑΑΑΑΑΑΑΑΑΑΑΑΑΑ
Interface (UPI)?
(a) Mobile wallets with not be necessary for online payments.
(b) Digital currency will totally replace the physical currency in about two decades.
(c) FDI inflows will drastically increase.
(d) Direct transfer of subsidies to poor people will become very effective.

ANSWERS A

EXPLANATION

fT+Α«ÞΑΑׁüØÄæΑÞüÞæ˜ÃΑ樁æΑ½½ËöÞΑÃËĘüΑæځÄÞ¢˜ÚΑ˜æö˜˜ÄΑÄüΑæöËΑÄºΑŽŽËêÄæÞΑüΑêÞ«Ä£ΑΑÞÁÚæα
רËĘΠΑfT+Α½½ËöÞΑΑŽêÞæËØÚΑæËΑׁüΑ”«Ú˜Žæ½üΑ¢ÚËÃΑΑÄºΑŽŽËêÄæΑæËΑ”«ą˜Ú˜ÄæΑØڎ¨ÄæÞΡΑËæ¨ΑËĽ«Ä˜ΑÄ”Α
Ëć«Ä˜ΡΑ ö«æ¨ËêæΑ 樘Α ¨ÞÞ½˜Α Ë¢Α æü׫ģΑ ŽÚ˜”«æΑ ŽÚ”Α ”˜æ«½ÞΡΑ +"[Α ŽË”˜ΡΑ ËÚΑ ĘæΑ Äº«Ä£έö½½˜æΑ ׁÞÞöËڔÞΠΑ
Hence option (a) is the correct answer.

ANALYSIS:

ÞטŽ«½½üΡΑ ޫϘΑ 樘Α æ«Ã˜Α Ë¢Α ˜ÃËĘæ«āæ«ËÄΑ Ä”Α ˜õ˜ÚΑ ޫϘΑ 樘Α #Ëõ˜ÚÄØÄæϐÞΑ ˜ąËÚæÞΑ æËΑ 恺˜Α Þæ˜×ÞΑ
æËöÚ”ÞΑΑϏÞ¨½˜ÞÞΑŽËÄËÃüϐΡΑTüØÄæΑŽËÞüÞæ˜ÃΑ¨ÞΑ˜˜ÄΑΑژŽêÚÚ«Ä£Α樘ØΠ

SOURCE : ¨ææ×΢έέöööΠ樘¨«Ä”êЎËÃݍêޫĘÞÞέŽËÄËÃüέr¨æα«ÞαfÄ«Ĉ˜”αTüα
ment-Interface/article14593189.ece
48

Year 2018 MERCHANT DISCOUNT RATE

Q3. Which one of the following best describes the term "Merchant Discount Rate" sometimes seen
in news?
(a) The incentive given by a bank to a merchant for accepting payments through debit cards pertaining to that
bank.
(b) The amount paid back by banks to their customers when they use debit cards for financial transactions for
purchaing goods or services.
(c) The charge to a merchant by a bank for accepting payments from his customers through the bank's debit cards.
(d) The incentive given by the Government to merchants for promoting digital payments by their customers
through Point of Sale (PoS) machines and debit cards.

ANSWERS C

EXPLANATION

A˜ÚŽ¨ÄæΑ«ÞŽËêÄæΑWæ˜Α«ÞΑΑ¢˜˜ΑŽ¨Ú£˜”Α¢ÚËÃΑΑØڎ¨ÄæΑüΑΑÄºΑ¢ËÚΑŽŽ˜×æ«Ä£ΑׁüØÄæÞΑ¢ÚËÃΑŽêÞæËÃα
˜ÚÞΑ æ¨ÚË꣨Α ŽÚ˜”«æΑ Ä”Α ”˜«æΑ ŽÚ”ÞΑ «ÄΑ 樘«ÚΑ ˜Þ恍½«Þ¨Ã˜ÄæÞΠΑ +æΑ ŽËÃטÄށæ˜ÞΑ 樘Α ŽÚ”Α «ÞÞê«Ä£Α ÄºΡΑ 樘Α
½˜Ä”˜ÚΑö¨«Ž¨Α×êæÞΑ樘ΑTË[Αæ˜Úëā½ΑÄ”ΑׁüØÄæΑ£æ˜öüÞΑÞꎨΑÞΑAÞæ˜ÚŽÚ”ΑËÚΑq«ÞΑ¢ËÚΑ樘«ÚΑޘÚõ«Ž˜ÞΠΑ
AWΑ Ž¨Ú£˜ÞΑ Ú˜Α êÞꁽ½üΑ Þ¨Ú˜”Α «ÄΑ ×Ú˜α£Ú˜˜”Α ×ÚË×ËÚæ«ËÄΑ ˜æö˜˜ÄΑ 樘Α ÄºΑ Ä”Α Α Øڎ¨ÄæΑ Ä”Α «ÞΑ
˜û×ژÞޘ”Α«ÄΑטڎ˜Ä恣˜ΑË¢ΑæځÄށŽæ«ËÄΑÃËêÄæΠΑHence option (c) is the correct answer.

ANALYSIS:

¢æ˜ÚΑ”˜ÃËĘæ«āæ«ËÄΑ«ÄΑBËõ˜Ã˜ÚΑ͉͋͊͏ΡΑ樘ΑêޘΑË¢ΑŽÞ¨½˜ÞÞΑÃ˔˜ΑË¢ΑׁüØÄæÞΑ«ÄŽÚ˜Þ˜”ΑÄ”ΑA˜ÚŽ¨ÄæΑ
«ÞŽËêÄæΑWæ˜Α¨ÞΑ½öüÞΑ˜˜ÄΑ«ÄΑ樘ΑĘöÞΠ

SOURCE : https://www.thehindu.com/business/Economy/rbi-reduces-mer-
chant-discount-rates-for-debit-cards/article21284463.ece

Please Note:
*Although the theme seems easy now (in 2023), it was not as prevalent a concept in 2018 as
it is today.
49

Year 2018 ATMs

Q4. Which one of the following links all the ATMs in India
(a) Indian Banks' Association
(b) National Securities Depository Limited
(c) National Payments Corporation of India
(d) Reserve Bank of India

ANSWERS C

EXPLANATION
Bæ«Ëā½Α"«ÄÄŽ«½Α[ö«æŽ¨ΑλB"[μΑbAΑĘæöËÚºΑ¨õ«Ä£Α͌͐ΑØ͘ÚÞΑÄ”ΑŽËÄʎæ«Ä£ΑËêæΑ͎͉Ρ͉͉͉ΑbAÞΑöÞΑ
恺˜ÄΑËõ˜ÚΑüΑBT+Α¢ÚËÃΑ+ÄÞæ«æêæ˜Α¢ËÚΑ˜õ˜½Ë×ØÄæΑÄ”ΑW˜Þ˜ÚŽ¨Α«ÄΑ Äº«Ä£Αb˜Ž¨Ä˽ˣüΑλ+W bμΑËÄΑ˜Ž˜Ãα
˜ÚΑ͍͊ΡΑ͉͉͋͒ΠΑb¨êÞΑB"[Α«ÞΑÚêÄΑüΑ樘ΑBæ«Ëā½ΑTüØÄæÞΑËÚ×Ëځæ«ËÄΑË¢Α+Ĕ«ΑλBT+μΠΑHence option (c) is
the correct answer.

Hõ˜ÚΑ樘ΑÞׁÄΑË¢Α¢˜öΑü˜ÚÞΡΑB"[ΑbAΑĘæöËÚºΑ¨ÞΑ£ÚËöÄΑÁÄüΑ¢Ë½”ÞΑÄ”Α«ÞΑÄËöΑ樘Α½˜”«Ä£ΑÃê½æ«½æ˜Ú½Α
bAΑĘæöËÚºΑ«ÄΑ樘ΑŽËêÄæÚüΠΑÞΑËÄΑ͌͊ÞæΑê£êÞæϐΑ͊͐ΡΑ樘ژΑö˜Ú˜Α͍͒͊ΑØ͘ÚÞΑ樁æΑ«ÄŽ½ê”˜ÞΑ͉͊͊Α«Ú˜ŽæΡΑ͐͐͏Α
[êΑØ͘ÚÞΡΑ͎͏ΑWW ÞΑÄ”Α͑Αr;HÞΑêÞ«Ä£ΑB"[ΑĘæöËÚºΑŽËÄĘŽæ˜”ΑæËΑÃËژΑ樁ÄΑ͋Π͌͐Α;ŽΑbAΠΑB"[Α¨ÞΑ
˜Þ恍½«Þ¨˜”Α Α ÞæÚËÄ£Α Ä”Α ÞêÞ恫ā½˜Α Ëטځæ«Ëā½Α Ã˔˜½Α ö«æ¨Α «Äα¨ËêޘΑ Ž×«½«æ«˜ÞΑ Ä”Α æ˔üΑ ŽÄΑ ˜Α
ŽËÃׁژ”ΑæΑׁÚΑö«æ¨ΑË樘ÚΑÁ¶ËÚΑÄ”Αö˜½½α˜Þ恍½«Þ¨˜”ΑÞö«æŽ¨ΑĘæöËÚºÞΠΑb¨˜ΑËטځæ«Ëā½Α¢êĎæ«ËÄÞΑÄ”Α
ޘÚõ«Ž˜ÞΑÚ˜ΑæΑׁÚΑö«æ¨ΑÃËÞæΑË¢Α樘Α£½Ë½ΑbAΑĘæöËÚºÞΠ

ANALYSIS:

Scope for elimination

Not much scope but still if the functions and roles of the other bodies are known to some extent, one
can easily eliminate options A, B and D.

SOURCE : https://www.thehindu.com/business/NPCI-tightens-securi-
ty-as-criminals-target-ATMs-e-banking/article60481538.ece

ÄΑ˜ûŽ˜Ú×æΑ¢ÚËÃΑ樘ΑËõ˜ΑÚ櫎½˜

Other links: https://www.live-


mint.com/Money/Gg9Z4hQBxS-
GKIjnY9qRP7N/Every-
thing-to-know-about-your-ATM.
html
50

Year 2018 BHIM

Q5. With reference to digital payments, consider the following statements:


1. BHIM app allows the user to tranfer money to anyone with a UPI-enabled bank account.
2. While a chip-pin debit card has four factors authentication, BHIM app has only two factors of authentication.
Which of the statements given above is/are correct?
(a) 1 only
(b) 2 only
(c) Both 1 and 2
(d) Neither 1 nor 2

ANSWERS A

EXPLANATION

Statement 1 is correct: ¨ÚæΑ+Äæ˜Ú¢Ž˜Α¢ËÚΑAËĘüΑλ (+AμΑ«ÞΑΑׁüØÄæΑ××Α樁æΑ½˜æÞΑüËêΑÁº˜ΑÞ«Ã×½˜ΡΑ


˜ÞüΑÄ”ΑÙ꫎ºΑæځÄށŽæ«ËÄÞΑêÞ«Ä£ΑfÄ«Ĉ˜”ΑTüØÄæÞΑ+Äæ˜Ú¢Ž˜ΑλfT+μΠΑxËêΑŽÄΑÁº˜Α”«Ú˜ŽæΑÄºΑׁüØÄæÞΑ
æËΑÄüËĘΑËÄΑfT+ΑêÞ«Ä£Α樘«ÚΑfT+Α+ΑËÚΑގÄÄ«Ä£Α樘«ÚΑVWΑö«æ¨Α樘Α (+AΑ××ΠΑxËêΑŽÄΑ½ÞËΑژÙê˜ÞæΑÃËĘüΑ
æ¨ÚË꣨Α樘Α××Α¢ÚËÃΑΑfT+Α+Π

Statement 2 is not correct:Α"ÚËÃΑΑŽËÄÞêØÚΑ×Ë«ÄæΑË¢Αõ«˜öΡΑ樘ژΑÚ˜Αæ¨Ú˜˜Α½˜õ˜½ÞΑË¢Αê樘Ä櫎æ«ËÄΑ樁æΑ


Ú˜ΑژÙê«Ú˜”Α«ÄΑ樫ÞΑ××ΠΑHĘΑ«ÞΑ樘Α”˜õ«Ž˜Α+ΑÄ”ΑÃˍ«½˜ΑÄê͘ÚΡΑ樘ÄΑ樘ΑÄºΑŽŽËêÄæΑö¨«Ž¨ΑüËêΑÚ˜Α
½«Äº«Ä£ΑæËΑ樫ÞΑ××ΡΑÄ”Α樘ΑĈā½½üΑ樘ΑfT+ΑT«ÄΑö¨«Ž¨Α«ÞΑʘ”˜”ΑæËΑŽËÃ×½˜æ˜Α樘ΑæځÄށŽæ«ËÄΠΑb¨˜Ú˜ΑÚ˜Αæ¨Ú˜˜Α
¢ŽæËÚÞΑË¢Αê樘Ä櫎æ«ËÄΑõ˜ÚÞêÞΑΑÄËÚÁ½ΑĘæΑÄº«Ä£Α××ΑËÚΑΑŽ¨«×α׫ÄΑ”˜«æΑŽÚ”Αö¨«Ž¨Αö«½½ΑËĽüΑ¨õ˜Α
two factors of authentication

ANALYSIS:

Scope for elimination

As since the demonetisation, the dependence on UPI-based payment system especially BHIM had
tremendously increased, 樫ÞΑ «ÞΑ ÃËژΑ ÄΑ ××½«Žæ«ËÄαæüטΑ Ë¢Α Ùê˜Þæ«ËÄΠΑ (˜ÄŽ˜Α «æΑ æ˜Þ昔Α 樘Α ×ځŽæ«Ž½Α
ºÄËö½˜”£˜ΑË¢Α樘ΑÞ׫ځÄæΠΑ[ææ˜Ã˜ÄæΑ͋Α«ÞΑˍõ«ËêÞ½üΑ«ÄŽËÚژŽæΑ˜õ˜ÄΑ«¢Αö˜Α¶êÞæΑºÄËöΑ樁æΑΑ”˜«æΑŽÚ”Α¨ÞΑΑ
͋὘õ˜½ΑޘŽêÚ«æüΑ”ÄΑÞææ˜Ã˜ÄæΑ͊Α«ÞΑŽËÚژŽæΠ

Motivation
ê˜Α æËΑ 樘Α ×ژõ½˜ÄŽ˜Α Ë¢Α ŽÞ¨½˜ÞÞΑ Ã˔˜ÞΑ Ë¢Α ׁüØÄæΑ Ä”Α ½ÞËΑ 樘Α ˜ąËÚæÞΑ Ë¢Α 樘Α £Ëõ˜ÚÄØÄæΑ æËΑ
lessen the use of hard cash.

How to cover? Reliance on practical knowledge. UPSC has repeatedly tested this.
51

Year 2019 STORAGE OF PAYMENT SYSTEM DATA

Q6. Consider the following statements:


The Reserve Bank of India’s recent directives relating to ‘Storage of Payment System Data’, popularly known as data
diktat, command the payment system providers that
1. they shall ensure that entire data relating to payment systems operated by them are store in a system only in
India.
2. they shall ensure that the systems are owned and operated by public sector enterprises.
3. they shall submit the consolidated system audit report to the Comptroller and Auditor General of India by the
end of the calendar year
Which of the statements given above is/are correct?
(a) 1 only
(b) 1 and 2 only
(c) 3 only
(d) 1, 2 and 3

ANSWERS A

EXPLANATION

b¨˜ΑW˜Þ˜Úõ˜Α ÄºÞΑ”«Ú˜Žæ«õ˜Αژ½æ˜”ΑæËΑ[æËځ£˜ΑË¢ΑTüØÄæΑ[üÞæ˜ÃΑæΑ×ÚËõ«”˜ÞΑ樁æ΢Α

All system providers shall ensure that the entire data relating to payment systems operated by them
are stored in a system only in India.Αb¨«ÞΑ”æΑÞ¨Ë꽔Α«ÄŽ½ê”˜Α樘Α¢ê½½Α˜Ä”αæËα˜Ä”ΑæځÄށŽæ«ËÄΑ”˜æ«½ÞΑέΑ
«Ä¢ËÚÁæ«ËÄΑŽË½½˜Žæ˜”ΑέΑŽÚÚ«˜”ΑέΑ×Úˎ˜Þޘ”ΑÞΑׁÚæΑË¢Α樘ΑØÞށ£˜ΑέΑׁüØÄæΑ«ÄÞæÚêŽæ«ËÄΠΑ"ËÚΑ樘Α¢Ëژ«£ÄΑ
½˜£ΑË¢Α樘ΑæځÄށŽæ«ËÄΡΑ«¢ΑÄüΡΑ樘Α”æΑŽÄΑ½ÞËΑ˜ΑÞæËژ”Α«ÄΑ樘Α¢Ëژ«£ÄΑŽËêÄæÚüΡΑ«¢ΑژÙê«Ú˜”Π

[üÞæ˜ÃΑ×ÚËõ«”˜ÚÞΑÞ¨½½Α˜ÄÞêژΑŽËÃ×½«ÄŽ˜ΑË¢Αλ«μΑËõ˜Αö«æ¨«ÄΑΑטګ˔ΑË¢ΑÞ«ûΑÃËÄæ¨ÞΑÄ”Αژ×ËÚæΑŽËÃ×½«ÄŽ˜Α
Ë¢Α樘ΑށØΑæËΑ樘ΑW˜Þ˜Úõ˜Α ÄºΑ½æ˜ÞæΑüΑHŽæˍ˜ÚΑ͎͊ΡΑ͉͋͊͑Π

[üÞæ˜ÃΑ ×ÚËõ«”˜ÚÞΑ Þ¨½½Α ÞêÃ«æΑ 樘Α [üÞæ˜ÃΑ ꔫæΑ W˜×ËÚæΑ λ[WμΑ ËÄΑ ŽËÃ×½˜æ«ËÄΑ Ë¢Α 樘Α ژÙê«Ú˜Ã˜ÄæΑ æΑ λ«μΑ
above. The audit should be conducted by CERT-IN empaneled auditors certifying completion of activi-
ty atΑλ«μΑËõ˜ΠΑ

b¨˜Α[WΑ”ê½üΑ××ÚËõ˜”ΑüΑ樘Α ˁڔΑË¢Α樘ΑÞüÞæ˜ÃΑ×ÚËõ«”˜ÚÞΑÞ¨Ë꽔Α˜ΑÞêÃ«æ昔ΑæËΑ樘ΑW˜Þ˜Úõ˜Α ÄºΑÄËæΑ


½æ˜ÚΑ樁ÄΑ˜Ž˜Ã˜ÚΑ͌͊ΡΑ͉͋͊͑ΠΑHence, only statement 1 is correct.

ANALYSIS:

It was widely covered in 樘ΑĘöÞΑ«ÄΑ͉͋͊͑α͊͒ΠΑBËæΑŽËõ˜Ú˜”Α«ÄΑÄüΑË¢Α樘ΑÞޘÄ櫁½ΑAæ˜Ú«½ΡΑêæΑÞꎨΑæË׫ŽÞΑ


Ë¢æ˜ÄΑ×טÚΑ«ÄΑ樘ΑĘöÞׁטÚÞΠΑÄΑŽËõ˜ÚΑ«æΑÞΑׁÚæΑË¢ΑêÚژÄæΑą«ÚÞΠ

SOURCE : ¨ææ×Þ΢έέöööΠڍ«ΠËÚ£Π«Äέގګ×æÞέBËæ«ĈŽæ«ËÄfޘÚЁÞ×ûΧ+”Ͻ͍͍͊͊͋
52

INFLATION AND RECESSION


(04 Questions)

Year 202o CPI/WPI

Q1. Consider the following statements:


1. The weightage of food in Consumer Price Index (CPI) is higher than that in Wholesale Price Index (WPI).
2. The WPI does not capture changes in the prices of services, which CPI does.
3. Reserve Bank of India has now adopted WPI as its key measure of inflation and to decide on changing the key
policy rates.
Which of the statements given above is/are correct?
(a) 1 and 2 only
(b) 2 only
(c) 3 only
(d) 1, 2 and 3

ANSWERS A

EXPLANATION

+Äĉæ«ËÄΑŽÄΑ˜Α”˜ÞŽÚ«˜”ΑÞΑ樘Α£˜Ä˜Ú½ΑګޘΑ«ÄΑ樘Α×Ú«Ž˜ΑË¢Α£Ë˔ÞΑÄ”ΑޘÚõ«Ž˜ÞΑ«ÄΑÄΑ˜ŽËÄËÃüΑËõ˜ÚΑæ«Ã˜ΠΑ
+æΑ«ÞΑŽ½Žê½æ˜”ΑüΑæځŽº«Ä£Α樘Α«ÄŽÚ˜Þ˜Α«ÄΑ×Ú«Ž˜ÞΑË¢Α˜ÞޘÄ櫁½ÞΠΑ

b¨˜ΑA˜æ¨Ë”ÞΑbËΑ½Žê½æ˜Α+Äĉæ«ËÄΑ

CPI:Αb¨˜Α×ګÁÚüΑ«Ä”˜ûΑ樁æΑæځŽºÞΑ樘ΑŽ¨Ä£˜Α«ÄΑژ恫½Α×Ú«Ž˜ÞΑË¢Α˜ÞޘÄ櫁½Α£Ë˔ÞΑÄ”ΑޘÚõ«Ž˜ÞΑŽËÄÞêؔΑ
üΑ+Ĕ«ÄΑ¨Ëêޘ¨Ë½”ÞΑ«ÞΑ樘ΑËÄÞêØÚΑTÚ«Ž˜Α+Ĕ˜ûΑËÚΑT+ΠΑ+æΑÞ¨ËöÞΑ樘Α«ÃׁŽæΑË¢Α«Äĉæ«ËÄΑËÄΑטË×½˜ΠΑ+æΑ«ÞΑ
½öüÞΑêޘ”Α¢ËÚΑ˜ÚĘÞÞΑ½½ËöÄŽ˜ΠΑBËöΡΑ«æΑ«ÞΑ½ÞËΑ˜«Ä£Αêޘ”Α¢ËÚΑÃËĘæÚüΑ×˽«ŽüΑ¢ËÚÃ꽁æ«ËÄΠΑ

[«ÄŽ˜Α͉͉͋͊ΡΑÞ˜”ΑËÄΑ樘ΑWÄ£Ú¶ÄΑËÃëæ昘ΣÞΑژ×ËÚæΡΑT+ΑλfW BμΡΑT+ΑλWfW;μΡΑT+ΑλHA +BμΑ


Ä”Α"T+αΑ¢Ë˔Α«Ä”«Ž˜ΑÚ˜ΑŽ½Žê½æ˜”Α樘ژ«ÄΠΑ+æΑŽËõ˜ÚÞΑËæ¨Α£Ë˔ÞΑÄ”ΑޘÚõ«Ž˜ÞΠ

͉͉͋Α#Ë˔ÞΑÄ”ΑޘÚõ«Ž˜ÞΑÚ˜Α恺˜ÄαΑ+æΑ«ÞΑΑޘ½˜Žæ˜”ΑÞº˜æΑË¢Α£Ë˔ÞΑÄ”ΑޘÚõ«Ž˜ÞΠΑb¨˜Αö˜«£¨æ£˜ΑË¢Α¢Ë˔Α
«ÄΑ樘ΑT+Α«ÞΑŽ½ËޘΑæËΑ͎͉АΑλb¨˜Αö˜«£¨æ£˜Α¢ËÚΑ¢Ë˔Α«ÞΑ½Ëö˜ÚΑ«ÄΑrT+μΠΑHence statement 1 is correct.

WPI:Αr¨«½˜Αژ恫½Α«Äĉæ«ËÄΑ½Ë˺ÞΑæΑ樘Α×Ú«Ž˜ΑæΑö¨«Ž¨Α樘ΑŽËÄÞêØÚΑêüÞΑ樘Α×Ú˔êŽæΡΑrT+Α«ÞΑ؁Þêژ”Α
Þ˜”ΑËÄΑ×Ú«Ž˜ÞΑæΑ樘Αö¨Ë½˜Þ½˜Α½˜õ˜½ΠΑrT+Α؁ÞêژÞΑ樘ΑŽ¨Ä£˜ÞΑ«ÄΑ樘Α×Ú«Ž˜ÞΑË¢Α£Ë˔ÞΑÞ˽”ΑÄ”Αæځ”˜”Α
«ÄΑê½ºΑüΑö¨Ë½˜Þ½˜ΑêޫĘÞޘÞΑæËΑË樘ÚΑêޫĘÞޘÞΠΑ+æΑژĉ˜ŽæÞΑ樘Α«Äĉæ«ËÄΑ«ÄΑ樘Α˜ŽËÄËÃüΑ«ÄΑ£˜Ä˜Ú½έΑ
£˜Ä˜Ú½Α½˜õ˜½ΑË¢Α«Äĉæ«ËÄΑËÚΑ×Ú«Ž˜Α½˜õ˜½Α«ÄΑ樘Α˜ŽËÄËÃüΠΑ

ÄË樘ÚΑ”«ą˜Ú˜ÄŽ˜Α˜æö˜˜ÄΑ樘ΑæöËΑ«Ä”«Ž˜ÞΑ«ÞΑ樁æΑ樘Αö¨Ë½˜Þ½˜ΑÁں˜æΑ«ÞΑËĽüΑ¢ËÚΑ£Ë˔ÞΡΑüËêΑŽÄÄËæΑ
êüΑޘÚõ«Ž˜ÞΑËÄΑΑö¨Ë½˜Þ½˜ΑÞ«ÞΠΑ[ËΑrT+Α”˘ÞΑÄËæΑ«ÄŽ½ê”˜ΑޘÚõ«Ž˜ÞΡΑö¨˜Ú˜ÞΑ樘Αژ恫½Α×Ú«Ž˜Α«Ä”˜ûΑ”˘ÞΠΑ
rT+ΡΑêĽ«º˜Α樘ΑËÄÞêØÚΑTÚ«Ž˜Α+Ĕ˜ûΑλT+μΡΑËĽüΑæځŽºÞΑ樘Α×Ú«Ž˜ÞΑË¢Α£Ë˔ÞΑ×êڎ¨Þ˜”ΑüΑŽËÄÞêØÚÞΠ
Hence statement 2 is correct.
53

͏͒͐Α£Ë˔ÞΑÚ˜Α恺˜Ä΢ΑTګÁÚüΑ£Ë˔ÞΡΑ"꘽ΑÄ”Α×Ëö˜ÚΑÄ”ΑAÄꢁŽæêÚ«Ä£Α£Ë˔ÞΡΑ樘Α½ææ˜ÚΑ˜«Ä£Α£«õ˜ÄΑ樘Α
Áû«ÃêÃΑö˜«£¨æ£˜Αλ͏͍Π͋͌АμΠΑ

r¨Ë½˜Þ½˜Α×Ú«Ž˜ÞΑÚ˜ΑŽËÄÞ«”˜Ú˜”ΠΑ Þ˜Αü˜ÚϽΑ͉͋͊͋Α Þ˜”ΑËÄΑ樘ΑÞ꣣˜Þæ«ËÄÞΑË¢Α樘ΑfÚ¶«æΑTæ˜½ΑŽËÃëæ昘ΡΑ


ÃËĘæÚüΑ×˽«ŽüΑλATμΑ«ÄΑ+Ĕ«Α«ÞΑæËΑ˜Α¢ËÚÃ꽁昔Α«ÄΑژ¢˜Ú˜ÄŽ˜ΑæËΑ樘ΑT+Αλ«ÄÞ昁”ΑË¢ΑrT+μΑÞ˜”Α«Äĉæ«ËÄΠΑ
λb¨˜ΑÞ꣣˜Þæ«ËÄΑöÞΑŽŽ˜×昔Α«ÄΑ͉͍͋͊Α«æޘ½¢ΠμΑb¨˜ΑژÞËÄΑ«ÞΑ樁æ΢Α«μΑT+ΑÞ¨ËöÞΑ樘Α«ÃׁŽæΑËÄΑ樘ΑטË×½˜ΑαΑ
rT+Α«Ä”«Žæ˜ÞΑ樘Αö¨Ë½˜Þ½˜Α×Ú«Ž˜ΡΑö¨˜Ú˜ÞΑT+ΑÞ¨ËöÞΑ樘Αژ恫½Α×Ú«Ž˜Α«Π˜ΠΑ樘Α×Ú«Ž˜ΑæΑö¨«Ž¨ΑטË×½˜ΑÁº˜Α
×êڎ¨Þ˜ÞΑ¢ÚËÃΑژ恫½ΑÁں˜æΠΑ

rT+Α”˘ÞΑÄËæΑÞ¨ËöΑ樘Α«ÃׁŽæΑË¢Α«Äĉæ«ËÄΑËÄΑ樘ΑטË×½˜ΠΑÄüΑ×˽«ŽüΑÞ¨Ë꽔ΑŽËÄÞ«”˜ÚΑ樘Α«ÃׁŽæΑËÄΑ樘Α
טË×½˜Α««μΑrT+Α”˘ÞΑÄËæΑŽŽËêÄæΑ¢ËÚΑ樘Α×Ú«Ž˜Α«ÄΑޘÚõ«Ž˜ÞΑ«««μΑ+æΑ«ÞΑÄΑ«Äæ˜Úāæ«Ëā½Α˜ÞæΑ×ځŽæ«Ž˜ΑαΑÃËÞæΑË¢Α
樘ΑŽËêÄæÚ«˜ÞΑ¨õ˜ΑÞ¨«¢æ˜”ΑæËΑT+Π Hence statement 3 is not correct.

ANALYSIS:
+æΑ«ÞΑΑÞæÄ”Ú”ΑÙê˜Þæ«ËÄΑژ½æ˜”ΑæËΑ«Äĉæ«ËÄ

Scope for elimination


[ææ˜Ã˜ÄæΑ͌Α«ÞΑ˜ÞüΑæËΑ˜½«Ã«Äæ˜ΠΑÞΑ樘ΑW +ΑêޘÞΑT+ΑÞΑ樘Α«Ä”˜ûΑÄ”ΑÄËæΑrT+ΡΑêæΑ£«ÄΡΑ«æΑژÙê«Ú˜ÞΑÞËØΑ
ÃËêÄæΑË¢ΑÞ«ŽΑºÄËö½˜”£˜ΑæËΑ˜½«Ã«Äæ˜Α樘ΑË×æ«ËÄΠ

SOURCE : Not covered in the NCERT Macroeconomics book, but can be covered from
http://arthapedia.in/index.php/Consumer_Price_Index

Year 2017 RECESSION

Q2. Which among the following steps is most likely to be taken at the time of an economic
recession?
(a) Cut in tax rates accompanied by increase in interest rate
(b) Increase in expenditure on public projects
(c) Increase in tax rates accompanied by reduction of interest rate
(d) Reduction of expenditure on public projects

ANSWERS B

EXPLANATION
W˜Ž˜ÞÞ«ËÄ΢Αb˜Ž¨Ä«Ž½ΑژŽ˜ÞÞ«ËÄΑ«ÞΑΑÞ«æêæ«ËÄΑö¨«Ž¨Α«ÞΑŽ¨ÚŽæ˜Ú«ā˜”ΑüΑĘ£æ«õ˜Α£ÚËöæ¨Αځæ˜ΑË¢Α#TΑ«ÄΑæöËΑ
Þꎎ˜ÞÞ«õ˜ΑÙêÚæ˜ÚÞΠΑ[ËØΑË¢Α樘Α«Ä”«ŽæËÚÞΑË¢ΑΑژŽ˜ÞÞ«ËÄΑ«ÄŽ½ê”˜ΑÞ½Ëö”ËöÄΑ«ÄΑ樘Α˜ŽËÄËÃüΡΑ¢½½Α«ÄΑ«Äõ˜Þæα
ØÄæÞΡΑ¢½½Α «ÄΑ樘Α Ëêæ×êæΑ Ë¢Α樘Α ˜ŽËÄËÃüΑ ˜æŽΠΑ +¢Α ÄΑ ˜ŽËÄËÃüΑ ˜ûטګ˜ÄŽ˜ÞΑ Α ژŽ˜ÞÞ«ËÄΑ Ä”Α #TΑ¢½½ÞΡΑæûΑ
ژõ˜Äê˜ÞΑ¢½½Α˜ŽêޘΑĈÚÃÞΑÄ”Α¨Ëêޘ¨Ë½”ÞΑׁüΑ½Ëö˜ÚΑæû˜ÞΑö¨˜ÄΑ樘üΑ˜ÚÄΑ½˜ÞÞΠΑ+æΑ«ÞΑ×ÚꔘÄæΑ¢ËÚΑ£Ëõ˜ÚÄα
ØÄæΑÞΑö˜½½ΑÞΑŽ˜Äæځ½ΑÄºΑæËΑ¢Ë½½ËöΑûׁÄÞ«ËāÚüΑĈގ½ΑÄ”ΑÃËĘæÚüΑ×˽«ŽüΑژÞטŽæ«õ˜½üΑæËΑÞæ«Ã꽁æ˜Α樘Α
˜ŽËÄËÃüΡΑêæΑº˜˜×«Ä£Α«ÄΑëĔΑ樘Α«Äĉæ«ËāÚüΑ×ژÞÞêژΠΑ
54

In all the given options- Cut in tax rates accompanied by increase in interest rate- Increase in interest
ځæ˜ΑژÞê½æÞΑ«ÄæËΑŽÚ˜”«æΑŽÚêϨΑ«ÄΑ樘Α˜ŽËÄËÃüΑö¨«Ž¨Α«ÞΑÄËæΑ”˜Þ«Ú½˜ΑæΑ樘Αæ«Ã˜ΑË¢ΑژŽ˜ÞÞ«ËÄΠΑIncrease in
expenditure on public projects-Α+æΑ«ÞΑËĘΑË¢Α樘ΑæË˽ΑæËΑÞæ«Ã꽁æ˜Α樘Α˜ŽËÄËÃüΑæΑ樘Αæ«Ã˜ΑË¢ΑژŽ˜ÞÞ«ËÄΡΑÞΑ
«æΑ æÚ«££˜ÚÞΑ 樘Α õ«ÚæêËêÞΑ ŽüŽ½˜Α Ë¢Α «Äõ˜ÞæØÄæΡΑ ö¨«Ž¨Α ½˜”ÞΑ æËΑ «ÄŽÚ˜Þ˜Α «ÄΑ #TΑ λ×Ú˔êŽæ«ËÄΑ Ë¢Α £Ë˔ÞΑ ϞΑ
ޘÚõ«Ž˜ÞμΑÄ”Α«ÄŽËØΑ«ÄΑ樘Α˜ŽËÄËÃüΑÄ”Α«ÄΑæêÚÄΑ«ÄŽÚ˜Þ˜Α«ÄΑ”˜ÃÄ”ΡΑÄ”Αæ¨êÞΑŽËÃ×½˜æ˜ÞΑ樘Αõ«ÚæêËêÞΑ
ŽüŽ½˜ΠΑIncrease in tax rates accompanied by reduction of interest rate- Increase in tax rate is not desir-
½˜ΑæΑ樘Αæ«Ã˜ΑË¢ΑژŽ˜ÞÞ«ËÄΑÞΑ«ÄŽËØΑ«ÞΑ¢½½«Ä£Α«ÄΑ樘Α˜ŽËÄËÃüΠΑReduction of expenditure on public
projects-Α+æΑ«ÞΑÄËæΑ”˜Þ«Ú½˜ΑæΑ樘Αæ«Ã˜ΑË¢ΑژŽ˜ÞÞ«ËÄΑÞΑ«æΑö«½½Α½˜”ΑæËΑ½˜ÞÞΑ£Ëõ˜ÚÄØÄæΑ˜ûטĔ«æêژΑ樘ژüΑ
ÄËæΑŽËÄæÚ«êæ«Ä£ΑÃꎨΑæËΑËêæ×êæΠΑHence (b) is the correct answer.

ANALYSIS:

SOURCE : Macroeconomics (NCERT Class 12)

Please Note:
AËÞæΑË¢Α樘Α؁ÞêژÞΑæËΑŽËÄæÚ˽Α«Äĉæ«ËÄΑŽËØΑêĔ˜ÚΑ"«ÞŽ½ΑT˽«ŽüΑËÚΑAËĘæÚüΑT˽«ŽüΡΑ樁æΑ«ÞΑ
ö¨üΑæËΑõË«”ΑŽËÄ¢êÞ«ËÄΑژ£Ú”«Ä£Α«æÞΑŽ½ÞÞĈ«Žæ«ËÄΡΑÞËØΑÙê˜Þæ«ËÄÞΑ¨õ˜Α˜˜ÄΑ×êæΑޘׁځ昽üΑ
under a topic called ‘INFLATION’

Year 2021 DEMAND-PULL INFLATION

V͌ΠΑr«æ¨Αژ¢˜Ú˜ÄŽ˜ΑæËΑ+Ĕ«ÄΑ˜ŽËÄËÃüΡΑ”˜ÃÄ”α×ê½½Α«Äĉæ«ËÄΑŽÄΑ˜ΑŽêޘ”έ«ÄŽÚ˜Þ˜”ΑüΑö¨«Ž¨ΑË¢Α
the following?
1. Expansionary policies
2. Fiscal stimulus
3. Inflation-indexing wages
4. Higher purchasing power
5. Rising interest rates
Select the correct answer using the code given below.
(a) 1, 2 and 4 only
(b) 3, 4 and 5 only
(c) 1, 2, 3 and 5 only
(d)1, 2, 3, 4 and 5

ANSWERS A
55

EXPLANATION

˜ÃÄ”ΑTê½½Α+Äĉæ«ËÄ΢Αb¨«ÞΑæüטΑË¢Α«Äĉæ«ËÄΑ«ÞΑŽêޘ”ΑüΑ«ÄŽÚ˜Þ˜Α«ÄΑ”˜ÃÄ”ΑÄ”Αö¨˜ÄΑ樘Α”˜ÃÄ”Α«ÄΑ樘Α
˜ŽËÄËÃüΑËêæ£ÚËöÞΑ樘ΑÞê××½üΑ«ÄΑ樘Α˜ŽËÄËÃüΠΑb¨«ÞΑº«Ä”ΑË¢Α«Äĉæ«ËÄΑŽÄΑ˜Α”˜ÞŽÚ«˜”ΑüΑύæËËΑÃꎨΑÃËĘüΑ
Ž¨Þ«Ä£ΑæËËΑ¢˜öΑ£Ë˔ÞώΠΑHĘΑË¢Α樘ΑژÞËÄÞΑ¢ËÚΑ”˜ÃÄ”Α×ê½½Α«Äĉæ«ËÄΑŽÄΑ˜Α樘Α«ÄŽÚ˜Þ˜Α«ÄΑÃËĘüΑÞê××½üΡΑ
üΑöüΑË¢Α«ÄŽÚ˜Þ˜”Αށ½ÚüΡΑ«ÄŽÚ˜Þ˜”Α£Ëõ˜ÚÄØÄæΑ˜ûטĔ«æêژΑ˜æŽΠΑ

ûׁÄÞ«ËāÚüΑ Ĉގ½Α ×˽«ŽüΑ «ÞΑ ”˜ĈʔΑ ÞΑ ÄΑ «ÄŽÚ˜Þ˜Α «ÄΑ £Ëõ˜ÚÄØÄæΑ ˜ûטĔ«æêژÞΑ Ä”έËÚΑ Α ”˜ŽÚ˜Þ˜Α «ÄΑ
æû˜ÞΠΑ ûׁÄÞ«ËāÚüΑ ÃËĘæÚüΑ ×˽«ŽüΑ «ÄŽÚ˜Þ˜ÞΑ 樘Α Þê××½üΑ Ë¢Α ÃËĘüΑ «ÄΑ ÄΑ ˜ŽËÄËÃüΑ üΑ Áº«Ä£Α ŽÚ˜”«æΑ
Þê××½üΑ˜Þ«½üΑõ«½½˜ΠΑÞËΑ˜ûׁÄÞ«ËāÚüΑ×˽«ŽüΑŽÄΑŽêޘΑ”˜ÃÄ”Α×ê½½Α«Äĉæ«ËÄΑ«ÄΑ樘Α˜ŽËÄËÃüΠΑHence
option 1 is correct.

"«ÞŽ½ΑÞæ«Ãê½êÞαΑ«æΑ«ÞΑ樘ΑׁÚæΑË¢Α˜ûׁÄÞ«ËāÚüΑ×˽«Ž«˜ÞΑË¢Α樘Α£Ëõ˜ÚÄØÄæΠΑ+æΑ«ÞΑêޘ”ΑüΑ樘Α£Ëõ˜ÚÄØÄæΑæËΑ
Þæ«Ã꽁æ˜Α樘Α˜ŽËÄËÃüΑüΑöüΑË¢ΑæûΑژæ˜ÞΡΑõÚ«ËêÞΑ«ÄŽ˜Äæ«õ˜ÞΡΑÞæ«Ã꽁æ˜Α×Ú«õæ˜ΑޘŽæËÚΑ˜ŽËÄËëŽΑŽæ«õ«α
櫘ÞΑæ¨êÞΑ¶ËΑŽÚ˜æ«ËÄΡΑ˜æŽΠΑ[ËΑ«æΑŽÄΑ½ÞËΑ½˜”ΑæËΑ˜ÃÄ”ΑTê½½Α+Äĉæ«ËÄΑ«ÄΑ樘Α˜ŽËÄËÃüΠΑHence option 2 is
correct.

+Äĉæ«ËÄα«Ä”˜û«Ä£Αö£˜ÞΡΑö£˜ÞΑ«ÄΑ樘Α˜ŽËÄËÃüΑ«ÞΑ½«Äº˜”ΑæËΑ樘Α«Äĉæ«ËÄΑö¨«Ž¨Α؁ÄÞΑö£˜ΑÃËõ˜ÞΑÞΑ
«Äĉæ«ËÄΑŽ¨Ä£˜ÞΑ«ÄΑ樘Α˜ŽËÄËÃüΠΑ[ꎨΑ«Ä”˜û«Ä£Α«ÞΑ×ÚËõ«”˜”ΑæËΑژ”ꎘΑ樘Α˜ą˜ŽæΑË¢Α«Äĉæ«ËÄΑËÄΑö£˜ÞΠΑ"ËÚΑ
˜ûÃ×½˜αΑΑöËÚº˜ÚΑ«ÞΑ£˜ææ«Ä£Α͉͉͊ΑÚÞΑÞΑΑö£˜ΑÄ”Α«Äĉæ«ËÄΑ«ÄΑ樘Α˜ŽËÄËÃüΑ«ÄŽÚ˜Þ˜ÞΑæËΑ͎АΡΑÞËΑö£˜ΑË¢Α樘Α
öËÚº˜ÚΑ«ÄŽÚ˜Þ˜ÞΑüΑ͎АΑ«Π˜ΠΑ͉͎͊ΠΑ[ËΑ˜ą˜Žæ«õ˜ΑŽ¨Ä£˜Α«ÄΑ樘Αö£˜ÞΑ«ÞΑā˜ÚËΑÄ”Α«æΑ”˘ÞΑÄËæΑ«ÄŽÚ˜Þ˜έ”˜ŽÚ˜Þ˜Α
×êڎ¨Þ«Ä£Α ×Ëö˜ÚΠΑ [ËΡΑ «æΑ ŽÄÄËæΑ ½˜”Α æËΑ ”˜ÃÄ”Α ×ê½½Α «Äĉæ«ËÄΑ «ÄΑ 樘Α ˜ŽËÄËÃüΠΑ Hence option 3 is not
correct.

+¢Α×êڎ¨Þ«Ä£Α×Ëö˜ÚΑ«ÄŽÚ˜Þ˜ÞΑ«ÄΑ樘Α˜ŽËÄËÃüΑλ¢ËÚΑ˜ûÃ×½˜Α×ژõ«ËêÞ½üΑΑ¨Ëêޘ¨Ë½”Α¨ÞΑÄΑ«ÄŽËØΑË¢Α͉͉͊Α
ÚÞΑÄ”ΑËêæΑË¢Α樁æΑ͎͉ΑÚÞΑ«ÞΑÞטĔ«Ä£ΡΑ«¢Α樘ΑÞטĔ«Ä£ΑŽ×Ž«æüΑË¢Α樫ÞΑ¨Ëêޘ¨Ë½”Α«ÄŽÚ˜Þ˜ÞΑæËΑ͏͉ΑÚÞΑ樘ÄΑ«æΑ
ŽÄΑ”˜ÃÄ”ΑÃËژΑ£Ë˔ÞέޘÚõ«Ž˜ÞΑ«ÄΑ樘Α˜ŽËÄËÃüΠμΑÄ”Α«æΑŽÄΑ½˜”ΑæËΑ”˜ÃÄ”Α×ê½½Α«Äĉæ«ËÄΑ«ÄΑ樘Α˜ŽËÄËα
ÃüΠΑHence option 4 is correct.

W«Þ«Ä£Α«Äæ˜Ú˜ÞæΑځæ˜Þ΢Α+æΑ”˜ŽÚ˜Þ˜ÞΑ樘ΑÃËĘüΑÞê××½üΑ«ÄΑ樘Α˜ŽËÄËÃüΠΑb¨«ÞΑÁüΑژÞê½æΑ«ÄΑŽÚ˜”«æΑŽÚêϨΑ«ÄΑ樘Α
˜ŽËÄËÃüΠΑ+æΑ«ÞΑŽËÞ潫˜ÚΑæËΑËÚÚËöΑÃËĘüΑ«ÄΑ樘Α˜ŽËÄËÃüΑÄ”Α«æΑ½˜”ÞΑæËΑ”˜ŽÚ˜Þ˜”ΑÃËĘüΑÞê××½üΠΑ[ËΡΑ«æΑ
ŽÄÄËæΑŽêޘΑ”˜ÃÄ”Α×ê½½Α«Äĉæ«ËÄΑ«ÄΑ樘Α˜ŽËÄËÃüΠ Hence option 5 is not correct.

ANALYSIS:

Scope for elimination


H×æ«ËÄΑ͎Α«ÞΑ˜ÞüΑæËΑ˜½«Ã«Äæ˜ΑÞΑϏÚ«Þ«Ä£Α«Äæ˜Ú˜ÞæΑځæ˜Þϐ«ÞΑêޘ”ΑÞΑΑ؁ÞêژΑæËΑ恎º½˜Α«Äĉæ«ËÄΑÄ”ΑŽÄÄËæΑ˜Α
ΑŽêޘΑË¢Α«Äĉæ«ËÄΠ

SOURCE : This concept has not been dealth with in the NCERT Macroeconomics book. One can refer to
Britannica, Investopedia or Wikipedia.
56

Year 2022 TACKLING INFLATION

Q4. In India, which one of the following is responsible for maintaining price stability by controlling
«Äĉæ«ËÄΧ
(a) Department of Consumer Affairs
(b) Expenditure Management Commission
(c) Financial Stability and Development Council
(d) Reserve Bank of India

ANSWERS D

EXPLANATION

b¨˜Α W˜Þ˜Úõ˜Α ÄºΑ Ë¢Α +Ĕ«Α λW +μΑ «ÞΑ õ˜Þ昔Α ö«æ¨Α 樘Α ژÞ×ËÄÞ««½«æüΑ Ë¢Α ŽËĔêŽæ«Ä£Α ÃËĘæÚüΑ ×˽«ŽüΠΑ b¨«ÞΑ
ژÞ×ËÄÞ««½«æüΑ«ÞΑ˜û×½«Ž«æ½üΑÁĔæ˜”ΑêĔ˜ÚΑ樘ΑW˜Þ˜Úõ˜Α ÄºΑË¢Α+Ĕ«ΑŽæΡΑ͍͊͒͌ΠΑb¨˜Α×ګÁÚüΑˍ¶˜Žæ«õ˜ΑË¢Α
ÃËĘæÚüΑ×˽«ŽüΑ«ÞΑæËΑÁ«Ä恫ÄΑ×Ú«Ž˜ΑÞ恍«½«æüΑö¨«½˜Αº˜˜×«Ä£Α«ÄΑëĔΑ樘Αˍ¶˜Žæ«õ˜ΑË¢Α£ÚËöæ¨ΠΑTÚ«Ž˜ΑÞ恍«½«æüΑ
«ÞΑΑʎ˜ÞށÚüΑ×ژŽËĔ«æ«ËÄΑæËΑÞêÞ恫ā½˜Α£ÚËöæ¨Π

b¨˜ΑATΑλAËĘæÚüΑT˽«ŽüΑËÃëæ昘μΑ”˜æ˜ÚëĘÞΑ樘Α×˽«ŽüΑ«Äæ˜Ú˜ÞæΑځæ˜ΑژÙê«Ú˜”ΑæËΑŽ¨«˜õ˜Α樘Α«Äĉæ«ËÄΑ
æÚ£˜æΠΑb¨˜ΑW˜Þ˜Úõ˜Α ÄºϐÞΑAËĘæÚüΑT˽«ŽüΑ˜×ÚæØÄæΑλATμΑÞÞ«ÞæÞΑ樘ΑATΑ«ÄΑ¢ËÚÃ꽁æ«Ä£Α樘ΑÃËĘα
æÚüΑ×˽«ŽüΠΑq«˜öÞΑË¢Αº˜üΑÞ恺˜¨Ë½”˜ÚÞΑ«ÄΑ樘Α˜ŽËÄËÃüΡΑÄ”ΑÄ½ü櫎½ΑöËÚºΑË¢Α樘ΑW˜Þ˜Úõ˜Α ÄºΑŽËÄæÚ«êæ˜Α
æËΑ樘Α×Úˎ˜ÞÞΑ¢ËÚΑÚÚ«õ«Ä£ΑæΑ樘Α”˜Ž«Þ«ËÄΑËÄΑ樘Α×˽«ŽüΑژ×ËΑځæ˜ΠΑb¨˜Α"«ÄÄŽ«½ΑAÚº˜æÞΑHטځæ«ËÄÞΑ˜×Úæα
ØÄæΑλ"AHμΑË¢ΑW +ΑËטځæ«Ëā½«Þ˜ÞΑ樘ΑÃËĘæÚüΑ×˽«ŽüΡΑÁ«Ä½üΑæ¨ÚË꣨Α”üαæËᔁüΑ½«Ù꫔«æüΑÁā£˜Ã˜ÄæΑ
Ëטځæ«ËÄÞΠΑb¨˜Α"«ÄÄŽ«½ΑAÚº˜æÞΑËÃëæ昘Αλ"AμΑؘæÞΑ”«½üΑæËΑژõ«˜öΑ樘Α½«Ù꫔«æüΑŽËĔ«æ«ËÄÞΑÞËΑÞΑæËΑ
˜ÄÞêژΑ樁æΑ樘ΑËטځæ«Ä£ΑæÚ£˜æΑË¢Α樘Αö˜«£¨æ˜”Αõ˜Ú£˜ΑŽ½½ΑÃËĘüΑځæ˜ΑλrWμΑ«ÞΑ½«£Ä˜”Αö«æ¨Α樘Αژ×ËΑ
rate.

b¨˜Ú˜ΑÚ˜Αޘõ˜Ú½Α”«Ú˜ŽæΑÄ”Α«Ä”«Ú˜ŽæΑ«ÄÞæÚêØÄæÞΑ樁æΑÚ˜Αêޘ”ΑüΑW +Α¢ËÚΑ«Ã×½˜Ã˜Äæ«Ä£ΑÃËĘæÚüΑ×˽«ŽüΑ
¢ËÚΑ×Ú«Ž˜ΑÞ恍«½«æüΑüΑº˜˜×«Ä£ΑŽ¨˜ŽºΑËÄΑ«Äĉæ«ËÄΑÞꎨΑÞΑW˜×ËΑځæ˜ΡΑW˜õ˜ÚޘΑژ×ËΡΑHטÄΑÁں˜æΑËטځæ«ËÄÞΡΑ
;«Ù꫔«æüΑ”¶êÞæÃĘæΑ¢Ž«½«æüΡΑ˜æŽΠΑ"ËÚΑ˜ûÃ×½˜ΡΑW +Α”˘ÞΑ«æΑüΑ«ÄŽÚ˜Þ«Ä£Α樘Αژ×ËΑځæ˜Α«ÄΑ樘Α˜ŽËÄËÃüΡΑ«¢Α
«Äĉæ«ËÄΑ«ÞΑ¨«£¨ΠΑ+ĎژÞ˜Α«ÄΑ«Äæ˜Ú˜ÞæΑځæ˜Α«ÞΑ˜½«˜õ˜”ΑæËΑÚ«Ä£Α”ËöÄΑ樘Α«Äĉæ«ËÄΑ½˜õ˜½Α«ÄΑ樘Α˜ŽËÄËÃüΑüΑ
ŽêÚ恫½«Ä£Α樘ΑÃËêÄæΑË¢Α˜ŽËÄËëŽΑŽæ«õ«æüΑö¨˜Ú˜ÞΑ”˜ŽÚ˜Þ«Ä£Α樘Α«Äæ˜Ú˜ÞæΑځæ˜Α«ÄŽÚ˜Þ˜ÞΑ樘Α«Äĉæ«ËÄΑ
½˜õ˜½ΑË¢Α樘Α˜ŽËÄËÃüΑüΑ«ÄŽÚ˜Þ«Ä£Α樘Α˜ŽËÄËëŽΑŽæ«õ«æüΠ Hence option (d) is the correct answer.

ANALYSIS:

Scope for elimination

Factual question. No scope for elimination.

SOURCE : https://www.rbi.org.in/scripts/FS_Overview.aspx?fn=2752
57

wwww.visionias.in
͍͑͏͉͉͑͋͋͋͋ΡΑ͉͉͒͊͒͏͏͉͏͏

phases - Prelims, Mains, and the Interview.


How to Prepare aspirants to navigate the evolving landscape of the civil services exam. With the right resources
and approach, candidates can transform this vast section into a strong advantage.

What is PT 365?

Comprehensive Coverage Infographics


Aids in understanding and retention.
UPSC relevant subjects like Polity, Economy, S&T, Techniques, methods, and processes displayed.
Features of PT365

Environment, IR, etc. Strategic use of maps to enhance learning.


Emphasis on topics likely to appear in upcoming
prelims exam. Government Schemes and Policies
In-depth coverage of key government schemes,
Clear and Concise Information policies, and initiatives.
Clear, concise presentation focusing on key issues.
New Elements
Information from credible sources.
Appendices for rapid revision. Tailored to reflect the
pattern of previous year's
QR based Smart quiz questions.
Interactive feature to assess aspirant's learning
and comprehension.
Scan this QR code
6LJQLǓFDQFHRI37 to watch video

Ease of Revision: Content categorisation is Crisp Material: Crisp points have been used in
based on subjects or topics to make it easier for the articles. It allows aspirants to easily do
aspirants to locate and revise specific areas multiple revisions in limited span of time.
Value Addition: Includes infographics, related Integrated Approach: Covers basic concepts
developments, or news, ensuring comprehensive
coverage of important information. manner, in line with previous trends of UPSC
questions. Also helps in integrating key current

PT 365 is a time and tested document. In the previous years, it has helped lakhs of

Scan this QR code


to Know More
58

FISCAL POLICY AND GOVERNMENT BUDGETING


(13 Questions)

Year 2017 GST

Q1. What is/are the most likely the advantages of implementing ‘Goods and Services Tax (GST)’?
1. It will replace multiple taxes collected by multiple authorities and will thus create a single market in India.
2. It will drastically reduce the ‘Current Account Deficit’ of India and will enable it to increase its foreign exchange
reserves.
3. It will enormously increase the growth and size of economy of India and will enable it to overtake China in the
near future.
Select the correct answer using the code given below.
(a) 1 only
(b) 2 and 3 only
(c) 1 and 3 only
(d) 1, 2 and 3

ANSWERS A

EXPLANATION

#[bΑ«ÞΑËĘΑ«Ä”«Ú˜ŽæΑæûΑ¢ËÚΑ樘Αö¨Ë½˜Αāæ«ËÄΡΑö¨«Ž¨Αö«½½ΑÁº˜Α+Ĕ«ΑËĘΑêÄ«Ĉ˜”ΑŽËÃÃËÄΑÁں˜æΠ Hence,
statement 1 is correct.

b¨˜Ú˜Α«ÞΑÄËΑ«Ä¢ËÚÁæ«ËÄΑö¨«Ž¨ΑށüÞΑ#[bΑö«½½Α”ځÞ櫎½½üΑژ”ꎘΑ樘ΑΑË¢Α+Ĕ«ΠΑB˜«æ¨˜ÚΑ«æΑ«ÞΑæÚê˜Α樁æΑ«æΑö«½½Α
˜Ä½˜Α+Ĕ«ΑæËΑ恺˜ΑËõ˜ÚΑ¨«ÄΠΑ(˜ÄŽ˜ΡΑstatements 2 and 3 are not correct.
59

Goods and Services Tax (GST)


What is it?
GST is a comprehensive, multi-stage, destination based single domestic tax
on the consumption of goods and services.

More about GST

First discussed Introduced Levied at Components of GST-


by the Kelkar through every value Central GST (CGST),
Task Force Constitution addition point. State GST (SGST),
in 2003 (101 Amendment) Union Territory GST
Act, 2016 (UTGST), and Integrated
GST (IGST)*

Concerning Law GST


Article 246A- Parliament and
Legislature of every State has State Taxes Central Taxes
power to make laws with respect
to goods and services tax VAT/Sales Tax Central Excise
imposed by the Union or by such Duty
State.
Entertainment Additional Excise
Tax Duty
Some key taxes out of Luxury Tax Additional
GST's ambit Customs duty

Tax on Lottery/ Special Additional


Excise Duty Stamp Duty and Betting/ duty of of Customs
on Alcohol Property Tax Gambling

Octroi Service Tax


Electricity Basic Custom
Purchase Tax Excise duty levied
Duty Duty
under Medicinal &
Petroleum crude, Diesel, Petrol, Toiletries
Aviation Turbine Fuel and preparations
Natural Gas
60

ANALYSIS:

Motivation
ÞΑ樘Α͉͊͊ÞæΑËÄÞæ«æêæ«Ëā½ΑØĔØÄæΑöÞΑ«ÄΑ樘ΑĘöÞΑÄ”Α#[bΑöÞΑö«”˜½üΑŽËõ˜Ú˜”Α«ÄΑ樘ΑĘöÞׁטÚÞΑ
«ÄΑ͉͋͊͐ΠΑ

Scope for elimination


b¨˜Α͌ڔΑÞææ˜Ã˜ÄæΑށüÞΑ樁æΑ«æΑö«½½Α¨˜½×Α+Ĕ«ΑËõ˜Ú恺˜Α¨«ÄΑ«ÄΑʁÚΑ¢êæêژΠΑ"«ÚÞæ½üΡΑŽËÄËÃüΑË¢Α¨«ÄΑ«ÞΑ
Þ«£Ä«ĈŽÄæ½üΑ½Ú£˜ÚΑλ«Π˜ΠΡΑ½ÃËÞæΑ͏Αæ«Ã˜ÞΑË¢Α+Ĕ«ϐÞμΑ樁ÄΑ+Ĕ«ΠΑ[˜ŽËĔ½üΡΑËĘΑÞ«Ä£½˜Αژ¢ËÚÃΑÞüÞæ˜ÃΑŽÄΑĘõ˜ÚΑ
«ÄŽÚ˜Þ˜Α樘Α£ÚËöæ¨ΑË¢ΑÄΑ˜ŽËÄËÃüΑ˜ÄËÚÃËêÞ½üΡΑ樁æΑæËËΑ«ÄΑ樘ΑʁÚΑ¢êæêÚ˜Π Therefore, 3rd statement
must be wrong. +ÄΑ樁æΑŽÞ˜ΡΑ樘ΑËĽüΑ×ËÞÞ«½˜ΑË×æ«ËÄΑ½˜¢æΑ«ÞΑË×æ«ËÄΑλμΠΑ

ÞΑ樘ΑBWbΑË˺ÞΑ×ژ”æ˜Α樘Α#[bΑژ¢ËÚÃÞΡΑ¨˜ÄŽ˜Α樘ΑŽËϘ×æÞΑöÞΑÄËæΑŽËõ˜Ú˜”Α«ÄΑÄüΑË¢Α樘ΑÞޘÄ櫁½Α
Aæ˜Ú«½Π

SOURCE : http://www.gstindia.com/about/

Year 2017 REVENUE AND EXPENDITURE


Q2. Consider the following statements:
1. Tax revenue as a percent of GDP of India has steadily increased in the last decade.
2. Fiscal deficit as a percent of GDP of India has steadily increased in the last decade.
Which of the statements given above is/are correct?
(a) 1 only
(b) 2 only
(c) Both 1 and 2
(d) Neither 1 nor 2

ANSWERS D

EXPLANATION

[˜Úõ«Ž˜Α æûΡΑ טÚÞËā½Α «ÄŽËØΑ æûΑ Ä”Α ŽËÚ×Ëځæ«ËÄΑ æûΑ ¨õ˜Α ˜˜ÄΑ ژ”ꎘ”Α «ÄΑ 樘Α ژŽ˜ÄæΑ ׁÞæΑ æËΑ ËËÞæΑΑ
££Ú˜£æ˜Α”˜ÃÄ”ΡΑÞËΑÞΑæËΑ×ÚË明æΑ樘Α˜ŽËÄËÃüΑ¢ÚËÃΑ£½Ë½ΑژŽ˜ÞÞ«ËÄΠΑ
ûŽ«Þ˜Α”êæüΑځæ˜ÞΑ¨õ˜Α˜˜ÄΑژ”ꎘ”Α«ÄΑ樘ΑژŽ˜ÄæΑׁÞæΑæËΑËËÞæΑ££Ú˜£æ˜Α”˜ÃÄ”ΡΑÞËΑÞΑæËΑ×ÚË明æΑ樘Α
˜ŽËÄËÃüΑ¢ÚËÃΑ£½Ë½ΑژŽ˜ÞÞ«ËÄΑ¨˜ÄŽ˜ΑæûΑژõ˜Äê˜ΑÞΑΑטڎ˜ÄæΑË¢Α#TΑË¢Α+Ĕ«Α¨ÞΑÞ昁”«½üΑÄËæΑ«ÄŽÚ˜Þ˜Α
in the last decade . Hence statement 1 is not correct.
"«ÞŽ½Α”˜ĈŽ«æΑÞΑΑטڎ˜ÄæΑË¢Α#TΑË¢Α+Ĕ«Α¨ÞΑÄËæΑÞ昁”«½üΑ«ÄŽÚ˜Þ˜”Α«ÄΑ樘Α½ÞæΑ”˜Ž”˜ΠΑHence statement
2 is not correct.
61

ANALYSIS:

#Ëõ˜ÚÄØÄæϐÞΑĈގ½ΑטڢËÚÁϘΑ«ÞΑ½öüÞΑŽËõ˜Ú˜”Α«ÄΑ”˜æ«½Α«ÄΑ樘ΑŽËÄËëŽΑ[êÚõ˜üÞΠΑ(˜ÄŽ˜Α£Ë«Ä£Α
through the survey of that year as thoroughly as possible is very important.

Scope for elimination


b¨˜ΑöËڔΑϏÞ昁”«½üϐΑÄ”Α樁æΑæËËΑ¢ËÚΑΑטګ˔ΑË¢Α͉͊Αü˜ÚÞΑ«ÞΑ樘Α«££˜ÞæΑŽ½ê˜ΠΑBËΑ؁ÞêژΑË¢ΑĈގ½Α×˽«ŽüΑŽÄΑ
טڢËÚÃΑ樁æΑÞ昁”«½üΑËõ˜ÚΑΑטګ˔ΑË¢Α͉͊Αü˜ÚÞΠ

SOURCE : http://www.gstindia.com/about/

Year 2018 EQUALIZATION TAX

Q3. With reference to India's decision to levy an equalization tax of 6% on online advertisement
ޘÚõ«Ž˜ÞΑËą˜Ú˜”ΑüΑÄËÄαژޫ”˜ÄæΑ˜Äæ«æ«˜ÞΡΑö¨«Ž¨ΑË¢Α樘Α¢Ë½½Ëö«Ä£ΑÞææ˜Ã˜ÄæÞΑ«ÞέÚ˜ΑŽËÚژŽæΧΑ
1. It is introduced as a part of the Income Tax Act.
2. Non-resident entities that offer advertisement services in India can claim a tax credit in their home country
under the "Double Taxation Avoidance Agreements".
Select the correct answer using teh code given below:
(a) 1 only
(b) 2 only
(c) Both 1 and 2
(d) Neither 1 nor 2

ANSWERS D

EXPLANATION

Statements 1 and 2 are not correct: Ùꁽ«Þæ«ËÄΑ;˜õüΑöÞΑ«ÄæÚ˔ꎘ”Α«ÄΑ+Ĕ«Α«ÄΑ͉͋͊͏ΡΑö«æ¨Α樘Α«Äæ˜Äæ«ËÄΑ


Ë¢Αæû«Ä£Α樘Α”«£«æ½ΑæځÄށŽæ«ËÄÞΑ«Π˜ΠΑ樘Α«ÄŽËØΑŽŽÚê«Ä£ΑæËΑ¢Ëژ«£ÄΑ˜αŽËÃØڎ˜ΑŽËÃׁī˜ÞΑ¢ÚËÃΑ+Ĕ«ΠΑ+t
is aimed at taxing business-to-business transactions.

As the levy was not introduced as part of the Income Tax Act but as a separate legislation under the
"«ÄÄŽ˜Α «½½ΡΑ£½Ë½ΑĈÚÃÞΑ樁æΑËą˜ÚΑÞꎨΑޘÚõ«Ž˜ÞΑ«ÄΑ+Ĕ«ΑŽÄÄËæΑŽ½«ÃΑΑæûΑŽÚ˜”«æΑ«ÄΑ樘«ÚΑ¨ËØΑŽËêÄæÚüΑ
êĔ˜ÚΑ樘Α”Ëꍽ˜Αæûæ«ËÄΑõË«”ÄŽ˜Α£Ú˜˜Ã˜ÄæÞΑ
62

ANALYSIS:

Motivation

b¨˜Α Ùê˜Þæ«ËÄΑ «ÞΑ ¢ŽæꁽΑ «ÄΑ āæêژΑ Ä”Α ½ÞËΑ Α ͋αÞææ˜Ã˜ÄæΑ Ùê˜Þæ«ËÄΡΑ ¨˜ÄŽ˜Α ËĘΑ «ÞΑ Þê××Ëޘ”Α æËΑ ¨õ˜Α
”˜Ùêæ˜ΑºÄËö½˜”£˜ΑæËΑ£˜æΑ樘ΑŽËÚژŽæΑÄÞö˜ÚΠ

Ùꁽ«Þæ«ËÄΑ;˜õüΑΑËÚΑ#"ΑæûΑλ#ËË£½˜ΡΑ××½˜ΡΑ"Ž˜Ë˺ΑÄ”ΑÁāËÄμΑΑ˜˜ÄΑ«ÄΑ樘ΑĘöÞΑޫϘΑ«æΑöÞΑ«ÄæÚËα
”ꎘ”Α«ÄΑ͉͋͊͏ΠΑ½ÞËΑ«æΑöÞΑژ×ËÚ昔Α樁æΑ樘Α˜ÄæژϐÞΑژõ˜Äê˜Α¢ÚËÃΑ˜Ùꁽ«Þæ«ËÄΑ½˜õüΑ«ÄΑ͉͋͊͐α͊͑ΑöÞΑËõ˜ÚΑ
WÞΑ͉͉͐ΑŽÚËژΡΑÃꎨΑ¨«£¨˜ÚΑ樁ÄΑ樘ΑWÞΑ͎͌͊ΑŽÚËژΑ«æΑ£ÚĘژ”Α¢ÚËÃΑ樘ΑæûΑ«ÄΑ樘Α×ژõ«ËêÞΑ͉͊αÃËÄæ¨Αטګ˔Π

SOURCE : From the standard newspapers


https://www.thehindu.com/business/Economy/tax-pan-
el-wants-68-levy-on-most-digital-services/article8381870.ece
¨ææ×Þ΢έέöööΠĈāĎ«½˜û×ژÞÞЎËÃέË׫īËÄݘÙꁽ«Þæ«ËÄ὘õüαÄËæαÞËα˜Ùꁽέ͎͉͍͎͋͐έ
https://economictimes.indiatimes.com/tech/internet/goo-
gle-and-other-digital-service-providers-generate-over-rs-560-cr-in-equalisation-levy/articl
eshow/63933584.cms

Year 2018 FRBM

Q4. Consider the following statements:


1. The Fiscal Responsibility and Budget Management (FRBM) Review Committee Report has recommended a debt
to GDP ratio of 60% for the general (combined) government by 2023, comprising 40% for the Central Government
and 20% for the State Governments.
2. The Central Government has domestic liabilities of 21% of GDP as compared to that of 49% of GDP of the State
Governments.
3. As per the Constitution of India, it is mandatory for a State to take the Central Government's consent for raising
any loan if the former owes any outstanding liabilities to the latter. Which of the statements given above is/are
correct?
(a) 1 only
(b) 2 and 3 only
(c) 1 and 3 only
(d) 1, 2 and 3

ANSWERS C
63

EXPLANATION

b¨˜Α "W AΑ W˜õ«˜öΑ ËÃëæ昘Α 먁«ÚטÚÞËÄ΢Α AÚΠΑ BΠ9ΠΑ [«Ä£¨μΑ ÞêÃ«æ昔Α «æÞΑ ژ×ËÚæΑ «ÄΑ 6ÄêÚüΑ ͉͋͊͐ΠΑ b¨˜Α
W˜×ËÚæΑöÞΑÁ”˜Α×ꍽ«ŽΑ«ÄΑ×Ú«½Α͉͋͊͐ΠΑb¨˜ΑËÃëæ昘ΑÞ꣣˜Þ昔ΑêÞ«Ä£Α”˜æΑÞΑ樘Α×ګÁÚüΑæÚ£˜æΑ¢ËÚΑĈގ½Α
×˽«ŽüΠΑΑ”˜æΑæËΑ#TΑځæ«ËΑË¢Α͏͉АΑÞ¨Ë꽔Α˜ΑæÚ£˜æ˜”Αö«æ¨ΑΑ͍͉АΑ½«Ã«æΑ¢ËÚΑ樘ΑŽ˜ÄæژΑÄ”Α͉͋АΑ½«Ã«æΑ¢ËÚΑ樘Α
Þææ˜ÞΠΑb¨˜ΑæÚ£˜æ˜”Α”˜æΑæËΑ#TΑځæ«ËΑÞ¨Ë꽔Α˜ΑŽ¨«˜õ˜”ΑüΑ͉͋͋͌ΠΑHence, statement 1 is correct.

b¨˜Α˜Äæځ½Α#Ëõ˜ÚÄØÄæΑ¨ÞΑ”ËØÞ櫎Α½««½«æ«˜ÞΑË¢Α͍͏Π͊АΑË¢Α#TΑλ͉͋͊͏α͊͐μΑÄ”ΑÞΑΑטÚΑŽ˜ÄæΑË¢Α#TΡΑ[ææ˜ÞϐΑ
½««½«æ«˜ÞΑ«ÄŽÚ˜Þ˜”ΑæËΑ͋͌Π͋ΑטÚΑŽ˜ÄæΑæΑ˜Ä”αAÚŽ¨Α͉͋͊͏ΠΑHence, statemenent 2 is not correct.

b¨˜Α ËÄÞæ«æêæ«ËÄΑ Ë¢Α +Ĕ«Α ˜Ã×Ëö˜ÚÞΑ [ææ˜Α #Ëõ˜ÚÄØÄæÞΑæËΑ ËÚÚËöΑ ËĽüΑ¢ÚËÃΑ ”ËØÞ櫎Α ÞËêڎ˜ÞΑ λÚ櫎½˜Α
͋͒͌λ͊μμΠΑ"êÚ樘ÚΡΑÞΑ½ËÄ£ΑÞΑΑ[ææ˜Α¨ÞΑËêæÞæÄ”«Ä£ΑËÚÚËö«Ä£ÞΑ¢ÚËÃΑ樘Α˜Äæځ½Α#Ëõ˜ÚÄØÄæΡΑ«æΑ«ÞΑژÙê«Ú˜”Α
æËΑˍ恫ÄΑ˜Äæځ½Α#Ëõ˜ÚÄØÄæϖÞΑ×Ú«ËÚΑ××ÚËõ½Α˜¢ËژΑ«ÄŽêÚÚ«Ä£Α”˜æΑλÚ櫎½˜Α͋͒͌Αλ͌μμΠΑHence, statement 3
is correct.

ANALYSIS:

Scope for elimination

½æ¨Ë꣨Α樘ΑÙê˜Þæ«ËÄΑ«ÞΑ¢ŽæꁽΑ«ÄΑāæêژΡΑÞææ˜Ã˜ÄæΑ͋Α«ÞΑ˜ÞüΑæËΑ˜½«Ã«Äæ˜ΑÞΑ樘Α˜ÄæژΑ«ÞΑ½öüÞΑæΑ樘Α
¨˜½ÃΑË¢Α+Ĕ«ϐÞΑ˜ŽËÄËÃüΡΑ«æÞΑ”ËØÞ櫎Α½««½«æ«˜ÞΑ¨õ˜ΑæËΑ˜Α£Ú˜æ˜ÚΠΑ

Motivation

b¨˜Α#Ëõ˜ÚÄØÄæΑ¨”Α”˜Ž«”˜”ΑæËΑÄËΑ½ËÄ£˜ÚΑæÚ£˜æΑژõ˜Äê˜Α”˜ĈŽ«æΑö¨«Ž¨ΑÚË꣨æΑ¨˜ΑB9Α[«Ä£¨ΑTÄ˜½ΑژŽËÃα
ØĔæ«ËÄÞΑ«ÄΑæËΑ¢ËŽêÞΠ

SOURCE : Almost all the standard newspapers covered the NK Singh report submission in 2017.
https://www.thehindu.com/business/Economy/the-hindu-ex-
plains-disciplining-government-debt/article18195556.ece
¨ææ×Þ΢έέ”˜Π£ËõΠ«ÄέÞ«æ˜Þέ”˜¢ê½æέĈ½˜Þέ[ææêÞА͉͋T×˜ÚА͉͋Ĉā½А͉͋͋͑Π͌Π͊͑Πה¢Αλb¨˜ΑÁ«ÄΑ
report)
64

Year 2018 GST

Q5. Consider the following items:


1. Cereals grains hulled
2. Chicken eggs cooked
3. Fish processed and canned
4. Newspapers containing advertising material
Which of the above items is/are exempted under GST (Goods and Services Tax)?
(a) 1 only
(b) 2 and 3 only
(c) 1, 2 and 4 only
(d) 1, 2, 3 and 4

ANSWERS C

EXPLANATION

The Goods and Services Tax (GST) in India was implemented on July 1, 2017.Αr¨«½˜ΑŽ˜Ú˜½ÞΡΑ˜££ÞΡΑ˜æŽΑÚ˜Α
˜û˜Ã×昔Α¢ÚËÃΑ#[bΡΑ樘ΑÙê˜Þæ«ËÄΑØÄæ«ËÄÞ 'cooked' and 'processed and canned.'Α(˜ÄŽ˜Α”ê˜ΑæËΑææ˜Äæ«ËÄΑ
ʘ”ÞΑ æËΑ ˜Α ׁ«”ΠΑ ÞΑ ÄΑ «æ˜ÃΑ «ÞΑ ŽÄʔΡΑ «æΑ «ÞΑ Þêژ½üΑ õ«½½˜Α ÞΑ Α ×Ú˔êŽæΑ «ÄΑ Áں˜æΑ Ä” thus surely
attracts GST. Thus using the elimination technique, we can get rid of option B and D. At the time of
樘Α˜ûÃΡΑ樘ژΑöÞΑΑ½ËæΑË¢ΑŽËÄ¢êÞ«ËÄΑ˜æö˜˜ÄΑΑÄ”ΑΠΑ êæΑ«ÄΑ«æÞΑËƎ«½Αº˜üΑfT[ΑŽËÄĈÚؔΑϏϐΑÞΑ
the answer.

½æ¨Ë꣨Αö«æ¨Αژ¢˜Ú˜ÄŽ˜ΑæËΑĘöÞׁטÚÞΑŽËÄ恫īģΑ”õ˜Úæ«Þ«Ã˜ÄæÞΡΑ£Ëõ˜ÚÄØÄæΑ¨”ΑژŽ˜Äæ½üΑ×ꍽ«Þ¨˜”ΑΑ
Ž½Ú«ĈŽæ«ËÄΑ樁æΑ樘ޘΑö«½½Α˜Αæû˜”ΑêĔ˜ÚΑ#[bΡΑêæΑ樘ΑÄÞö˜ÚΑ£«õ˜ÄΑöÞΑΠΑb¨˜Α£Ëõ˜ÚÄØÄæΑ¨”ΑŽ½Ú«Ĉ˜”Α
樁æΑ樘ΑĘöÞׁטÚÞΑöË꽔Α¨õ˜ΑæËΑׁüΑ͎ΑטÚΑŽ˜ÄæΑ#[bΑËÄΑ樘Αژõ˜Äê˜Α˜ÚʔΑ¢ÚËÃΑÞׁŽ˜Αޘ½½«Ä£ΑêæΑŽÄΑ
õ«½ΑË¢Α«Ä×êæΑæûΑŽÚ˜”«æΑ¢ËÚΑ樘ΑæûΑׁ«”ΑüΑ樘Α”õ˜Úæ«Þ˜Ã˜ÄæΑ£˜ÄŽüΑËÄΑŽËÃëÞÞ«ËÄΑژŽ˜«õ˜”Π

ANALYSIS:
This question was askedΑ˜ŽêޘΑ樘Α#[bΑöÞΑ×ژõ½˜Äæ½üΑ«ÄΑ樘ΑĘöÞΑËö«Ä£ΑæËΑ«æÞΑŽËëģΑ«ÄæËΑ¢Ëڎ˜Α
¢ÚËÃΑ6ê½üΑ͊ΡΑ͉͋͊͐Π

Scope for elimination


BËΑގËטΠΑ+æΑöÞΑ”«ĆŽê½æΑæËΑØÃËګޘΑ樘Α½«ÞæΑË¢Α#[bα˜û˜Ã×昔Α«æ˜ÃΑ½ÞËΑ樘Α˜½«Ã«Äæ«ËÄΑ明¨Ä«Ùê˜ΑŽË꽔Α
˜½«Ã«Äæ˜ΑËĽüΑæöËΑË×æ«ËÄÞΠ

SOURCE : Although one cannot memorize the list of items, just for the reference, the following link can be
used
http://www.msmedinewdelhi.gov.in-
/PDF2017-18/GST/List%20of%20Exempted%20Goods%20under%20GST%20with%20HSN%20
Code.pdf
65

Year 2018 EXTERNAL DEBT

Q6. Consider the following statements:


1. Most of India’s external debt is owed by governmental entities.
2. All of India’s external debt is denominated in US dollars.
Which of the statements given above is/are correct?
(a) 1 only
(b) 2 only
(c) Both 1 and 2
(d) Neither 1 nor 2

ANSWERS D

EXPLANATION

ÞΑטÚΑ樘Α#Ëõ˜ÚÄØÄæΑژ×ËÚæΑËÄΑûæ˜Úā½Α”˜æΑüΑ˜Ž˜Ã˜ÚΑ͉͋͊͑αËÃØڎ«½ΑËÚÚËö«Ä£ÞΑÚ˜Α樘Α½Ú£˜ÞæΑ
ŽËÃ×ËĘÄæΑË¢Α˜ûæ˜Úā½Α”˜æΑö«æ¨ΑΑÞ¨Ú˜ΑË¢Α͌͐Π͊Αטڎ˜ÄæΡΑ¢Ë½½Ëö˜”ΑüΑBW+Α”˜×ËÞ«æÞΑλ͋͌Π͒Αטڎ˜ÄæμΑÄ”ΑÞ¨ËÚæΑ
æ˜ÚÃΑæځ”˜ΑŽÚ˜”«æΑλ͊͒Π͒Αטڎ˜ÄæμΠ Hence statement 1 is not correct.

f[Α”˽½ÚΑ”˜ÄËëā昔Α”˜æΑŽËÄæ«ÄꘔΑæËΑ˜Α樘Α½Ú£˜ÞæΑŽËÃ×ËĘÄæΑË¢Α+Ĕ«ϐÞΑ˜ûæ˜Úā½Α”˜æΑö«æ¨ΑΑÞ¨Ú˜Α
Ë¢Α͍͎Π͒Αטڎ˜ÄæΑæΑ˜Ä”Α˜Ž˜Ã˜ÚΑ͉͋͊͑ΡΑ¢Ë½½Ëö˜”ΑüΑ樘Α+Ĕ«ÄΑÚêט˜Αλ͍͋Π͑Αטڎ˜ÄæμΡΑ[WΑλ͎Π͊Αטڎ˜ÄæμΡΑü˜ÄΑ
λ͍Π͒Αטڎ˜ÄæμΑÄ”Α˜êÚËλ͌Π͊Αטڎ˜ÄæμΠΑHence, Statement 2 is not correct.

ANALYSIS:

½öüÞΑ«ÄΑ樘ΑĘöÞΡΑÞËØæ«Ã˜ÞΑŽËõ˜Ú˜”Α«ÄΑ£Ú˜æΑ”˜æ«½Α«ÄΑ樘ΑŽËÄËëŽΑ[êÚõ˜üÞΠ

Scope for elimination


"ÚËÃΑËêÚΑ½«Ã«æ˜”ΑêĔ˜ÚÞæÄ”«Ä£Αö˜ΑŽÄΑ«Ä¢˜ÚΑ樁æΑ½½ΑË¢Α+Ĕ«ϐÞΑ”˜æΑö«½½ΑÄËæΑ˜Α”˜ÄËëā昔Α«ÄΑËĘΑŽêÚژÄα
ŽüΠΑ"ÚËÃΑ樁æΑö˜ΑŽÄΑ˜½«Ã«Äæ˜ΑÞææ˜Ã˜ÄæΑ͋Π

SOURCE : ¨ææ×Þ΢έέ”˜Π£ËõΠ«ÄέÞ«æ˜Þέ”˜¢ê½æέĈ½˜Þέ+Ĕ«А͋͐ÞА͉͋ûæ˜Úα
nal%20Debt%20as%20at%20the%20end%20December%202018.pdf
66

Economic Survey 2022-23 Statistical Appendix


67

Year 2020 NON-FINANCIAL DEBT

V͐ΠΑ+ÄΑ樘ΑŽËÄæ˜ûæΑË¢Α樘Α+Ĕ«ÄΑ˜ŽËÄËÃüΡΑÄËÄαĈāĎ«½Α”˜æΑ«ÄŽ½ê”˜ÞΑö¨«Ž¨ΑË¢Α樘Α¢Ë½½Ëö«Ä£ΧΑ
1. Housing loans owed by households
2. Amounts outstanding on credit cards
3. Treasury bills
Select the correct answer using the code given below:
(a) 1 only
(b) 1 and 2 only
(c) 3 only
(d) 1, 2 and 3

ANSWERS D

EXPLANATION
BËÄαĈāĎ«½Α ”˜æΑ ŽËÄÞ«ÞæÞΑ Ë¢Α ŽÚ˜”«æΑ «ÄÞæÚêØÄæÞΑ «ÞÞꘔΑ üΑ £Ëõ˜ÚÄØÄ恽Α ˜Äæ«æ«˜ÞΡΑ ¨Ëêޘ¨Ë½”ÞΑ Ä”ΑΑ
êޫĘÞޘÞΑ樁æΑÚ˜ΑÄËæΑ«ÄŽ½ê”˜”Α«ÄΑ樘ΑĈāĎ«½ΑޘŽæËÚΠΑλb¨˜ΑĈāĎ«½ΑޘŽæËÚΑŽËÃ×ګޘÞΑŽËÃØڎ«½ΑÄºÞΡΑ
«ÄÞêځϘΑ ŽËÃׁī˜ÞΡΑ ÄËčÄº«Ä£ΑĈāĎ«½Α ŽËÃׁī˜ÞΡΑ ŽËËטځæ«õ˜ÞΡΑ טÄÞ«ËÄΑ¢êĔÞΡΑ ÃêæꁽΑ¢êĔÞΑ Ä”Α
Ë樘ÚΑÞÁ½½˜ÚΑĈāĎ«½Α˜Äæ«æ«˜ÞμΠΑBËÄαĈāĎ«½Α”˜æΑ«ÄŽ½ê”˜ÞΑ¨Ëêޘ¨Ë½”ΑËÚΑŽËÃØڎ«½Α½ËÄÞΡΑbژÞêÚüΑ«½½ÞΑ
Ä”Α ŽÚ˜”«æΑ ŽÚ”Α ½ÄŽ˜ÞΠΑ b¨˜üΑ Þ¨Ú˜Α ÃËÞæΑ Ë¢Α 樘Α ށØΑ Ž¨ÚŽæ˜Ú«Þ櫎ÞΑ ö«æ¨Α ĈāĎ«½Α ”˜æΡΑ ˜ûŽ˜×æΑ 樘Α
«ÞÞê˜ÚÞΑÚ˜ΑÄËÄαĈāĎ«½ΠΑΑHence option (d) is the correct answer

ANALYSIS:

Source/How to Cover? – ¨ææ×Þ΢έέöööΠށ׽«Ä£ΠŽËÃέ͏͏͎͌͐͌͋έÄËÄĈāĎ«½α”˜æΑ


68

Year 2021 EFFECTS OF DEFICIT

V͑ΠΑr¨«Ž¨ΑËĘΑË¢Α樘Α¢Ë½½Ëö«Ä£Α«ÞΑ½«º˜½üΑæËΑ˜Α樘ΑÃËÞæΑ«Äĉæ«ËāÚüΑ«ÄΑ«æÞΑ˜ą˜ŽæÞΧΑ
(a) Repayment of public debt
(b) Borrowing from the public to finance a budget deficit
(c) Borrowing from the banks to finance a budget deficit
(d) Creation of new money to finance a budget deficit

ANSWERS D

EXPLANATION

ËÚÚËö«Ä£Α¢ÚËÃΑ ×ꍽ«ŽΑ Ä”Α ÄºÞΑö«½½Α ½˜”ΑæËΑ Α ”˜ŽÚ˜Þ˜Α «ÄΑ樘Α ÃËĘüΑ Þê××½üΑ «ÄΑ Áں˜æΑ ÞΑ «ÄΑ Ëæ¨Α樘Α
Ë×æ«ËÄÞΑ ÃËĘüΑ «ÄΑ ¨Ä”Α «ÞΑ ژ”ꎘ”Α¢ËÚΑ ×ꍽ«ŽΑ Ä”Α ÃËĘüΑæËΑ ½˜Ä”Α «ÞΑ ژ”ꎘ”Α¢ËÚΑ ÄºÞΠΑ ژæ«ËÄΑ Ë¢Α ĘöΑ
ÃËĘüΑæËΑĈāϘΑΑê”£˜æΑ”˜ĈŽ«æΑö«½½Α¨õ˜ΑÃËژΑ«Äĉæ«ËāÚüΑ˜ą˜ŽæΑ樁ÄΑژׁüØÄæΑË¢Α”˜æΡΑÞΑ«æΑö«½½Α½˜”Α
æËΑÄΑ«ÄŽÚ˜Þ˜Α«ÄΑæË恽ΑÃËĘüΑÞê××½üΑ«ÄΑ樘ΑÁں˜æΠλÞΑĘöΑÃËĘüΑ«ÞΑ˜«Ä£ΑŽÚ˜æ˜”μΠΑHence option (d) is
the correct answer.

êÚ«Ä£Α½ÞæΑĈގ½Αü˜ÚΡΑË×æ«ËÄΑË¢ΑÃËĘæ«āæ«ËÄΑË¢Α”˜ĈŽ«æΑöÞΑ˜û×½Ëژ”ΑêæΑ«”˜ΑöÞΑ”ÚË×ט”Α”ê˜ΑæËΑ½«º˜½üΑ
«Äĉæ«ËāÚüΑ×ژÞÞêژΠ

ANALYSIS:

The question featured in the CSE that year asΑÃËĘæ«Þæ«ËÄΑË¢Α”˜ĈŽ«æΑ¨ÞΑ˜˜ÄΑژŽêÚÚ«Ä£Α«ÄΑĘöÞΑ«ÄΑ樘Α


ŽËÄæ˜ûæΑË¢ΑËõ«”͊͒Π

Source: Standard newspapers

Year 2021 DEBTS AND SAVINGS

Q9. With reference to the Indian economy, consider the following statements :
1. A share of the household financial savings goes towards government borrowings.
2. Dated securities issued at market-related rates in auctions form a large component of internal debt.
Which of the above statements is/are correct ?
(a) 1 only
(b) 2 only
(c) Both l and 2
(d) Neither 1 nor 2

ANSWERS C
69

EXPLANATION

(Ëêޘ¨Ë½”ΑĈāĎ«½Αށõ«Ä£ÞΑژ¢˜ÚΑæËΑŽêÚژĎüΡΑÄºΑ”˜×ËÞ«æÞΡΑ”˜æΑޘŽêګ櫘ÞΡΑÃêæꁽΑ¢êĔÞΡΑטÄÞ«ËÄΑ¢êĔÞΡΑ
«ÄÞêځϘΡΑÄ”Α«Äõ˜ÞæØÄæÞΑ«ÄΑÞÁ½½Αށõ«Ä£ÞΑގ¨˜Ã˜ÞΑüΑ¨Ëêޘ¨Ë½”ÞΠΑb¨˜ΑĘæΑ¨Ëêޘ¨Ë½”ΑĈāĎ«½Αށõ«Ä£ÞΑ
öÞΑ͊͊Π͎Αטڎ˜ÄæΑË¢Α#B+λΑ£ÚËÞÞΑāæ«Ëā½Α”«Þ×Ëށ½Α«ÄŽËØμΑ«ÄΑ͉͉͋͋α͋͊ΠΑΑׁÚæΑË¢Α樫ÞΑĈāĎ«½Αށõ«Ä£Α£Ë˜ÞΑ
æËöÚ”Α£Ëõ˜ÚÄØÄæΑËÚÚËö«Ä£ΠÞΑ£Ëõ˜ÚÄØÄæΑËÚÚËöÞΑæ¨ÚË꣨Α樘Α«ÞÞê˜ΑË¢Α£Ëõ˜ÚÄØÄæΑޘŽêګ櫘ÞΑŽ½½˜”Α
#αޘŽÞΑÄ”ΑbژÞêÚüΑ «½½ÞΠ+æΑËÚÚËöÞΑ¢ÚËÃΑ樘ΑÁں˜æΡΑÞÁ½½Αށõ«Ä£ÞΑ¢êĔÞΡΑÞææ˜Α×ÚËõ«”˜ÄæΑ¢êĔÞΡΑ˜ûæ˜Úā½Α
ÞÞ«ÞæÄŽ˜ΑÄ”ΑÞ¨ËÚæαæ˜ÚÃΑËÚÚËö«Ä£ÞΠΑÄüΑ”õ˜ÚޘΑÃËõ˜Ã˜ÄæΑ«ÄΑ樘Α¨Ëêޘ¨Ë½”Αށõ«Ä£ÞΑö«½½Α¨õ˜ΑΑÞ«£Ä«Ĉα
ŽÄæΑ˜Ú«Ä£ΑËÄΑÄºÞΡΑ«ÄÞêځϘΑŽËÃׁī˜ÞΑÄ”ΑÃêæꁽέ×ÚËõ«”˜ÄæΑ¢êĔÞΡΑö¨ËΡΑ«ÄΑæêÚÄΡΑÚ˜Αº˜üΑ«Äõ˜ÞæËÚÞΑ
«ÄΑ£Ëõ˜ÚÄØÄæΑޘŽêګ櫘ÞΠΑHence statement 1 is correct.

b¨˜Α˜Äæځ½Α#Ëõ˜ÚÄØÄæΑ˜æΑ«ÄŽ½ê”˜ÞΑ½½Α½««½«æ«˜ÞΑË¢Α˜Äæځ½Α#Ëõ˜ÚÄØÄæΑŽËÄæځŽæ˜”Α£«ÄÞæΑ樘ΑËÄÞ˽«α
”æ˜”Α "êĔΑ Ë¢Α +Ĕ«Α λ”˜ĈʔΑ ÞΑ Tꍽ«ŽΑ ˜æμΠΑ Tꍽ«ŽΑ ”˜æΑ «ÞΑ¢êÚ樘ÚΑ Ž½ÞÞ«Ĉ˜”Α «ÄæËΑ «Äæ˜Úā½Α Ä”Α ˜ûæ˜Úā½Α
”˜æΠΑ+Äæ˜Úā½Α”˜æΑŽËÄÞ«ÞæÞΑË¢ΑÁں˜æ½˜Α”˜æΑÄ”ΑÄËÄαÁں˜æ½˜Α”˜æΠΑAÚº˜æ½˜Α”˜æΑŽËÃ×ګޘÞΑË¢Α
#Ëõ˜ÚÄØÄæΑ”æ˜”ΑޘŽêګ櫘ÞΑÄ”ΑbژÞêÚüΑ «½½ÞΡΑ«ÞÞꘔΑæ¨ÚË꣨ΑêŽæ«ËÄÞΠΑBËÄαÁں˜æ½˜Α”˜æΑŽËÃ×ګޘÞΑ
Ë¢Α«Äæ˜Úؔ«æ˜

bژÞêÚüΑ «½½ÞΑλ͍͊Α”üÞΑ+b ÞμΑ«ÞÞꘔΑæËΑ[ææ˜Α#Ëõ˜ÚÄØÄæÞέfbÞΑÞΑö˜½½ΑÞΑޘ½˜ŽæΑ˜Äæځ½Α ÄºÞΡΑÞטŽ«½ΑޘŽêα


ګ櫘ÞΑ«ÞÞꘔΑ£«ÄÞæΑÞÁ½½Αށõ«Ä£ÞΡΑÞטŽ«½ΑޘŽêګ櫘ÞΑ«ÞÞꘔΑæËΑ×ꍽ«ŽΑޘŽæËÚΑÄºÞέw+AΑ ÄºΡΑޘŽêګ櫘ÞΑ
«ÞÞꘔΑæËΑ«Äæ˜Úāæ«Ëā½ΑĈāĎ«½Α«ÄÞæ«æêæ«ËÄÞΡΑÄ”ΑŽËÃטÄށæ«ËÄΑÄ”ΑË樘ÚΑËĔÞΠ

½½Α Áں˜æ½˜Α ޘŽêګ櫘ÞΑ «Π˜Α æ˜”Α ޘŽêګ櫘ÞΑ Ä”ΑbژÞêÚüΑ «½½ÞΑ Ú˜Α «ÞÞꘔΑ æ¨ÚË꣨Α êŽæ«ËÄÞΑ ÞΑ טÚΑ 樘Α
ގ¨˜”꽘Α ÄËæ«Ĉ˜”Α æ¨ÚË꣨Α ¨½¢ü˜Ú½üέÙêÚæ˜Ú½üΑ êŽæ«ËÄΑ Ž½˜Ä”ÚÞΠΑ ÞΑ æΑ ˜Ä”αAÚŽ¨Α ͉͋͋͊ΡΑ ËêæÞæÄ”«Ä£Α
ÃËêÄæÞΑêĔ˜ÚΑ”æ˜”ΑޘŽêګ櫘ÞΑÞæË˔ΑæΑ͐͊Π͐Α½º¨ΑŽÚËژΑλ͌͏Π͌ΑטÚΑŽ˜ÄæΑË¢Α#TμΑÄ”ΑŽŽËêÄ昔Α¢ËÚΑ͏͑Π͊ΑטÚΑ
Ž˜ÄæΑË¢Α樘ΑæË恽ΑTꍽ«ŽΑ˜æΠΑHence statement 2 is correct.

ANALYSIS:

+Äæ˜Úā½Α”˜æΑϞΑ(Ëêޘ¨Ë½”ΑĈāĎ«½Αށõ«Ä£ÞΑÚ˜Α«Ã×ËÚæÄæΑæË׫ŽÞΑÄ”ΑË¢æ˜ÄΑÚ˜Α«ÄΑ樘ΑĘöÞΠ

Scope for elimination


Not much scope is there, especially because it is a 2-statement question

SOURCE : ¨ææ×Þ΢έέ”˜Π£ËõΠ«ÄέÞ«æ˜Þέ”˜¢ê½æέĈ½˜Þέ"«Ä½А͉͋[ææêÞАׁ͉͋טÚА͉͉͉͋͋͋α͋͊Πה¢
Page 8
¨ææ×Þ΢έέöööΠĈāĎ«½˜û×ژÞÞЎËÃέË׫īËÄέ¨ËöαŽÚ«æ«α
cal-are-household-savings-for-government-borrowing/1542170/
https://rbidocs.rbi.org.in/rdocs/AnnualReport/PDFs/0RBI-
AR2021226AD1119FF6674A13865C988DF70B4E1A.PDF
70

Year 2021 CAPITAL EXPENDITURE

Q10. With reference to the expenditure made by an organisation or a company, which of the
following statements is/are correct?
1. Acquiring new technology is capital expenditure.
2. Debt financing is considered capital expenditure, while equity financing is considered revenue expenditure.
Select the correct answer using the code given below :
(a) 1 only
(b) 2 only
(c) Both 1 and 2
(d) Neither 1 nor 2

ANSWERS A

EXPLANATION

×«æ½Α˜ûטĔ«æêژÞΑλ×ûμΑÚ˜Α¢êĔÞΑêޘ”ΑüΑΑŽËÃׁÄüΑæËΑŽÙê«Ú˜ΡΑêףځ”˜ΡΑÄ”ΑÁ«Ä恫ÄΑרüÞ«Ž½Α
ÞޘæÞΑÞꎨΑÞΑ×ÚËטÚæüΡΑ×½ÄæÞΡΑê«½”«Ä£ÞΡΑ明¨Ä˽ˣüΡΑËÚΑ˜Ùê«×ØÄæΠΑ×ûΑ«ÞΑË¢æ˜ÄΑêޘ”ΑæËΑêĔ˜Ú恺˜Α
ĘöΑ×Ú˶˜ŽæÞΑËÚΑ«Äõ˜ÞæØÄæÞΑüΑΑŽËÃׁÄüΠΑËÞæÞΑæËΑêףځ”˜ΑËÚΑ×êڎ¨Þ˜ΑÞË¢æöÚ˜ΡΑ«Äõ˜Þæ«Ä£Α«ÄΑĘöΑ明¨α
Ä˽ˣüΑÄ”ΑŽËÃ×êæ˜ÚΑ˜Ùê«×ØÄæΡΑÚ˜ΑŽËÄÞ«”˜Ú˜”ΑׁÚæΑË¢Α×«æ½Α˜ûטĔ«æêژΠΑÞΑ樘üΑÚ˜ΑË¢æ˜ÄΑ˜Ã×½Ëü˜”Α
æËΑ«Ã×ÚËõ˜ΑËטځæ«Ëā½Α˜ĆŽ«˜ÄŽüΡΑ«ÄŽÚ˜Þ˜Αژõ˜Äê˜Α«ÄΑ樘Α½ËÄ£Αæ˜ÚÃΡΑËÚΑÁº˜Α«Ã×ÚËõ˜Ã˜ÄæÞΑæËΑ樘Α˜û«Þæ«Ä£Α
ÞޘæÞΑË¢ΑΑŽËÃׁÄüΠΑHence statement 1 is correct.

r¨˜ÄΑΑŽËÃׁÄüΑËÚÚËöÞΑÃËĘüΑæËΑ˜Αׁ«”ΑŽºΑæΑΑ¢êæêژΑ”æ˜Αö«æ¨Α«Äæ˜Ú˜ÞæΑ«æΑ«ÞΑºÄËöÄΑÞΑ”˜æΑĈāĎα
«Ä£ΠΑ+æΑˎŽêÚÞΑö¨˜ÄΑΑĈÚÃΑޘ½½ÞΑĈû˜”Α«ÄŽËØΑ×Ú˔êŽæÞΡΑÞꎨΑÞΑËĔÞΡΑ«½½ÞΡΑËÚΑÄËæ˜ÞΠΑ+æΑŽË꽔Α˜Α«ÄΑ樘Α¢ËÚÃΑ
Ë¢ΑΑޘŽêژ”ΑÞΑö˜½½ΑÞΑÄΑêÄޘŽêژ”Α½ËÄΠΑ˜æΑĈāĎ«Ä£Α«ÞΑ樘ΑË××ËÞ«æ˜ΑË¢Α˜Ùê«æüΑĈāĎ«Ä£ΡΑö¨«Ž¨Α˜Ä恫½ÞΑ
«ÞÞê«Ä£ΑÞæˎºΑæËΑځ«Þ˜ΑÃËĘüΠΑ"ËÚΑ˜ûÃ×½˜ΡΑژ½«ÄŽ˜ΑŽÄΑŽ¨ËËޘΑ”˜æΑĈāĎ«Ä£ΡΑö¨«Ž¨Α˜Ä恫½ÞΑޘ½½«Ä£ΑĈû˜”Α
«ÄŽËØΑ ×Ú˔êŽæÞΡΑ ÞꎨΑ ÞΑ ËĔÞΡΑ «½½ÞΡΑ ËÚΑ ÄËæ˜ÞΡΑæËΑ «Äõ˜ÞæËÚÞΑæËΑ ˍ恫ÄΑ樘Α Ž×«æ½Α ʘ”˜”ΑæËΑ £ÚËöΑ Ä”Α
˜ûׁĔΑ«æÞΑËטځæ«ËÄÞΠΑ Ëæ¨Α”˜æΑĈāĎ«Ä£ΑÄ”Α˜Ùê«æüΑĈāĎ«Ä£ΑÚ˜ΑŽËÄÞ«”˜Ú˜”ΑÞΑׁÚæΑË¢ΑŽ×«æ½ΑژŽ˜«×æÞΑ
¢ËÚΑ樘ΑŽËÃׁÄüΡΑÞΑŽ×«æ½ΑژŽ˜«×æÞΑÚ˜ΑژŽ˜«×æÞΑ樁æΑŽÚ˜æ˜Α½««½«æ«˜ÞΑËÚΑژ”ꎘΑĈāĎ«½ΑÞޘæÞΠΑ"êĔÞΑ
¢ÚËÃΑ樘ޘΑöË꽔Α˜Αêޘ”ΑüΑŽËÃׁÄüΑ¢ËÚΑŽ×«æ½Α˜ûטĔ«æêژΑÞꎨΑÞΑæËΑ£ÚËöΑËÚΑ˜ûׁĔΑ«æÞΑËטځæ«ËÄÞΠΑ
Hence statement 2 is not correct.

ANALYSIS:

Please Note:
Capital & revenue expenditure fundamental topic. Although we study that from the
£Ëõ˜ÚÄØÄæΑ ê”£˜æ«Ä£Α ×Ë«ÄæΑ Ë¢Αõ«˜öΡΑ樫ÞΑ Ùê˜Þæ«ËÄΑöÞΑ¢ÚËÃΑ樘Α טÚÞטŽæ«õ˜Α Ë¢Α ΑĈÚÃΠΑ
This question is normally covered in the Accountancy.
71

SOURCE : ¨ææ×Þ΢έέöööΠ«Äõ˜ÞæËט”«ΠŽËÃέæ˜ÚÃÞέ”έ”˜æĈāĎ«Ä£ΠÞ×ά΢Ѕ΢æ˜ûα
æϽ˜æА͉͋ĈāĎ«Ä£А͉͋ˎŽêÚÞА͉͋ö¨˜ÄА͉͋А͉͋ŽËÃׁÄüА͉͋ځ«Þ˜ÞА͉͋ÃËĘüА͉͋üА͉͋ޘ
lling,bonds%2C%20bills%2C%20or%20notes.
https://www.business-standard.com/about/what-is-capital-re-
Ž˜«×æÞά΢Ѕ΢æ˜ûæϽ×«æ½А͉͋ژŽ˜«×æÞА͉͋Ú˜А͉͋ژŽ˜«×æÞА͉͋樁æюÚꎫ½Аׁ͉͋ÚæА͉͋Ë¢А͉͋Ž×«
tal%20receipts.
https://www.investopedia.com/ask/an-
swers/112814/what-are-some-examples-main-types-capital-expenditures-capex.asp

Year 2022 INDIRECT TRANSFER


V͊͊ΠΑr¨«Ž¨ΑËĘΑË¢Α樘Α¢Ë½½Ëö«Ä£ΑÞ«æêæ«ËÄÞΑ˜ÞæΑژĉ˜ŽæÞΑϕ+Ĕ«Ú˜ŽæΑbځÄÞ¢˜ÚÞϕΑË¢æ˜ÄΑ恽º˜”ΑËêæΑ«ÄΑ
media recently with reference to India ?
(a) An Indian company investing in a foreign enterprise and paying taxes to the foreign country on the profits
arising out of its investment
(b) A foreign company investing in India and paying taxes to the country of its base on the profits arising out of its
investment
(c) An Indian company purchases tangible assets in a foreign country and sells such assets after their value increas-
es and transfers the proceeds to India
(d) A foreign company transfers shares and such shares derive their substantial value from assets located in India

ANSWERS D

EXPLANATION

+Ĕ«Ú˜ŽæΑæځÄÞ¢˜ÚÞΑژ¢˜ÚΑæËΑÞ«æêæ«ËÄÞΑö¨˜Ú˜Αö¨˜ÄΑ¢Ëژ«£ÄΑ˜Äæ«æ«˜ÞΑËöÄΑÞ¨Ú˜ÞΑËÚΑÞޘæÞΑ«ÄΑ+Ĕ«ΡΑ樘ΑÞ¨Ú˜ÞΑ
Ë¢ΑÞꎨΑ¢Ëژ«£ÄΑ˜Äæ«æ«˜ÞΑÚ˜ΑæځÄÞ¢˜Úژ”Α«ÄÞ昁”ΑË¢ΑΑ”«Ú˜ŽæΑæځÄÞ¢˜ÚΑË¢Α樘ΑêĔ˜Ú½ü«Ä£ΑÞޘæÞΑ«ÄΑ+Ĕ«ΠΑHence
option (d) is the corerct answer.

b¨˜ΑËÚ«£«ÄΑË¢ΑژæÚËÞטŽæ«õ˜Αæûæ«ËÄΑŽÄΑ˜ΑæځŽ˜”ΑŽº˜”ΑæËΑ͉͋͊͋ΡΑr¨˜ÄΑq˔¢ËĘΑ;æ”ΠΑöÞΑژæÚËÞטŽæ«õ˜½üΑ
æû˜”ΑüΑ樘Α+Ĕ«ÄΑæûΑêæ¨Ëګ櫘ÞΑ¢ËÚΑΑ͉͉͋͐Α”˜½ΠΑb¨˜Α͉͋͊͋ΑŽæΑ¨”ΑÃ˜Ä”˜”Α樘Α+bΑŽæΑæËΑ«Ã×ËޘΑæûΑ½««½α
«æüΑËÄΑ樘Α«ÄŽËØΑ˜ÚʔΑ¢ÚËÃΑ樘Αށ½˜ΑË¢ΑÞ¨Ú˜ÞΑË¢ΑΑ¢Ëژ«£ÄΑŽËÃׁÄüΑËÄΑΑژæÚËÞטŽæ«õ˜ΑÞ«ÞΑλ«Π˜ΠΡΑ½ÞËΑ
××½«Ž½˜ΑæËΑ樘ΑæځÄށŽæ«ËÄÞΑ”ËĘΑ˜¢ËژΑAüΑ͋͑ΡΑ͉͋͊͋μΠΑ

b¨˜ΑÃ˜Ä”ØÄæÞΑÁ”˜ΑüΑ樘Α͉͋͊͋ΑŽæΑŽ½Ú«Ĉ˜”Α樁æΑ«¢ΑΑŽËÃׁÄüΑ«ÞΑژ£«Þæ˜Ú˜”ΑËÚΑ«ÄŽËÚ×Ëځ昔ΑËêæÞ«”˜Α
+Ĕ«ΡΑ «æÞΑ Þ¨Ú˜ÞΑ ö«½½Α ˜Α ”˜˜Ã˜”Α æËΑ ˜Α ËÚΑ ¨õ˜Α ½öüÞΑ ˜˜ÄΑ Þ«æêæ˜”Α «ÄΑ +Ĕ«Α «¢Α 樘üΑ ”˜Ú«õ˜Α 樘«ÚΑ õ½ê˜Α
ÞêÞæÄ櫁½½üΑ¢ÚËÃΑ樘ΑÞޘæÞΑ½ËŽæ˜”Α«ÄΑ+Ĕ«ΠΑÞΑΑژÞê½æΡΑ樘ΑטÚÞËÄÞΑö¨ËΑÞ˽”ΑÞꎨΑÞ¨Ú˜ÞΑË¢Α¢Ëژ«£ÄΑ
ŽËÃׁī˜ÞΑ˜¢ËژΑ樘Α˜ÄŽæØÄæΑË¢Α樘ΑŽæΑλ«Π˜ΠΡΑAüΑ͋͑ΡΑ͉͋͊͋μΑ½ÞËΑ˜ŽÃ˜Α½«½˜ΑæËΑׁüΑæûΑËÄΑ樘Α«ÄŽËØΑ
˜ÚʔΑ¢ÚËÃΑÞꎨΑށ½˜Π

b¨˜Αbûæ«ËÄΑ;öÞΑλØĔØÄæμΑŽæΡΑ͉͋͋͊ΑÄê½½«Ĉ˜ÞΑ樘ΑϏژæÚËÞטŽæ«õ˜Αæûæ«ËÄϐΑ樁æΑöÞΑ«ÄæÚ˔ꎘ”Αö«æ¨Α
樘Α"«ÄÄŽ˜ΑŽæΑË¢Α͉͋͊͋ΠΑ+æΑÄê½½«Ĉ˜ÞΑ樫ÞΑæûΑ½««½«æüΑ«Ã×Ëޘ”ΑËÄΑÞꎨΑטÚÞËÄÞΑ×ÚËõ«”˜”Α樘üΑ¢ê½Ĉ½ΑŽ˜Ú恫ÄΑΑ
conditions.
72
These conditions are:
+¢Α樘Α טÚÞËÄΑ ¨ÞΑĈ½˜”Α ÄΑ ×ט½Α ËÚΑ טæ«æ«ËÄΑ «ÄΑ樫ÞΑ ژ£Ú”ΡΑ «æΑ ÃêÞæΑ ˜Αö«æ¨”ځöÄΑ ËÚΑ樘Α טÚÞËÄΑ ÃêÞæΑ
ÞêÃ«æΑÄΑêĔ˜Ú恺«Ä£ΑæËΑö«æ¨”ځöΑ«æΡ
«¢Α樘ΑטÚÞËÄΑ¨ÞΑ«Ä«æ«æ˜”ΑËÚΑ£«õ˜ÄΑÄË櫎˜Α¢ËÚΑÄüΑÚ«æځæ«ËÄΡΑŽËĎ«½«æ«ËÄΡΑËÚΑؔ«æ«ËÄΑ×Úˎ˜˜”«Ä£ÞΑ«ÄΑ
樫ÞΑ ژ£Ú”ΡΑ 樘Α ÄË櫎˜ÞΑ ËÚΑ Ž½«ÃÞΑ êĔ˜ÚΑ ÞꎨΑ ×Úˎ˜˜”«Ä£ÞΑ ÃêÞæΑ ˜Α ö«æ¨”ځöÄΑ ËÚΑ 樘Α טÚÞËÄΑ ÃêÞæΑ
ÞêÃ«æΑÄΑêĔ˜Ú恺«Ä£ΑæËΑö«æ¨”ځöΑ樘ÃΡ
樘ΑטÚÞËÄΑÃêÞæΑÞêÃ«æΑÄΑêĔ˜Ú恺«Ä£ΑæËΑö«õ˜Α樘ΑÚ«£¨æΑæËΑޘ˜ºΑËÚΑ×êÚÞê˜ΑÄüΑژؔüΑËÚΑŽ½«ÃΑ«ÄΑ樫ÞΑ
ژ£Ú”ΡΑö¨«Ž¨ΑÁüΑË樘Úö«Þ˜Α˜Αõ«½½˜ΑêĔ˜ÚΑÄüΑ½öΑ«ÄΑ¢Ëڎ˜ΑËÚΑÄüΑ«½æ˜Ú½Α£Ú˜˜Ã˜ÄæΡΑÄ”
Ë樘ÚΑŽËĔ«æ«ËÄÞΡΑÞΑÁüΑ˜Α×ژގګ˜”Π
b¨˜ΑŽæΑ˜ÄÞêژÞΑ樁æΑ樘ژΑŽÄÄËæΑ˜û«ÞæΑÄüΑ¢êæêژΑ”˜ÃÄ”ΑüΑ樘Α£Ëõ˜ÚÄØÄæΑ¢ËÚΑ樘ΑŽË½½˜Žæ«ËÄΑË¢Αæû˜ÞΑ
ËÄΑ 樘Α Þ«ÞΑ Ë¢Α ÄΑ Ã˜Ä”ØÄæΑ ö«æ¨Α ژæÚËÞטŽæ«õ˜Α ˜ą˜ŽæΠΑb¨«ÞΑŽæΑ Áº˜ÞΑ 樘Α æûΑ ژ£«Ã˜Α Ë¢Α +Ĕ«Α ÃËژΑ
×ژ”«Žæ½˜ΡΑ«ÄŽÚ˜Þ«Ä£Α樘ΑގËטΑË¢Α¢Ëژ«£ÄΑ«Äõ˜ÞæØÄæΑ«ÄæËΑ樘ΑŽËêÄæÚüΑÞΑ«æΑŽ½Ú«Ĉ˜ÞΑ樘ΑÞæÄŽ˜ΑË¢Α+Ĕ«ÄΑ
#Ëõ˜ÚÄØÄæΑËÄΑ«Ã×ËÞ«æ«ËÄΑË¢ΑژæÚËÞטŽæ«õ˜Αæûæ«ËÄΠ

ANALYSIS:

+æΑöÞΑ«ÄΑ樘ΑĘöÞΑ˜ŽêޘΑË¢Α樘Αbûæ«ËÄΑ;öÞΑλØĔØÄæμΑŽæΡΑ͉͋͋͊Αö¨«Ž¨ΑÄê½½«Ĉ˜”Α樘ΑژæÚËÞטŽæ«õ˜Α
æûæ«ËÄΑژ½æ˜”ΑæËΑ«Ä”««Ú˜ŽæΑæځÄÞ¢˜ÚÞΑ樁æΑöÞΑ«ÄæÚ˔ꎘ”Αö«æ¨Α樘Α"«ÄÄŽ˜ΑŽæΑË¢Α͉͋͊͋Π

SOURCE : https://prsindia.org/billtrack/the-taxation-laws-amendment-bill-2021
https://economictimes.indiatimes.com/indus-
try/indl-goods/svs/cement/adani-holcim-deal-and-the-legacy-vodafone-tax-issue/articlesh
ow/91619826.cms?from=mdr
https://www2.deloitte.com/content/dam/Deloitte/in/Docu-
ments/tax/Global%20Business%20Tax%20Alert/in-tax-gbt-alert-indirect-transfer-provisions
-to-apply-prospectively-from-28-may-2012-noexp.pdf
https://www.nishithdesai.com/Content/document/pdf/Arti-
cles/171023_A_Indirect-Transfer-Taxation-in-India.pdf
73

Year 2023 FINANCE COMMISSION

Q12. Consider the following:


1. Demographic performance
2. Forest and ecology
3. Governance reforms
4. Stable government
5. Tax and fiscal efforts
For the horizontal tax devolution, the Fifteenth Finance Commission used how many of the above as criteria other
than population area and income distance?
(a) Only two
(b) Only three
(c) Only four
(d) All five

ANSWERS B

EXPLANATION
Þ˜”ΑËÄΑ×ګĎ«×½˜ÞΑË¢Αʘ”ΡΑ˜Ùê«æüΑÄ”ΑטڢËÚÁϘΡΑËõ˜Ú½½Α”˜õ˽êæ«ËÄΑ¢ËÚÃ꽁Α«ÞΑÞΑ¢Ë½½ËöÞΠ

Criteria Weight(%)

TË×꽁æ«ËÄΑαΑ͎͊Π͉Α

ژΑαΑ͎͊Π͉

Forest & ecology - 10

+ĎËØΑ”«ÞæÄŽ˜ΑαΑ͍͎Α

bûΑϞΑĈގ½Α˜ąËÚæÞΑαΑ͋Π͎Α

Demographic performance - 12.5

HÄΑ¨ËÚ«āËÄ恽Α”˜õ˽êæ«ËÄΡΑö¨«½˜Αwq"Α£Ú˜˜”Α樁æΑ樘Α˜ÄÞêÞΑ͉͋͊͊Α×Ë×꽁æ«ËÄΑ”æΑ˜ææ˜ÚΑژ×ژޘÄæÞΑ樘Α
×ژޘÄæΑʘ”ΑË¢Α[ææ˜ÞΡΑæËΑ˜Α¢«ÚΑæËΡΑÞΑö˜½½ΑÞΑژöÚ”ΡΑ樘Α[ææ˜ÞΑö¨«Ž¨Α¨õ˜Α”ËĘΑ˜ææ˜ÚΑËÄΑ樘Α”˜ÃËα
£Ú×¨«ŽΑ¢ÚËÄæΡΑwq"Α¨ÞΑÞÞ«£Ä˜”ΑΑ͊͋Π͎ΑטÚΑŽ˜ÄæΑö˜«£¨æΑæËΑ樘Α”˜Ãˣځר«ŽΑטڢËÚÁϘΑŽÚ«æ˜Ú«ËÄΠΑwq"Α
¨ÞΑ ژ«ÄæÚ˔ꎘ”Α æûΑ ˜ąËÚæΑ ŽÚ«æ˜Ú«ËÄΑ æËΑ ژöÚ”Α Ĉގ½Α טڢËÚÁϘΠΑ Hence option (b) is the correct
answer.

ANALYSIS:

b¨˜ΑĈ¢æ˜˜Äæ¨ΑĈāϘΑŽËÃëÞÞ«ËÄΑÞêÃ«æ昔Α«æÞΑژ×ËÚæΑ«ÄΑBËõ˜Ã˜ÚΑ͉͉͋͋ΠΑ(˜ÄŽ˜Α«æΑ«ÞΑ«Ã×ËÚæÄæΑ¢ËÚΑ樘Α
Þ׫ځÄæÞΑæËΑ£ËΑæ¨ÚË꣨Α樘Α«Ã×ËÚæÄæΑژŽËÃØĔæ«ËÄÞΑÄ”Α樘ΑŽÚ«æ˜Ú«Α樁æΑ«æΑޘæÞΑ¢ËÚΑ¢êĔΑ”˜õ˽êæ«ËÄΑ
between the states.
74

SOURCE : PIB (instead of reading the entire report)


https://pib.gov.in/PressReleasePage.aspx?PRID=1693868

Year 2023 INTANGIBLE INVESTMENTS

Q13. Consider the investments in the following assets:


1. Brand recognition
2. Inventory
3. Intellectual property
4. Mailing list of clients
How many of the above are considered intangible investments?
(a) Only one
(b) Only two
(c) Only three
(d) All four

ANSWERS C

EXPLANATION

ÄΑ«ÄæÄ£«½˜ΑÞޘæΑ«ÞΑÄΑ«”˜Äæ«Ĉ½˜ΑÄËÄαÃËĘæÚüΑÞޘæΡΑö«æ¨ËêæΑרüÞ«Ž½ΑÞêÞæÄŽ˜ΡΑ¨˜½”Α¢ËÚΑêޘΑ«ÄΑ樘Α
×Ú˔êŽæ«ËÄΑËÚΑÞê××½üΑË¢Α£Ë˔ÞΑËÚΑޘÚõ«Ž˜ÞΡΑ¢ËÚΑژÄ恽ΑæËΑË樘ÚÞΡΑËÚΑ¢ËÚΑ”ëīÞæځæ«õ˜Α×êÚ×ËޘÞΠΑ

Äæ˜Ú×ګޘÞΑ ¢Ú˜Ùê˜Äæ½üΑ ˜ûטĔΑ ژÞËêڎ˜ÞΡΑ ËÚΑ «ÄŽêÚΑ ½««½«æ«˜ÞΡΑ ËÄΑ 樘Α ŽÙê«Þ«æ«ËÄΡΑ ”˜õ˜½Ë×ØÄæΡΑΑ
Á«Äæ˜ÄÄŽ˜ΑËÚΑ˜Ä¨ÄŽ˜Ã˜ÄæΑË¢Α«ÄæÄ£«½˜ΑژÞËêڎ˜ÞΑÞꎨΑÞΑގ«˜Äæ«ĈŽΑËÚΑ明¨Ä«Ž½ΑºÄËö½˜”£˜ΡΑ”˜Þ«£ÄΑ
Ä”Α«Ã×½˜Ã˜Äææ«ËÄΑË¢ΑĘöΑ×Úˎ˜ÞޘÞΑËÚΑÞüÞæ˜ÃÞΡΑ½«Ž˜ÄŽ˜ÞΡΑ«Ä昽½˜ŽæꁽΑ×ÚËטÚæüΡΑmarket knowledge and
trademarks (including brand names and publishing titles).

ËÃÃËÄΑ ˜ûÃ×½˜ÞΑ Ë¢Α «æ˜ÃÞΑ ˜ÄŽËÃׁÞޘ”Α üΑ 樘ޘΑ Úˁ”Α ¨˜”«Ä£ÞΑ Ú˜Α ŽËÃ×êæ˜ÚΑ ÞË¢æöÚ˜ΡΑ patents,
copyrights,Α ÃËæ«ËÄΑ ׫ŽæêژΑ Ĉ½ÃÞΡΑ customer lists,Α ÃËÚ棁£˜Α ޘÚõ«Ž«Ä£Α Ú«£¨æÞΡΑ ĈÞ¨«Ä£Α ½«Ž˜ÄŽ˜ÞΡΑ «Ã×ËÚæΑ
ÙêËæÞΡΑ ¢ÚÄŽ¨«Þ˜ÞΡΑ ŽêÞæËØÚΑ ËÚΑ Þê××½«˜ÚΑ ژ½æ«ËÄÞ¨«×ÞΡΑ ŽêÞæËØÚΑ ½Ëü½æüΡΑ Áں˜æΑ Þ¨Ú˜Α Ä”Α Áں˜æ«Ä£Α
rights.

#Ë˔ö«½½Α«ÞΑÄË樘ÚΑ˜ûÃ×½˜ΑË¢ΑÄΑ«æ˜ÃΑË¢Α«ÄæÄ£«½˜ΑāæêژΑö¨«Ž¨Α˜«æ¨˜ÚΑÚ«Þ˜ÞΑËÄΑŽÙê«Þ«æ«ËÄΑËÚΑ«ÞΑ«Äæ˜Úā½α
½üΑ£˜Ä˜Úæ˜”ΠΑbÄ£«½˜ΑÞޘæÞΑÚ˜Αæü׫Ž½½üΑרüÞ«Ž½ΑÞޘæÞΑËÚΑ×ÚËטÚæüΑËöʔΑüΑΑŽËÃׁÄüΡΑÞꎨΑÞΑ˜Ùê«×α
ØÄæΡΑê«½”«Ä£ÞΡΑÄ”Α«Äõ˜ÄæËÚüΠΑHence only option 2 is not correct.
75

Please Note:
Normally this topic is covered under ACCOUNTANCY.

This is a question that is covered in Accountancy and not normally studies under economics, but if an
aspirant understands the meaning of the word tangible (things that can be touched and felt), then
ö¨æΑ «ÞΑ «ÄæÄ£«½˜Α «ÞΑ ÄËæΑ ¨Ú”Α æËΑ £ê˜ÞÞΠΑ b¨˜Α öËڔΑ «Äõ˜ÄæËÚüΑ ¨ÞΑ ˜˜ÄΑ ”˜ĈʔΑ «ÄΑ ¨×æ˜ÚΑ ͋Α Ë¢Α 樘Α
Macroeconomics textbook (meant for Class 12).

The following excerpt (screenshot) is from the same chapter

ûŽ˜Ú×æΑ¢ÚËÃΑBWb
b¨êÞΡΑ«æΑ«ÞΑÄËæΑΑ¨Ú”Α½Ë£«ŽΑæËΑ××½üΠΑb¨˜ΑÙê˜Þæ«ËÄΑÞ¨Ë꽔ΑÄËæΑæ¨êÞΑ˜Αژ£Ú”˜”ΑÞΑ”«ĆŽê½æΠ

+æΑ«ÞΑ”«ĆŽê½æΑæËΑ׫Ä×Ë«ÄæΑËĘΑژÞËÄΑÞΑæËΑö¨üΑ樫ÞΑÙê˜Þæ«ËÄΑŽÃ˜Α«ÄΑ樘Α[Α͉͋͋͌ΡΑêæΑfT[Α¨ÞΑ˜˜ÄΑæ˜Þæ«Ä£Α
Ä”ΑÁüΑæ˜ÞæΑ«ÄΑ樘Α¢êæêژΑöÄæΑæËΑæ˜ÞæΑ樘ΑŽêÚ«ËÞ«æüΑË¢ΑÞ׫ځÄæΑ«ÄΑºÄËö«Ä£ΑËêæΑ樘ΑÞ«ŽΑ¢êĎæ«ËÄ«Ä£ΑË¢ΑΑ
ĈÚÃΑËÚΑŽËÃׁÄüΠ

+Ď½ê”˜ÞΑŽËÃ×ژ¨˜ÄÞ«õ˜ΑŽËõ˜Ú£˜ΑË¢Α½½ΑæË׫ŽÞΑ¢ËÚΑ½½Α樘Α¢ËêÚΑׁטÚÞΑË¢Α#[ΑA«ÄÞΡΑ#[ΑTژ½«ÃÞΡΑ[bΑ
“You are as strong Ä”ΑÞށü
+Ď½ê”˜ΑTژΑ"ËêĔæ«ËÄΑŽ½ÞޘÞΫΑλ͉͐ΑŽ½ÞޘÞμΑ¢ËÚΑêĔ˜ÚÞæÄ”«Ä£ΑË¢Α9˜üΑ#[ΑŽËϘ×æÞΑϞΑ¢êĔÃ˜Ä恽Α
as your Foundation” Ë¢ΑBWbÞΑϞΑ Þ«ŽΑ Ë˺Þ
+Ď½ê”˜Α½½Α+Ĕ«Α#[ΑA«ÄÞΡΑTژ½«ÃÞΡΑ[bΑÄ”ΑÞށüΑb˜ÞæΑ[˜Ú«˜ÞΑ͉͎͋͋

Ž½ÞޘÞΑλAWμΡΑA«ÄÞΑ͌͏͎ΑϞΑTb͌͏͎
T˜ÚÞËā½«Þ˜”ΑA˜ÄæËÚÞΑ¢ËÚΑÞޘÞÞ«Ä£ΑטڢËÚÁϘΑË¢Α樘Α
ÞæꔘÄæÞΑژ£ê½Ú½üΑ

ŽŽ˜ÞÞΑæËΑ;«õ˜ΑÞΑö˜½½ΑÞΑW˜ŽËڔ˜”ΑŽ½ÞޘÞΑËÄΑüËêÚΑטÚÞËā½Α
ËĽ«Ä˜Α[æꔘÄæΑT½æ¢ËÚÃΑ
+Ď½ê”˜ÞΑT˜ÚÞËā½«æüΑ˜õ˜½Ë×ØÄæΑTÚˣځÃØΑ

|
PRELIMS CUM MAINS

2025, 2026 & 2027


76

In a world facing unprecedented environmental challenges, staying


informed and empowered is more crucial than ever. VisionIAS brings you
‘The Planet Vision’ʟ˔˦˜ˠˣ˟˜Ѓ˘˗ʟ˜ˡ˙ˢ˥ˠ˔˧˜˩˘ʟ˔ˡ˗˜ˡ˧˘˥˔˖˧˜˩˘ˠ˔˚˔˭˜ˡ˘ to
delve into the complexities of the environment.
With the belief, that individual efforts and awareness are the key to a
˦˨˦˧˔˜ˡ˔˕˟˘ ˙˨˧˨˥˘ʟ ˧˛˘ ˠ˔˚˔˭˜ˡ˘ ˦˘˘˞˦ ˧ˢ ˜ˡ˦ˣ˜˥˘ ˔ˡ˗ ˘˗˨˖˔˧˘ ˣ˘ˢˣ˟˘ ˧ˢ
develop a deeper understanding and appreciation for the environment,
nature & planet.

Objectives of ‘The Planet Vision’

Sensitise the young generation: Highlighting pressing environmental issues and their
multifaceted impacts.

Inspire Action and Promote Sustainable Lifestyle: Inspiring stories and case studies to motivate
readers to make environmentally conscious choices.

Bridge Environmental Science and Public Understanding:˃˥˘˦˘ˡ˧˜ˡ˚˖ˢˠˣ˟˘˫˦˖˜˘ˡ˧˜Ѓ˖˖ˢˡ˖˘ˣ˧˦


in a simple and interactive manner.

Showcase Solutions: Spotlight innovative technologies, projects, and initiatives that offer solutions
to environmental challenges.

Highlight Local Efforts: Showcase local conservation efforts, community initiatives, and grassroots
projects that make a positive impact on the environment.

Who is the magazine for?


ˇ˛˘ˀ˔˚˔˭˜ˡ˘˜˦˗˘˦˜˚ˡ˘˗˙ˢ˥˦˧˨˗˘ˡ˧˦ʟ˘˖ˢʠ˖ˢˡ˦˖˜ˢ˨˦˜ˡ˗˜˩˜˗˨˔˟˦ʟ˘˗˨˖˔˧ˢ˥˦ʟ˘ˡ˩˜˥ˢˡˠ˘ˡ˧˔˟˜˦˧˦ʟ˔ˡ˗˔ˡˬˢˡ˘
who cares about the health of our planet.

Key elements of the ‘The Planet Vision’

Cover Stories: ˇ˛ˢ˨˚˛˧ʠˣ˥ˢ˩ˢ˞˜ˡ˚ ˔˥˧˜˖˟˘˦ Environment and You: Illustrating ways to


about a critical ongoing environmental make environmentally conscious choices in
issue, along with the mitigation strategies everyday life.
adopted at the national and international
levels. Green Tech: New and emerging technologies
˜ˡ˧˛˘Ѓ˘˟˗ˢ˙˘ˡ˩˜˥ˢˡˠ˘ˡ˧ʡ
ʵ˥˜˘Ѓˡ˚˔ˡ˗ʷ˘˩˘˟ˢˣˠ˘ˡ˧˦ʭStay informed
about the latest environmental news, Interactive elements:
trends, and solutions. Snapshot: Telling a story through
capturing images.
Protect and Preserve: Inspirational stories
of local conversation efforts. Quizzes and crosswords: To test your
understanding and knowledge as a
reader.

“ Look deep into nature, and then you


will understand everything better.
“ Scan the QR code,
to download and
read the Magazine:
ʠʴ˟˕˘˥˧ʸ˜ˡ˦˧˘˜ˡ
77

EXTERNAL SECTOR
λ͋͑ΑVê˜Þæ«ËÄÞμ

Year 2017 GLOBAL INFRASTRUCTURE FACILITY

Q1. The Global Infrastructure Facility is a/an


λμΑ[BΑ«Ä«æ«æ«õ˜ΑæËΑêףځ”˜Α«Ä¢ÚÞæÚêŽæêژΑ«ÄΑÞ«ΑÄ”ΑĈāϘ”ΑüΑŽÚ˜”«æΑ¢ÚËÃΑ樘ΑÞ«ÄΑ˜õ˜½Ë×ØÄæΑ
Bank.
λμΑrËÚ½”Α ÄºΑŽË½½Ëځæ«ËÄΑ樁æΑ¢Ž«½«æ«˜ÞΑ樘Α×ژׁځæ«ËÄΑÄ”ΑÞæÚêŽæêÚ«Ä£ΑË¢ΑŽËÃ×½˜ûΑ«Ä¢ÚÞæÚêŽæêژΑ
Tꍽ«ŽαTÚ«õæ˜ΑTÚæĘÚÞ¨«×ΑλTTTÞμΑæËΑ˜Ä½˜ΑÃˍ«½«āæ«ËÄΑË¢Α×Ú«õæ˜ΑޘŽæËÚΑÄ”Α«ÄÞæ«æêæ«Ëā½Α«Äõ˜ÞæËÚΑ
Ž×«æ½Π
λŽμΑ˽½Ëځæ«ËÄΑÃËÄ£Α樘ΑÁ¶ËÚΑÄºÞΑË¢Α樘ΑöËÚ½”ΑöËÚº«Ä£Αö«æ¨Α樘ΑHΑÄ”Α«Ä¢ÚÞæÚêŽæêژΑ×Ú˶˜ŽæÞΑ
樁æΑ¨õ˜Α樘Α×Ëæ˜Ä櫁½ΑæËΑÃˍ«½«ā˜Α×Ú«õæ˜Α«Äõ˜ÞæØÄæΠ
λ”μΑfBbΑ¢êĔ˜”Α«Ä«æ«æ«õ˜Α樁æΑޘ˜ºÞΑæËΑĈāϘΑÄ”Α¢Ž«½«ææ˜Α«Ä¢ÚÞæÚêŽæêژΑ”˜õ˜½Ë×ØÄæΑ«ÄΑ樘Α
rËÚ½”Π

ANSWERS B

EXPLANATION
b¨˜Α #½Ë½Α +ĢځÞæÚêŽæêژΑ "Ž«½«æüΑ λ#+"μΑ Þê××ËÚæÞΑ #Ëõ˜ÚÄØÄæÞΑ «ÄΑ Ú«Ä£«Ä£Α ö˜½½αÞæÚêŽæêژ”Α Ä”Α Äº½˜Α
«Ä¢ÚÞæÚêŽæêژΑ ×Ú˶˜ŽæÞΑ æËΑ Áں˜æΠΑ #+"ϐÞΑ ×Ú˶˜ŽæΑ Þê××ËÚæΑ ŽÄΑ ŽËõ˜ÚΑ 樘Α ÞטŽæÚêÃΑ Ë¢Α ”˜Þ«£ÄΡΑ ×ژׁځæ«ËÄΡΑ
ÞæÚêŽæêÚ«Ä£Α Ä”Α æځÄށŽæ«ËÄΑ «Ã×½˜Ã˜Äææ«ËÄΑ Žæ«õ«æ«˜ÞΡΑ ”ځö«Ä£Α ËÄΑ 樘Α ŽËͫʔΑ ˜ûטÚæ«Þ˜Α Ë¢Α 樘Α #+"ϐÞΑ
b˜Ž¨Ä«Ž½ΑÄ”Α”õ«ÞËÚüΑTÚæĘÚÞΑÄ”Α¢ËŽêÞ«Ä£ΑËÄΑÞæÚêŽæêژÞΑ樁æΑÚ˜Α½˜ΑæËΑææځŽæΑΑö«”˜Αځģ˜ΑË¢Α×Ú«õæ˜Α
investors.
½ËÄ£Αö«æ¨Α樘ΑrËÚ½”Α ÄºΑ#ÚËê×ΡΑÄ”Α«ÞΑŽËᎨ«ÚΑË¢Α樘Α#+"ϐÞΑ#Ëõ˜ÚÄ«Ä£ΑËêĎ«½ΠΑHence option (b) is the
correct answer.

ANALYSIS:
ANALYSIS:
b¨«ÞΑæË׫ŽΑÄ”Αޫ뽁ÚΑæË׫ŽÞΑژ½æ˜”ΑæËΑ½˜ÞޘÚαºÄËöÄΑËÚ£Ä«Þæ«ËÄÞΑÚ˜ΑÄËæΑŽËõ˜Ú˜”Α«ÄΑÄüΑË¢Α樘ΑÞޘÄ櫁½Α
Aæ˜Ú«½ΡΑêæΑË¢æ˜ÄΑ×טÚΑ«ÄΑ樘ΑĘöÞׁטÚÞΠΑ[æꔘÄæÞΑŽÄΑ«”˜Äæ«¢üΑº˜üΑ£½Ë½Α樘ØÞΑ½«º˜Α£½Ë½ΑĈāϘΡΑ
Ž½«Ãæ˜ΑŽ¨Ä£˜Α˜æŽΠΑö¨«Ž¨ΑÚ˜Αژ½˜õÄæΑ¢ËÚΑfT[ΑÄ”Α˜ΑŽËÄގ«ËêÞΑË¢ΑË”«˜ÞΑÄ”ΑĘöÞΑŽËÄĘŽæ˜”Αö«æ¨Α樘ÃΠΑ

http://www.thehindu.com/business/Economy/gif-launched-to-help-india-bridge-infra-
SOURCE :
”˜ĈŽ«æ݁Ú櫎½˜͏͍͑͏͍͍͉ИŽ˜

+æΑöÞΑ«ÄΑ樘ΑĘöΑ樘ÄΡΑÄ”ΑÄΑ
˜ûטŽæ˜”Α 樘ØΑ ŽËÄÞ«”˜Ú«Ä£Α
樘Α ¢ËŽêÞΑ ËÄΑ «Ä¢ÚÞæÚêŽæêژΑ
Ä”Α«æÞΑĈāĎ«Ä£Α«ÄΑ+Ĕ«ΠΑ
78

Year 2017 BTIA

V͋ΠΑ Úˁ”αÞ˜”Αbځ”˜ΑÄ”Α+Äõ˜ÞæØÄæΑ£Ú˜˜Ã˜ÄæΑλ b+μϐΑ«ÞΑÞËØæ«Ã˜ÞΑޘ˜ÄΑ«ÄΑ樘ΑĘöÞΑ«ÄΑ樘Α


context of negotiations held between India and
(a) European Union
(b) Gulf Cooperation Council
(c) Organization for Economic Cooperation and Development
(d) Shanghai Cooperation Organization

ANSWERS A

EXPLANATION
b¨˜Α"bΑ恽ºÞΑö«æ¨ΑfΑ¨õ˜Α˜˜ÄΑ”˜”½ËŽº˜”ΑޫϘΑ͉͋͊͌Α¢æ˜ÚΑ͊͏ΑÚËêĔÞΑË¢ΑĘ£Ë櫁æ«ËÄÞΠΑ;ÞæΑü˜ÚΑfΑځ«Þ˜”Α
«æÞΑŽËϘÚÄΑËõ˜ÚΑö¨æΑ«æΑŽ½½˜”Α+Ĕ«ϐÞΑύêÄ«½æ˜Ú½Αæ˜Úëāæ«ËÄώΑË¢Α˜û«Þæ«Ä£Α «½æ˜Ú½Α+Äõ˜ÞæØÄæΑbژæ«˜ÞΑλ +bÞμΑ
ö«æ¨ΑύΑÞ«£Ä«ĈŽÄæΑÄê͘ÚΑË¢ώΑfαØ͘ÚΑŽËêÄæÚ«˜ÞΠΑHence option (a) is the correct answer.

ANALYSIS:

SOURCE : The FTA negotiations with EU has remained continuously in news.


¨ææ×΢έέ׫ΠÄ«ŽΠ«ÄέĘöÞ«æ˜έTÚ«ÄæW˜½˜Þ˜ΠÞ×ûΧژ½«”Ͻ͊͏͉͊͌͊Α
http://www.thehindu.com/business/Industry/India-EU-aim-to-break-Free-Trade-
£Ú˜˜Ã˜Äæα«ÃׁÞÞ˜έÚ櫎½˜͍͍͊͌͐͑͌͑ИŽ˜

Year 2017 WTO

V͌ΠΑËÄÞ«”˜ÚΑ樘Α¢Ë½½Ëö«Ä£ΑÞææ˜Ã˜ÄæÞ΢
1. India has ratified to Trade Facilitation Agreement (TFA) of WTO.
2. TFA is a part of WTO’s Bali Ministerial Package of 2013.
3. TFA came into force in January 2016.
Which of the statements given above is/are correct?
(a) 1 and 2 only
(b) 1 and 3 only
(c) 2 and 3 only
(d) 1, 2 and 3

ANSWERS A
79

EXPLANATION
b¨˜Αbځ”˜Α "Ž«½«ææ«ËÄΑ£Ú˜˜Ã˜ÄæΑ λb"μΑ «ÞΑ樘Α rbHϐÞΑ ĈÚÞæα˜õ˜ÚΑ Ãê½æ«½æ˜Ú½Α ŽŽËڔΑ樁æΑ «ÃÞΑæËΑ Þ«Ã×½«¢üΑ
ŽêÞæËÃÞΑ ژ£ê½æ«ËÄÞΑ ¢ËÚΑ 樘Α ŽÚËÞÞαËڔ˜ÚΑ ÃËõ˜Ã˜ÄæΑ Ë¢Α £Ë˔ÞΠΑ +æΑ öÞΑ ËêæŽËØΑ Ë¢Α rbHϐÞΑ ͒æ¨Α ½«Α
λ+ĔËĘޫμΑëīÞæ˜Ú«½ΑׁŽº£˜ΑË¢Α͉͋͊͌ΠΑHence statement 2 is correct.
+Ĕ«Αځæ«Ĉ˜”Α«æΑ«ÄΑ×Ú«½Α͉͋͊͏ΠΑb¨˜Αb"Α«ÃÞΑæËΑ˜ûט”«æ˜Α樘ΑÃËõ˜Ã˜ÄæΑÄ”ΑŽ½˜ÚÄŽ˜ΑË¢Α£Ë˔ÞΠΑb¨˜Αb"ΑöÞΑ
¨Ä”˜”Α Ëõ˜ÚΑ æËΑ 樘Α rbHΑ «Ú˜ŽæËÚα#˜Ä˜Ú½Α üΑ +Ĕ«ΠΑ b¨˜Α b"Α öÞΑ Þê××Ëޘ”Α æËΑ ˜Äæ˜ÚΑ «ÄæËΑ ¢Ëڎ˜Α ËϘΑ
æöËα樫ڔÞΑË¢Α樘ΑrbHΑØ͘ÚÞΑ¨”ΑŽËÃ×½«˜”Αö«æ¨Α樘Α”ËØÞ櫎ΑژŽæ«ĈŽæ«ËÄΑ×Úˎ˜ÞÞΠΑHence statement
1 is correct.
+æΑŽÃ˜Α«ÄæËΑ¢Ëڎ˜Α«ÄΑ"˜ÚêÚüΑ͉͋͊͐ΠΑHence statement 3 is not correct. Hence option (a) is the correct
answer.
ANALYSIS:
ANALYSIS:
India and WTO has always have been the favourite topic of the UPSC. Time and again, UPSC has asked
questions from this area.

SOURCE : http://pib.nic.in/newsite/PrintRelease.aspx?relid=159992
http://www.thehindu.com/business/Economy/india-tables-legally-vetted-proposal
αËÄα£½Ë½αޘÚõ«Ž˜ÞαׁŽæαæαöæË݁Ú櫎½˜͎͎͍͊͐͌͐͌ИŽ˜

Motivation
TFA has remained regularly in news for last 3 years.

Year 2017 DOMESTIC CONTENT REQUIREMENT

V͍ΠΑb¨˜Αæ˜ÚÃΑϏËØÞ櫎ΑËÄæ˜ÄæΑW˜Ùê«Ú˜Ã˜ÄæϐΑ«ÞΑÞËØæ«Ã˜ÞΑޘ˜ÄΑ«ÄΑ樘ΑĘöÞΑö«æ¨Αژ¢˜Ú˜ÄŽ˜ΑæË
(a) Developing solar power production in our country
(b) Granting licenses to foreign T.V. channels in our country
(c) Exporting our food products to other countries
(d) Permitting foreign educational institutions to set up their campuses in our country

ANSWERS A

EXPLANATION

ϏËØÞ櫎ΑËÄæ˜ÄæΑW˜Ùê«Ú˜Ã˜ÄæϐΑλWμΡΑ¨ÞΑË¢æ˜ÄΑ˜˜ÄΑ«ÄΑ樘ΑĘöÞΡΑÄ”Α«ÞΑژ½æ˜”ΑæËΑ”˜õ˜½Ë×ØÄæΑË¢ΑÞ˽ÚΑ
×Ëö˜ÚΑ×Ú˔êŽæ«ËÄΠ
ËØÞ櫎Α ËÄæ˜ÄæΑ W˜Ùê«Ú˜Ã˜ÄæÞΑ λWμΑ Ú˜Α £˜Ä˜Ú½½üΑ ×ÚËÃê½£æ˜”Α ¢ËÚΑ ×ÚË明æ«Ä£Α «Ä¢ÄæΑ «Ä”êÞæÚ«˜ÞΡΑ
˜ÞטŽ«½½üΑ«ÄΑ”˜õ˜½Ë׫ģΑŽËêÄæÚ«˜ÞΡΑêÄ櫽Α樘üΑŽÄΑŽËÃטæ˜ΑËÄΑ樘Α«Äæ˜Úāæ«Ëā½ΑÁں˜æΠΑ+æΑ”˘ÞΑÄËæΑ½˜”ΑæËΑ
½Ëö˜Ú«Ä£ΑË¢Α×Ú«Ž˜ÞΑË¢Α樘ޘΑŽËÃÃ˔«æ«˜ÞΠ
b¨˜ΑWorld Trade Organization (WTO) ¨”ΑÚ꽘”Α£«ÄÞæΑ+Ĕ«ϖÞΑ”ËØÞ櫎ΑŽËÄæ˜ÄæΑ×˽«ŽüΑ¢ËÚΑÞ˽ÚΑŽ˜½½ÞΑÄ”Α
Ã˔꽘ÞΣΑ+Ĕ«Α×ט½˜”Α樘ΑÚ꽫ģΑ«ÄΑΑ«”ΑæËΑº˜˜×ΑWΑÚ꽘ÞΑ«ÄΑ×½Ž˜Α¢ËÚΑ£Ëõ˜ÚÄØÄæΑ×ÚˎêژØÄæΠΑb¨˜Α
ŽÞ˜Α £«ÄÞæΑ +Ĕ«ΑöÞΑ ËÚ«£«Ä½½üΑĈ½˜”Α «ÄΑ ͉͋͊͌ΡΑ¢Ë½½Ëö«Ä£Α樘Α ÄÄËêĎ˜Ã˜ÄæΑ Ë¢Α WΑ «ÄΑ 6BB[AΑ T¨Þ˜Α ++Α
×˽«ŽüΑÄ”Α¢æ˜ÚΑ+Ĕ«Α”˜Ž«”˜”ΑæËΑĈ½˜ΑÄΑÄæ«α”êÃ׫ģΑŽÞ˜Α£«ÄÞæΑ樘ΑfΠ[ΠΡΑ¨«ÄΡΑA½üÞ«ΑÄ”Αb«öÄΠ
80

ANALYSIS:
ANALYSIS:
The WTO ruling against India was in the news then

SOURCE : http://economictimes.indiatimes.com/industry/energy/power/removal-of-domestic-content
αژÙê«Ú˜Ã˜Äæα«ÄαÞ˽ÚαÄËæαα«£α½Ëö݁Ú櫎½˜Þ¨Ëöέ͎͍͍͎͊͋͒͊ЎÃÞ

Year 2017 INTELLECTUAL PROPERTY RIGHTS

V͎ΠΑr«æ¨Αژ¢˜Ú˜ÄŽ˜ΑæËΑ樘ΑϏBæ«Ëā½Α+Ä昽½˜ŽæꁽΑTÚËטÚæüΑW«£¨æÞΑT˽«ŽüϐΡΑŽËÄÞ«”˜ÚΑ樘Α¢Ë½½Ëö«Ä£ΑÞææ˜α
ments:
1. It reiterates India’s commitment to the Doha Development Agenda and the TRIPS Agreement.
2. Department of Industrial Policy and Promotion is the nodal agency for regulating intellectual property rights in
India.
Which of the above statements is/are correct?
(a) 1 only
(b) 2 only
(c) Both 1 and 2
(d) Neither 1 nor 2

ANSWERS C

EXPLANATION

b¨˜ΑfÄ«ËÄΑ«Ä˜æΑ××ÚËõ˜”Α樘ΑBæ«Ëā½Α+Ä昽½˜ŽæꁽΑTÚËטÚæüΑW«£¨æÞΑλ+TWμΑT˽«ŽüΑ«ÄΑ͉͋͊͏Α樁æΑö«½½Α½üΑ樘Α
¢êæêژΑ Úˁ”Á×Α ¢ËÚΑ «Ä昽½˜ŽæꁽΑ ×ÚËטÚæüΑ «ÄΑ +Ĕ«ΠΑ b¨«ÞΑ ×˽«ŽüΑ Þ¨½½Α ö˜õ˜Α «ÄΑ 樘Α Þæژģæ¨ÞΑ Ë¢Α 樘Α
#Ëõ˜ÚÄØÄæΡΑ ژޘÚŽ¨Α Ä”Α ”˜õ˜½Ë×ØÄæΑ ËÚ£Ä«āæ«ËÄÞΡΑ ˜”ꎁæ«Ëā½Α «ÄÞæ«æêæ«ËÄÞΡΑ ŽËÚ×Ëځæ˜Α ˜Äæ«æ«˜ÞΑ
«ÄŽ½ê”«Ä£Α A[AÞΡΑ ÞæÚæαê×ÞΑ Ä”Α Ë樘ÚΑ Þ恺˜¨Ë½”˜ÚÞΑ «ÄΑ 樘Α ŽÚ˜æ«ËÄΑ Ë¢Α ÄΑ «ÄÄËõæ«ËÄαŽËĔꎫõ˜Α
˜Äõ«ÚËÄØÄæΡΑ ö¨«Ž¨Α Þæ«Ã꽁æ˜ÞΑ ŽÚ˜æ«õ«æüΑ Ä”Α «ÄÄËõæ«ËÄΑ ŽÚËÞÞΑ ޘŽæËÚÞΡΑ ÞΑ ½ÞËΑ ¢Ž«½«ææ˜ÞΑ Α Þ恍½˜ΡΑ
æځÄÞׁژÄæΑÄ”ΑޘÚõ«Ž˜αËÚ«˜Ä昔Α+TWΑ”ëīÞæځæ«ËÄΑ«ÄΑ樘ΑŽËêÄæÚüΠΑ
b¨˜ΑT˽«ŽüΑژŽË£Ä«ā˜ÞΑ樁æΑ+Ĕ«Α¨ÞΑΑö˜½½α˜Þ恍½«Þ¨˜”ΑbW+T[αŽËÃ×½«ÄæΑ½˜£«Þ½æ«õ˜ΡΑ”ëīÞæځæ«õ˜ΑÄ”Α
¶ê”«Ž«½Α ¢ÚÃ˜öËÚºΑ æËΑ ށ¢˜£êÚ”Α +TWÞΡΑ ö¨«Ž¨Α ؘæÞΑ «æÞΑ «Äæ˜Úāæ«Ëā½Α ˍ½«£æ«ËÄÞΑ ö¨«½˜Α ê櫽«ā«Ä£Α 樘Α
ĉ˜û««½«æ«˜ÞΑ×ÚËõ«”˜”Α«ÄΑ樘Α«Äæ˜Úāæ«Ëā½Αژ£«Ã˜ΑæËΑ””Ú˜ÞÞΑ«æÞΑ”˜õ˜½Ë×ØÄ恽ΑŽËϘÚÄÞΠΑ+æΑژ«æ˜Úæ˜ÞΑ+Ĕ«ϐÞΑ
ŽËÃëæØÄæΑæËΑ樘Α˨Α˜õ˜½Ë×ØÄæΑ£˜Ä”ΑÄ”Α樘ΑbW+T[Α£Ú˜˜Ã˜ÄæΠΑHence statement 1 is correct.
b¨˜Þ˜Α ˍ¶˜Žæ«õ˜ÞΑ Ú˜Α ÞË꣨æΑ æËΑ ˜Α Ž¨«˜õ˜”Α æ¨ÚË꣨Α ”˜æ«½˜”Α Žæ«ËÄΑ ×Ë«ÄæÞΠΑ b¨˜Α Žæ«ËÄΑ üΑ ”«ą˜Ú˜ÄæΑ
A«Ä«ÞæÚ«˜ÞέΑ˜×ÚæØÄæÞΑÞ¨½½Α˜ΑÃËÄ«æËژ”ΑüΑ+TTΑö¨«Ž¨ΑÞ¨½½Α˜Α樘ΑÄ˔½Α”˜×ÚæØÄæΑæËΑŽËËڔ«Äæ˜ΡΑ
£ê«”˜Α Ä”Α Ëõ˜Úޘ˜Α «Ã×½˜Ã˜Äææ«ËÄΑ Ä”Α ¢êæêژΑ ”˜õ˜½Ë×ØÄæΑ Ë¢Α +TWÞΑ «ÄΑ +Ĕ«ΠΑ Hence statement 2 is
correct.

ANALYSIS:
ANALYSIS:

SOURCE : ¨ææ×΢έέ׫ΠÄ«ŽΠ«ÄέĘöÞ«æ˜έTÚ«ÄæW˜½˜Þ˜ΠÞ×ûΧژ½«”Ͻ͊͌͑͑͒͊
81

Year 2017 ASIA-PACIFIC MINISTERIAL CONFERENCE

V͏ΠΑ r«æ¨Α ژ¢˜Ú˜ÄŽ˜Α æËΑ ϏÞ«Α TŽ«ĈŽΑ A«Ä«Þæ˜Ú«½Α ËÄ¢˜Ú˜ÄŽ˜Α ËÄΑ (ËêÞ«Ä£Α Ä”Α fڍÄΑ ˜õ˜½Ë×ØÄæΑ
λTA(fμϐΡΑŽËÄÞ«”˜ÚΑ樘Α¢Ë½½Ëö«Ä£ΑÞææ˜Ã˜ÄæÞ΢
1. The first APMCHUD was held in India in 2006 on the theme ‘Emerging Urban Forms – Policy Responses and
Governance Structure’.
2. India hosts all the Annual Ministerial Conferences in partnership with ADB, APEC and ASEAN.
Which of the statements given above is/are correct?
(a) 1 only
(b) 2 only
(c) Both 1 and 2
(d) Neither 1 nor 2

ANSWERS D

EXPLANATION
b¨˜Α͊ÞæΑTA(fΑöÞΑ¨˜½”Α«ÄΑB˜öΑ˜½¨«ΡΑ+Ĕ«Α¢ÚËÃΑ͊͌æ¨α͊͏æ¨Α˜Ž˜Ã˜ÚΡΑ͉͉͋͏ΑËÄΑ樘Α樘ØΑË¢ΑϏΑq«Þ«ËÄΑ
¢ËÚΑ[êÞ恫ā½˜ΑfڍÄ«āæ«ËÄΑ«ÄΑ樘ΑÞ«αTŽ«ĈŽΑüΑ͉͉͋͋ϐΠΑϏØڣ«Ä£ΑfڍÄΑ"ËÚÃÞΑγΑT˽«ŽüΑW˜Þ×ËÄޘÞΑÄ”Α
#Ëõ˜ÚāϘΑ [æÚêŽæêژϐΑ öÞΑ 樘Α 樘ØΑ Ë¢Α 樘Α ͏æ¨Α Þ«Α TŽ«ĈŽΑ ËÄ¢˜Ú˜ÄŽ˜Α ËÄΑ (ËêÞ«Ä£Α Ä”Α fڍÄΑ
˜õ˜½Ë×ØÄæΑ¨˜½”Α«ÄΑ˜Ž˜Ã˜ÚΑ͉͋͊͏ΠΑHence statement 1 is not correct.
͋ĔΑ TA(fΑ öÞΑ ¨˜½”Α «ÄΑ b˜¨ÚÄΡΑ +Þ½Ã«ŽΑ W˜×ꍽ«ŽΑ Ë¢Α +ځÄΡΑ ͌ڔΑ «ÄΑ [˽ËΡΑ W˜×ꍽ«ŽΑ Ë¢Α +ĔËĘޫΡΑ ͍æ¨Α «ÄΑ
ÃÁÄΡΑ(Þ¨˜Ã«æ˜Α9«Ä£”ËÃΑË¢Α6ËڔÄΑÄ”Α͎æ¨Α«ÄΑ[˜Ëê½ΡΑW˜×ꍽ«ŽΑË¢Α9ËژΠΑHence statement 2 is not
correct.
"êÚ樘ÚΡΑTA(fΑ«ÞΑΑŽËÄÞê½ææ«õ˜Α؎¨Ä«ÞÃΑËÄΑ樘Α×ÚËÃËæ«ËÄΑË¢ΑÞêÞ恫ā½˜Α”˜õ˜½Ë×ØÄæΑË¢Α(ËêÞ«Ä£Α
Ä”Α fڍÄΑ ˜õ˜½Ë×ØÄæΑ «ÄΑ 樘Α Þ«αTŽ«ĈŽΑ W˜£«ËÄΑ ˜Þ恍½«Þ¨˜”Α êĔ˜ÚΑ 樘Α ˜£«ÞΑ Ä”Α Þê××ËÚæΑ Ë¢Α
fBα(«ææΠ

ANALYSIS:

Scope for elimination

b¨˜Ú˜Α«ÞΑÄËΑގËטΑ¢ËÚΑ˜½«Ã«Äæ«ËÄΑæ¨Ë꣨ΑÞææ˜Ã˜ÄæΑ͋ΑÞËêĔÞΑÞ½«£¨æ½üΑêč˜½«˜õ½˜ΑÞΑËĘΑŽËêÄæÚüΑŽÄÄËæΑ
¨ËÞæΑ½½Α樘ΑÄÄꁽΑŽËÄ¢˜Ú˜ÄŽ˜ÞΡΑêæΑ«æΑÞ櫽½ΑŽÚÚ«˜ÞΑŽ˜Ú恫ÄΑÃËêÄæΑË¢ΑÚ«ÞºΠΑ[ꎨΑÙê˜Þæ«ËÄÞΑŽÄΑ˜Α˜Þ«½üΑ½˜¢æΑ
êāÄÞö˜Ú˜”Α«¢ΑËĘΑ¨ÞΑÄËΑŽ½ê˜Π

SOURCE : ¨ææ×΢έέèê×Π£ËõΠ«ÄέöÚ«æ˜Ú˜””æέ͏æ¨[«εTŽ«ĈŽεq«£üÄ卨öÄΠה¢

V͐ΠΑr¨«Ž¨ΑË¢Α樘Α¢Ë½½Ëö«Ä£Α£«õ˜ÞΑϏ#½Ë½Α#˜Ä”˜ÚΑ#×Α+Ĕ˜ûϐΑځĺ«Ä£ΑæËΑ樘ΑŽËêÄæÚ«˜ÞΑË¢Α樘ΑöËÚ½”Χ
(a) World Economic Forum
(b) UN Human Rights Council
(c) UN Women
(d) World Health Organization

ANSWERS A
82

EXPLANATION
#½Ë½Α#˜Ä”˜ÚΑ#×Α+Ĕ˜ûΑ«ÞΑژ½˜Þ˜”ΑüΑrËÚ½”ΑŽËÄËëŽΑ"ËÚêÃΠΑÞΑטÚΑ樘Α#½Ë½Α#˜Ä”˜ÚΑ#×ΑW˜×ËÚæΑ͉͋͊͏ΡΑ
+Ĕ«ΑځĺÞΑæΑ͑͐æ¨Α«ÄΑژÞטŽæΑË¢Α#½Ë½Α#˜Ä”˜ÚΑ#×Α+Ĕ˜ûΑλ##+μΑÃËÄ£Α͍͍͊ΑŽËêÄæÚ«˜ÞΑË¢Α樘ΑrËÚ½”ΠΑÞΑטÚΑ
樘Α##WΡΑ+Ĕ«ΑŽ½«Ã˜”Α͋͊ΑÞ×ËæÞΑæËΑځĺΑ͑͐æ¨Α«ÄΑ͉͋͊͏ΡΑö¨«Ž¨Α«ÞΑÄΑ«Ã×ÚËõ˜Ã˜ÄæΑ¢ÚËÃΑ˜«Ä£Αځ嘔ΑæΑ͉͊͑æ¨Α
«ÄΑ͉͎͋͊Π

ANALYSIS:
ANALYSIS:

SOURCE : ¨ææ×΢έέ׫ΠÄ«ŽΠ«ÄέĘöÞ«æ˜έTÚ«ÄæW˜½˜Þ˜ΠÞ×ûΧژ½«”Ͻ͎͎͉͊͊͌

Motivation
India's performance in Global Gender Gap Report has always been discussed and also it was one of the important
global indicators alongside HDI, MPI Index, Ease of doing business (EoDB) etc.

Additional Information

ORGANISATION REPORT
WEF (World Economic Forum) 1. Global Competitiveness Report (GCR)
2. Global Gender Gap Report
3. Travel and Tourism Competitiveness Report
4. Global Information Technology Report

UNCTAD (United Nations Conference on Trade 1. World Investment Report


and Development)

IMF (International Monetary Fund) 1. World Economic Outlook


2. Global Financial Stability Report
3. Fiscal Monitor

Financial Action Task Force 1. Global Money Laundering Report

WIPO (World Intellectual Property Organization) 1. World Intellectual Property Report (WIPR)

World Bank 1. Ease of Doing Business


2. World Development Report

International Labour Organisation (ILO) 1. World Social Protection Report Global


2. Wage Report
3. World Employment and Social Outlook
4. World of Work Report

United Nations Development Programme 1. Human Development Report


(UNDP)
83

Year 2017 WTO (GOODS ACT)

V͑ΠΑΑ+Ĕ«Α˜ÄŽæ˜”Α樘Α#˜Ë£Ú×¨«Ž½Α+Ĕ«Žæ«ËÄÞΑË¢Α#Ë˔ÞΑλW˜£«Þæځæ«ËÄΑÄ”ΑTÚË明æ«ËÄμΑŽæΡΑ͊͒͒͒Α«ÄΑ
order to comply with the obligations to
(a) ILO
(b) IMF
(c) UNCTAD
(d) WTO

ANSWERS D

EXPLANATION

#˜Ë£Ú×¨«Ž½Α +Ĕ«Žæ«ËÄÞΑ Ë¢Α #Ë˔ÞΑ Ú˜Α ”˜ĈʔΑ ÞΑ 樁æΑ ÞטŽæΑ Ë¢Α «Ä”êÞæÚ«½Α ×ÚËטÚæüΑ ö¨«Ž¨Α ژ¢˜ÚΑ æËΑ 樘Α
£˜Ë£Ú×¨«Ž½Α«Ä”«Žæ«ËÄΑژ¢˜ÚÚ«Ä£ΑæËΑΑŽËêÄæÚüΑËÚΑæËΑΑ×½Ž˜ΑÞ«æêæ˜”Α樘ژ«ÄΑÞΑ˜«Ä£Α樘ΑŽËêÄæÚüΑËÚΑ×½Ž˜Α
Ë¢ΑËÚ«£«ÄΑË¢Α樁æΑ×Ú˔êŽæΠ Typically, such a name conveys an assurance of quality and distinctiveness
ö¨«Ž¨Α«ÞΑ˜ÞޘÄ櫁½½üΑææÚ«ê恍½˜ΑæËΑ樘Α¢ŽæΑË¢Α«æÞΑËÚ«£«ÄΑ«ÄΑ樁æΑ”˜ĈʔΑ£˜Ë£Ú×¨«Ž½Α½ËŽ½«æüΡΑژ£«ËÄΑËÚΑ
country. fĔ˜ÚΑ Ú櫎½˜ÞΑ ͊Α λ͋μΑ Ä”Α ͉͊Α Ë¢Α 樘Α TÚ«ÞΑ ËÄõ˜Äæ«ËÄΑ ¢ËÚΑ 樘Α TÚË明æ«ËÄΑ Ë¢Α +ĔêÞæÚ«½Α TÚËטÚæüΡΑ
£˜Ë£Ú×¨«Ž½Α«Ä”«Žæ«ËÄÞΑÚ˜ΑŽËõ˜Ú˜”ΑÞΑÄΑ˜½˜Ã˜ÄæΑË¢Α+TWÞΠΑb¨˜üΑÚ˜Α½ÞËΑŽËõ˜Ú˜”ΑêĔ˜ÚΑÚ櫎½˜ÞΑ͋͋ΑæËΑ͍͋Α
Ë¢Α樘Αbځ”˜Α W˜½æ˜”ΑÞטŽæÞΑ Ë¢Α +Ä昽½˜ŽæꁽΑ TÚËטÚæüΑ W«£¨æÞΑ λbW+T[μΑ£Ú˜˜Ã˜ÄæΡΑö¨«Ž¨ΑöÞΑ ׁÚæΑ Ë¢Α樘Α
£Ú˜˜Ã˜ÄæÞΑŽËĎ½ê”«Ä£Α樘ΑfÚê£êüΑWËêĔΑË¢Α#bbΑĘ£Ë櫁æ«ËÄÞΠΑ+Ĕ«ΡΑÞΑΑØ͘ÚΑË¢Α樘ΑrËÚ½”Αbځ”˜Α
HÚ£Ä«āæ«ËÄΑλrbHμΡΑ˜ÄŽæ˜”Α樘Α#˜Ë£Ú×¨«Ž½Α+Ĕ«Žæ«ËÄÞΑË¢Α#Ë˔ÞΑλW˜£«Þæځæ«ËÄΑϞΑTÚË明æ«ËÄμŽæΡΑ͊͒͒͒Α
¨ÞΑŽËØΑ«ÄæËΑ¢Ëڎ˜Αö«æ¨Α˜ą˜ŽæΑ¢ÚËÃΑ͎͊æ¨Α[˜×æ˜Ã˜ÚΑ͉͉͋͌ΠΑHence option (d) is the correct answer.

ANALYSIS:

#+Α恣£«Ä£ΑË¢Α+Ĕ«ÄΑ£Ë˔ÞΑ¨ÞΑ½öüÞΑ˜˜ÄΑ«ÄΑ樘ΑĘöÞΠΑb¨Ë꣨Α#+Α恣£˜”Α×Ú˔êŽæÞΑÚ˜ΑŽËõ˜Ú˜”ΑêĔ˜ÚΑ
+Ĕ«ÄΑ ÚæΑ Ä”Α ê½æêژΡΑ +TWΑ ŽËØÞΑ êĔ˜ÚΑ ŽËÄËëŽÞΠΑ b¨«ÞΑ Ùê˜Þæ«ËÄΑ «ÞΑ õ˜ÚüΑ Þ«ŽΑ «ÄΑ āæêژΑ Ä”Α «æΑ «ÞΑ Α
ژŽêÚÚ«Ä£Α樘ØΑ«ÄΑ樘ΑĘöÞׁטÚÞΠ
T½˜Þ˜ΑĈĔΑ˜½ËöΑÄΑ˜ûŽ˜Ú×æΑ¢ÚËÃΑb¨˜Α(«Ä”ê
84

Year 2019 AIIB

V͒ΠΑr«æ¨Αژ¢˜Ú˜ÄŽ˜ΑæËΑÞ«ÄΑ+ĢځÞæÚêŽæêژΑ+Äõ˜ÞæØÄæΑ ÄºΑλ++ μΡΑŽËÄÞ«”˜ÚΑ樘Α¢Ë½½Ëö«Ä£ΑÞææ˜Ã˜ÄæÞ΢Α


1. AIIB has more than 80 member nations.
2. India is the largest shareholder in AIIB.
3. AIIB does not have any members from outside Asia.
Which of the statements given above is/are correct?
(a) 1 only
(b) 2 and 3 only
(c) 1 and 3 only
(d) 1, 2 and 3

ANSWERS A

EXPLANATION

Statement 1 is correct and statement 3 is not correct:Α +æΑ «ÞΑ Α Ãê½æ«½æ˜Ú½Α ”˜õ˜½Ë×ØÄæΑ ÄºΑ ö«æ¨Α Α
ëÞÞ«ËÄΑ æËΑ «Ã×ÚËõ˜Α Þˎ«½Α Ä”Α ˜ŽËÄËëŽΑ ËêæŽËØÞΑ «ÄΑ Þ«Α Ä”Α ˜üËĔΠΑ êÚژÄæ½üΑ «æΑ ¨ÞΑ ͒͐Α ××ÚËõ˜”Α
Ø͘ÚÞΑö«æ¨Α+Ĕ«ΑÞΑΑ¢ËêĔ«Ä£ΑØ͘ÚΠ
Statement 2 is not correct: +Ĕ«Α«ÞΑ樘ΑޘŽËĔΑ½Ú£˜ÞæΑÞ¨Ú˜¨Ë½”˜ÚΑ«ÄΑ++ Αö«æ¨Α͐Π͎АΑõËæ«Ä£ΑÞ¨Ú˜ÞΑö¨«½˜Α
¨«ÄΑ¨Ë½”ÞΑ͋͏Π͉͏АΑõËæ«Ä£ΑÞ¨Ú˜ÞΑ˜«Ä£Α樘Α½Ú£˜ÞæΠ

ANALYSIS:

b¨˜ΑژÞËÄΑö¨üΑ樫ÞΑÙê˜Þæ«ËÄΑöÞΑÞº˜”ΑöÞΑژŽ˜Äæ½üΡΑ樫ڔΑÄÄꁽΑؘæ«Ä£ΑË¢Α++ ΑöÞΑ¨˜½”Α«ÄΑAế«Α
ö¨˜Ú˜ΑÞ«ÄΑ+ĢځÞæÚêŽæêژΑ"ËÚêÃΑöÞΑ½êϨ˜”ΠΑ
½ÞËΡΑ «æΑ «ÞΑ «Ã×ËÚæÄæΑ æËΑ £ËΑ æ¨ÚË꣨Α 樘Α Þ«ŽΑ «Ä¢ËÚÁæ«ËÄΑ ژ½æ˜”Α æËΑ £½Ë½½üΑ «Ã×ËÚæÄæΑ ĈāĎ«½Α
ËÚ£Ä«āæ«ËÄÞΑ ½«º˜Α 樘Α rËÚ½”Α ÄºΡΑ 樘Α +A"ΡΑ 樘Α rbHΡΑ 樘Α  Α λÞ«ÄΑ ˜õ˜½Ë×ØÄæΑ ÄºμΡΑ 樘Α B˜öΑ
˜õ˜½Ë×ØÄæΑ ÄºΑ덁Žº˜”ΑüΑ W+[μΡΑ樘Α++ ΡΑ˜ÞטŽ«½½üΑæ¨ËޘΑËÚ£Ä«āæ«ËÄÞΑË¢Αö¨«Ž¨Α+Ĕ«Α«ÞΑΑØ͘ÚΠ
Scope for elimination: b¨˜ΑĈÚÞæΑÞææ˜Ã˜ÄæΑÄ”Α樘Α樫ڔΑÞææ˜Ã˜ÄæΑŽËÄæځ”«ŽæΑ˜Ž¨ΑË樘ÚΑÞΑ«¢Α++ Α¨ÞΑ͉͑Α
Ø͘ÚÞΑ樘ÄΑ樘üΑ½½ΑŽÄÄËæΑ˜Α¢ÚËÃΑËêæÞ«”˜ΑÞ«Αλb¨˜Ú˜ΑÚ˜ΑËĽüΑ͍͑ΑŽËêÄæÚ«˜ÞΑ«ÄΑÞ«μΠΑb¨êÞΡΑ͊ΑÄ”Α͌Α
ŽÄÄËæΑ˜ΑËæ¨ΑŽËÚژŽæΠΑ;Ë£«Ž½½üΡΑË×æ«ËÄΑλμΑÄ”ΑË×æ«ËÄΑλμΑŽÄΑ˜Α˜½«Ã«Äæ˜”ΠΑÄË樘ÚΑ½Ë£«ŽΑŽÄΑ˜Α××½«˜”Α
λêæΑ«æΑ¨ÞΑ˜½˜Ã˜ÄæΑË¢ΑÚ«ÞºμΑö¨«Ž¨Α«ÞΑ樘ژΑÚ˜ΑæöËΑÞ«ÄΑ×Ëö˜ÚÞΑ«££˜ÚΑ樁ÄΑ+Ĕ«Αö¨«Ž¨ΑÚ˜Α¨«ÄΑÄ”Α
6×ÄΠΑb¨êÞΡΑ+Ĕ«ΑŽÄÄËæΑ˜Α˜½«˜õ˜”ΑæËΑ˜Α樘Α½Ú£˜ÞæΑÞ¨Ú˜¨Ë½”˜ÚΠΑ(˜ÄŽ˜ΑË×æ«ËÄΑλ μΑŽÄΑ˜Α˜½«Ã«Äæ˜”ΠΑ
r˜ΑÚ˜Α½˜¢æΑö«æ¨ΑËĽüΑËĘΑË×æ«ËÄΑ樁æΑ«ÞΑË×æ«ËÄΑλμΑÄ”Α«æΑ«ÞΑ樘ΑŽËÚژŽæΑÄÞö˜ÚΠΑ

SOURCE : https://www.aiib.org/en/about-aiib/index.html
85

Year 2019 REPORTS

V͉͊ΠΑr¨«Ž¨ΑËĘΑË¢Α樘Α¢Ë½½Ëö«Ä£Α«ÞΑÄËæΑΑÞêα«Ä”˜ûΑË¢Α樘ΑrËÚ½”Α ÄºϐÞΑϏÞ˜ΑË¢ΑË«Ä£Α êޫĘÞÞΑ+Ĕ˜ûϐΧ


(a) Maintenance of law and order
(b) Paying taxes
(c) Registering property
(d) Dealing with construction permits

ANSWERS A

EXPLANATION
Ë«Ä£Α êޫĘÞÞΑ ؁ÞêژÞΑ ژ£ê½æ«ËÄÞΑ
ą˜Žæ«Ä£Α͊͊ΑÚ˜ÞΑË¢Α樘Α½«¢˜ΑË¢ΑΑêޫĘÞÞΠΑ
b˜ÄΑ Ë¢Α 樘ޘΑ Ú˜ÞΑ Ú˜Α «ÄŽ½ê”˜”Α «ÄΑ 樫ÞΑ
Ë«Ä£Α êޫĘÞÞΑ͉͋͊͒Αځĺ«Ä£ΑËÄΑ樘Α˜Þ˜ΑË¢Α
”Ë«Ä£Α êޫĘÞÞ΢Α ÞæÚæ«Ä£Α Α êޫĘÞÞΡΑ
”˜½«Ä£Αö«æ¨ΑŽËÄÞæÚêŽæ«ËÄΑטÚëæÞΡΑ£˜ææ«Ä£Α
˜½˜ŽæÚ«Ž«æüΡΑ ژ£«Þæ˜Ú«Ä£Α ×ÚËטÚæüΡΑ £˜ææ«Ä£Α
ŽÚ˜”«æΡΑ ×ÚË明æ«Ä£Α ëÄËÚ«æüΑ «Äõ˜ÞæËÚÞΡΑ
ׁü«Ä£Α æû˜ÞΡΑ æځ”«Ä£Α ŽÚËÞÞΑ Ëڔ˜ÚÞΡΑ
˜Ä¢Ëڎ«Ä£Α ŽËÄæځŽæÞΑ Ä”Α ژÞ˽õ«Ä£Α
«ÄÞ˽õ˜ÄŽüΠΑ Ë«Ä£Α êޫĘÞÞΑ ½ÞËΑ ؁ÞêژÞΑ
½ËêÚΑ Áں˜æΑ ژ£ê½æ«ËÄΡΑ ö¨«Ž¨Α «ÞΑ ÄËæΑ
«ÄŽ½ê”˜”Α «ÄΑ 樫ÞΑ ü˜ÚϐÞΑ ځĺ«Ä£ΠΑ Hence
option (a) is the correct answer.

ANALYSIS:
ANALYSIS:
b¨˜ΑË«Ä£Α êޫĘÞÞΑW˜×ËÚæΑ͉͋͊͒ΑöÞΑژ½˜Þ˜”ΑژŽ˜Äæ½üΠ

SOURCE : ¨ææ×΢έέöööΠ”Ë«Ä£êޫĘÞÞΠËÚ£έ˜Äέژ×ËÚæÞέ£½Ë½αژ×ËÚæÞέ”Ë«Ä£αêޫĘÞÞα͉͋͊͒

Year 2019 REPORTS

Q11. The Global Competitiveness Report is published by the


(a) International Monetary Fund
(b) United Nations Conference on Trade and Development
(c) World Economic Forum
(d) World Bank

ANSWERS C
86

EXPLANATION
b¨˜Α#½Ë½ΑËÃטæ«æ«õ˜Ä˜ÞÞΑW˜×ËÚæΑλ#WμΑ«ÞΑΑü˜Ú½üΑژ×ËÚæΑ×ꍽ«Þ¨˜”ΑüΑ樘ΑrËÚ½”ΑŽËÄËëŽΑ"ËÚêÃΠΑ

ANALYSIS:

[æꔘÄæÞΑÞ¨Ë꽔ΑŽËõ˜ÚΑ樫ÞΑÞΑׁÚæΑË¢ΑA¶ËÚΑ#½Ë½Α+Ĕ«Ž˜ÞΑ«ÄΑĘöÞΑÄ”ΑŽËÄĘŽæ˜”ΑæËΑ+Ĕ«Π
(˜ÄŽ˜ΑË×æ«ËÄΑλŽμΑ«ÞΑ樘ΑŽËÚژŽæΑÄÞö˜ÚΠ

SOURCE : ¨ææ×Þ΢έέöööΠ樘¨«Ä”êêޫĘÞÞ½«Ä˜ΠŽËÃݘŽËÄËÃüέ«Ä”«αځĺÞα͎͑æ¨αÃËÞæαŽËÃטæ«æ«õ˜αΑ˜ŽËÄËα
Ãüα«Äαö˜¢α«Ä”˜û݁Ú櫎½˜͎͍͍͋͋͏͍͉ИŽ˜

Year 2019 CURRENCY CRISIS

V͊͋ΠΑ+ÄΑ樘ΑŽËÄæ˜ûæΑË¢Α+Ĕ«ΡΑö¨«Ž¨ΑË¢Α樘Α¢Ë½½Ëö«Ä£Α¢ŽæËÚÞΑ«ÞέÚ˜ΑŽËÄæÚ«êæËÚÞݎËÄæÚ«êæËÚÞΑæËΑ
reducing the risk of a currency crisis?
1. The foreign currency earnings of India’s IT sector
2. Increasing the government expenditure
3. Remittances from Indians abroad
Select the correct answer using the code given below.
(a) 1 only
(b) 1 and 3 only
(c) 2 only
(d) 1, 2 and 3

ANSWERS B
Α ŽêÚژĎüΑ ŽÚ«Þ«ÞΑ «ÞΑ ÚË꣨æΑ ËÄΑ üΑ Α ”˜Ž½«Ä˜Α «ÄΑ 樘Α õ½ê˜Α Ë¢Α Α ŽËêÄæÚüϖÞΑ ŽêÚژĎüΠΑ b¨«ÞΑ ”˜Ž½«Ä˜Α «ÄΑ õ½ê˜Α
Ę£æ«õ˜½üΑ ą˜ŽæÞΑ ÄΑ ˜ŽËÄËÃüΑ üΑ ŽÚ˜æ«Ä£Α «ÄÞ恍«½«æ«˜ÞΑ «ÄΑ ˜ûŽ¨Ä£˜Α ځæ˜ÞΡΑ ؁īģΑ 樁æΑ ËĘΑ êÄ«æΑ Ë¢Α Α
Ž˜Ú恫ÄΑŽêÚژĎüΑÄËΑ½ËÄ£˜ÚΑêüÞΑÞΑÃꎨΑÞΑ«æΑêޘ”ΑæËΑ«ÄΑÄË樘ÚΑŽêÚژĎü

Α ÞêÞæÄ櫁½Α ÃËêÄæΑ Ë¢Α ¢Ëژ«£ÄΑ ˜ûŽ¨Ä£˜Α ژޘÚõ˜ÞΑ ŽÄΑ ¨˜½×Α æËΑ ŽêÞ¨«ËÄΑ £«ÄÞæΑ ÄüΑ Ú«ÞºÞΑ Ë¢Α ŽêÚژĎüΑ
crisis.
b¨˜Α¢Ëژ«£ÄΑŽêÚژĎüΑ˜ÚÄ«Ä£ÞΑË¢Α+Ĕ«ϖÞΑ+bΑޘŽæËÚΑÄ”ΑژëææÄŽ˜ÞΑ¢ÚËÃΑÚˁ”ΑöË꽔Α½˜”ΑæËΑÃËژΑ«ÄĉËöΑ
Ë¢Α¢Ëژ«£ÄΑŽêÚژĎ«˜ÞΑ«ÄΑ樘Α˜ŽËÄËÃüΑÄ”ΑËËÞæΑ樘Α¢Ëژ«£ÄΑ˜ûŽ¨Ä£˜ΑژޘÚõ˜ÞΠ Hence statements 1 and
3 are correct.
Statement 2 is not correct ÞΑ«ÄŽÚ˜Þ«Ä£Α樘Α£Ëõ˜ÚÄØÄæΑ˜ûטĔ«æêژΑ«ÞΑÄËæΑژ½æ˜”ΑæËΑŽ¨Ä£˜ÞΑ«ÄΑ¢Ëژ«£ÄΑ
˜ûŽ¨Ä£˜ΑژޘÚõ˜ÞΑËÚΑÄüΑŽêÚژĎüΑĉêŽæêæ«ËÄÞΠ
ANALYSIS:
ANALYSIS:

Scope for elimination


r«æ¨Α ËêÚΑ £˜Ä˜Ú½Α êĔ˜ÚÞæÄ”«Ä£Α ö˜Α ŽÄΑ «Ä¢˜ÚΑ 樁æΑ «¢Α £ËõæΑ «ÄŽÚ˜Þ˜ÞΑ 樘Α ˜ûטĔ«æêژΑ 樘ÄΑ 樘Α õ½ê˜Α Ë¢Α
ÃËĘüΑö«½½Α”˜ŽÚ˜Þ˜ΑÞΑÞê××½üΑ«ÞΑÃËژΡΑÞËΑ¨˜Ú˜Α樘Α͋ĔΑÞææ˜Ã˜ÄæΑ«ÞΑŽËÄæځ”«Žæ«Ä£Αö«æ¨Α樘ΑÞ˽êæ«ËÄΑ樘Α
Ùê˜Þæ«ËÄΑ«ÞΑÞº«Ä£ΠΑΑ[ËΑ樘ΑޘŽËĔΑÞææ˜Ã˜ÄæΑŽÄΑ˜Α˜½«Ã«Äæ˜”Π

SOURCE : AŽÚ˘ŽËÄËëŽÞΑλBWbΑ½ÞÞΑ͊͋μ
87

Year 2019 PPP

Q13. Consider the following statements:


1. Purchasing Power Parity (PPP) exchange rates are calculated by comparing the prices of the same basket of goods
and services in different countries.
2. In terms of PPP dollars, India is the sixth-largest economy in the world.
Which of the statements given above is/are correct?
(a) 1 only
(b) 2 only
(c) Both 1 and 2
(d) Neither 1 nor 2

ANSWERS A

EXPLANATION
In purchasing power parity (PPP) exchange rate — the rate at which the currency of one country would
have to be converted into that of another country to buy the same amount of goods and services in each
country. For example-If a burger is selling in India for Rs100 and in New York for $2, this would imply a PPP
exchange rate of 1 dollar to Rs50. Hence statement 1 is correct.
On the PPP basis, China is world's largest economy in 2018. Total wealth of china is estimated at 25.3
æÚ«½½«ËÄΑ «Äæ˜Úāæ«Ëā½Α ”˽½ÚΠΑ ¨«ÄΑ «ÞΑ ¢Ë½½Ëö˜”Α üΑ fī昔Α [ææ˜ÞΑ ö«æ¨Α Ĉ£êژΑ ͊͒Π͍Α æÚ«½½«ËÄΠΑ +Ĕ«Α «ÞΑ 樘Α 樫ڔΑ
largest economy , in terms of PPP dollars. Hence, Statement 2 is not correct.

ANALYSIS:
PPP should be covered under basic of economics, it is a very important topic and theme is also repetitive.
Scope for elimination: If one knows the simple fact that in terms of PPP, India’s rank is number 3, then 2
Ë×æ«ËÄÞΑŽÄΑ˜Α˜½«Ã«Äæ˜”ΠΑ½ÞËΡΑ樘Α”˜ĈÄ«æ«ËÄΑË¢ΑTTTΑ«ÞΑΑÞ«ŽΑ¢ŽæΠ

SOURCE : https://www.businesstoday.in/current/economy-politics/india-pips-japan-in-gdp-based
-on-purchasing-power-parity/story/282226.html
88

Year 2020 FINANCIAL CRISIS

V͍͊ΠΑΑ+¢ΑÄË樘ÚΑ£½Ë½ΑĈāĎ«½ΑŽÚ«Þ«ÞΑ¨×טÄÞΑ«ÄΑ樘ΑʁÚΑ¢êæêژΡΑö¨«Ž¨ΑË¢Α樘Α¢Ë½½Ëö«Ä£Α
action/policies are most likely to give some immunity to India?
1. Not depending on short-term foreign borrowings
2. Opening up to more foreign banks
3. Maintaining full capital account convertibility
Select the correct answer using the code given below:
(a) 1 only
(b) 1 and 2 only
(c) 3 only
(d) 1, 2 and 3

ANSWERS A

EXPLANATION

+¢ΑΑ£½Ë½ΑĈāĎ«½ΑŽÚ«Þ«ÞΑ¨×טÄÞΑ«ÄΑ樘Α¢êæêژΡΑ½˜ÞޘÚΑ˜û×ËÞêژΑæËΑ樘Α¢Ëژ«£ÄΑĈāĎ«½ΑÁں˜æÞΑ«ÞΑ½«º˜½üΑæËΑ
give some immunity to India. Hence option 1 is correct:
Short-term borrowings would lead to the burden of paying back the debt, and could result in stressful
conditions for the borrowing economy/ India. Hence option 2 is not correct.
Opening up to more foreign banks would lead to an enhanced exposure to the global economy, and hence
an increased risk. Hence option 3 is not correct.
Currency convertibility refers to a situation in which a currency can be converted into a foreign currency,
and vice-versa at the prevailing exchange rate without any government intervention. In India, we cannot
completely convert rupees to dollars as there are restrictions to the same.
Now, capital account convertibility is the freedom to convert domestic currency into a foreign currency,
and vice -versa wrt capital account transactions of the Balance of Payments accounts. It could also be the
¢Ú˜˜”ËÃΑæËΑŽËÄõ˜ÚæΑ”ËØÞ櫎ΑĈāĎ«½ΑÞޘæÞΑ뽫º˜ΑÚêט˜ÞμΑέΑ½««½«æ«˜ÞΑ«ÄæËΑ¢Ëژ«£ÄΑÞޘæÞΑέΑ½««½«æ«˜ÞΑÄ”Α
vice -versa. It is more risky, as the foreign investors can withdraw all their money at once which called
Ž×«æ½Αĉ«£¨æΠΑ
Risks associated with full capital account convertibility:
It increases the vulnerability of the domestic economy to external economic shocks. Hence the economy
becomes unstable. For example, during the American recession - countries that were more connected
ö˜Ú˜Αą˜Žæ˜”ΑÃËژΠΑ
Flight of capital: i.e. the withdrawal of huge foreign exchange within a short period of time. If full
ŽËÄõ˜Ú櫍«½«æüΑ «ÞΑ ÄËæΑ 樘ژΡΑ 樘Α W +Α ŽÄΑ Ž¨˜ŽºΑ 樘Α ŽËÃ×½˜æ˜Α ö«æ¨”ځö½Α Ë¢Α ¢êĔÞΠΑ λb¨«ÞΑ ĉ«£¨æΑ Ë¢Α Ž×«æ½Α
¨×טʔΑ«ÄΑ樘Α[Ëêæ¨ΑÞæΑÞ«ÄΑŽÚ«Þ«ÞΑË¢Α͊͒͒͐ΠΑ"ËÚΑ樘ޘΑÞ«ÄΑæ«£˜ÚÞΑαΑ樘Α¢Þæ˜ÞæΑ£ÚËö«Ä£Α˜ŽËÄËë˜ÞΑË¢Α
樘ΑöËÚ½”ΑαΑ樘ΑŽêÚژÄæΑŽŽËêÄæΑöÞΑ«ÄΑΑ”˜ĈŽ«æΡΑêæΑ樘ژΑöÞΑÞ«£Ä«ĈŽÄæΑ«ÄĉËöΑ«ÄΑ樘«ÚΑŽ×«æ½ΑŽŽËêÄæÞΠΑ
However, as interest rates in America increased, people withdrew their money and foreign exchange
began to deplete. The people began to fear that they would not be able to take their money out. Out of this
fear, everybody started to withdraw their investments- FDI was also withdrawn and the recession set in-
they collapsed in one go!)
+æΑ«ÄŽÚ˜Þ˜ÞΑõ˽æ«½«æüΑ«ÄΑ樘Α”ËØÞ櫎ΑĈāĎ«½ΑÁں˜æÞΑ
The capital Account is potentially more volatile than the current account- People cannot start
consuming double/ triple/ four times the previous amount. But investment in shares accumulates, and
the entire money.
89

ANALYSIS:

b¨˜Α#½Ë½Α"«ÄÄŽ«½ΑÚ«Þ«ÞΑÄ”Α樘ΑÚ«××½˜Α˜ą˜ŽæΑö˜Ú˜ΑŽËÄÞ«”˜Ú˜”ΑæËΑ˜ΑΑĘ£æ«õ˜Α½Ä”ÁںΑ«ÄΑ樘ΑöËÚ½”Α
-politico - economic history. Scope for Elimination: In this question, we need to understand that less
˜û×ËÞêژΑæËΑ£½Ë½ΑĈāĎ«½ΑÁں˜æÞΑö«½½Α˜ÄÞêژΑÞËØΑ«ÃÃêÄ«æüΑæËΑ+Ĕ«ΑγΑ+ÄΑ樫ÞΑŽÞ˜ΡΑË×æ«ËÄÞΑ͋ΑÄ”Α͌Α(refer
to more exposure) are opposite to option 1 (as it highlights less exposure), therefore, statement 1 cannot
come together with statement 2 and 3, hence we can easily eliminate options (b) and (c), and option (a) will
be the answer.

SOURCE : Macroeconomics (NCERT Class 12) can be referred to for some help

Year 2020 IMF (GOLD TRANCHE)

Q15. "Gold Tranche" (Reserve Tranche) refers to


(a) a loan system of the World Bank
(b) one of the operations of a Central Bank
(c) a credit system granted by WTO to its members
(d) a credit system granted by IMF to its members

ANSWERS D

EXPLANATION
A reserve tranche is a portion of the required quota of currency each member country must provide to
the International Monetary Fund (IMF) that can be utilized for its own purposes without a service fee
or economic reform conditions.
The IMF is funded through its members and their quota contributions. The reserve tranche is basically an
emergency account that IMF members can access at any time without agreeing to conditions or paying a
service fee.
b¨˜ΑژޘÚõ˜ΑæځϨ˜ÞΑ樁æΑŽËêÄæÚ«˜ÞΑ¨Ë½”Αö«æ¨Α樘Α+A"ΑÚ˜ΑŽËÄÞ«”˜Ú˜”Α樘«ÚΑ¢Ž«½«æ«˜ÞΑË¢ΑĈÚÞæΑژÞËÚæΡΑ؁īģΑ
they will tap into the reserve tranche at a short notice before seeking a formal credit tranche. The reserve
tranche represents the member’s unconditional drawing right on the IMF, created by the foreign exchange
portion of the quota subscription, plus increase (decrease) through the IMF’s sale (repurchase) of the
member’s currency to meet the demand for use of IMF resources by other members in need of balance of
ׁüØÄæÞΑĈāĎ«Ä£Π

ANALYSIS:
This topic remains has always remained in the news. Hence, with respect to international bodies, this
question should have been prepared.

Scope for elimination


No scope for elimination as it is a direct question and in such question, we either know the answer or don’t
know the answer.

SOURCE : ¨ææ×Þ΢έέöööΠ«Ã¢ΠËÚ£έ˜ûæ˜Úā½έ×êÞέ¢æݍË×έ͉͋͊͑έה¢έ½Ú«ĈŽæ«ËÄ͉͋͊͑Πה¢
90

Year 2020 BILATERAL TRADE

Q16. Consider the following statements:


1. The value of Indo-Sri Lanka trade has consistently increased in the last decade.
2. "Textile and textile articles" constitute an important item of trade between India and Bangladesh.
3. In the last five years, Nepal has been the largest trading partner of India in South Asia.
Which of the statements given above is/are correct?
(a) 1 and 2 only
(b) 2 only
(c) 3 only
(d) 1, 2 and 3

ANSWERS B

EXPLANATION

«½æ˜Ú½Α æځ”˜Α ˜æö˜˜ÄΑ +Ĕ«Α Ä”Α [Ú«Α ;ÄºΑ ¨ÞΑ «ÄŽÚ˜Þ˜”Α üΑ ÚËêĔΑ ͒Α æ«Ã˜ÞΑ ˜æö˜˜ÄΑ ͉͉͉͋α͉͊Α Ä”Α
͉͋͊͑α͊͒ΠΑbË恽Αæځ”˜Α˜æö˜˜ÄΑ樘ΑæöËΑŽËêÄæÚ«˜ÞΑöÞΑf[ϮΑ͏Π͋Α«½½«ËÄΑ«ÄΑ͉͋͊͑α͊͒ΡΑËêæΑË¢Αö¨«Ž¨Α+Ĕ«ϐÞΑ˜û×ËÚæÞΑ
æËΑ[Ú«Α;ÄºΑö˜Ú˜Αf[ϮΑ͍Π͐Α«½½«ËÄΑÄ”Α«Ã×ËÚæÞΑö˜Ú˜Αf[ϮΑ͊Π͎Α«½½«ËÄΠΑ½æ¨Ë꣨Α+Ĕ«Α¨ÞΑ½öüÞΑ¨”ΑΑæځ”˜Α
ÞêÚ×½êÞΑö«æ¨Α[Ú«Α;ÄºΡΑ樘Α£ÞΑ¨ÞΑö«”˜Ä˜”ΑޫϘΑ͉͉͋͑α͉͒ΠΑ+ÄΑ͉͋͊͋α͊͌ΑÄ”Α͉͋͊͏α͊͐Α樘Αæځ”˜ΑÞ½êÃט”ΡΑ
thus disturbing the steady increase in the graph. Hence statement 1 is not correct.
Bangladesh is India’s biggest trade partner in South Asia. Bilateral trade between India and Bangladesh
¨ÞΑ£ÚËöÄΑÞ昁”«½üΑËõ˜ÚΑ樘Α½ÞæΑ”˜Ž”˜ΠΑ+Ĕ«ϐÞΑ˜û×ËÚæÞΑæËΑ Ä£½”˜Þ¨Α«ÄΑ"xΑ͉͋͊͑α͊͒ΑÞæË˔ΑæΑϮ͒Π͋͊Α«½½«ËÄΑ
Ä”Α«Ã×ËÚæÞΑ”êÚ«Ä£Α樘ΑށØΑטګ˔ΑöÞΑæΑϮ͊Π͉͍Α«½½«ËÄΠ Hence statement 3 is not correct.
According to the World Bank, India exports $2.25 billion-worth textile and clothing products to
Ä£½”˜Þ¨ΠΑ +ÄΑ æêÚÄΡΑ «æΑ «Ã×ËÚæÞΑ Ϯ͌͌͏Α ë½½«ËÄαöËÚæ¨Α æ˜û櫽˜Α Ä”Α Ž½Ë樫ģΑ ×Ú˔êŽæÞΑ ¢ÚËÃΑ ¨ºΠΑ Hence
statement 2 is correct.

ANALYSIS:
ANALYSIS:

Scope for elimination

One could have applied a simple logic to eliminate ‘statement 1’. It is rare for two countries like India and
Sri Lanka (although neighbours but Sri Lanka’s inclination towards China is well-known and the maritime
issues) to have a bilateral trade relationship that grew every year.Thus the word ‘consistently’ can be used
as a hint to infer that the statement is not correct. Thus options (a) and (d) are eliminated. Statement 2
seems more believable as one must have seen the ‘Made in Bangladesh’ tag in many textile items (again
there is some amount of risk involved). Hence option (b) is the correct answer. But one could be more
sure about the error in statement 3 as India and Nepal relations got embittered during the Nepali
Constitutional Crisis.

SOURCE : https://diplomatist.com/2020/03/03/india-sri-lanka-trade-relations/
https://economictimes.indiatimes.com/news/economy/foreign-trade/india-plans-to-
enhance-trade-with-bangladesh/articleshow/76881534.cms
91

Year 2020 FDI

V͊͐ΠΑr«æ¨Αژ¢˜Ú˜ÄŽ˜ΑæËΑ"Ëژ«£ÄΑ«Ú˜ŽæΑ+Äõ˜ÞæØÄæΑ«ÄΑ+Ĕ«ΡΑö¨«Ž¨ΑËĘΑË¢Α樘Α¢Ë½½Ëö«Ä£Α«ÞΑŽËÄÞ«”˜Ú˜”Α«æÞΑ
major characteristic?
(a) It is the investment through capital instruments essentially in a listed company.
(b) It is a largely non-debt creating capital flow.
(c) It is the investment which involves debt-servicing.
(d) It is the investment made by foreign institutional investors in the Government Securities.

ANSWERS B

EXPLANATION
Foreign Direct Investment (FDI) is the investment by a non-resident entity/person resident outside India
in the capital of an Indian company under Foreign Exchange Management (Transfer or Issue of Security by
ΑT˜ÚÞËÄΑW˜Þ«”˜ÄæΑHêæÞ«”˜Α+Ĕ«μΑW˜£ê½æ«ËÄÞΡΑ͉͋͊͐ΠΑb¨˜Α«Äõ˜ÞæØÄæΑ«ÞΑ”ËĘΑæ¨ÚË꣨ΑŽ×«æ½Α«ÄÞæÚêØÄæÞΑ«ÄΑ
(1) an unlisted Indian company; or (2) 10% or more of the post issue paid-up equity capital on a fully diluted
basis of a listed Indian company. Hence, option a is incorrect.
The investment can be made in equities or equity linked instruments or debt instruments issued by the
company. Thus, FDI isn’t directly associated with government securities, and hence option (d) is
incorrect.
Generally, FDI takes place when an investor establishes foreign business operations or acquires foreign
business assets, including establishing ownership or controlling interest in a foreign company
(investments linked with equities), transfer of technology. This means they aren’t just bringing money with
them, but also knowledge, skills and technology. Debt servicing is the regular repayment of interest and
principal on a debt for a particular period. Hence option (c) is incorrect.
ΑÄËÄα”˜æΑŽÚ˜æ«Ä£ΑŽ×«æ½ΑĉËöΑ«ÞΑ樘ΑËĘΑö¨˜Ú˜Α樘ژΑ«ÞΑÄËΑ”«Ú˜ŽæΑژׁüØÄæΑˍ½«£æ«ËÄΑ¢ËÚΑ樘Αژޫ”˜ÄæÞΠΑ
"+Α«ÞΑ½Ú£˜½üΑΑÄËÄα”˜æΑŽÚ˜æ«Ä£ΑŽ×«æ½ΑĉËöΡΑÄ”Αtherefore option (b) is correct.

ANALYSIS:
ANALYSIS:
It is again a basic term which is used in common day to day discussions on Economy. Hence, it is one of the
most simple terms that any student must know.

SOURCE : https://www.investopedia.com/terms/f/fdi.asp
92

Year 2020 TRIMS

V͊͑ΠΑr«æ¨Α ژ¢˜Ú˜ÄŽ˜ΑæËΑbځ”˜αW˜½æ˜”Α +Äõ˜ÞæØÄæΑ A˜ÞêژÞΑ λbW+A[μΡΑö¨«Ž¨Α Ë¢Α樘Α¢Ë½½Ëö«Ä£Α Þææ˜α


ments is/are correct?
1. Quantitative restrictions on imports by foreign investors are prohibited.
2. They apply to investment measures related to trade in both goods and services.
3. They are not concerned with the regulation of foreign investment.
Select the correct answer using the code given below:
(a) 1 and 2 only
(b) 2 only
(c) 1 and 3 only
(d) 1, 2 and 3

ANSWERS C

EXPLANATION

Under the Agreement on Trade - Related Investment Measures of the World Trade Organization (WTO),
(TRIMs Agreement), WTO members have agreed not to apply certain investment measures related to trade
in goods that restrict or distort trade. (TRIMs Agreement is a multilateral agreement on trade in goods, and
not services). Hence statement 2 is not correct.
The TRIMs Agreement prohibits certain measures that violate the national treatment (Article III) and
ÙêÄæ«ææ«õ˜ΑژÞæÚ«Žæ«ËÄÞΑژÙê«Ú˜Ã˜ÄæÞΑλÚ櫎½˜Αw+μΑË¢Α樘Α#˜Ä˜Ú½Α£Ú˜˜Ã˜ÄæΑËÄΑbÚ«ąÞΑÄ”Αbځ”˜Αλ#bbμΑ
͍͊͒͒ΠΑb¨˜Α½«ÞæΑË¢ΑbW+AÞΑ£Ú˜˜”ΑæËΑ˜Α«ÄŽËÄÞ«Þæ˜ÄæΑö«æ¨Α樘ޘΑÚ櫎½˜ÞΑ«ÄŽ½ê”˜ÞΑ؁ÞêژÞΑö¨«Ž¨ΑژÙê«Ú˜΢ΑαΑ
particular levels of local procurement by an enterprise (“local content requirements”), - restrict the
volume or value of imports such an enterprise can purchase, - use to an amount related to the level of
products it exports (“trade balancing requirements”). Hence statement 1 is correct.
TRIMs Agreement stipulates that certain measures adopted by Governments to regulate FDI can cause
æځ”˜αژÞæÚ«Žæ«õ˜ΑÄ”Α”«ÞæËÚæ«Ä£Α˜ą˜ŽæÞΠΑ(Ëö˜õ˜ÚΡΑ樘Α£Ú˜˜Ã˜ÄæΑ«ÞΑËĽüΑŽËϘÚʔΑö«æ¨Α樘Αæځ”˜Α˜ą˜ŽæÞΑ
of investment measures. It is not intended to deal with the regulation of investment as such and does not
impact directly on WTO members’ ability to regulate and place conditions upon the entry and
establishment of foreign investment. Hence statement 3 is correct

ANALYSIS:

SOURCE : https://www.wto.org/english/tratop_e/invest_e/trims_e.htm
93

Year 2020 WEST TEXAS INTERMEDIATE

V͊͒ΠΑb¨˜Αæ˜ÚÃΑϖr˜ÞæΑb˜ûÞΑ+Äæ˜Úؔ«æ˜ϖΡΑÞËØæ«Ã˜ÞΑ¢ËêĔΑ«ÄΑĘöÞΡΑژ¢˜ÚÞΑæËΑΑ£Ú”˜ΑË¢Α
(a) Crude oil
(b) Bullion
(c) Rare earth elements
(d) Uranium

ANSWERS A

EXPLANATION

r˜ÞæΑ b˜ûÞΑ +Äæ˜Úؔ«æ˜Α λrb+μΑ ŽÚꔘΑ Ë«½Α «ÞΑ Α ÞטŽ«ĈŽΑ £Ú”˜Α Ë¢Α ŽÚꔘΑ Ë«½Α Ä”Α ËĘΑ Ë¢Α 樘Α Á«ÄΑ æ¨Ú˜˜Α
benchmarks in oil pricing, along with Brent and Dubai Crude. WTI is known as a light sweet oil because it
ŽËÄ恫ÄÞΑ͉Π͍͋АΑÞê½¢êÚΑÄ”Α¨ÞΑΑ½ËöΑ”˜ÄÞ«æüΡΑÁº«Ä£Α«æΑϕ½«£¨æΠϕΑHence option (a) is the correct answer.

ANALYSIS:

SOURCE : https://www.investopedia.com/terms/w/wti.asp

Year 2021 FDI

Q20. Consider the following:


1. Foreign currency convertible bonds
2. Foreign institutional investment with certain conditions
3. Global depository receipts
4. Non-resident external deposits
Which of the above can be included in Foreign Direct Investments?
(a) 1, 2 and 3
(b) 3 only
(c) 2 and 4
(d) 1 and 4

ANSWERS A
94

EXPLANATION
In Capital Account of Balance of Payment, we can classify into Investment, Borrowings and External
ÞÞ«ÞæÄŽ˜ΠΑ+Äõ˜ÞæØÄæΑ«ÄŽ½ê”˜ÞΑÙê«æüΑĉËöΑ«ÄΑ樘Α˜ŽËÄËÃüΠΑ"Ëژ«£ÄΑêÚژĎüΑËÄõ˜Ú櫍½˜Α ËĔÞΑλ" μΡΑ
"Ëژ«£ÄΑ+ÄÞæ«æêæ«Ëā½Α+Äõ˜ÞæØÄæΑö«æ¨ΑŽ˜Ú恫ÄΑŽËĔ«æ«ËÄÞΑλÞꍶ˜ŽæΑæËΑ樘ΑËõ˜Ú½½Α½«Ã«æΑË¢Α͍͋АμΡΑÄ”Α#½Ë½Α
Depository Receipts (GDR) are the instruments for the foreign investment in India. Hence options 1, 2, and
3 are correct.
BËÄαW˜Þ«”˜ÄæΑ ˜ûæ˜Úā½Α ”˜×ËÞ«æÞΑ Ú˜Α Α Ϗ”˜æΑ ŽÚ˜æ«Ä£ϖΑ ĉËöΑ «ÄΑ ½ÄŽ˜Α Ë¢Α ׁüØÄæÞΑ ŽŽËêÄæÞ and
therefore, not part of Foreign Direct investments. Hence statement 4 is not correct.

ANALYSIS:
ANALYSIS:
Conceptual question to check the basic understanding of terms in Economics.

SOURCE : The question is based on the conceptual understanding of FDI. Hence, it can be covered from
multiple sources including.
Excerpt from NCERT
95

Year 2021 CURRENCY DEVALUATION

Q21. Consider the following statements:


The effect of devaluation of a currency is that it necessarily
1. improves the competitiveness of the domestic exports in the foreign markets.
2. increases the foreign value of domestic currency.
3. improves the trade balance.
Which of the above statements is/are correct?
(a) 1 only
(b) 1 and 2 only
(c) 3 only
(d) 2 and 3 only

ANSWERS A

EXPLANATION

˜õ½êæ«ËÄΑ ؁ÄÞΑ ËƎ«½Α ½Ëö˜Ú«Ä£Α Ë¢Α 樘Α õ½ê˜Α Ë¢Α Α ŽËêÄæÚüϖÞΑ ŽêÚژĎüΑ ö«æ¨«ÄΑ Α Ĉû˜”Α ˜ûŽ¨Ä£˜Α ځæ˜Α
ÞüÞæ˜ÃΠΑ ˜õ½êæ«ËÄΑ Ë¢Α Α ŽêÚژĎüΑ ¨×טÄÞΑ «ÄΑ ŽËêÄæÚ«˜ÞΑö«æ¨Α ΑĈû˜”Α ˜ûŽ¨Ä£˜Α ځæ˜Α λËÚΑ ½ÞËΑö¨˜Ú˜Α «æΑ «ÞΑ
Áā£˜”Αĉˁæ«Ä£Αځæ˜μΠΑûÃ×½˜αΑ;˜æΑêÞΑÞÞêØΑ樁æΑ×ژõ«½«Ä£Α˜ûŽ¨Ä£˜Αځæ˜ΑË¢ΑϮ͊Α«ÞΑ͉͊ΑÚÞΠΑ[ËΑŽêÚژÄæ½üΑ
1 rs is worth of $0.1. If devaluation of currency is done and now echange rate of $1 is 20 rs, this means 1 rs
is worth of $0.05. So the vaule of domestic currency (rs) is decreased in terms of value of foregin currency
($). Hence, statement 2 is not correct.
;˜æΑêÞΑ恺˜ΑÄË樘ÚΑ˜ûÃ×½˜ΑæËΑêĔ˜ÚÞæÄ”ΑÞææ˜Ã˜ÄæΑ͊αΑ+¢ΑΑÞ¨«ÚæΑŽËÞæÞΑϮ͑Α«ÄΑ樘Αf[ΑÄ”ΑWÞΑ͍͉͉Α«ÄΑ+Ĕ«ΡΑ
樘ΑÚêט˜α”˽½ÚΑ˜ûŽ¨Ä£˜Αځæ˜ΑÞ¨Ë꽔Α˜ΑWÞΑ͎͉ΠΑbËΑޘ˜Αö¨üΡΑæΑÄüΑځæ˜Α¨«£¨˜ÚΑ樁ÄΑWÞΑ͎͉ΡΑށüΑWÞΑ͏͉ΡΑ«æΑ
ŽËÞæÞΑWÞΑ͍͉͑ΑטÚΑÞ¨«ÚæΑ«ÄΑ樘Αf[ΑêæΑËĽüΑWÞΑ͍͉͉Α«ÄΑ+Ĕ«ΠΑ+ÄΑ樁æΑŽÞ˜ΡΑ½½Α¢Ëژ«£ÄΑŽêÞæËØÚÞΑöË꽔ΑêüΑ
shirts from India. So devaluation of currency improves the competitveness of the domestic exports (India)
in the foreign markets. Hence, statement 1 is correct.
½ÄŽ˜ΑË¢Αbځ”˜Αλ HbμΑ«ÞΑ樘Α”«ą˜Ú˜ÄŽ˜Α˜æö˜˜ÄΑ樘Αõ½ê˜ΑË¢Α˜û×ËÚæÞΑÄ”Αõ½ê˜ΑË¢Α«Ã×ËÚæÞΑË¢Α£Ë˔ÞΑË¢ΑΑ
country in a given period of time. Regarding third statement, with the devaluation of currency
competitveness of the export improves, but trade balance depends upon both export as well as imports
and it is not necessarily true that devaluation of currency improves trade balance. (Let us take a possibility
in Indian economy, our economy is haveily dendent upon the import of crude oil, and at the time of
devaluation of currency, export increases but for the same period, demand for crude oil also increases due
to increased growth in the economy, and the value of this increase in import is more than the value of
«ÄŽÚ˜Þ˜Α«ÄΑ˜û×ËÚæΠΑ[ËΡΑ«ÄΑ樫ÞΑގ˜ÄÚ«ËΑ£«ÄΑö˜Α¨õ˜Α”˜ĈŽ«æΑæځ”˜Α½ÄŽ˜μΠΑHence statement 3 is not
correct.

ANALYSIS:

Exchange rate has always been in the news. Dollar-rupee comparison is a one of the important indicator
of the working of Indian economy.

SOURCE : Macroeconomics (NCERT Class 12)


96
Excerpt from NCERT

Year 2021 WATER CREDIT

V͋͋ΠΑr«æ¨Αژ¢˜Ú˜ÄŽ˜ΑæËΑϖræ˜ÚΑژ”«æϖΡΑŽËÄÞ«”˜ÚΑ樘Α¢Ë½½Ëö«Ä£ΑÞææ˜Ã˜ÄæÞ΢Α
1. It puts microfinance tools to work in the water and sanitation sector.
2. It is a global initiative launched under the aegis of the World Health Organization and the World Bank.
3. It aims to enable the poor people to meet their water need without depending on subsidies.
Which of the statements given above are correct?
(a) 1 and 2 only
(b) 2 and 3 only
(c) 1 and 3 only
(d) 1, 2 and 3

ANSWERS C

EXPLANATION

ræ˜ÚΠËÚ£Α«ÞΑΑ£½Ë½ΑÄËÄα×ÚËĈæΑËÚ£Ä«āæ«ËÄΑöËÚº«Ä£ΑæËΑÚ«Ä£Αöæ˜ÚΑÄ”Αށīææ«ËÄΑæËΑ樘ΑöËÚ½”ΠΑ+æΑÞæÚ昔Α
樘Α«Ä«æ«æ«õ˜ΑË¢Αræ˜ÚΑژ”«æΑ½ËÄΑ×ÚˣځÃΑæËΑ””Ú˜ÞÞΑ樘ΑÚÚ«˜ÚΑË¢ΑąËڔ½˜ΑĈāĎ«Ä£Α¢ËÚΑށ¢˜Αöæ˜ÚΑÄ”Α
sanitation. Hence, statement 2 is not correct.
ræ˜ÚΑژ”«æΑ«ÞΑΑ×Ëö˜Ú¢ê½ΑÞ˽êæ«ËÄΑÄ”Α樘ΑĈÚÞæΑæËΑ×êæΑëŽÚËĈāϘΑæË˽ÞΑæËΑöËÚºΑ«ÄΑ樘Αöæ˜ÚΑÄ”Α
sanitation sector.Αræ˜ÚΑژ”«æΑ¨˜½×ÞΑÚ«Ä£ΑÞÁ½½Α½ËÄÞΑæËΑæ¨ËޘΑö¨ËΑʘ”ΑŽŽ˜ÞÞΑæËΑąËڔ½˜ΑĈāĎ«Ä£Α
and expert resources to make household water and toilet solutions a reality.
97

EXPLANATION
How it works
+æΑ «”˜Äæ«Ĉ˜ÞΑ Α ژ£«ËÄΑ ö¨˜Ú˜Α טË×½˜Α ʘ”Α ŽŽ˜ÞÞΑ æËΑ öæ˜ÚΑ Ä”Α ށīææ«ËÄΑ Ä”Α 樁æΑ «ÞΑ ژ”üΑ ¢ËÚΑ Α
ëŽÚËĈāϘΑÞ˽êæ«ËÄΠΑ
+æΑׁÚæĘÚÞΑö«æ¨ΑŽÚ˜¢ê½½üΑޘ½˜Žæ˜”Α«ÄÞæ«æêæ«ËÄÞΑæËΑ×ÚËõ«”˜ΑąËڔ½˜ΑĈāĎ«Ä£Α¢ËÚΑöæ˜ÚΑÄ”Αށīææ«ËÄΑæËΑ
families in need.
b¨˜Þ˜Α ëŽÚËĈāϘΑ ׁÚæĘÚÞΑ ˜Þ恍½«Þ¨Α öæ˜ÚΑ Ä”Α ށīææ«ËÄΑ ½ËÄÞΑ «ÄΑ 樘«ÚΑ ×ËÚæ¢Ë½«ËΑ Ë¢Α Ëą˜Ú«Ä£ÞΠΑ +æΑ
supports them by providing technical assistance, connections and resources to get them started.
T˜Ë×½˜Α«ÄΑʘ”ΑêޘΑ樘ޘΑÞÁ½½ΡΑąËڔ½˜Α½ËÄÞΑæËΑ×êæΑΑæ×ΑËÚΑæË«½˜æΑ«ÄΑ樘«ÚΑ¨ËØÞΑÄ”ΑŽŽ˜ÞÞΑ½ËŽ½Α
resources to do the work. It enables poor households to meet their water and sanitation needs without
depending on subsidies.
Every repaid loan can be lent to another family in need of safe water or sanitation.
Thus statements 1 and 3 are correct. Hence option (c) is the correct answer.

ANALYSIS:

b¨«ÞΑ樘ØΑöÞΑÞ½«£¨æ½üΑËą˜æΑÞΑ«æΑ«ÞΑÄËæΑΑõ˜ÚüΑ×Ë×꽁ÚΑËÚ£Ä«Þæ«ËÄΑ½«º˜Α樘ΑrbHΑËÚΑ樘Α+A"ΠΑ;Ë£«Ž½½üΡΑ
these kind of topics can be prepared. One can only apply his intelligence on the spot.

SOURCE : https://water.org/solutions/watercredit/

Please Note:
If one goes by the theme of this question, it is slightly dilemmatic to classify it. It could
˜Α ށ¢˜½üΑ Ž½ÞÞ«Ĉ˜”Α êĔ˜ÚΑ Α ¨˜”Α ϏA«ÞŽ˜½½Ä˜ËêÞϐΡΑ êæΑ ÞΑ 樘Α «Ä«æ«æ«õ˜Α «ÞΑ 樁æΑ Ë¢Α ÄΑ
international organisation, we have put it under ‘External Sector: International
Organisations’

Year 2022 IMF

Q23. "Rapid Financing Instrument" and "Rapid Credit Facility" are related to the provisions of lend-
ing by which one of the following?
(a) Asian Development Bank
(b) International Monetary Fund
(c) United Nations Environment Programme Finance Initiative
(d) World Bank

ANSWERS B
98

EXPLANATION
Recent context: The Finance minister of Sri Lanka has started bailout talks with the lender of last resort
λ+A"μΑ Ä”Α ޘ˜º«Ä£Α 樘Α +A"ϐÞΑ ځ׫”Α ĈāĎ«½Α ÞÞ«ÞæÄŽ˜Α æËΑ 恎º½˜Α £ÚËö«Ä£Α Þ¨ËÚ恣˜ÞΑ Ë¢Α ¢Ë˔ΡΑ ¢ê˜½Α Ä”Α
medicines.
The Rapid Financing Instrument (RFI)Α ×ÚËõ«”˜ÞΑ ځ׫”Α ĈāĎ«½Α ÞÞ«ÞæÄŽ˜ΡΑ ö¨«Ž¨Α «ÞΑ õ«½½˜Α æËΑ ½½Α
member countries facing an urgent balance of payments need. The RFI was created as part of a broader
ژ¢ËÚÃΑ æËΑ Áº˜Α 樘Α +A"ϐÞΑ ĈāĎ«½Α Þê××ËÚæΑ ÃËژΑ ĉ˜û«½˜Α æËΑ ””Ú˜ÞÞΑ 樘Α ”«õ˜ÚޘΑ ʘ”ÞΑ Ë¢Α Ø͘ÚΑ
countries. The RFI replaced the IMF’s previous emergency assistance policy and can be used in a wide
range of circumstances. Financial assistance under the RFI is provided in the form of outright purchases
ö«æ¨ËêæΑ樘Αʘ”Α¢ËÚΑΑ¢ê½½αĉ˜”£˜”Α×ÚˣځÃΑËÚΑژõ«˜öÞΠΑΑØ͘ÚΑŽËêÄæÚüΑژÙê˜Þæ«Ä£ΑW"+ΑÞÞ«ÞæÄŽ˜Α«ÞΑ
ژÙê«Ú˜”ΑæËΑ ŽËËטځæ˜Αö«æ¨Α樘Α +A"ΑæËΑ Áº˜Α ˜ąËÚæÞΑæËΑ Þ˽õ˜Α «æÞΑ ½ÄŽ˜Α Ë¢Α ׁüØÄæÞΑ ”«ĆŽê½æ«˜ÞΑ Ä”ΑæËΑ
describe the general economic policies that it proposes to follow. Prior actions may be required where
warranted.
The Rapid Credit Facility (RCF) of IMF ×ÚËõ«”˜ÞΑ ځ׫”Α ŽËϘÞÞ«Ëā½Α ĈāĎ«½Α ÞÞ«ÞæÄŽ˜ΑæËΑ ½Ëöα«ÄŽËØΑ
countries (LICs) facing an urgent balance of payments (BoP) need with no ex post conditionality where a
¢ê½½αĉ˜”£˜”Α˜ŽËÄËëŽΑ×ÚˣځÃΑ«ÞΑĘ«æ¨˜ÚΑʎ˜ÞށÚüΑÄËÚΑ¢˜Þ«½˜ΠΑb¨˜ΑW"ΑöÞΑŽÚ˜æ˜”ΑêĔ˜ÚΑ樘ΑTËõ˜ÚæüΑ
W˜”êŽæ«ËÄΑÄ”Α#ÚËöæ¨ΑbÚêÞæΑλTW#bμΑÞΑׁÚæΑË¢ΑΑÚˁ”˜ÚΑژ¢ËÚÃΑæËΑÁº˜Α樘Α"êĔϐÞΑĈāĎ«½ΑÞê××ËÚæΑÃËژΑ
ĉ˜û«½˜ΑÄ”Α˜ææ˜ÚΑ恫½Ëژ”ΑæËΑ樘Α”«õ˜ÚޘΑʘ”ÞΑË¢Α;+ÞΡΑ«ÄŽ½ê”«Ä£Α«ÄΑæ«Ã˜ÞΑË¢ΑŽÚ«Þ«ÞΠ
Hence option (b) is the current answer.

ANALYSIS:

Recently these provisions of lending were in news, due to Sri Lankan economic crisis.

SOURCE : https://www.imf.org/en/About/Fact-
sheets/Sheets/2016/08/02/19/55/Rapid-Financing-Instrument#:~:text=The%20Rapid%20
"«ÄÄŽ«Ä£А͉͋+ÄÞæÚêØÄæА͉͋λW"+Ρ”«õ˜ÚޘА͉͋ʘ”ÞА͉͋Ë¢А͉͋Ø͘ÚА͉͋ŽËêÄæÚ«˜ÞΠ
https://www.imf.org/en/About/Fact-
sheets/Sheets/2016/08/02/21/08/Rapid-Credit-Facility
https://economictimes.indiatimes.com/news/internation-
al/world-news/technical-level-discussions-with-sri-lanka-will-continue-imf/articleshow/9
1487174.cms?from=mdr

Year 2022 REER/NEER

V͍͋ΠΑr«æ¨Αژ¢˜Ú˜ÄŽ˜ΑæËΑ樘Α+Ĕ«ÄΑ˜ŽËÄËÃüΡΑŽËÄÞ«”˜ÚΑ樘Α¢Ë½½Ëö«Ä£ΑÞææ˜Ã˜ÄæÞΑ΢
1. An increase in Nominal Effective Exchange Rate (NEER) indicates the appreciation of rupee.
2. An increase in the Real Effective Exchange Rate (REER) indicates an improvement in trade competitiveness.
3. An increasing trend in domestic inflation relative to inflation in other countries is likely to cause an increasing diver-
gence between NEER and REER.
Which of the above statements are correct?
(a) 1 and 2 only
(b) 2 and 3 only
(c) 1 and 3 only
(d) 1, 2 and 3

ANSWERS C
99

BËëā½Α ą˜Žæ«õ˜Α ûŽ¨Ä£˜Α Wæ˜Α λBWμ is a measure of the value of a currency against a weighted
average of several foreign currencies. The nominal exchange rate is the amount of domestic currency
needed to purchase foreign currency. If a domestic currency increases against a basket of other
ŽêÚژĎ«˜ÞΑ«ÄÞ«”˜ΑΑĉˁæ«Ä£Α˜ûŽ¨Ä£˜Αځæ˜Αژ£«Ã˜ΡΑBWΑ«ÞΑށ«”ΑæËΑ××ژŽ«æ˜ΠΑ+¢Α樘Α”ËØÞ櫎ΑŽêÚژĎüΑ
falls against the basket, the NEER depreciates. An increase in NEER indicates an appreciation of the local
currency against the weighted basket of currencies of its trading partners. Hence statement 1 is correct.
b¨˜ΑW˜½Αą˜Žæ«õ˜ΑûŽ¨Ä£˜ΑWæ˜ΑλWWμΑ«ÞΑ樘Αژ½Α˜ą˜Žæ«õ˜Α˜ûŽ¨Ä£˜Αځæ˜ΑλΑ؁ÞêژΑË¢Α樘Αõ½ê˜ΑË¢ΑΑ
ŽêÚژĎüΑ£«ÄÞæΑΑö˜«£¨æ˜”Αõ˜Ú£˜ΑË¢Αޘõ˜Ú½Α¢Ëژ«£ÄΑŽêÚژĎ«˜ÞμΑ”«õ«”˜”ΑüΑΑ×Ú«Ž˜Α”˜ĉæËÚΑËÚΑ«Ä”˜ûΑË¢Α
ŽËÞæÞΠΑ +ÄΑ Þ«Ã×½˜ΑöËڔÞΡΑ Α āæ«ËÄϖÞΑ ÄËëā½Α ˜ą˜Žæ«õ˜Α ˜ûŽ¨Ä£˜Α ځæ˜Α λBWμΡΑ ”¶êÞ昔Α¢ËÚΑ «Äĉæ«ËÄΑ «ÄΑ樘Α
¨ËØΑŽËêÄæÚüΡΑ˜ÙꁽÞΑ«æÞΑژ½Α˜ą˜Žæ«õ˜Α˜ûŽ¨Ä£˜Αځæ˜ΑλWWμΠΑÄΑ«ÄŽÚ˜Þ˜Α«ÄΑWWΑ«Ã×½«˜ÞΑ樁æΑ˜û×ËÚæÞΑ
become more expensive and imports become cheaper; therefore, an increase indicates a loss in trade
competitiveness. Hence statement 2 is not correct.
WWΑ«ÞΑ樘ΑBWΑ¢æ˜ÚΑ¢ŽæËÚ«Ä£Α«ÄΑژ½æ«õ˜Α«Äĉæ«ËÄΑλŽËÄÞêØÚΑ×Ú«Ž˜αÞ˜”Α«Ä”˜ûμΑêÞ«Ä£ΑÞËØΑ؁ÞêژΑË¢Α
relative prices or costs; changes in the REER thus take into account both nominal exchange rate changes
Ä”Α樘Α«Äĉæ«ËÄΑ”«ą˜Ú˜Ä櫁½Αõ«Þα†αõ«ÞΑæځ”«Ä£ΑׁÚæĘÚÞΠΑ[ˁګģΑ«Äĉæ«ËÄΑö«½½Α«ÃׁŽæΑWWΡΑö¨«Ž¨ΡΑ«ÄΑæêÚÄΡΑ
öË꽔Α«Ä˜õ«æ½üΑ×êÞ¨Αê×Α樘ΑŽËÞæΑË¢ΑØڎ¨Ä”«Þ˜ΑÄ”Αą˜ŽæΑ樘ΑŽËÃטæ«æ«õ˜Ä˜ÞÞΑË¢Α+Ĕ«ÄΑ˜û×ËÚæÞΠΑb¨êÞΡΑ
«¢Α«Äĉæ«ËÄΑ«ÞΑ«ÄΑÄΑ«ÄŽÚ˜Þ«Ä£ΑæژĔΑ«ÄΑ樘Α”ËØÞ櫎Αāæ«ËÄÞΑژ½æ«õ˜ΑæËΑ«Äĉæ«ËÄΑ«ÄΑË樘ÚΑŽËêÄæÚ«˜ÞΡΑ«æΑ«ÞΑ½«º˜½üΑ
to be an increasing divergence between NEER and REER. For instance, considering NEER and REER values
˜æö˜˜ÄΑ×Ú«½Α͉͋͊͒ΑÄ”ΑAüΑ͉͋͋͊Α«ÄΑ+Ĕ«ΡΑ樘ΑBWΑ¨ÞΑ˜˜ÄΑÃËÞæ½üΑ”˜Ž½«Ä«Ä£ΡΑö¨˜Ú˜ÞΡΑWWΑژÁ«ÄÞΑ«ÄΑ
ÞüĎΑö«æ¨Α樘Α«Äĉæ«ËāÚüΑæژĔÞδ樘Αê×öÚ”Α«Þ˜ÞΑ«ÄΑWWΑ”ê˜ΑæËΑ«Äĉæ«ËÄΑö˜Ú˜Α½Ú˜”üΑ˜«Ä£Α¢˜½æΑ«ÄΑAüΑ
͉͋͋͊ΠΑb¨˜Α«ÄŽÚ˜Þ«Ä£Α”«ą˜Ú˜ÄŽ˜Α˜æö˜˜ÄΑæژĔÞΑË¢ΑBWΑÄ”ΑWWΑ«ÄΑ樘Α½ÞæΑ͋͏ΑÃËÄæ¨ÞΑöÞΑ”ê˜ΑæËΑ+Ĕ«ϐÞΑ
”ËØÞ櫎Α «Äĉæ«ËÄΑ ˜«Ä£Α ¨«£¨˜ÚΑ ژ½æ«õ˜Α æËΑ 樘Α Þ«ûΑ Á¶ËÚΑ ŽêÚژĎ«˜ÞΑ ŽËÄÞ«”˜Ú˜”ΠΑ Hence statement 3 is
correct.

ANALYSIS:

SOURCE : ¨ææ×Þ΢έέ”æ¨˜½×Π«Ã¢ΠËÚ£έºÄËö½˜”£˜Þ˜έÚ櫎½˜Þέ͎͍͌͐͏͒αö¨æα«ÞαÄËëā½α˜ą˜Žæ«õ˜α˜ûŽ¨
nge-rate-neer
¨ææ×Þ΢έέ”æ¨˜½×Π«Ã¢ΠËÚ£έºÄËö½˜”£˜Þ˜έÚ櫎½˜Þέ͎͍͌͐͐͋αö¨æα«Þαژ½α˜ą˜Žæ«õ˜α˜ûŽ¨Ä£˜
-rate-reer

Please Note:
Certain concepts have been well explained in websites like Investopedia or one can
ژ¢˜ÚΑæËΑ樘ΑËƎ«½Αö˜Þ«æ˜ÞΑË¢Α+A"Α¢ËÚΑŽËϘ×æÞΑژ½æ˜”ΑæËΑ˜ûŽ¨Ä£˜Αځæ˜ΡΑ˜æŽΠ
100

Year 2023 UNOPS (GLOBAL ORGANISATIONS)

Q25. Consider the following infrastructure sectors:


1. Affordable housing
2. Mass rapid transport
3. Health care
4. Renewable energy
On how many of the above does UNOPS Sustainable Investments in Infrastructure and Innovation (S3i) initiative focus
for its investments?
(a) Only one
(b) Only two
(c) Only three
(d) All four

ANSWERS C

EXPLANATION

Sustainable Investments in Infrastructure and Innovation (‘S3i’) In March 2020, Sustainable Investments
in Infrastructure and Innovation (S3i) was formally established as a stand-alone business unit in the
fBHT[Α£Ëõ˜ÚāϘΑÞæÚêŽæêژΡΑÁº«Ä£ΑfBHT[Α樘ΑĈÚÞæΑfī昔ΑBæ«ËÄÞΑËÚ£Ä«āæ«ËÄΑ½˜ΑæËΑÁº˜Α”«Ú˜ŽæΑ
investments from its own balance sheet.
+ÄΑ½«Ä˜Αö«æ¨Α樘ΑfBHT[Α[æځ昣«ŽΑT½ÄΡΑ͉͋͋͋α͉͎͋͋ΡΑ樘Α[͌«ΑËƎ˜Αö«½½Αޘ˜ºΑæËΑ˜Ä¨ÄŽ˜ΑÄ”ΑŽŽ˜½˜Úæ˜Α樘Α
˜ąËÚæΑ Ë¢Α ˜Ä££«Ä£Α ×ꍽ«ŽΑ Ä”Α ×Ú«õæ˜Α ޘŽæËÚΑ «Äõ˜ÞæËÚÞΑ æËΑ öËÚºΑ ŽË½½˜Žæ«õ˜½üΑ æËΑ ގ½˜Α ê×Α «Ä¢ÚÞæÚêŽæêژΑ
«Äõ˜ÞæØÄæÞΑÄ”ΑŽËÄÞ«”˜ÚΑŽËαŽÚ˜æ«Ä£Α«ÄÄËõæ«õ˜ΑĈāĎ«Ä£ΑË×æ«ËÄÞΠΑb¨˜ΑfBHT[Α[͌«Αö«½½ΑŽËÄæ«Äê˜ΑÚ˽½«Ä£Α
ËêæΑ樘Α«Ä«æ«æ«õ˜ΑŽÚËÞÞΑ«æÞΑæ¨Ú˜˜Α¢ËŽêÞαÚ˜Þ΢ΑλμΑąËڔ½˜Α¨ËêÞ«Ä£ΣΑλμΑژĘö½˜Α˜Ä˜Ú£üΣΑÄ”ΑλŽμΑ¨˜½æ¨Α
infrastructure. Hence only three are correct.

ANALYSIS:
ANALYSIS:

Scope for elimination


If one applies logic and is ready take some risk, then one should see that ‘rapid mass transport’ as a need
or requirement is one step higher than the basic requirements in the form of Hosuing, Health and Energy.
Otherwise, his is a tough theme because it was not widely covered by the standard newspapers.

Under the S3i scheme, a British businessman was in the news as he was the sole businessman dealing
with the S3i related housing sector in Goa and there was a scandal reported in 2022. Although the Indian
Express covered it, but the criteria (in the question) were not mentioned.

SOURCE : https://indianexpress.com/article/india/in-60-million-dollar-un-scandal-a-2-5-million-
dollar-housing-investment-in-goa-and-no-house-to-show-for-it-7919071/
101

Year 2023 TRADE

Q26. Consider the following statements:


Statement-I : India accounts for 3.2% of global export of goods.
Statement-II : Many local companies and some foreign companies operating in India have taken advantage of India's
'Production-linked Incentive' scheme.
Which one of the following is correct in respect of the above statements?
(a) Both Statement-I and Statement-II are correct and Statement-II is the correct explanation for Statement-I
(b) Both Statement-I and Statement-II are correct and Statement-II is not the correct explanation for Statement-I
(c) Statement-I is correct but Statement-II is incorrect
(d) Statement-I is incorrect but Statement-II is correct

ANSWERS D

EXPLANATION
Statement I is incorrect: +Ĕ«ϐÞΑÞ¨Ú˜Α«ÄΑ£½Ë½ΑØڎ¨Ä”«Þ˜Αæځ”˜Α«ÞΑËĽüΑ͊Π͑АΑÄ”Α͍АΑ«ÄΑ£½Ë½ΑޘÚõ«Ž˜ÞΠΑ
+Ĕ«Α×½ÄÞΑæËΑ«ÄŽÚ˜Þ˜Α«æÞΑ˜û×ËÚæΑÞ¨Ú˜Α«ÄΑ£½Ë½Αæځ”˜Α¢ÚËÃΑ͋Π͊АΑæËΑ͌АΑüΑ͉͋͋͐ΑÄ”Α͉͊АΑüΑ͉͍͋͐ΠΑ
Statement II is correct: The PLI scheme is open to both domestic and international manufacturers.
[ÃÞêÄ£ΑÞΑö˜½½ΑÞΑ+Ĕ«ÄΑĈÚÃÞΑÞꎨΑÞΑ«ûËÄΑb˜Ž¨Ä˽ˣ«˜ÞΡΑfb;ΡΑB˜Ë½üĎÞΡΑ;õΑ+Äæ˜Úāæ«Ëā½ΡΑH×櫘ÃêÞΑ
Electronics and Micromax are also expanding their factories to take advantage of the PLI scheme.

ANALYSIS:
ANALYSIS:
Scope for elimination: On the face of it there is no scope for elimination, but if the aspirants are able to
Ĉ£êژΑËêæΑ樁æΑÄüΑË¢Α樘ΑæöËΑÞææ˜Ã˜ÄæÞΑ«ÞΑ«ÄŽËÚژŽæΡΑ樘ÄΑ樘üΑŽÄΑ˜Þ«½üΑ£˜æΑ樘ΑŽËÚژŽæΑÄÞö˜ÚΠΑ+ÄΑ樫ÞΑ
question, the second statement has a word, ‘some’ which in general is suggestive of its correctness.
Caution: That assumption also carries an element of risk.
On the reasons why it was asked could be the PLI scheme whichhas always been in the news and India’s
trade with the world is also an important theme. India’s global share in service exports is considerable
λ͍Π͍АμΠΑb¨«ÞΑü˜ÚΑ£Ë˔ϐÞΑ˜û×ËÚæÞΑöÞΑÞº˜”ΡΑ«ÄΑ樘ΑŽËëģΑü˜ÚÞϐΑޘÚõ«Ž˜Α˜û×ËÚæÞΑÁüΑ˜ΑÞº˜”Π

SOURCE : Standard newspapers


https://www.hindustantimes.com/india-news/indias-merchan-
dise-exports-to-seven-top-trading-partners-contract-101668627916014.html

ʷ˜˙Ѓ˖˨˟˧ˬ˟˘˩˘˟ʭʷ˜˙Ѓ˖˨˟˧ʛ˔˦˙˔˖˧˨˔˟˜ˡˡ˔˧˨˥˘ʜ
102

Year 2023 TRADE

Q27. Consider the following statements:


Statement-I: Recently, the United States of America (USA) and the European Union (EU) have launched the 'Trade and
Technology Council'
Statement-II: The USA and the EU claim that through this they are trying to bring technological progress and physical
productivity under their control.
Which one of the following is correct in respect of the above statement?
(a) Both Statement-I and Statement-II are correct and Statement-II is the correct explanation for Statement-I
(b) Both Statement-I and Statement-II are correct and Statement-II is not the correct explanation for Statement-I
(c) Statement-I is correct but Statement-II is incorrect
(d) Statement-I is incorrect but Statement-II is correct

ANSWERS C

EXPLANATION

Recently the United States of America and the European Union have launched the Trade and Technology
Council, a strategic coordination mechanism to tackle challenges at the nexus of trade, trusted
technology and security. Hence statement 1 is correct. It serves as a forum for the United States and
European Union to coordinate approaches to key global trade, economic, and technology issues and to
deepen transatlantic trade and economic relations based on these shared values.
Through the Council, the EU and the US are working together to: ensure that trade and technology serve
our societies and economies, while upholding our common values strengthen our technological and
industrial leadership expand bilateral trade and investment The aim is not about bringing technological
progress and physical productivity under their control, but rather about coordinating approaches with
respect to same. Hence statement 2 is not correct.

ANALYSIS:
ANALYSIS:
This is not a conventional theme that is covered in Economics but it was asked as recently EU set up a
Trade and Technology Council with India along the lines of the EU-US Trade and Technology Council. The
same was covered in The Hindu (Feb., 2023)

Statement II hints at it being a wrong statement as the two cannot claim to bring technological progress
under control. (It is an extreme statement).

Scope for elimination

If one can sense the extremeness of the statement and thus its incorrectness, then 3 options can be got
rid of, A, B and D
103

Year 2023 GLOBAL INSTITUTIONS

Q28. Consider the following statements:


The 'Stability and Growth Pact' of the European Union is a treaty that
1. limits the levels of the budgetary deficit of the countries of the European Union
2. makes the countries of the European Union to share their infrastructure facilities
3. enables the countries of the European Union to share their technologies
How many of the above statements are correct?
(a) Only one
(b) Only two
(c) All three
(d) None

ANSWERS A

EXPLANATION

The Stability and Growth Pact (SGP) is a set of rules designed to ensure that countries in the European
fÄ«ËÄΑ×êÚÞê˜ΑÞËêĔΑ×ꍽ«ŽΑĈāϘÞΑÄ”ΑŽËËڔ«Äæ˜Α樘«ÚΑĈގ½Α×˽«Ž«˜ÞΠΑ˜ÞŽÚ«˜”ΑÞΑ樘ΑêÚËāËĘϖÞΑĈގ½Α
Ú꽘Ë˺ΡΑ樘Α[#TΑ«ÞΑΑޘæΑË¢ΑĈގ½ΑÚ꽘ÞΑ”˜Þ«£Ä˜”ΑæËΑ×ژõ˜ÄæΑfΑŽËêÄæÚ«˜ÞΑ¢ÚËÃΑÞטĔ«Ä£Α˜üËĔΑ樘«ÚΑ
؁ÄÞΠΑ[«ÄŽ˜Α͊͒͒͐ΡΑØ͘ÚΑÞææ˜ÞΑ¨õ˜Α£Ú˜˜”Α樁æΑ樘üΑÞ¨Ë꽔Αº˜˜×Α#TΑځæ«ËÞΑË¢Α”˜ĈŽ«æΑÄ”Α”˜æΑ˜½ËöΑ
͌АΑÄ”Α͏͉АΑژÞטŽæ«õ˜½üΠΑb¨˜ΑêÚËטÄΑËÃëÞÞ«ËÄΑÄ”Α樘ΑËêĎ«½ΑĈāϘΑëīÞæ˜ÚÞΑ¨õ˜Α樘ΑÁ«ÄΑ”êæüΑ
of surveillance. They issue an annual recommendation on policy measures every year and also monitor EU
countries to ensure each nation is compliant with budget regulations. Countries that break the rules for
æ¨Ú˜˜ΑŽËÄޘŽêæ«õ˜Αü˜ÚÞΑÚ˜ΑĈʔΑΑÁû«ÃêÃΑË¢Α͉Π͎АΑË¢Α樘«ÚΑ#TΠΑb¨˜Α[#TΑ”˘ÞΑÄËæΑÁº˜ΑŽËêÄæÚ«˜ÞΑË¢Α樘Α
European Union share their infrastructure facilities or technologies. Hence only statement 1 is correct
and option (a) is the correct answer.

ANALYSIS:
ANALYSIS:
The European Union (EU) has been tested by three great external events, the Syrian refugee crisis, the
Coronavirus, and the Russian invasion of Ukraine. Each of these events challenges the EU’s Stability and
#ÚËöæ¨ΑTŽæΑλ[#TμΑ樁æΑށĎæ«ËÄÞΑØ͘ÚΑÞææ˜ÞΑ«ÄŽêÚÚ«Ä£Α˜ûŽ˜ÞÞ«õ˜Αê”£˜æÚüΑ”˜ĈŽ«æÞΠΑ(˜ÄŽ˜Α樫ÞΑöÞΑ
in the news.
As the standard newspapers did not widely cover the topic, this theme was lesser-know and slightly
”«ĆŽê½æΠ
Scope for elimination: Not applicable

SOURCE : https://www.euronews.com/busi-
ness/2022/01/18/what-is-the-eu-s-stability-and-growth-pact-and-why-is-it-important
104
104

FINANCIAL MARKET
(11 Questions)

Year 2018 T–BILLS

Q1. Consider the following statements:


1. The Reserve Bank of India manages and services Government of India Securities but not any State Government Securi-
ties.
2. Treasury bills are issued by the Government of India and there are no treasury bills issued by the statement Govern-
ments.
3. Treasury bills offer are issued at a discount from the par value.
Which of the statements given above is/are correct?
(a) 1 and 2 only
(b) 3 only
(c) 2 and 3 only
(d) 1, 2 and 3

ANSWERS C

EXPLANATION

The Reserve Bank of India (RBI) manages public debt and issues Indian currency denominated loans on behalf of the
central and the state governments under the powers derived from the Reserve Bank of India Act. The RBI is the debt
manager for both the Central Government and the State Governments. The RBI manages the debt of state
governments on the basis of separate agreements. Hence, statement 1 is not correct.
Treasury bills or T-bills, which are money market instruments, are short term debt instruments issued by the
Government of India and are presently issued in three tenors, namely, 91 day, 182 day and 364 day. Treasury bills are
zero coupon securities and pay no interest. They are issued at a discount and redeemed at the face value at maturity.
For example, a 91 day Treasury bill of 100/- (face value) may be issued at say  98.20, that is, at a discount of say, 1.80
and would be redeemed at the face value of 100/-. The return to the investors is the difference between the maturity
value or the face value (that is 100). Hence, statements 2 and 3 are also correct.

ANALYSIS:
ANALYSIS:
Although it is a topic categorised under macroeconomics, it is not covered in the Introductory Macroeconomics book,
but it has comprehensively covered in the chapter Financial Markets of the Business Studies Class 12 book.
Kindly refer to the image shared below.
105

SOURCE : Business Studies (NCERT Class 12)


Excerpt from NCERT
106

Year 2019 PARTICIPATORY NOTES

Q2. Which of the following is issued by registered foreign portfolio investors to overseas investors who
want to be part of the Indian stock market without registering themselves directly?
(a) Certificate of Deposits
(b) Commercial Paper
(c) Promissory Note
(d) Participatory Note

ANSWERS D

EXPLANATION

A Participatory Note (or P-Note) is a derivative instrument issued in foreign jurisdictions, by a


SEBI-registered Foreign Institutional Investor (FII) or its sub-accounts or one of its associates, against
underlying Indian securities. The underlying Indian security instrument may be equity, debt, derivatives or
may even be an index.
A Promissory NoteΑ«ÞΑΑĈāĎ«½Α«ÄÞæÚêØÄæΑ樁æΑŽËÄ恫ÄÞΑΑöÚ«ææ˜ÄΑ×ÚËëޘΑüΑËĘΑׁÚæüΑæËΑׁüΑÄË樘ÚΑ
ׁÚæüΑΑ”˜ĈÄ«æ˜ΑÞêÃΑË¢ΑÃËĘüΡΑ˜«æ¨˜ÚΑËÄΑ”˜ÃÄ”ΑËÚΑæΑΑÞטŽ«Ĉ˜”Α¢êæêژΑ”æ˜Π
Commercial Paper (CP) is an unsecured money market instrument issued in the form of a promissory
note. CPs are short-term instruments and the maturity period varies from seven days to up to one year. It
öÞΑ «ÄæÚ˔ꎘ”Α æËΑ ˜Ä½˜Α ¨«£¨½üΑ ځ昔Α ŽËÚ×Ëځæ˜Α ËÚÚËö˜ÚÞΑ æËΑ ”«õ˜ÚÞ«¢üΑ 樘«ÚΑ ÞËêڎ˜ÞΑ Ë¢Α Þ¨ËÚæαæ˜ÚÃΑ
ËÚÚËö«Ä£ÞΑÄ”Α½ÞËΑæËΑ×ÚËõ«”˜ΑÄΑ””«æ«Ëā½Α«ÄÞæÚêØÄæΑæËΑ«Äõ˜ÞæËÚÞΠ
˜Úæ«ĈŽæ˜Α Ë¢Α ˜×ËÞ«æÞΑ «ÞΑ Α ށõ«Ä£Α Ž˜Úæ«ĈŽæ˜Αö«æ¨Α ΑĈû˜”Α ÁæêÚ«æüΑ ”æ˜Α æΑ樘ΑĈû˜”Α «Äæ˜Ú˜ÞæΑ ځæ˜ΠΑ +æΑ «ÞΑ
«ÞÞꘔΑ üΑ ŽËÃØڎ«½Α ÄºÞΑ Ä”Α ĈāĎ«½Α «ÄÞæ«æêæ«ËÄÞΠΑ +æΑ «ÞΑ «ÞÞꘔΑ «ÄΑ 樘Α ¢ËÚÃΑ Ë¢Α Α ×ÚËëÞÞËÚüΑ ÄËæ˜Α «ÄΑ
˜ûŽ¨Ä£˜ΑË¢Α¢êĔÞΑ”˜×Ëޫ昔Α«ÄΑÄºÞΑ¢ËÚΑΑÞטŽ«Ĉ˜”Αטګ˔ΠΑHence option (d) is the correct answer.

ANALYSIS:
ANALYSIS:
TαBËæ˜ÞΑ«ÞΑΑÞ«ŽΑŽËϘ×æΑ«ÄΑ樘Α"«ÄÄŽ«½ΑAÚº˜æΠΑÞΑĈāĎ«½ΑÁں˜æΡΑÞΑΑŽ¨×æ˜ÚΡΑ«ÞΑÄËæΑ樘ژΑ«ÄΑ樘Α
Macroeconomics NCERT book, the same can be studied from arthapedia

SOURCE : http://www.arthapedia.in/index.php/Participatory_Notes_(PNs)
107

Year 2020 MISCELLANEOUS (FINANCIAL MARKET)

Q3. With reference to the Indian economy, consider the following statements:
1. 'Commercial Paper' is a short-term unsecured promissory note.
2. 'Certificate of Deposit' is a long-term instrument issued by the Reserve Bank of India to a corporation.
3. 'Call Money' is a short-term finance used for interbank transactions.
4. 'Zero-Coupon Bonds are the interest bearing short-term bond issued by the Scheduled Commercial Banks to
corporations.
Which of the statements given above is/are correct?
(a) 1 and 2 only
(b) 4 only
(c) 1 and 3 only
(d) 2, 3 and 4 only

ANSWERS C

EXPLANATION

Commercial Paper (CP) is an unsecured money market instrument issued in the form of a promissory note. It can be
issued for maturities between a minimum of 7 days and a maximum of up to one year from the date of issue
(short-term). Hence statement 1 is correct.
Certificate of Deposit (CD) is a negotiable money market instrument and is issued in dematerialised form against funds
deposited at a bank or other eligible financial institution for a specified time period. Issued by the Federal Deposit
Insurance Corporation (FDIC) and regulated by the Reserve Bank of India, the CD is a promissory note, the interest on
which is paid by the financial institution. Hence statement 2 is not correct.
Call money rate is the rate at which short term funds are borrowed and lent in the money market among banks on a
day-today basis. Banks resort to this type of loan to fill the asset liability mismatch, comply with the statutory CRR and
SLR requirements and to meet the sudden demand of funds. Hence statement 3 is correct.
Bonds are a type of debt instrument. Zero Coupon Bonds are issued at a discount and redeemed at par. No interest
payment is made on such bonds at periodic intervals before maturity. Hence statement 4 is not correct.

ANALYSIS:

Scope for elimination


b¨˜ΑÙê˜Þæ«ËÄΑ«ÞΑŽ¨˜Žº«Ä£Α樘Α¢êĔÃ˜Ä恽ΑºÄËö½˜”£˜ΑËêæΑ樘ΑÞ«ŽΑæ˜ÚÃÞΑ«ÄΑΑ£«õ˜ÄΑæË׫ŽΠΑ+æΑ«ÞΑÃËژΑË¢ΑΑ
¢ŽæꁽΑÙê˜Þæ«ËÄΑÄ”Α樘ژ¢ËژΡΑ樘ژΑ«ÞΑÄËΑގËטΑË¢Α˜½«Ã«Äæ«ËÄΑêÄ櫽ΑÄ”ΑêĽ˜ÞÞΑËĘΑºÄËöÞΑÞËØΑË¢Α樘ޘΑ
terms.
108

SOURCE : Business Studies (NCERT Class 12)


Excerpt from NCERT
109

Year 2021 BOND MARKET

V͍ΠΑ+Ĕ«ÄΑ#Ëõ˜ÚÄØÄæΑ ËĔΑx«˜½”ÞΑÚ˜Α«Äĉê˜ÄŽ˜”ΑüΑö¨«Ž¨ΑË¢Α樘Α¢Ë½½Ëö«Ä£ΧΑ
1. Actions of the United States Federal Reserve
2. Actions of the Reserve bank of India
3. Inflation and short-term interest rates
Select the correct answer using the code given below.
(a) 1 and 2 only
(b) 2 only
(c) 3 only
(d) 1, 2 and 3

ANSWERS D

EXPLANATION
Bond yield is the return an investor gets on that bond or on a particular government security. It
”˜×˜Ä”ÞΑËÄΑ樘Α×Ú«Ž˜ΑË¢Α樘ΑËĔΑö¨«Ž¨Α«ÞΑ«ÃׁŽæ˜”ΑüΑ«æÞΑ”˜ÃÄ”ΠΑb¨˜ΑÁ¶ËÚΑ¢ŽæËÚÞΑą˜Žæ«Ä£Α樘Αü«˜½”Α«ÞΑ
the monetary policy of the Reserve Bank of India,Α ˜ÞטŽ«½½üΑ 樘Α ŽËêÚޘΑ Ë¢Α «Äæ˜Ú˜ÞæΑ ځæ˜ÞΡΑ 樘Α Ĉގ½Α
×ËÞ«æ«ËÄΑË¢Α樘Α£Ëõ˜ÚÄØÄæΑÄ”Α«æÞΑËÚÚËö«Ä£Α×ÚˣځÃØΡΑ£½Ë½ΑÁں˜æÞΡΑ˜ŽËÄËÃüΡΑÄ”Α«Äĉæ«ËÄΠΑ
Žæ«ËÄÞΑ Ë¢Α 樘Α fī昔Α [ææ˜ÞΑ "˜”˜Ú½Α W˜Þ˜Úõ˜Α ŽÄΑ «ÃׁŽæΑ 樘Α «Äõ˜ÞæØÄæÞΑ ĉËö«Ä£Α «ÄΑ +Ĕ«ΠΑ b¨˜Α
«Äõ˜ÞæØÄæÞΑüΑ¢Ëژ«£ÄΑ×½ü˜ÚÞΑ«ÄΑ£Ëõ˜ÚÄØÄæΑޘŽêګ櫘ÞΑŽÄΑ˜Α˜ą˜Žæ˜”ΑüΑ樫ÞΠΑb¨«ÞΑö«½½Α½˜”ΑæËΑŽ¨Ä£˜Α
in demand of government securities and thereby impacting its yield. Hence option 1 is correct.
Actions of Reserve bank determine the liquidity and also the cost of funds available in the economy
æ¨ÚË꣨Α «æÞΑ õÚ«ËêÞΑ «Äĉæ«ËÄΑ Áā£˜Ã˜ÄæΑ æË˽ÞΠΑ b¨˜Α ŽËÞæΑ Ë¢Α ¢êĔÞΑ ö«½½Α ”«Ú˜Žæ½üΑ «ÃׁŽæΑ 樘Α ”˜ÃÄ”Α Ë¢Α
£Ëõ˜ÚÄØÄæΑޘŽêګ櫘ÞΑ«ÄΑ樘ΑÁں˜æΑÄ”Α樘ژüΑ«Äĉê˜ÄŽ«Ä£Α«æÞΑü«˜½”ΠΑHence option 2 is correct.
+Äĉæ«ËÄΑÄ”ΑÞ¨ËÚæΑæ˜ÚÃΑځæ˜ÞΑ”˜æ˜ÚëĘΑ樘Α×êڎ¨Þ«Ä£ΑŽ×Ž«æüΑË¢Α樘ΑטË×½˜Α«ÄΑ樘Α˜ŽËÄËÃüΠΑb¨˜Ú˜¢ËژΡΑ
樫ÞΑ½ÞËΑ¨ÞΑ«ÃׁŽæΑËÄΑ樘Α”˜ÃÄ”ΑÄ”Α×Ú«Ž˜ΑË¢Α樘Α£Ëõ˜ÚÄØÄæΑαޘŽêګ櫘ÞΑ樘ژüΑ«Äĉê˜ÄŽ«Ä£Α樘Αü«˜½”ΠΑ
Hence option 3 is correct.

ANALYSIS:
ANALYSIS:

SOURCE : https://indianexpress.com/article/explained/explained-what-rise-in-bond-yield-means-for-
investors-and-govt-7205054/ , NCERT – Introductory Macroeconomics, Chapter – Money and
Banking, Page number – 45, 46
Excerpt from NCERT

Please Note:
Yield is a general term that relates to the return on the capital you invest in a bond. Price and
ü«˜½”ΑÚ˜Α«Äõ˜Úޘ½üΑژ½æ˜”΢ΑÞΑ樘Α×Ú«Ž˜ΑË¢ΑΑËĔΑ£Ë˜ÞΑê×ΡΑ«æÞΑü«˜½”Α£Ë˜ÞΑ”ËöÄΡΑÄ”Αõ«Ž˜Αõ˜ÚށΠ
110

Year 2021 G-SECS

Q5. With reference to India, consider the following statements:


1. Retail investors through demat account can invest in 'Treasury Bills' and 'Government of India Debt Bonds' in prima-
ry market.
2. The 'Negotiated Dealing System-Order Matching' is a government securities trading platform of the Reserve Bank
of India.
3. The 'Central Depository Services Ltd' is jointly promoted by the Reserve Bank of India and the Bombay Stock
Exchange.
Which of the statements given above is/are correct?
(a) 1 only
(b) 1 and 2
(c) 3 only
(d) 2 and 3

ANSWERS B

EXPLANATION

+ÄΑ "˜ÚêÚüΡΑ ͉͋͋͊ΡΑ W +Α ½½Ëö˜”Α ژ恫½Α «Äõ˜ÞæËÚÞΑ æËΑ ”«Ú˜Žæ½üΑ ×êڎ¨Þ˜Α £Ëõ˜ÚÄØÄæΑ ËĔÞΑ üΑ ËטīģΑ £«½æΑ
ŽŽËêÄæÞΑ ö«æ¨Α W +ΠΑ W +Α ¨ÞΑ ×ÚËõ«”˜”Α ژ恫½Α «Äõ˜ÞæËÚÞΑ ö«æ¨Α ËĽ«Ä˜Α ŽŽ˜ÞÞΑ æËΑ 樘Α £Ëõ˜ÚÄØÄæΑ ޘŽêګ櫘ÞΑ
market (primary and secondary) through the RBI (Retail Direct). Hence statement 1 is correct.
Previously, retail investors can purchase government bonds by registering themselves on stock exchanges
for non-competitive bids. Other route for retail investors is to buy government bonds is government
ޘŽêګ櫘ÞΑ 룫½æμΑ ÃêæꁽΑ ¢êĔÞΠΑ b¨˜Þ˜Α Ú˜Α ÃêæꁽΑ ¢êĔÞΑ ö¨«Ž¨Α «ÄΑ æêÚÄΑ «Äõ˜ÞæΑ «ÄΑ £Ëõ˜ÚÄØÄæΑ ޘŽêګ櫘ÞΠΑ
Negotiated Dealing System-Order Matching (NDS-OM) is a screen based electronic anonymous order
Á推«Ä£ΑÞüÞæ˜ÃΑ¢ËÚΑޘŽËĔÚüΑÁں˜æΑæځ”«Ä£Α«ÄΑ#Ëõ˜ÚÄØÄæΑޘŽêګ櫘ÞΑËöʔΑüΑW +ΠΑb¨˜ΑØ͘ÚÞ¨«×ΑË¢Α
the system is open to entities like Banks, Primary Dealers, Insurance Companies, Mutual Funds etc. i.e.
˜Äæ«æ«˜ÞΑö¨ËΑÁ«Ä恫ÄΑ[#;ΑŽŽËêÄæÞΑö«æ¨ΑW +ΠΑHence, statement 2 is correct.
˜Äæځ½Α˜×ËÞ«æËÚüΑ[˜Úõ«Ž˜ÞΑ;æ”Αλ[;μΑöÞΑ×ÚËÃË昔ΑüΑ [Α;æ”ΠΑ¶Ë«Äæ½üΑö«æ¨Α½˜”«Ä£ΑÄºÞΑÞꎨΑÞΑ[ææ˜Α
Bank of India, Bank of India, Bank of Baroda, HDFC Bank, Standard Chartered Bank and Union Bank of India.
[;ΑöÞΑޘæΑê×Αö«æ¨Α樘Αˍ¶˜Žæ«õ˜ΑË¢Α×ÚËõ«”«Ä£ΑŽËÄõ˜Ä«˜ÄæΡΑ”˜×˜Ä”½˜ΑÄ”ΑޘŽêژΑ”˜×ËÞ«æËÚüΑޘÚõ«Ž˜ÞΑæΑ
ąËڔ½˜Α ŽËÞæΑæËΑ ½½Α Áں˜æΑ ׁÚ櫎«×ÄæÞΠΑΑ ˜×ËÞ«æËÚüΑ¢Ž«½«ææ˜ÞΑ ¨Ë½”«Ä£Α Ë¢Α ޘŽêګ櫘ÞΑ «ÄΑ樘Α ˜½˜ŽæÚËÄ«ŽΑ
form and enables securities transactions to be processed by book entry. Hence, statement 3 is not
correct.

ANALYSIS:
ANALYSIS:
TÚ櫎«×æ«ËÄΑ Ë¢Α ژ恫½Α «Äõ˜ÞæËÚÞΑ «ÄΑ ×ګÁÚüΑ Áں˜æΑ ¨ÞΑ ˜˜ÄΑ «ÄΑ ĘöÞΠΑ b¨˜Α Äê͘ÚÞΑ Ú˜Α ˜û×ËĘÄ櫁½½üΑ
increasing in 2023 also.

SOURCE : ¨ææ×Þ΢έέöööΠ½«õ˜Ã«ÄæЎËÃέÃËĘüέטÚÞËā½αĈāĎ˜έ½½αüËêαʘ”αæËαºÄËö၍ËêæαΑ£Ëõ˜ÚÄα
ment-bonds-purchase-11612514003080.html
https://m.rbi.org.in/scripts/FAQView.aspx?Id=86
111
112

Year 2022 ,QɪDWLRQLQGH[HG%RQGV

V͏ΠΑr«æ¨Αژ¢˜Ú˜ÄŽ˜ΑæËΑ樘Α+Ĕ«ÄΑ˜ŽËÄËÃüΡΑö¨æΑÚ˜Α樘Α”õÄ恣˜ÞΑË¢Αϕ+Äĉæ«ËÄα+Ĕ˜û˜”Α ËĔÞΑ
(IIBs)"?
1. Government can reduce the coupon rates on its borrowing by way of IIBs.
2. IIBs provide protection to the investors from uncertainty regarding inflation.
3. The interest received as well as capital gains on IIBs are not taxable.
Which of the statements given above are correct?
(a) 1 and 2 only
(b) 2 and 3 only
(c) 1 and 3 only
(d) 1, 2 and 3

ANSWERS A

EXPLANATION

+Äĉæ«ËÄα«Ä”˜û˜”Α ËĔÞΑ «ÄΑ +Ĕ«Α ö˜Ú˜Α «ÞÞꘔΑ üΑ 樘Α W˜Þ˜Úõ˜Α ÄºΑ Ë¢Α +Ĕ«Α λW +μΑ «ÄΑ ͉͋͊͌Α Ä”Α ö˜Ú˜Α
benchmarked to Wholesale Price Index (WPI).
+Äĉæ«ËÄα«Ä”˜û˜”ΑËĔÞΑÚ˜ΑĈāĎ«½Α«ÄÞæÚêØÄæÞΑ樁æΑææ˜Ã×æΑæËΑ×ÚË明æΑ樘ΑËĔÞϖΑ×êڎ¨Þ«Ä£Α×Ëö˜ÚΑüΑ
æü«Ä£Α«Äæ˜Ú˜ÞæΑÄ”Α×ګϫׁ½ΑׁüØÄæÞΑæËΑÄΑ«Ä”˜ûΑË¢Α×Ú«Ž˜ΑŽ¨Ä£˜ÞΠΑ+Ĕ˜û˜”ΑËĔÞΑ«ÄŽ½ê”˜ΑæöËΑæüטÞΑË¢Α
ŽËÃטÄށæ«ËÄΡΑ Α ژ½Α ځæ˜Α Ë¢Α ژæêÚÄΑ ×½êÞΑ Α ŽËÃטÄށæ«ËÄΑ¢ËÚΑ樘Α ˜ÚËÞ«ËÄΑ Ë¢Α ×êڎ¨Þ«Ä£Α ×Ëö˜ÚΠΑ +Äĉæ«ËÄΑ
ŽËÃ×ËĘÄæΑËÄΑ×ګϫׁ½Αö«½½ΑÄËæΑ˜Αׁ«”Αö«æ¨Α«Äæ˜Ú˜ÞæΑêæΑ樘ΑށØΑöË꽔Α˜Α”¶êÞ昔Α«ÄΑ樘Α×ګϫׁ½ΑüΑ
Ãê½æ«×½ü«Ä£Α ×ګϫׁ½Α ö«æ¨Α «Ä”˜ûΑ ځæ«ËΑ λ+WμΠΑ æΑ 樘Α æ«Ã˜Α Ë¢Α ژ”˜Ã×æ«ËÄΡΑ ”¶êÞ昔Α ×ګϫׁ½Α ËÚΑ 樘Α ¢Ž˜ΡΑ
ö¨«Ž¨˜õ˜ÚΑ«ÞΑ¨«£¨˜ÚΡΑöË꽔Α˜Αׁ«”ΠΑ+Äæ˜Ú˜ÞæΑځæ˜Αö«½½Α˜Α×ÚËõ«”˜”Α×ÚË明æ«ËÄΑ£«ÄÞæΑ«Äĉæ«ËÄΑüΑׁü«Ä£Α
Ĉû˜”ΑŽËê×ËÄΑځæ˜ΑËÄΑ樘Α×ګϫׁ½Α”¶êÞ昔Α£«ÄÞæΑ«Äĉæ«ËÄΠΑHence statement 2 is correct.
ŽËÄËëÞæÞΑ¨õ˜ΑÚ£ê˜”Α樁æΑ«Äĉæ«ËÄΑ«Ä”˜û˜”ΑËĔÞΑŽË꽔Αژ”ꎘΑ£Ëõ˜ÚÄØÄæΑËÚÚËö«Ä£ΑŽËÞæÞΠΑ+¢Α樘Α
Áں˜æΑ Ëõ˜Ú˜Þæ«Ãæ˜ÞΑ ¢êæêژΑ «Äĉæ«ËÄΡΑ £Ëõ˜ÚÄØÄæΑ ö«½½Α ژ”ꎘΑ ËÚÚËö«Ä£Α ŽËÞæÞΑ üΑ «ÞÞê«Ä£Α «Äĉæ«ËÄΑ
indexed bonds rather than nominal bonds. This may occur because, for example, investors• expectations
Ú˜Α ÄËæΑ ŽËÃ×½˜æ˜½üΑ ¢ËÚöÚ”α½Ë˺«Ä£Α ËÚΑ ځæ«Ëā½ΠΑ ½æ˜Úāæ«õ˜½üΡΑ 樘Α £Ëõ˜ÚÄØÄæΡΑ ˜ŽêޘΑ «æΑ «ÞΑ ½˜Α æËΑ
«Äĉê˜ÄŽ˜Α«Äĉæ«ËÄΑæ¨ÚË꣨Α«æÞΑ×˽«Ž«˜ÞΡΑÁüΑ¨õ˜Α˜ææ˜ÚΑ«Ä¢ËÚÁæ«ËÄΑËêæΑ樘Α¢êæêژΑŽËêÚޘΑË¢Α«Äĉæ«ËÄΡΑËÚΑ
perhaps has more faith in its commitment to contain it than the public does. In these cases a treasury can
½Ëö˜ÚΑ«æÞΑŽËÞæÞΑüΑ«ÞÞê«Ä£Α«Ä”˜û˜”ΑËĔÞΠΑ"ËÚΑ˜ûÃ×½˜ΡΑ«¢ΑŽËê×ËÄΑځæ˜ΑË¢Α++ ÞΑ«ÞΑĈû˜”Α͊Π͎ΑАΑËõ˜ΑrT+Αλr¨Ë½˜Α
ށ½˜Α×Ú«Ž˜Α«Ä”˜ûμΑÄ”ΑŽêÚژÄæΑrT+Α«ÞΑ͍АΡΑÞËΑ˜ą˜Žæ«õ˜Αځæ˜Αö«½½Α˜Α͎Π͎АΑλ͍Ϲ͊Π͎μΠΑ+ÄΑ¢êæêژΡΑö¨˜ÄΑrT+Α¢½½ÞΑ¢ÚËÃΑ
͍АΑæËΑ͋АΡΑ樘ÄΑ˜ą˜Žæ«õ˜ΑŽËê×ËÄΑځæ˜Αö«½½Α˜ŽËØΑ͌Π͎АΑλ«Π˜Α͋Ϲ͊Π͎μΑÄ”Αæ¨êÞΑ£Ëõ˜ÚÄØÄæΑŽÄΑژ”ꎘΑ樘Α
ŽËê×ËÄΑځæ˜ÞΑËÄΑ«æÞΑËÚÚËö«Ä£ΑüΑöüΑË¢Α++ ÞΑæ¨ÚË꣨Αژ”ꎫģΑ«Äĉæ«ËāÚüΑæژĔÞΠΑHence statement 1 is
correct.
ûæÄæΑæûΑ ×ÚËõ«Þ«ËÄÞΑö«½½Α ˜Α ××½«Ž½˜Α ËÄΑ «Äæ˜Ú˜ÞæΑ ׁüØÄæΑ Ä”Α Ž×«æ½Α £«ÄÞΑ ËÄΑ ++ ÞΠΑb¨˜Ú˜Αö«½½Α ˜Α ÄËΑ
special tax treatment for these bonds. Hence statement 3 is not correct.

ANALYSIS:

++ ÞΑö˜Ú˜Α«ÄΑĘöÞΡΑÞΑW +ΑæÚ«˜ÞΑæËΑ£ê£˜Α”˜ÃÄ”Α¢ËÚΑ樘ΑށØΑ”ê˜ΑæËΑ«ÄŽÚ˜Þ«Ä£Α«Äĉæ«ËÄΠ

SOURCE : https://m.rbi.org.in/Scripts/FAQView.aspx?Id=91
113

Year 2022 CREDIT RATING AGENCY

Q7. Consider the following statements:


1. In India, credit rating agencies are regulated by Reserve Bank of India.
2. The rating agency popularly known as ICRA is a public limited company.
3. Brickwork Ratings is an Indian credit rating agency.
Which of the statements given above are correct.
(a) 1 and 2 only
(b) 2 and 3 only
(c) 1 and 3 only
(d) 1, 2 and 3

ANSWERS B

EXPLANATION

Credit rating agencies in India are regulated by SEBI. The SEBI (Credit Rating Agencies) Regulations, 1999
govern the credit rating agencies and provide for eligibility criteria for registration of credit rating
£˜ÄŽ«˜ÞΡΑÃËÄ«æËÚ«Ä£ΑÄ”Αژõ«˜öΑË¢Αځæ«Ä£ÞΡΑژÙê«Ú˜Ã˜ÄæÞΑ¢ËÚΑΑ×ÚËטÚΑځæ«Ä£Α×Úˎ˜ÞÞΡΑõË«”ÄŽ˜ΑË¢ΑŽËÄĉ«ŽæΑ
of interest and inspection of rating agencies by SEBI, amongst other things. Hence statement 1 is not
correct.
+WΑ;«Ã«æ˜”Αλ¢ËÚØڽüΑ+Äõ˜ÞæØÄæΑ+Ä¢ËÚÁæ«ËÄΑÄ”Αژ”«æΑWæ«Ä£Α£˜ÄŽüΑË¢Α+Ĕ«Α;«Ã«æ˜”μΑöÞΑޘæΑê×Α«ÄΑ
͊͒͒͊ΑüΑ½˜”«Ä£ΑĈāĎ«½έ«Äõ˜ÞæØÄæΑ«ÄÞæ«æêæ«ËÄÞΡΑŽËÃØڎ«½ΑÄºÞΑÄ”ΑĈāĎ«½ΑޘÚõ«Ž˜ÞΑŽËÃׁī˜ÞΑÞΑ
an independent and professional investment Information and Credit Rating Agency. Today, ICRA and its
ÞêÞ«”«Ú«˜ÞΑæË£˜æ¨˜ÚΑ¢ËÚÃΑ樘Α+WΑ#ÚËê×ΑË¢ΑËÃׁī˜ÞΑλ#ÚËê×Α+WμΠΑ+WΑ«ÞΑΑTꍽ«ŽΑ;«Ã«æ˜”ΑËÃׁÄüΡΑ
ö«æ¨Α«æÞΑÞ¨Ú˜ÞΑ½«Þ昔ΑËÄΑ樘Α Ë́üΑ[æˎºΑûŽ¨Ä£˜ΑÄ”Α樘ΑBæ«Ëā½Α[æˎºΑûŽ¨Ä£˜ΠΑHence statement
2 is correct.
Ú«ŽºöËÚºΑ Wæ«Ä£ÞΑ λ rWμΡΑ «ÞΑ Α [ +Α ژ£«Þæ˜Ú˜”Α +Ĕ«ÄΑ ژ”«æΑ Wæ«Ä£Α £˜ÄŽüΑ ¢ËêĔ˜”Α üΑ Äº˜ÚÞΡΑ ŽÚ˜”«æΑ
ځæ«Ä£Α×ÚË¢˜ÞÞ«Ëā½ÞΡΑ¢ËÚØÚΑژ£ê½æËÚÞΑÞΑö˜½½ΑÞΑ×ÚË¢˜ÞÞËÚÞΡΑ«ÞΑŽËÃëæ昔ΑæËΑ×ÚËÃËæ«Ä£Α"«ÄÄŽ«½Α;«æ˜ÚŽüΠΑ
Ú«ŽºöËÚºΑWæ«Ä£ÞΑ¨ÞΑÄÚΑ ÄºΡΑΑ½˜”«Ä£ΑTꍽ«ŽΑ[˜ŽæËÚΑ ÄºΡΑÞΑ«æÞΑ×ÚËÃËæ˜ÚΑÄ”ΑÞæځ昣«ŽΑׁÚæĘÚΠΑ
Hence statement 3 is correct.

ANALYSIS:
W˜Ž˜Äæ½üΡΑ[bΑޘæÞΑÞ«”˜ΑWÞΑ͊ΑŽÚËژΑטā½æüΑ«Ã×Ëޘ”ΑËÄΑ Ú«ŽºöËÚºΑWæ«Ä£ÞΑüΑ樘Α[ +

Scope for elimination


+æΑ«ÞΑÄËæΑ”«ĆŽê½æΑæËΑ£ê˜ÞÞΑ樁æΑŽÚ˜”«æΑځæ«Ä£Α£˜ÄŽ«˜ÞΑ«ÄΑ+Ĕ«ΑÚ˜Αژ£ê½æ˜”ΑêĔ˜ÚΑ樘Α[ +ΑλÞΑ«æΑ«ÞΑژ½æ˜”ΑæËΑ
the capital market). Thus, there is evidently a scope of elimination.

SOURCE : ¨ææ×Þ΢έέöööΠޘ«Π£ËõΠ«Äέޘ«ε”æέ¢ÙĈ½˜Þέˎæα͉͋͋͊έ͊͏͍͉͌͒͋͏͏͍͌͐͊Πה¢

https://www.business-standard.com/article/companies/sat-sets-aside-rs-1-cr-penalty-
imposed-on-brickwork-ratings-by-sebi-122032801153_1.html

https://www.brickworkratings.com/Aboutus.aspx

¨ææ×Þ΢έέöööΠ«ŽÚΠ«Äέ(ËØέTÚËĈ½˜ά΢Ѕ΢æ˜ûæϽ+WА͉͋«ÞА͉͋А͉͋Tꍽ«ŽА͉͋;«Ã«æ˜”ΡÄ”АΑ
͉͋樘А͉͋Bæ«Ëā½А͉͋[æˎºА͉͋ûŽ¨Ä£˜
114

Year 2022 CONVERTIBLE BONDS

Q8. With reference to Convertible Bonds, consider the following statements:


1. As there is an option to exchange the bond for equity, Convertible Bonds pay a lower rate of interest.
2. The option to convert to equity affords the bondholder a degree of indexation to rising consumer prices.
Which of the statements given above is/are correct?
(a) 1only
(b) 2 only
(c) Both 1 and 2
(d) Neither 1 nor 2

ANSWERS C

EXPLANATION

ΑŽËÄõ˜Ú櫍½˜ΑËĔΑ«ÞΑΑĈû˜”α«ÄŽËØΑŽËÚ×Ëځæ˜Α”˜æΑޘŽêÚ«æüΑ樁æΑü«˜½”ÞΑ«Äæ˜Ú˜ÞæΑׁüØÄæÞΡΑêæΑŽÄΑ˜Α
ŽËÄõ˜Ú昔Α«ÄæËΑΑ×ژ”˜æ˜ÚëʔΑÄê͘ÚΑË¢ΑŽËÃÃËÄΑÞæˎºΑËÚΑ˜Ùê«æüΑÞ¨Ú˜ÞΠΑ+æΑËą˜ÚÞΑ«Äõ˜ÞæËÚÞΑΑæüטΑË¢Α
¨üÚ«”ΑޘŽêÚ«æüΑ樁æΑ¨ÞΑ¢˜æêژÞΑË¢ΑΑËĔΡΑÞꎨΑÞΑ«Äæ˜Ú˜ÞæΑׁüØÄæÞΡΑö¨«½˜Α½ÞËΑ¨õ«Ä£Α樘ΑË×æ«ËÄΑæËΑ
ËöÄΑ樘ΑêĔ˜Ú½ü«Ä£ΑÞæˎºΠ
+ÞÞê«Ä£ΑŽËÄõ˜Ú櫍½˜ΑËĔÞΑŽÄΑ¨˜½×ΑŽËÃׁī˜ÞΑëīëā˜ΑĘ£æ«õ˜Α«Äõ˜ÞæËÚΑޘÄæ«Ã˜ÄæΑ樁æΑöË꽔ΑÞêÚÚËêĔΑ
˜Ùê«æüΑ«ÞÞêÄŽ˜Π"êÚ樘ÚΡΑ«ÞÞê«Ä£ΑŽËÄõ˜Ú櫍½˜ΑËĔÞΑŽÄΑ½ÞËΑ¨˜½×Α×ÚËõ«”˜Α«Äõ˜ÞæËÚÞΑö«æ¨ΑÞËØΑޘŽêÚ«æüΑ«ÄΑ
樘Α˜õ˜ÄæΑË¢Α”˜¢ê½æΠΑΑŽËÄõ˜Ú櫍½˜ΑËĔΑ×ÚË明æÞΑ«Äõ˜ÞæËÚÞϖΑ×ګϫׁ½ΑËÄΑ樘Α”ËöÄÞ«”˜ΡΑêæΑ½½ËöÞΑ樘ÃΑæËΑ
ׁÚ櫎«×æ˜Α«ÄΑ樘Αê×Þ«”˜ΑÞ¨Ë꽔Α樘ΑêĔ˜Ú½ü«Ä£ΑŽËÃׁÄüΑÞꎎ˜˜”ΠΑ(Ëö˜õ˜ÚΡΑŽËÄõ˜Ú櫍½˜ΑËĔÞΑæ˜Ä”ΑæËΑ
Ëą˜ÚΑΑ½Ëö˜ÚΑŽËê×ËÄΑځæ˜ΑËÚΑځæ˜ΑË¢ΑژæêÚÄΑ«ÄΑ˜ûŽ¨Ä£˜Α¢ËÚΑ樘Αõ½ê˜ΑË¢Α樘ΑË×æ«ËÄΑæËΑŽËÄõ˜ÚæΑ樘ΑËĔΑ«ÄæËΑ
ŽËÃÃËÄΑÞæˎºΠΑËÃׁī˜ÞΑ˜Ä˜ĈæΑޫϘΑ樘üΑŽÄΑ«ÞÞê˜Α”˜æΑæΑ½Ëö˜ÚΑ«Äæ˜Ú˜ÞæΑځæ˜ÞΑ樁ÄΑö«æ¨Αæځ”«æ«Ëā½Α
ËĔΑËą˜Ú«Ä£ÞΠΑ(Ëö˜õ˜ÚΡΑÄËæΑ½½ΑŽËÃׁī˜ÞΑËą˜ÚΑŽËÄõ˜Ú櫍½˜ΑËĔÞΠΑHence statement 1 is correct.
b¨˜ΑË×æ«ËÄΑæËΑŽËÄõ˜ÚæΑæËΑ˜Ùê«æüΑąËڔÞΑ樘ΑËĔ¨Ë½”˜ÚΑΑ”˜£Ú˜˜ΑË¢Α«Ä”˜ûæ«ËÄΑæËΑÚ«Þ«Ä£ΑŽËÄÞêØÚΑ×Ú«Ž˜ÞΠΑ
ÞΑ «Ä”˜ûæ«ËÄΑ ö«½½Α ˜ÄÞêژΑ 樁æΑ ×Ú«Ž˜ÞΑ Ú˜Α ”¶êÞ昔Α ö«æ¨Α «Äĉæ«ËÄΑ Ëõ˜ÚΑ Α טګ˔Α Ë¢Α æ«Ã˜Πr«æ¨Α 樘Α ¨˜½×Α Ë¢Α
«Ä”˜ûæ«ËÄΡΑ ËĔ¨Ë½”˜ÚÞΑö«½½Α ˜Α ½˜ΑæËΑ ½Ëö˜ÚΑ樘«ÚΑ ½ËÄ£αæ˜ÚÃΑ Ž×«æ½Α £«ÄÞΑ λÞΑ樘«ÚΑ «Äõ˜ÞæØÄæΑö«½½Α ˜Α
”¶êÞ昔Αö«æ¨Α«Äĉæ«ËÄμΑ˜õ˜ÄΑö¨˜ÄΑŽËÄõ˜Úæ«Ä£ΑËĔÞΑ«ÄæËΑ˜Ùê«æüΡΑö¨«Ž¨ΑÚ«Ä£ÞΑ”ËöÄΑ樘«ÚΑæû½˜Α«ÄŽËØΠ
Hence statemnet 2 is correct.

ANALYSIS:
ANALYSIS:
There is no scope for elimination as it is a 2-statement question.

SOURCE : ¨ææ×Þ΢έέöööΠ«Äõ˜ÞæËט”«ΠŽËÃέæ˜ÚÃÞݎݎËÄõ˜Ú櫍½˜ËÄ”ΠÞ×ά΢Ѕ΢α
æ˜ûæϽ(Ëö˜õ˜ÚА͋А͉͋ŽËÄõ˜Ú櫍½˜А͉͋ËĔÞА͉͋æ˜Ä”А͉͋æËс½½А͉͋ŽËÃׁī˜ÞА͉͋Ëą˜ÚА͉͋Ž
ËÄõ˜Ú櫍½˜А͉͋ËĔÞΠ

¨ææ×Þ΢έέöööИŽËÄËëÞæЎËÃέĈāϘαÄ”α˜ŽËÄËÃα
ics/2021/07/10/why-convertible-bonds-are-the-asset-class-for-the-times
115

Year 2023 CAPITAL MARKET

Q9. Consider the following markets:


1. Government Bond Market
2. Call Money Market
3. Treasury Bill Market
4. Stock Market
How many of the above are included in capital markets?
(a) Only one
(b) Only two
(c) Only three
(d) All four

ANSWERS B

EXPLANATION

ΑŽ×«æ½ΑÁں˜æΑ«ÞΑΑĈāĎ«½ΑÁں˜æΑö¨˜Ú˜Α½ËÄ£αæ˜ÚÃΑ”˜æΑËÚΑ˜Ùê«æüፁŽº˜”ΑޘŽêګ櫘ÞΑÚ˜ΑË꣨æΑÄ”Α
sold. It includes 1. Government Bonds - In India, fall under the broad category of government securities
λ#α[˜ŽμΑÄ”ΑÚ˜Α×ګÁګ½üΑ½ËÄ£Αæ˜ÚÃΑ«Äõ˜ÞæØÄæΑæË˽ÞΑ«ÞÞꘔΑ¢ËÚΑטګ˔ÞΑځģ«Ä£Α¢ÚËÃΑ͎ΑæËΑ͍͉Αü˜ÚÞΠΑ+æΑŽÄΑ
be issued by both Central and State governments of India. 2. Capital market refers to a broad spectrum of
æځ”˜½˜Α ÞޘæÞΑ 樁æΑ «ÄŽ½ê”˜ÞΑ 樘Α ÞæˎºΑ Áں˜æΑ ÞΑ ö˜½½Α ÞΑ Ë樘ÚΑ õ˜Äê˜ÞΑ ¢ËÚΑ æځ”«Ä£Α ”«ą˜Ú˜ÄæΑ ĈāĎ«½Α
products. Treasury bills are money market instruments issued by the Government of India and call
money market is an essential part of the Indian Money Market, ö¨˜Ú˜Α 樘Α ”üαæËᔁüΑ ÞêÚ×½êÞΑ ¢êĔÞΑ
(mostly of banks) are traded.
Hence option (b) is the correct answer.

ANALYSIS:
ANALYSIS:

Scope for elimination


b¨«ÞΑ «ÞΑ Α Þ«ŽΑ ½˜õ˜½Α Ùê˜Þæ«ËÄΣΑ ¨˜ÄŽ˜Α 樘Α æöËΑ ÃËĘüΑ Áں˜æΑ «ÄÞæÚêØÄæÞΑ ŽÄΑ ˜Α ˜Þ«½üΑ ˜½«Ã«Äæ˜”ΠΑ b¨˜Α
question intends to test the basic understanding of the candidate.

SOURCE : Business Studies (NCERT Class 12)


Excerpt from NCERT
116

Year 2023 TERMS USED IN FINANCIAL MARKET

V͉͊ΠΑ+ÄΑ樘ΑŽËÄæ˜ûæΑË¢ΑĈāϘΡΑ樘Αæ˜ÚÃΑϖ˜æϖΑژ¢˜ÚÞΑæËΑ
(a) the process of simultaneous buying and selling of an asset from difference platforms.
(b) an investment strategy of a portfolio manager to balance risk versus reward.
(c) a type of systemic risk that arises where perfect hedging is not possible.
(d) a numeric value that measures the fluctuations of a stock to changes in the overall stock market.

ANSWERS D

EXPLANATION
Beta (β) is a measure of the volatility—or systematic risk—of a security or portfolio compared to the market
ÞΑΑö¨Ë½˜ΠΑÙê«æ«˜ÞΑ¨õ«Ä£ΑΑ˜æΑõ½ê˜Α½Ú£˜ÚΑ樁ÄΑËĘΡΑËÚΑ¨«£¨Α˜æΑÞæˎºÞΡΑÚ˜ΑË¢æ˜ÄΑºÄËöÄΑÞΑõ˽æ«½˜Α
ÞæˎºÞΠΑb¨˜ΑÞ½«£¨æ˜ÞæΑ”¶êÞæØÄæÞΑ«ÄΑÞæˎºΑÁں˜æΑ«Ä”«ŽæËÚÞΑ¨õ˜ΑΑ«£Α«Äĉê˜ÄŽ˜ΑËÄΑ樘ÃΠΑΑޘŽêÚ«æüΑ樁æΑ«ÞΑ
ŽËÃׁځæ«õ˜½üΑÃËژΑÞ恍½˜Α«ÞΑΑ½ËöΑ˜æΑÞæˎºΑ«Π˜ΠΑ¨ÞΑΑ˜æΑځæ«Ä£Α˜½ËöΑ͊ΠΑHence option (d) is the correct
answer.

ANALYSIS:
ANALYSIS:

Scope for elimination


No scope as it is a factual question.
+æΑ¨ÞΑ½öüÞΑ˜˜ÄΑ«ÄΑ樘ΑĘöÞΑêæΑÞꎨΑæ˜ÚÃÞΑÄ”ΑŽËϘ×æÞΑ˜Þ«½üΑ˜ÞŽ×˜Α樘ΑÄË櫎˜ΑË¢Α樘ΑÞ׫ځÄæÞΑÞΑ
‘stock markets’ is not conventionally studied under UPSC-related economics, but as the craze for stock
Áں˜æÞΑÄ”ΑÃêæꁽΑ¢êĔÞΑÚ˜Α«ÄŽÚ˜Þ«Ä£ΑÄ”Αæ¨ΑüËêæ¨Α«ÞΑĈĔ«Ä£Α«æΑÃËژΑ×ט½«Ä£ΡΑæ¨êÞΑfT[Α¨ÞΑ˜£êÄΑ
making foray into that domain.

SOURCE : 7KH+LQGXKDGH[SODLQHGWKHWHUPȏEHWDȐH[WHQVLYHO\EXWORQJDJRLQDUWLFOHLQDQDUWLFOHLQ-XQH
¨ææ×Þ΢έέöööΠ樘¨«Ä”êЎËÃέË׫īËÄέË×α˜”έö¨æα”˘Þፘæα؁ÄÞα«ÄαĈāĎ˜έÚ櫎½˜Α
19165973.ece?homepage=true

Year 2023 ,QIUDVWUXFWXUH,QYHVWPHQW7UXVWV ,QY,7V

Q11. Consider the following statements:


Statement-I: Interest income from the deposits in Infrastructure Investment Trusts (InvITs) distributed to their investors
is exempted from tax, but the dividend is taxable.
Statement-II: InvITs are recognized as borrowers under the 'Securitization and Reconstruction of Financial Assets and
Enforcement of Security Interest Act, 2002'.
Which one of the following is correct in respect of the above statements?
(a) Both Statement-I and Statement-II are correct and Statement-II is the correct explanation for Statement-I
(b) Both Statement-I and Statement-II are correct and Statement-II is not the correct explanation for Statement-I
(c) Statement-I is correct but Statement-II is incorrect
(d) Statement-I is incorrect but Statement-II is correct

ANSWERS D
117

EXPLANATION
Statement I is incorrect: The Finance (No.2) Act, 2014 introduced a special taxation regime for Real Estate
Investment Trust (REIT) and Infrastructure Investment Trust (InVIT) (commonly referred to as business
æÚêÞæÞμΠΑ+Äõ+bÞΑ×ÚËõ«”˜ΑæöËΑ”«ą˜Ú˜ÄæΑæüטÞΑË¢ΑژæêÚÄÞΑæËΑ«Äõ˜ÞæËÚÞΑγΑ«õ«”˜Ä”Α+ĎËØΑÄ”Α×«æ½Α#«ÄÞΠΑ
Ž¨ΑæüטΑË¢Α«ÄŽËØΑ¨ÞΑΑ”«ą˜Ú˜ÄæΑæûΑæژæØÄæΠ Any dividend or interest income that you get from an
InvIT is completely taxable as per your Income Tax Slab rate. This income has to be declared every year in
your Income Tax Return under the head “Income from Other Sources”.
Statement II is correct: As per the Securitization and Reconstruction of Financial Assets and
Ä¢Ëڎ˜Ã˜ÄæΑË¢Α[˜ŽêÚ«æüΑ+Äæ˜Ú˜ÞæΑŽæΡΑ͉͉͋͋ΡΑύËÚÚËö˜ÚώΑ؁ÄÞΑοÄüΑטÚÞËÄΑö¨ËΡΑËÚΑΑ×Ë˽˜”Α«Äõ˜ÞæØÄæΑ
õ˜¨«Ž½˜ΑÞΑ”˜ĈʔΑ«ÄΑŽ½êޘΑ딁μΑË¢ΑޘŽæ«ËÄΑ͋ΑË¢Α樘Α[˜Žêګ櫘ÞΑËÄæځŽæÞΑλW˜£ê½æ«ËÄμΑŽæΡΑ͎͊͒͏Αλ͍͋ΑË¢Α͎͊͒͏μΑ
ö¨«Ž¨ΡπΑ¨ÞΑ˜˜ÄΑ£ÚÄ昔ΑĈāĎ«½ΑÞÞ«ÞæÄŽ˜ΑüΑÄüΑÄºΑËÚΑĈāĎ«½Α«ÄÞæ«æêæ«ËÄΑËÚΑö¨ËΑ¨ÞΑ£«õ˜ÄΑÄüΑ
£êÚÄ昘ΑËÚΑŽÚ˜æ˜”ΑÄüΑÃËÚ棁£˜ΑËÚΑ×½˜”£˜ΑÞΑޘŽêÚ«æüΑ¢ËÚΑ樘ΑĈāĎ«½ΑÞÞ«ÞæÄŽ˜Α£ÚÄ昔ΑüΑÄüΑÄºΑ
ËÚΑ ĈāĎ«½Α «ÄÞæ«æêæ«ËÄΑ οÄ”Α «ÄŽ½ê”˜ÞΑ Α טÚÞËÄΑ ö¨ËΡΑ ËÚΑ Α ×Ë˽˜”Α «Äõ˜ÞæØÄæΑ õ˜¨«Ž½˜Α ö¨«Ž¨ΡπΑ ˜ŽËØÞΑ
ËÚÚËö˜ÚΑË¢ΑΑοÞޘæΑژŽËÄÞæÚêŽæ«ËÄΑŽËÃׁÄüπΑŽËÄޘÙê˜ÄæΑê×ËÄΑŽÙê«Þ«æ«ËÄΑüΑ«æΑË¢ΑÄüΑÚ«£¨æÞΑËÚΑ«Äæ˜Ú˜ÞæΑ
Ë¢ΑÄüΑÄºΑËÚΑĈāĎ«½Α«ÄÞæ«æêæ«ËÄΑ«ÄΑژ½æ«ËÄΑæËΑÞꎨΑĈāĎ«½ΑÞÞ«ÞæÄŽ˜Π

ANALYSIS:
ANALYSIS:

Scope for elimination


£«ÄΡΑ˜½«Ã«Äæ«ËÄΑ明¨Ä«Ùê˜Α«ÞΑÃËژΑ”˜×˜Ä”˜ÄæΑËÄΑºÄËö½˜”£˜ΡΑ«¢ΑËĘΑºÄËöÞΑ樁æΑÞææ˜Ã˜ÄæΑ+Α«ÞΑöÚËÄ£Α樁æΑ
«ÞΑËæ¨ΑϏ«Äæ˜Ú˜ÞæΑ«ÄŽËØϐΑÄ”ΑϏ”«õ«”˜Ä”ϐΑژŽ˜«õ˜Α樘ΑށØΑæûΑæژæØÄæΡΑ樘ÄΑ«æΑ«ÞΑöÚËÄ£ΠΑĔΑ«ÞΑ[ææ˜Ã˜ÄæΑ+Α
«ÞΑöÚËÄ£ΡΑË×æ«ËÄΑλ”μΑ«ÞΑ樘ΑŽËÚژŽæΑÄÞö˜ÚΠ
How to cover? InVITs and REITS are neither covered in the Introductory Macroeconomics book (NCERT
Class 12) nor in the NCERT Business Studies book. However, standard newspapers cover them from
time to time.

Additional Information
What is interest income and dividend income?
r¨«½˜Α«Äæ˜Ú˜ÞæΑ«ÄŽËØΑ«ÞΑ˜ÚʔΑüΑΑ½˜Ä”˜ÚΑ¢ÚËÃΑΑËÚÚËö˜ÚΑλŽËÃׁÄüμΑËÄΑ樘Α
funds given by the lender, dividend is the amount received by the shareholder of
ΑŽËÃׁÄüΑ¢ÚËÃΑ樘Α×ÚËĈæÞΑÁ”˜ΑüΑΑŽËÃׁÄüΠ
«ą˜Ú˜ÄŽ˜Α˜æö˜˜ÄΑ+Äõ+bÞΑÄ”ΑW+bÞ
+ÄΑÞ«Ã×½˜ΑöËڔÞΡΑËæ¨Α+Äõ+bÞΑÄ”ΑW+bÞΑÚ˜ΑÃêæꁽΑ¢êĔÞΑ½«º˜Α«ÄÞæÚêØÄæÞΑö¨«Ž¨Α
恺˜Α ÃËĘüΑ ¢ÚËÃΑ 樘Α «Äõ˜ÞæËÚÞΑ Ä”Α «Äõ˜ÞæΑ 樘ÃΑ «ÄΑ ”«ą˜Ú˜ÄæΑ ×Ú˶˜ŽæÞΠΑ +Äõ+bÞΑ
«Äõ˜ÞæΑ樘ÃΑ«ÄΑ«Ä¢ÚÞæÚêŽæêژΑ×Ú˶˜ŽæÞΑ½«º˜ΑÚˁ”ΑŽËÄÞæÚêŽæ«ËÄΡΑژĘö½˜Α˜Ä˜Ú£üΑ
×Ú˶˜ŽæÞΑö¨«½˜ΑW+bÞΑ«Äõ˜ÞæΑ樘ÃΑ«ÄΑ樘ΑŽËÄÞæÚêŽæ«ËÄΑË¢Αê«½”«Ä£ÞΡΑËƎ˜ÞΡΑ˜æŽΠ
118

INDIAN ECONOMY
LAND REFORMS
(1 Questions)

Year 2019 LAND REFORMS AFTER INDEPENDENCE

EXPLANATION
119

ANALYSIS:

Scope for elimination

AGRICULTURE
(17 Questions)

Year 2017 SOIL HEALTH CARD

Q1. The nation-wide ‘Soil Health Card Scheme’ aims at


1. expanding the cultivable area under irrigation.
2. enabling the banks to assess the quantum of loans to be granted to farmers on the basis of soil quality.
3. checking the overuse of fertilizers in farmlands.
Which of the above statements is/are correct?
(a) 1 and 2 only
(b) 3 only
(c) 2 and 3 only
(d) 1, 2 and 3

EXPLANATION
Launched by the central government in February 2015, the scheme is tailor-made to issue ‘Soil card’ to
farmers which will carry crop-wise recommendations of nutrients and fertilizers required for the individual
farms. This is aimed to help farmers to improve productivity through judicious use of inputs. Hence only
statement 3 is correct.
Under the scheme, the government issues Soil Health Cards (SHC) to farmers. A SHC provide soil nutrient
status of his holding to each farmer and advise him/ her on the dosage of fertilizers and also the needed
soil amendments that should be applied to maintain soil health in the long run.

ANALYSIS:
ANALYSIS:
Soil Health Card was a recent theme
then. And also the phase I of the
scheme ended in 2017. Students are
advised to maintain a list of Flagship
Schemes and Policies of the Govern-
ment. Follow developments associat-
ed with them and thoroughly go
through their features.
120

SOURCE : http://pib.snic.in/newsite/PrintRelease.aspx?relid=159441
https://informatics.nic.in/uploads/pdfs/1a479603_soilhealthcard.pdf

Year 2017 NATIONAL AGRICULTURAL MARKET

Q2. What is/are the advantage/advantages of implementing the ‘National Agriculture Market’
scheme?
1. It is a pan-India electronic trading portal for agricultural commodities.
2. It provides the farmers access to nationwide market, with prices commensurate with the quality of their produce.
Select the correct answer using the code given below:
(a) 1 only
(b) 2 only
(c) Both 1 and 2
(d) Neither 1 nor 2

ANSWERS C

EXPLANATION
National Agriculture Market (NAM) is a pan-India electronic trading portal which networks the existing
TAΑ ÁĔ«ÞΑ æËΑ ŽÚ˜æ˜Α Α êÄ«Ĉ˜”Α āæ«Ëā½Α Áں˜æΑ ¢ËÚΑ £Ú«Žê½æêځ½Α ŽËÃÃ˔«æ«˜ÞΠΑ Hence, statement 1 is
correct.
BAΑ×ÚËÃËæ˜ÞΑêÄ«¢ËÚëæüΡΑÞæژÃ½«Ä«Ä£ΑË¢Α×Úˎ˜”êژÞΑŽÚËÞÞΑ樘Α«Ä昣ځ昔ΑÁں˜æÞΡΑژÃËõ˜ÞΑ«Ä¢ËÚÁæ«ËÄΑ
ÞüÃØæÚüΑ˜æö˜˜ÄΑêü˜ÚÞΑÄ”Αޘ½½˜ÚÞΑÄ”Α×ÚËÃËæ˜ÞΑژ½Αæ«Ã˜Α×Ú«Ž˜Α”«ÞŽËõ˜ÚüΡΑÞ˜”ΑËÄΑŽæꁽΑ”˜ÃÄ”Α
Ä”ΑÞê××½üΡΑ×ÚËÃËæ˜ÞΑæځÄÞׁژĎüΑ«ÄΑêŽæ«ËÄΑ×Úˎ˜ÞÞΡΑÄ”ΑŽŽ˜ÞÞΑæËΑΑāæ«ËÄö«”˜ΑÁں˜æΑ¢ËÚΑ樘Α¢ÚØÚΡΑ
ö«æ¨Α×Ú«Ž˜ÞΑŽËÃØÄÞêځæ˜Αö«æ¨ΑÙꁽ«æüΑË¢Α¨«ÞΑ×Ú˔ꎘΑÄ”ΑËĽ«Ä˜ΑׁüØÄæΑÄ”Αõ«½«½«æüΑË¢Α˜ææ˜ÚΑÙꁽ«æüΑ
×Ú˔ꎘΑÄ”ΑæΑÃËژΑژÞËā½˜Α×Ú«Ž˜ÞΑæËΑ樘ΑŽËÄÞêØÚΠΑHence, statement 2 is correct.

ANALYSIS:
"ÚØÚÞΑ”«ÞæژÞÞΑ¨ÞΑ½öüÞΑ˜˜ÄΑ«ÄΑ樘ΑĘöÞΠΑΑb¨˜«ÚΑ½ŽºΑË¢Α£˜ææ«Ä£Α”˜Ùêæ˜Α×Ú«Ž˜ÞΑ¢ËÚΑ樘«ÚΑ×Ú˔ꎘΑ«ÞΑΑ
טژÄÄ«½Α×Úˍ½˜ÃΠ
ANALYSIS:
SOURCE : https://www.enam.gov.in/web/
121

Year 2017 AGRICULTURAL TRADE

Q3. Consider the following statements:


1. The quantity of imported edible oils is more than the domestic production of edible oils in the last five years.
2. The Government does not impose any customs duty on all the imported edible oils as a special case.
Which of the statements given above is/are correct?
(a) 1 only
(b) 2 only
(c) Both 1 and 2
(d) Neither 1 nor 2

ANSWERS A

EXPLANATION
Please note: Dynamic tools like taxes, duties, expenditure keep changing from year to year. The
Ùê˜Þæ«ËÄΑ öÞΑ Þº˜”Α «ÄΑ ͉͋͊͑ΠΑ (˜ÄŽ˜ΡΑ «ÄΑ 樫ÞΑ ŽËÄæ˜ûæΡΑ Ϗ½ÞæΑ Ĉõ˜Α ü˜ÚÞϐΑ ؁ÄÞΑ ͉͋͊͌α͉͋͊͑Α ËÚΑ ͉͋͊͋α͊͌Α æËΑ
͉͋͊͐α͊͑Π
Statement 1 is correct: +Ĕ«Α ˎŽê׫˜ÞΑ Α ×ÚËëĘÄæΑ ×ËÞ«æ«ËÄΑ «ÄΑ 樘Α öËÚ½”Α Ë«½Þ˜˜”ÞΑ «Ä”êÞæÚüΑ ö«æ¨ΑΑ
ŽËÄæÚ«êæ«ËÄΑË¢ΑÚËêĔΑ͉͊АΑ«ÄΑöËÚ½”ö«”˜Α×Ú˔êŽæ«ËÄΠΑ êæΑ樘Α”˜ÃÄ”ΑË¢Α˜”«½˜ΑË«½ÞΑλ˜ûæځŽæ˜”Α¢ÚËÃΑË«½Þ˜˜”ÞΑ
«ÄΑ””«æ«ËÄΑæËΑׁ½ÃΑË«½μΑ«ÞΑÞ«£Ä«ĈŽÄæ½üΑ¨«£¨˜ÚΑ樁ÄΑ樘Α”ËØÞ櫎Α×Ú˔êŽæ«ËÄΡΑ½˜”«Ä£ΑæËΑΑΑΑΑΑΑΑΑΑΑΑΑΑΑΑΑΑΑΑΑΑΑΑΑΑΑΑΑΑΑΑΑ”˜×˜Äα
”˜ÄŽ˜ΑËÄΑ«Ã×ËÚæÞΑλËêæΑ͎͏ΑАΑË¢Α”ËØÞ櫎Α”˜ÃÄ”ΑË¢Α˜”«½˜ΑË«½ÞΑ«ÞΑØæΑæ¨ÚË꣨Α«Ã×ËÚæÞΑËêæΑË¢Αö¨«Ž¨Αׁ½ÃΑ
Ë«½έׁ½Ã˽˜«ÄΑŽËÄÞæ«æêæ˜ÞΑËêæΑ͎͍АΠμΠΑ
Statement 2 is not correct:Α#Ëõ˜ÚÄØÄæΑ«Ã×ËޘÞΑŽêÞæËÃÞΑ”êæüΑËÄΑ˜”«½˜ΑË«½ÞΑæËΑށ¢˜£êÚ”Α樘Α«Äæ˜Ú˜ÞæÞΑ
Ë¢Α”ËØÞ櫎ΑË«½ΑŽÚêÞ¨«Ä£Α«Ä”êÞæÚüΠΑb¨˜Α”êæüΑËÄΑæöËΑÁ¶ËÚΑ˜”«½˜ΑË«½ÞΡΑāؽüΑŽÚꔘΑÞêÄĉËö˜ÚΑޘ˜”ΑË«½ΑÄ”Α
ŽÚꔘΑŽÄ˽έځטޘ˜”έÃêÞæÚ”Α«ÞΑ͎͋ΑטÚΑŽ˜ÄæΑλ«ÄΑ͉͋͊͑μΡΑö¨«½˜ΑŽÚꔘΑÞËü˜ÄΑË«½ΑææځŽæÞΑ͉͌ΑטÚΑŽ˜ÄæΑ
”êæüΑλ«ÄΑ͉͋͊͑μΠΑb¨˜ΑÞ«ŽΑ”êæüΑËÄΑW˜ĈʔΑ[Ëü˜ÄΑË«½ΑÄ”ΑW˜ĈʔΑ[êÄĉËö˜ÚΑH«½ΑöÞΑژ”ꎘ”ΑæËΑ͊͐Π͎АΑ¢ÚËÃΑ
͌͋Π͎АΑÄ”Α樘ΑÞ«ŽΑ”êæüΑËÄΑW˜ĈʔΑT½ÃΑH«½ÞΑöÞΑژ”ꎘ”Α¢ÚËÃΑ͊͐Π͎АΑæËΑ͊͋Π͎АΑËÄΑ͋͊Π͊͋Π͉͋͋͊ΠΑThis duty
¨ÞΑ˜˜ÄΑ˜ûæ˜Ä”˜”Αê×ΑæËΑ͌͊ÞæΑAÚŽ¨ΡΑ͉͍͋͋Π

ANALYSIS:
ANALYSIS:
”«½˜Α Ë«½Α «Ã×ËÚæΑ ގ˜ÄÚ«ËΑ «ÄΑ +Ĕ«Α ¨ÞΑ ˜˜ÄΑ ژ£ê½Ú½üΑ ŽËõ˜Ú˜”Α üΑ 樘Α ÞæÄ”Ú”Α ĘöÞׁטÚÞΠΑ (˜Ú˜Α «ÞΑ ÄΑ
˜ûŽ˜Ú×æΑ¢ÚËÃΑ樘Α(«Ä”êΑλBËõ˜Ã˜ÚΑ͒ΡΑ͉͋͊͐μ

+ÄΑ͉͋͊͒ΡΑfT[Α¨”ΑÞº˜”ΑΑÙê˜Þæ«ËÄΑژ½æ˜”ΑæËΑ樘ΑöËÚ½”ϐÞΑ½Ú£˜ÞæΑÚ«Ž˜Α˜û×ËÚæ˜ÚΑλÄÞö˜Ú΢Α+Ĕ«μΠΑ+æΑ«ÞΑ«Ã×ËÚæÄæΑ
æËΑژØ͘ÚΑ樘ΑÞ«ŽΑ¢ŽæÞΑژ½æ˜”ΑæËΑ樘Α£ÚËᘎËÄËëŽÞΑ˜Π£ΠΑ˜”«½˜ΑË«½ÞΡΑ×ê½Þ˜ÞΑλ+Ĕ«Α«ÞΑ樘Α½Ú£˜ÞæΑ×Ú˔êŽα
˜ÚΑÄ”ΑÞêÚ×Ú«Þ«Ä£½üΑ樘Α½Ú£˜ÞæΑ«Ã×ËÚæ˜ÚΑË¢Α×ê½Þ˜ÞμΡΑÚ«Ž˜ΠΑA«½½˜æÞΡΑ樘ΑŽê½æ«õæ«ËÄΑË¢Αö¨«Ž¨Α«ÞΑ˜«Ä£Α×ÚËÃË昔Α«ÞΑ
ÄË樘ÚΑ«Ã×ËÚæÄæΑ樘ØΠΑfT[ΑÁüΑÞºΑÃËژΑÙê˜Þæ«ËÄÞΑËÄΑ樫ÞΑ樘ØΑ«ÄΑ樘Αü˜ÚÞΑæËΑŽËØΠ
[ꎨΑæË׫ŽÞΑŽÄΑ˜ΑŽËõ˜Ú˜”ΑÃËÞæ½üΑæ¨ÚË꣨Α樘ΑĘöÞׁטÚÞΑλb¨˜Α(«Ä”êΑÄ”Αb¨˜Α+Ĕ«ÄΑû×ژÞÞΑÄ”Α¢ÚËÃΑ樘Α
ŽËÄËëŽΑ[êÚõ˜üμ
122

Year 2018 MSP

Q4. Consider the following:


1. Areca nut
2. Barley
3. Coffee
4. Finger millet
5. Groundnut
6. Sesamum
7. Turmeric
The Cabinet Committee on Economic Affairs has announced the Minimum Support Price for which of the above?
(a) 1, 2, 3 and 7 only
(b) 2, 4, 5 and 6 only
(c) 1, 3, 4, 5 and 6 only
(d) 1, 2, 3, 4, 5, 6 and 7

ANSWERS B

EXPLANATION

A«Ä«ÃêÃΑ [ê××ËÚæΑ TÚ«Ž˜Α λA[TμΑ «ÞΑ Α ¢ËÚÃΑ Ë¢Α Áں˜æΑ «Äæ˜Úõ˜Äæ«ËÄΑ üΑ 樘Α
#Ëõ˜ÚÄØÄæΑË¢Α+Ĕ«ΑæËΑ«ÄÞêژΑ£Ú«Žê½æêځ½Α×Ú˔ꎘÚÞΑ£«ÄÞæΑÄüΑÞ¨Ú×Α¢½½Α«ÄΑ
¢ÚÃΑ×Ú«Ž˜ÞΠ
b¨˜ΑA[TΑ«ÞΑĈû˜”ΑËÄΑ樘ΑژŽËÃØĔæ«ËÄÞΑË¢Α樘ΑËÃëÞÞ«ËÄΑ¢ËÚΑ£Ú«Žê½æêځ½Α
ËÞæÞΑÄ”ΑTÚ«Ž˜ÞΑλTμΠΑ
b¨˜Α TΑ «ÞΑ Α ÞææêæËÚüΑ Ë”üΑ Ä”Α ÞêÃ«æÞΑ ޘׁځæ˜Α ژ×ËÚæÞΑ ژŽËÃØĔ«Ä£Α
×Ú«Ž˜ÞΑ¢ËÚΑ9¨Ú«¢ΑÄ”ΑW«ΑޘÞËÄÞΠΑ
+ī櫁½½üΡΑA[TΑŽËõ˜Ú˜”Αׁ””üΡΑÚ«Ž˜ΡΑö¨˜æΡΑ¶ËöÚΡΑ¶ÚΡΑÁ«ā˜ΡΑځ£«Αλ"«Ä£˜ÚΑA«½½˜æμΡΑ
Ú½˜üΡΑ £ÚÃΡΑ æêÚΡΑ ÃËËÄ£ΡΑ êځ”ΡΑ ÞꣁڎÄ˜ΡΑ £ÚËêĔÄêæΡΑ ÞËü˜ÄΡΑ ÞêÄĉËö˜ÚΑ
ޘ˜”ΡΑځטޘ˜”ΑÄ”ΑÃêÞæÚ”ΡΑŽËææËÄΡΑ¶êæ˜ΑÄ”ΑæˍŽŽËΠΑ
"ÚËÃΑ ͍͊͒͒α͎͒Α ËÄöÚ”ÞΡΑ B«£˜Úޘ˜”Α Ä”Α [˜ÞÃ˜Α ö˜Ú˜Α «ÄŽ½ê”˜”Α êĔ˜ÚΑ A[TΑ
[Ž¨˜Ã˜Α Ë¢Α TΡΑ «ÄΑ ””«æ«ËÄΑ æËΑ 樘Α ˜”«½˜Α Ë«½Þ˜˜”ÞΑ ½Ú˜”üΑ ŽËõ˜Ú˜”Α üΑ 樘Α
ËÃëÞÞ«ËÄΠΑ[«Ã«½Ú½üΡΑ”êÚ«Ä£Α͉͉͋͊α͉͉͋͋ΡΑ樘Α£Ëõ˜ÚÄØÄæΑ˜Ä¨ÄŽ˜”Α樘Αæ˜ÚÃÞΑ
Ë¢Αژ¢˜Ú˜ÄŽ˜ΑË¢Α樘ΑËÃëÞÞ«ËÄΑüΑ«ÄŽ½ê”«Ä£Α½˜Ä櫽ΑλÁÞêÚμΠ Hence option (b) is
the correct answer.

ANALYSIS:
ANALYSIS:
Scope for elimination
Ëą˜˜Α«ÞΑÄËæΑËĘΑË¢Α樘Α͋͌ΑŽÚË×ÞΑ樁æΑÚ˜ΑŽËõ˜Ú˜”ΑêĔ˜ÚΑ樘ΑA[TΠΑb¨êÞΑË×æ«ËÄÞΑλμΡΑ
λŽμΑÄ”Αλ”μΑŽË꽔Α˜Α˜½«Ã«Äæ˜”Π

SOURCE : https://vikaspedia.in/agriculture/market-information/minimum-support-price
123

Year 2018 AGRICULTURAL SURVEY

Q5. As per the NSSO 70th Round "Situation Assessment Survey of Agricultural Households", consider
the following statements:
1. Rajasthan has the highest percentage share of agricultural households among its rural households.
2. Out of the total agricultural households in the country, a little over 60 percent belong to OBCs.
3. In Kerala, a little over 60 percent of agricultural households reported to have received maximum income from
source other than agricultural activities.
Which of the statements given above is/are correct?
(a) 2 and 3 only
(b) 2 only
(c) 1 and 3 only
(d) 1, 2 and 3

ANSWERS C

EXPLANATION

Statement 1 is correct:Α ÃËÄ£Α 樘Α Á¶ËÚΑ [ææ˜ÞΡΑ W¶Þ樁ÄΑ ¨”Α ¨«£¨˜ÞæΑ טڎ˜Ä恣˜Α Ë¢Α £Ú«Žê½æêځ½Α
¨Ëêޘ¨Ë½”ÞΑ λ͐͑Π͍Α טڎ˜ÄæμΑ ÃËÄ£Α «æÞΑ Úêځ½Α ¨Ëêޘ¨Ë½”ÞΑ ¢Ë½½Ëö˜”Α üΑ fææÚΑ Tځ”˜Þ¨Α λ͍͐Π͑Α טڎ˜ÄæμΑ Ä”Α
A”¨üΑTځ”˜Þ¨Αλ͉͐Π͑Αטڎ˜ÄæμΠΑ9˜Ú½Α¨”Α樘Α½˜ÞæΑטڎ˜Ä恣˜ΑÞ¨Ú˜ΑË¢Α£Ú«Žê½æêځ½Α¨Ëêޘ¨Ë½”ÞΑλ͋͐Π͌Α
טڎ˜ÄæμΑ «ÄΑ «æÞΑ Úêځ½Α ¨Ëêޘ¨Ë½”ÞΑ ×ژŽ˜”˜”Α üΑ Ë樘ÚΑ ÞËê樘ÚÄΑ [ææ˜ÞΑ ½«º˜ΑbÃ«½Α B”êΑ λ͍͌Π͐Α טڎ˜ÄæμΑ Ä”Α
Ĕ¨ÚΑTځ”˜Þ¨Αλ͍͊Π͎Αטڎ˜ÄæμΠΑ
Statement 2 is not correct:ΑêÚ«Ä£Α樘Αژ¢˜Ú˜ÄŽ˜Αטګ˔ΑË¢Α6ê½üΑ͉͋͊͋αΑ6êĘΑ͉͋͊͌ΡΑËêæΑ͍͎Αטڎ˜ÄæΑËêæΑ
Ë¢Α樘ΑæË恽Α£Ú«Žê½æêځ½Α¨Ëêޘ¨Ë½”ÞΑ«ÄΑ樘ΑŽËêÄæÚüΑ˜½ËÄ£˜”ΑæËΑH樘ÚΑ ŽºöÚ”Α½ÞޘÞΑλH μΠΑËêæΑ͊͏Α
טڎ˜ÄæΑË¢Α£Ú«Žê½æêځ½Α¨Ëêޘ¨Ë½”ÞΑö˜Ú˜Α¢ÚËÃΑ[Ž¨˜”꽘”ΑÞæ˜ÞΑλ[μΑÄ”Α͊͌Αטڎ˜ÄæΑö˜Ú˜Α¢ÚËÃΑ[Ž¨˜”꽘”Α
bÚ«˜ÞΑλ[bμΠ
[ææ˜Ã˜ÄæΑ͌Α«ÞΑŽËÚژŽæ΢Α£Ú«Žê½æêځ½ΑŽæ«õ«æüΑλŽê½æ«õæ«ËÄΡΑ½«õ˜ÞæˎºΑÄ”ΑË樘ÚΑ£Ú«Žê½æêځ½ΑŽæ«õ«æ«˜ÞμΑöÞΑ
ژ×ËÚ昔ΑæËΑ˜Α樘Α×ګϫׁ½ΑÞËêڎ˜ΑË¢Α«ÄŽËØΑ¢ËÚΑÁ¶ËÚ«æüΑË¢Α樘Α¨Ëêޘ¨Ë½”ÞΑ«ÄΑ½½Α樘ΑÁ¶ËÚΑ[ææ˜ÞΡΑ˜ûŽ˜×æΑ
9˜Ú½Αö¨˜Ú˜ΑËêæΑ͏͊Αטڎ˜ÄæΑË¢Α樘Α£Ú«Žê½æêځ½Α¨Ëêޘ¨Ë½”ÞΑژ×ËÚ昔ΑæËΑ¨õ˜Α˜ÚʔΑÁû«ÃêÃΑ«ÄŽËØΑ
¢ÚËÃΑÞËêڎ˜ÞΑË樘ÚΑ樁ÄΑ£Ú«Žê½æêځ½ΑŽæ«õ«æ«˜ÞΠ

ANALYSIS:
ANALYSIS:
Scope for elimination

b¨«ÞΑ«ÞΑÙê˜Þæ«ËÄΑ«ÞΑæË꣨Α˜ŽêޘΑ樘Α«Ä¢ËÚÁæ«ËÄΑêޘ”Α«ÄΑ樘ΑÞææ˜Ã˜ÄæÞΑË¢Α樘ΑÙê˜Þæ«ËÄΑö˜Ú˜ΑÄËæΑ¢ÚËÃΑ
ÄüΑË¢Α樘ΑŽËÄõ˜Äæ«Ëā½ΑÞËêڎ˜ÞΡΑêæΑ«¢Α”ê˜Αææ˜Äæ«ËÄΑ«ÞΑ£«õ˜ÄΡΑ樘ÄΑ樘ΑޘŽËĔΑÞææ˜Ã˜ÄæΑ«ÞΑ«ÄŽËÚژŽæΑÄ”Α
Ë×æ«ËÄÞΑλμΡΑλμΑÄ”Αλ”) can be eliminatedΑÞΑλ͏͉ΑϹμАΑH Α£Ú«Žê½æêځ½Α¨Ëêޘ¨Ë½”ÞΑöË꽔Α½Ë˺Αޘ˜Ã«Ä£½üΑ
”Ëêæ¢ê½ΑæΑΑæ«Ã˜Αö¨˜ÄΑژޘÚõæ«ËÄΑËą˜Ú˜”ΑüΑ樘ΑŽ˜Äæځ½Α£Ëõ˜ÚÄØÄæΑ«ÞΑÚËêĔΑ͋͐АΠ

SOURCE : ¨ææ×Þ΢έέÃËÞ×«Π£ËõΠ«ÄέÞ«æ˜Þέ”˜¢ê½æέĈ½˜Þέāæ«Ëā½ε”æεÄºέĔαÚ×æÞα͉͐Π¨æÃ
124

Year 2018 ORGANIC FARMING

Q6. With reference to organic farming in India, consider the following statements:
1. 'The National Programme for Organic Production' (NPOP) is operated under the guidelines and directions of the
Union Ministry of Rural Development.
2. 'The Agricultural and Processed Food Productds Export Development Authority' (APEDA) functions as the Secreta-
tiat for the implementation of NPOP.
3. Sikkim has become India's first fully organic State.
Which of the statements given above is/are corect?
(a) 1 and 2 only
(b) 2 and 3 only
(c) 3 only
(d) 1, 2 and 3

ANSWERS B

EXPLANATION
A«Ä«ÞæÚüΑ Ë¢Α ËÃØڎ˜Α ¨ÞΑ «Ã×½˜Ã˜Ä昔Α 樘Α Bæ«Ëā½Α TÚˣځÃØΑ ¢ËÚΑ HÚ£Ä«ŽΑ TÚ˔êŽæ«ËÄΑ λBTHTμΑ ޫϘΑ
͉͉͋͊ΠΑHence statement 1 is not correct.
£Ú«Žê½æêځ½Α Ä”Α TÚˎ˜Þޘ”Α "Ë˔Α TÚ˔êŽæÞΑ û×ËÚæΑ ˜õ˜½Ë×ØÄæΑ êæ¨ËÚ«æüΑ λTμΑ ¢êĎæ«ËÄÞΑ ÞΑ 樘Α
[˜ŽÚ˜æÚ«æΑ¢ËÚΑ樘Α«Ã×½˜Ã˜Äææ«ËÄΑË¢Α樘ΑBTHTΠ Hence statement 2 is correct.
[«ºº«ÃΑ¨ÞΑ˜ŽËØΑ+Ĕ«ϐÞΑĈÚÞæΑ¢ê½½üΑËÚ£Ä«ŽΑÞææ˜ΡΑ«ÄΑ͉͋͊͏ΡΑüΑ«Ã×½˜Ã˜Äæ«Ä£ΑËÚ£Ä«ŽΑ×ځŽæ«Ž˜ÞΑËÄΑÚËêĔΑ
͎͐Ρ͉͉͉Α¨˜ŽæÚ˜ÞΑË¢Α£Ú«Žê½æêځ½Α½Ä”ΠΑ(˜ÄŽ˜ΑÞææ˜Ã˜ÄæΑ͌Α«ÞΑŽËÚژŽæΠ

ANALYSIS:
ANALYSIS:
bË׫Ž΢Α£Ú«Žê½æêژΑÄ”Α"Ë˔α×Úˎ˜ÞÞ«Ä£
[êαæË׫Ž΢ΑHÚ£Ä«ŽΑ"Ë˔ΑTÚ˔êŽæ«ËÄΑÄ”ΑT
Scope for elimination: Statement 1 is incorrectΑÞΑ˜õ˜ÄΑ«¢ΑËĘΑ”˘ÞΑÄËæΑºÄËöΑö¨«Ž¨Α«ÞΑ樘Α˜ûŽæΑëīÞæÚüΑ
λA«Ä«ÞæÚüΑË¢ΑËÃØڎ˜ΑËÚΑ樘ΑA«Ä«ÞæÚüΑË¢Α£Ú«Žê½æêژμΑËĘΑÞêژ½üΑºÄËöÞΑö¨«Ž¨ΑëīÞæÚ«˜ÞΑŽÄÄËæΑ˜ΠΑ#Ë«Ä£Α
üΑ樁æΑ½Ë£«ŽΡΑö˜ΑŽÄΑ˜Þ«½üΑ˜½«Ã«Äæ˜ΑË×æ«ËÄÞΑË×æ«ËÄÞΑλμΑÄ”Αλ”μΡΑêæΑö˜ΑŽÄÄËæΑ£ËΑ¢êÚ樘ÚΑÞΑ«æΑ«ÞΑ”«ĆŽê½æΑ
æËΑ×ژ”«ŽæΑ樘ΑŽËÚژŽæĘÞÞΑË¢ΑÞææ˜Ã˜ÄæΑ͋ΠΑ
b¨êÞΑ”«ĆŽê½æüΑ½˜õ˜½΢ΑA˜”«êÃΑλÄËæΑ˜Þüμ
λAËæ«õæ«ËÄΡΑö¨üΑÞº˜”Χμ΢ΑΑ+æΑ¨ÞΑ½öüÞΑ˜˜ÄΑ«ÄΑ樘ΑĘöÞΑ˜ÞטŽ«½½üΑ樘Α¢ŽæΑ樁æΑ[«ºº«ÃΑ˜ŽÃ˜Α+Ĕ«ϐÞΑ
ĈÚÞæΑËÚ£Ä«ŽΑÞææ˜Π

SOURCE : ¨ææ×Þ΢έέöööΠ樘¨«Ä”êЎËÃέĘöÞέāæ«Ëā½έ½˜½αÁĔæËÚüα¢ËÚα¢ËË”αŽ˜Úæ«Ĉ˜”αΑÞαËÚ£Äα
ic-from-july/article22378459.ece
¨ææ×Þ΢έέöööΠ樘¨«Ä”êЎËÃέĘöÞέāæ«Ëā½έ[«ºº«ÃፘŽËØÞα+Ĕ«А͋А͉͑А͒͒ÞαĈÚÞæαΑËÚ£Äα
ic-state/article13999445.ece
125

Year 2019 FOOD CORPORATION OF INDIA (FCI)

Q7. The economic cost of food grains to the Food Corporation of India is the Minimum Support Price
and bonus (if any) paid to the farmers plus
(a) transportation cost only
(b) interest cost only
(c) procurement incidentals and distribution cost
(d) procurement incidentals and charges for godowns

ANSWERS C

EXPLANATION

The Public Distribution System (or PDS) in India aims at targeting and curing the problem of food
scarcity.Α+æΑ«ÞΑ¶Ë«Äæ½üΑÁā£˜”ΑüΑ樘Α˜ÄæژΑÄ”Α樘ΑÞææ˜Α£Ëõ˜ÚÄØÄæÞΠΑr¨«½˜Α樘ΑŽ˜ÄæژΑ¨Ä”½˜ÞΑ樘Α¢Ë˔Α
×ÚˎêژØÄæΡΑÞæËځ£˜ΡΑæځÄÞ×ËÚæΑÄ”Αê½ºΑ”«ÞæÚ«êæ«ËÄΑׁÚæΡΑ樘ΑÞææ˜Α£Ëõ˜ÚÄØÄæÞΑ¨õ˜Α樘ΑËטځæ«Ëā½Α
ژÞ×ËÄ«Þ««½æüΑ Ä”Α «ÄæځαÞææ˜Α ½½ËŽæ«Ëā½Α ژÞ×ËÄÞ««½«æ«˜ÞΠΑ +æΑ «ÞΑ 樘Α ژÞ×ËÄÞ««½«æ«˜ÞΑ Ë¢Α 樘Α Þææ˜Α
£Ëõ˜ÚÄØÄæÞΑæËΑ«ÞÞê˜Α樘Αځæ«ËÄΑŽÚ”ÞΑÄ”ΑÞêטÚõ«Þ˜Α樘Α¢êĎæ«ËÄ«Ä£ΑË¢Α樘Α¢«ÚΑ×Ú«Ž˜ΑÞ¨Ë×ÞΠ
b¨˜Α˜ŽËÄËëŽΑŽËÞæΑŽËÃ×ګޘÞΑ樘Α×ÚˎêژØÄæΑ×Ú«Ž˜ΑË¢Α¢Ë˔£Ú«ÄÞΡΑŽËÞæÞΑژ½æ˜”ΑæËΑ×ÚˎêژØÄæΑλÞꎨΑÞΑ
ÞææêæËÚüΑæû˜ÞΡΑ½ËêÚΑŽËÞæÞΡΑÁĔ«Α¢˜˜ÞΡΑÄ”ΑÞËΑËÄμΑÄ”ΑŽËÞæÞΑË¢Α”«ÞæÚ«êæ«ËÄΑλ«ÄŽ½ê”«Ä£Α¢Ú˜«£¨æΡΑÞæËځ£˜Α
Ä”Α”ëīÞæځæ«ËÄμΠΑΑHence option (c) is the correct answer.

ANALYSIS:
ANALYSIS:
BËæΑŽËõ˜Ú˜”Α«ÄΑ樘ΑBWbΑË˺ÞΡΑêæΑÞꎨΑæË׫ŽÞΑË¢æ˜ÄΑ×טÚΑ«ÄΑ樘ΑĘöÞׁטÚÞΠΑ½ÞËΡΑ«æΑöÞΑ«ÄΑ樘ΑB˜öÞΑ
«ÄΑ樁æΑü˜ÚΑÄ”ΑöÞΑΑÁ¶ËÚΑ×Ë«ÄæΑË¢Α”«ÞŽêÞÞ«ËÄΑ«ÄΑ樘Α˜½˜Žæ«ËÄΑÁī¢˜ÞæËΠ
Scope of elimination: Α ÄËÚÁ½Α ××ÚˁŽ¨ΑæËΑ ژŽ¨Α樘Α ŽËÚژŽæΑ ÄÞö˜ÚΑ «ÞΑæËΑ āÚÚËöΑ ”ËöÄΑ樘Α¢ËŽêÞΑ ËÄΑ
ޫ뽁ÚΑ Ë×æ«ËÄÞΑ λæ¨Ë꣨Α 樘ژΑ «ÞΑ ÞËØΑ ˜½˜Ã˜ÄæΑ Ë¢Α Ú«ÞºμΠΑ +ÄΑ 樫ÞΑ Ùê˜Þæ«ËÄΡΑ Α 樘ژΑ Ú˜Α æöËΑ ޫ뽁ÚΑ Ë×æ«ËÄÞΡΑ
Ë×æ«ËÄÞΑλŽμΑÄ”Αλ”μΠΑb¨êÞΑæöËΑË×æ«ËÄÞΑŽÄΑ˜Α˜Þ«½üΑ˜½«Ã«Äæ˜”Π

SOURCE : ¨ææ×Þ΢έέöööΠĈāĎ«½˜û×ژÞÞЎËÃέË׫īËÄέژ¢ËÚÃ«Ä£α¢Ž«αÄËæαα׫˜Ž˜αË¢αŽº˜έ͎͊͑͏͉͌͑έ

Additional Information MSP FORMULA


b¨˜ΑBæ«Ëā½ΑËÃëÞÞ«ËÄΑË¢Α"ÚØÚÞέ[öÃ«Äæ¨ÄΑËÃëÞÞ«ËÄΑژŽËÃØĔ˜”Α樁æ
樘ΑA[TΑÞ¨Ë꽔ΑæΑ½˜ÞæΑ˜Α͎͉ΑטÚΑŽ˜ÄæΑÃËژΑ樁ÄΑ樘Αö˜«£¨æ˜”Αõ˜Ú£˜ΑËTΡΑö¨«Ž¨Α«æ
ژ¢˜ÚÞΑæËΑÞΑ樘Α͋ΑŽËÞæΠΑ
b¨˜Α£Ëõ˜ÚÄØÄæΑÁ«Ä恫ÄÞΑ樁æΑ樘ΑA[TΑöÞΑĈû˜”ΑæΑΑ½˜õ˜½ΑË¢ΑæΑ½˜ÞæΑ͊Π͎Αæ«Ã˜Þ
Ë¢Α樘Α½½α+Ĕ«Αö˜«£¨æ˜”Αõ˜Ú£˜ΑËTΡΑêæΑ«æΑŽ½Žê½æ˜ÞΑ樫ÞΑŽËÞæΑÞΑ͊Π͎Αæ«Ã˜ÞΑË¢Α͋Ϲ";Π
(Ëö˜õ˜ÚΑ¢ÚØÚÞΑ¢ÚËÃΑæ«Ã˜ΑæËΑæ«Ã˜Α”˜ÃÄ”˜”ΑæËΑ«Ã×½˜Ã˜ÄæΑ[öÃ«Äæ¨ÄΑA˜æ¨Ë”ΑË¢
͋Α«ÄÞ昁”ΑË¢Α×ژޘÄæΑØæ¨Ë”˽ˣüΑË¢Α͋Ϲ";ΠΑ
ËÃ×ËĘÄæÞΑêĔ˜ÚΑ樘ޘΑØæ¨Ë”˽ˣüΑÚ˜΢Α
͋΢Α ËÞæÞΑ «ÄŽêÚژ”Α üΑ 樘Α ¢ÚØÚΑ «ÄΑ ×Ú˔êŽæ«ËÄΑ Ë¢Α Α ׁÚ櫎꽁ÚΑ ŽÚË×ΠΑ +æΑ «ÄŽ½ê”˜ÞΑ ޘõ˜Ú½Α «Ä×êæÞΑ ÞꎨΑ ÞΑ
˜ûטĔ«æêژΑËÄΑޘ˜”ÞΡΑ¢˜Ú櫽«Þ˜ÚÞΡΑטÞ櫎«”˜ÞΡΑ½˜Þ˜”α«ÄΑ½Ä”ΡΑ¨«Ú˜”Α½ËêÚΡΑÁŽ¨«Ä˜ÚüΑÄ”Α¢ê˜½Α
͋Ϲ";΢ΑËÞæÞΑ«ÄŽêÚژ”ΑüΑ樘Α¢ÚØÚΑÄ”Α樘Αõ½ê˜ΑË¢Α¢Ã«½üΑ½ËêÚΑ
͋΢ΑΑŽËÃ×ژ¨˜ÄÞ«õ˜ΑŽËÞæΡΑö¨«Ž¨Α«ÞΑ͋Ϲ";ΑŽËÞæΑ×½êÞΑ«Ã×ê昔ΑژÄ恽Αõ½ê˜ΑË¢ΑËöʔΑ½Ä”Α×½êÞΑ«Äæ˜Ú˜ÞæΑËÄΑ
Ĉû˜”ΑŽ×«æ½ΡΑژÄæΑׁ«”Α¢ËÚΑ½˜Þ˜”α«ÄΑ½Ä”Π
126

Year 2019 AGRICULTURAL TRADE

Q8. Among the agricultural commodities imported by India, which one of the following accounts for
樘Α¨«£¨˜ÞæΑ«Ã×ËÚæÞΑ«ÄΑæ˜ÚÃÞΑË¢Αõ½ê˜Α«ÄΑ樘Α½ÞæΑĈõ˜Αü˜ÚÞΧ
(a) Spices
(b) Fresh fruits
(c) Pulses
(d) Vegetable oils

ANSWERS D

EXPLANATION
b¨˜Α Ùê˜Þæ«ËÄΑ ÞºÞΑ ËêæΑ樘Αõ½ê˜Α Ë¢Α £Ú«Žê½æêځ½Α «Ã×ËÚæÞΑ «ÄΑ樘Α Ϗ½ÞæΑĈõ˜Αü˜ÚÞϐΠΑÞΑ樫ÞΑ Ùê˜Þæ«ËÄΑöÞΑ
Þº˜”Α«ÄΑ͉͋͊͒ΡΑ樁æΑ؁ÄÞΑ樘Αæ«Ã˜Αטګ˔Α«ÄΑ樘ΑŽËÄæ˜ûæΑ«ÞΑ͉͍͋͊α͉͋͊͒Π
b¨˜ΑæË恽Αõ½ê˜ΑË¢Α£Ú«Žê½æêځ½ΑŽËÃÃ˔«æ«˜ÞΑ«Ã×ËÚ昔Α”êÚ«Ä£Α樘ΑށØΑæ«Ã˜Αטګ˔Α«ÞΑÚË꣨½üΑ˜æö˜˜ÄΑ(Rs
͊Ρ͉͋Ρ͉͉͉ΑαΑ͊Ρ͍͉Ρ͉͉͉ΑŽÚËژÞμ
b¨˜Αõ½ê˜ΑË¢Α×ê½Þ˜ÞΑ«Ã×ËÚ昔Α”êÚ«Ä£Α樁æΑæ«Ã˜Αטګ˔Α«ÞΑÚË꣨½üΑ˜æö˜˜ÄΑλWÞΑ͎͉͉͉͊ΑαΑWÞΑ͉͉͉͉͌ΑŽÚËژÞμΠ
b¨˜Αõ½ê˜ΑË¢Α¢Ú˜Þ¨Α¢Úê«æÞΑ«Ã×ËÚ昔Α”êÚ«Ä£Α樘ΑށØΑæ«Ã˜Αטګ˔Α«ÞΑÚË꣨½üΑ˜æö˜˜Ä (Rs 15000 crores).
b¨˜Αõ½ê˜ΑË¢ΑÞ׫Ž˜ÞΑ«Ã×ËÚ昔Α”êÚ«Ä£Α樘ΑށØΑæ«Ã˜Αטګ˔Α«ÞΑÚË꣨½üΑ˜æö˜˜ÄΑλWÞΑ͎͉͉͉ΑαΑ͉͉͉͊͊ΑŽÚËژÞμΠ
b¨˜Αõ½ê˜ΑË¢Αõ˜£˜æ½˜ΑË«½ÞΑ«Ã×ËÚ昔Α”êÚ«Ä£Α樘ΑށØΑæ«Ã˜Αטګ˔Α«ÞΑÚË꣨½üΑ˜æö˜˜ÄΑλWÞΑ͏͎͉͉͉ΑαΑ͎͉͉͉͑Α
crores. Hence option (d) is the correct answer.

ANALYSIS:
ANALYSIS:
Scope for elimination
HĽüΑ Α ½«æ潘Α «æΑ Ë¢Α ގËטΑ ¢ËÚΑ ˜½«Ã«Äæ«ËÄΑ öÞΑ 樘ژΡΑ Þ׫Ž˜ÞΑ
Ä”Α¢Ú˜Þ¨Α¢Úê«æÞΑŽË꽔Α¨õ˜Α˜˜ÄΑ˜½«Ã«Äæ˜”ΡΑêæΑö«æ¨ËêæΑÄΑ
˜ûŽæΑ«”˜ΡΑŽ¨ËËÞ«Ä£Α˜æö˜˜ÄΑæöËΑË×æ«ËÄÞΑŽË꽔Α¨õ˜Α˜˜ÄΑ
”«ĆŽê½æΡΑ×ê½Þ˜ÞΑÄ”Αõ˜£˜æ½˜ΑË«½ÞΠ
+Ĕ«Α «ÞΑ 樘Α ½Ú£˜ÞæΑ «Ã×ËÚæ˜ÚΑ Ë¢Α 樘Α ˜”«½˜Α Ë«½ÞΠΑ b¨˜Α Ë樘ÚΑ
important commodities in relation to which questions can
˜Α Þº˜”Α Ú˜Α ë½½˜æÞ (regarding pulses there was a
question in 2018).
It is common knowledge that India is the world's largest
«Ã×ËÚæ˜ÚΑË¢Αõ˜£˜æ½˜ΑË«½ÞΑö«æ¨ΑÄΑÄÄꁽΑ«Ã×ËÚæΑõ˽êØΑË¢Α
͍͊α͎͊Αë½½«ËÄΑæËÄĘÞΑõ½ê˜”ΑæΑϮ͊͋α͊͌Α«½½«ËÄΑÄ”Α樁æΑ樘Α
ŽËêÄæÚüΑ ¢Ž˜ÞΑ ŽËÃ×ê½Þ«ËÄΑ æËΑ «Ã×ËÚæΑ ˜ŽêޘΑ ”ËØÞ櫎Α
×Ú˔êŽæ«ËÄΑ¨ÞΑטÚÞ«Þæ˜Äæ½üΑ¢«½˜”ΑæËΑؘæΑ樘ΑêÚ£˜ËÄ«Ä£Α
ŽËÄÞêÃ×æ«ËÄΑ ”˜ÃÄ”ΠΑ r˜Α ½ÞËΑ ºÄËöΑ 樁æΑ +Ĕ«Α «ÞΑ 樘Α
½Ú£˜ÞæΑ «Ã×ËÚæ˜ÚΑ Ë¢Α ×ê½Þ˜ÞΠΑ Α ½Ë£«ŽΑ ŽË꽔Α ¨õ˜Α ˜˜ÄΑ
××½«˜”ΡΑ ÞΑ ϏË«½ϐΑ «ÞΑ Α ÃËژΑ «Ã×ËÚæÄæΑ «Ä£Ú˜”«˜ÄæΑ 樁ÄΑ
Ϗ×ê½Þ˜ÞϐΑÞΑö«æ¨ËêæΑϏË«½ϐΑÄËΑŽË˺«Ä£ΑŽÄΑ˜Α”ËĘΠ

SOURCE : Please note Economic Survey did not cover all the agricultural commodities. Refer to the Agricul-
tural Statistics At a Glance https://desagri.gov.in/wp-content/uploads/2021/07/Agricultur-
al-Statistics-at-a-Glance-2021-English-version.pdf
127

Year 2019 AGRICULTURAL TRADE

V͒ΠΑÃËÄ£Α樘Α¢Ë½½Ëö«Ä£Αö¨«Ž¨ΑËĘΑ«ÞΑ樘Α½Ú£˜ÞæΑ˜û×ËÚæ˜ÚΑË¢ΑÚ«Ž˜Α«ÄΑ樘ΑöËÚ½”Α«ÄΑ樘Α½ÞæΑĈõ˜Αü˜ÚÞΧ
(a) China
(b) India
(c) Myanmar
(d) Vietnam

ANSWERS B

EXPLANATION
+Ĕ«Α¨ÞΑ˜˜ÄΑ樘ΑöËÚ½”ϐÞΑæË×ΑÚ«Ž˜Α˜û×ËÚæ˜ÚΑޫϘΑ樘Α˜£«ÄÄ«Ä£ΑË¢Α樫ÞΑ”˜Ž”˜ΠΑλ͉͋͊͊α͊͋μ
+Ĕ«ϐÞΑÞ¨Ú˜Α«ÄΑöËÚ½”Α˜û×ËÚæÞΑË¢ΑÚ«Ž˜Α«ÄΑژŽ˜ÄæΑü˜ÚÞΑλ͉͍͋͊α͊͑μΑ¨ÞΑÞæü˜”ΑæΑ͎͋α͋͏ΑטÚΑŽ˜ÄæΡΑb¨«½Ä”ϐÞΑ¨ÞΑ
ĉêŽæêæ˜”Α˜æö˜˜ÄΑ͋͋ΑÄ”Α͎͋ΑטÚΑŽ˜ÄæΡΑÄ”Αq«˜æāÃϐÞΑ˜æö˜˜ÄΑ͊͌ΑÄ”Α͊͏ΑטÚΑŽ˜ÄæΠ
Hence option (b) is the correct answer.

ANALYSIS:

Scope for elimination


½æ¨Ë꣨Α樫ÞΑ«ÞΑΑ¢ŽæꁽΑÙê˜Þæ«ËÄΡΑޫϘΑ樘Α˜û×ËÚæÞΑË¢ΑËæ¨Α ÞÁæ«ΑÄ”ΑÄËÄα ÞÁæ«Α£Ú”˜ΑÚ«Ž˜Α¨ÞΑ
˜˜ÄΑõ˜ÚüΑ¨«£¨ΡΑ+Ĕ«Α«ÞΑ樘Α½Ú£˜ÞæΑ˜û×ËÚæ˜ÚΑË¢ΑÚ«Ž˜ΠΑb¨«ÞΑ¨ÞΑ½öüÞΑ˜˜ÄΑ«ÄΑ樘ΑĘöÞΠ

SOURCE : https://www.thehindubusinessline.com/opinion/columns/c-p-chan-
drasekhar/the-dynamics-of-indias-rice-export-boom/article25994349.ece

Also refer to the Agricultural Statistics At A Glace (15 to 20 pages are important and not the
entire book)
https://desagri.gov.in/wp-content/uploads/2021/07/Agricultur-
al-Statistics-at-a-Glance-2021-English-version.pdf
128

Year 2020 FERTILIZERS

Q10. With reference to chemical fertilizers in India, consider the following statements:
1. At present, the retail price of chemical fertilizers is market-driven and not administered by the Government.
2. Ammonia, which is an input of urea, is produced from natural gas.
3. Sulphur, which is a raw material for phosphoric acid fertilizer, is a by -product of oil refineries.
Which of the statements given above is/are correct?
(a) 1 only
(b) 2 and 3 only
(c) 2 only
(d) 1, 2 and 3

ANSWERS B

EXPLANATION

b¨˜Α#Ëõ˜ÚÄØÄæΑË¢Α+Ĕ«ΑÞêÞ«”«ā˜ÞΑ¢˜Ú櫽«ā˜ÚÞΑæËΑ˜ÄÞêژΑ樁æΑ¢˜Ú櫽«ā˜ÚÞΑÚ˜Α˜Þ«½üΑõ«½½˜ΑæËΑ¢ÚØÚÞΑÄ”Α
樘ΑŽËêÄæüΑژÁ«ÄÞΑޘ½¢ΑαÞêƎ«˜ÄæΑ«ÄΑ£Ú«Žê½æêژΠΑb¨˜ΑށØΑ¨ÞΑ˜˜ÄΑŽ¨«˜õ˜”Α½Ú£˜½üΑüΑŽËÄæÚ˽½«Ä£Α樘Α
×Ú«Ž˜Α Ë¢Α ¢˜Ú櫽«ā˜ÚΑ Ä”Α 樘Α ÃËêÄæΑ Ë¢Α ×Ú˔êŽæ«ËÄΑ "ËÚΑ ˜ûÃ×½˜ΡΑ ÞΑ טÚΑ 樘Α B˜öΑ fژΑ T˽«ŽüΑ Ë¢Α ͉͎͋͊ΡΑ 樘Α
£Ëõ˜ÚÄØÄæΑĈû˜ÞΑ樘ΑÁں˜æΑ×Ú«Ž˜ΑË¢ΑêژΠΑ½ÞËΡΑ樘ژΑ«ÞΑΑĈû˜”ΑÞêÞ«”üΑŽËÃ×ËĘÄæΑÞΑö˜½½ΠΑ[«Ã«½Ú½üΡΑ¢ËÚΑ
T¨ËÞרËÚËêÞΑ Ä”Α TËæÞÞ«êÃΡΑ ÞΑ טÚΑ 樘Α BêæÚ«˜ÄæΑ Þ˜”Α [êÞ«”üΑ [Ž¨˜Ã˜Α Ë¢Α ͉͉͋͊ΡΑ ÞêÞ«”üΑ «ÞΑ ×ÚËõ«”˜”Α
Þ˜”ΑËÄΑÄêæÚ«˜ÄæΑŽËÄæ˜ÄæΑטÚΑº£ΑË¢Α¢˜Ú櫽«ā˜ÚΠΑHence statement 1 is not correct.
"˜Ú櫽«ā˜ÚΑ ×Ú˔êŽæ«ËÄΑ êޘÞΑ ͊Π͋АΑ Ë¢Α 樘Α öËÚ½”ϐÞΑ æË恽Α ˜Ä˜Ú£üΑ ËêæΑ Ë¢Α ö¨«Ž¨Α ͉͒АΑ «ÞΑ êޘ”Α ¢ËÚΑ ÃÃËÄ«Α
×Ú˔êŽæ«ËÄΡΑö¨«Ž¨Α«ÞΑΑº˜üΑ«Ä£Ú˜”«˜ÄæΑ«ÄΑ樘Α×Ú˔êŽæ«ËÄΑË¢ΑÄ«æÚË£˜ÄΑ¢˜Ú櫽«ā˜ÚÞΠΑÃÃËÄ«ΑŽÄΑ˜Α×Ú˔ꎘ”Α
¢ÚËÃΑāæêځ½Α£ÞΠ Hence statement 2 is correct
[ê½¢êÚΑ«ÞΑΑÁ¶ËÚΑüα×Ú˔êŽæΑË¢ΑË«½ΑژĈÄ«Ä£ΑÄ”Α£ÞΑ×Úˎ˜ÞÞ«Ä£ΠΑAËÞæΑŽÚꔘΑË«½Α£Ú”˜ÞΑŽËÄ恫ÄΑÞËØΑÞê½¢êÚΡΑ
ÃËÞæΑË¢Αö¨«Ž¨ΑÃêÞæΑ˜ΑژÃËõ˜”Α”êÚ«Ä£Α樘ΑژĈÄ«Ä£Α×Úˎ˜ÞÞΑæËΑؘæΑÞæÚ«ŽæΑÞê½¢êÚΑŽËÄæ˜ÄæΑ½«Ã«æÞΑ«ÄΑژĈʔΑ
×Ú˔êŽæÞΠΑ+ĔêÞæÚ«˜ÞΡΑ¢ËÚΑ«ÄÞæÄŽ˜ΡΑ樘ΑAæ¨êځΑË«½ΑژĈĘÚüΡΑ¨õ˜Α˜˜ÄΑژÞ×ËÄÞ«½˜Α¢ËÚΑ×Ú˔ꎫģΑ×˽½êæÄæÞΑ
½«º˜Α Þê½×¨êÚΑ ”«Ëû«”˜Α Ä”Α Ä«æÚË£˜ÄΑ ”«Ëû«”˜ΠΑ½ÞËΡΑ [ê½×¨êÚΑ «ÞΑ êޘ”Α «ÄΑ רËÞרËÚ«ŽΑ Ž«”Α ¢˜Ú櫽«ā˜ÚΑ λb¨˜Ú˜Α «ÞΑ Α
×Úˎ˜ÞÞΑºÄËöÄΑÞΑϖb¨˜Αr˜æΑTÚˎ˜ÞÞϖΑ¢ËÚΑ×Ú˔ꎫģΑ樘ΑށØμΠΑ(˜ÄŽ˜ΑÞææ˜Ã˜ÄæΑ͌Α«ÞΑŽËÚژŽæΠΑ

ANALYSIS:
"˜Ú櫽«ā˜ÚΑ ÞêÞ«”üΑ «ÞΑ ÄΑ «Ã×ËÚæÄæΑ ŽËÃ×ËĘÄæΑ Ë¢Α樘Α Þü½½êÞΠΑ Α½ÞËΡΑ «æΑ «ÞΑ ËĘΑ Ë¢Α樘Α ژ£ê½Ú½üΑ ”«ÞŽêÞޘ”Α
æË׫ŽÞΑ«ÄΑrbHΑ먁ÞΑ½öüÞΑ˜˜ÄΑ«ÄΑ樘ΑĘöÞμΠΑΑ

Scope for elimination


"ÚËÃΑËêÚΑêĔ˜ÚÞæÄ”«Ä£ΑË¢Α£Ú«Žê½æêژΑ«ÄΑ+Ĕ«ÄΑŽËÄËÃüΡΑö˜ΑºÄËöΑ樁æΑÃËÞæΑ¢˜Ú櫽«Þ˜ÚÞΑÚ˜Αõ«½½˜ΑæËΑ樘Α
¢ÚØÚÞΑ æΑ Α ÞêÞ«”«Þ˜”Α ځæ˜ΠΑ (˜ÄŽ˜ΡΑ Áں˜æΑ ”Ú«õ˜ÄΑ ×Ú«Ž˜ÞΑ Ú˜Α ÄËæΑ ×ËÞÞ«½˜Α +Á£«Ä˜Α ”êÚ«Ä£Α 樘Α ÞËö«Ä£Α
ޘÞËÄΡΑ”˜ÃÄ”Αö«½½Α«ÄŽÚ˜Þ˜ΑÄ”ΑÃËÞæΑ×Úˍ½üΑ樘Α×Ú«Ž˜Αö«½½Α½ÞËΑ«ÄŽÚ˜Þ˜ΑγΑö¨«Ž¨Α루æΑ£ËΑËõ˜Α樘Α
×êڎ¨Þ«Ä£Α×Ëö˜ÚΑË¢Α樘Α¢ÚØÚΠΑ+ÄΑÞꎨΑΑŽÞ˜ΡΑ«æΑö«½½Α¨õ˜ΑΑ”«Ú˜ŽæΑ«ÃׁŽæΑËÄΑ樘Α¢Ë˔Α×Ú˔êŽæ«ËÄΑË¢Α樘Α
ü˜ÚΠΑb¨æΑ«ÞΑö¨üΡΑÁÄüΑځöΑÁæ˜Ú«½ÞΑ«ÄŽ½ê”«Ä£Α¢˜Ú櫽«Þ˜ÚÞΑÚ˜ΑÁ”˜Αõ«½½˜ΑæËΑ樘Α¢ÚØÚÞΑæΑΑÞêÞ«”«Þ˜”Α
ځæ˜ΠΑ(˜ÄŽ˜ΡΑÞææ˜Ã˜ÄæΑ͊Α«ÞΑöÚËÄ£ΠΑb¨˜Ú˜¢ËژΡΑ樘ΑÄÞö˜ÚΑöË꽔Α˜Α˜«æ¨˜ÚΑË×æ«ËÄΑλμΑËÚΑλŽμΑγΑö˜Αʘ”ΑæËΑæ˜ÞæΑ
樘ΑŽÚ˜”««½«æüΑË¢ΑÞææ˜Ã˜ÄæΑ͌ΑËĽüΠΑ
129

SOURCE : https://pib.gov.in/PressReleasePage.aspx?PRID=1580828

Year 2020 AGRICULTURAL INVESTMENT

Q11. In India, which of the following can be considered as public investment in agriculture?
1. Fixing Minimum Support Price for agricultural produce of all crops.
2. Computerization of Primary Agricultural Credit Societies
3. Social Capital development
4. Free electricity supply to farmers
5. Waiver of agricultural loans by the banking system
6. Setting up of cold storage facilities by the governments.
Select the correct answer using the code given below.
(a) 1, 2 and 5 only
(b) 1, 3, 4 and 5 only
(c) 2, 3 and 6 only
(d) 1, 2, 3, 4, 5 and 6

ANSWERS C
130
EXPLANATION
Tꍽ«ŽΑ+Äõ˜ÞæØÄæΑ¨˜Ú˜Αژ¢˜ÚÞΑæËΑ樘ΑŽÚ˜æ«ËÄΑË¢Α˜«æ¨˜ÚΑרüÞ«Ž½Α«Ä¢ÚÞæÚêŽæêژΑËÚΑ«ÄæÄ£«½˜ΑŽ×«æ½ΠΑ
(˜ÄŽ˜Α«ÄΑ樫ÞΑŽËÄæ˜ûæΡΑ«Äõ˜ÞæØÄæΑ«ÞΑêĔ˜ÚÞæË˔ΑÞΑ˜«æ¨˜ÚΑ«Ä¢ÚÞæÚêŽæêژαژ½æ˜”ΑŽ×«æ½ΑÞΑ£«õ˜ÄΑ«ÄΑË×æ«ËÄÞΑ
͋ΑÄ”Α͏ΑËÚΑÞˎ«½ΑŽ×«æ½ΑÞΑ£«õ˜ÄΑ«ÄΑË×æ«ËÄΑ͌ΠΑ
[êÞ«”«˜ÞΑ Ä”Α ½ËÄΑ ö«õ˜ÚÞΑ Ú˜Α ÄËæΑ «Äõ˜ÞæØÄæÞΠΑ Hence option 1 is not correctΑ λb¨˜Ú˜Α «ÞΑ ގËטΑ ¢ËÚΑ
˜½«Ã«Äæ«ËÄΑ¨˜Ú˜μΠΑ(˜ÄŽ˜ΑË×æ«ËÄΑλŽμΑ«ÞΑ樘ΑŽËÚژŽæΑÄÞö˜ÚΠΑ[«Ã«½Ú½üΡΑË×æ«ËÄΑ͍ΑλÞꍫޔüμΑÄ”Α͎ΑÚ˜ΑÄËæΑŽËÚژŽæΠ

ANALYSIS:

Scope for elimination


(˜Ú˜ΡΑ«¢Αö˜ΑÚ˜Α½˜ΑæËΑ”«Þæ«Ä£ê«Þ¨Α˜æö˜˜ÄΑ×ꍽ«ŽΑ«Äõ˜ÞæØÄæΑÄ”ΑÞêÞ«”«˜ÞΡΑ樘ÄΑö˜ΑŽÄΑ˜Þ«½üΑĈĔΑ樘Α
ÄÞö˜ÚΑæ¨ÚË꣨Α˜½«Ã«Äæ«ËÄΠΑ+Äõ˜ÞæØÄæΑ£˜Ä˜Ú½½üΑژ½æ˜ÞΑæËΑ樘ΑŽÚ˜æ«ËÄΑË¢ΑÄΑ«Ä¢ÚÞæÚêŽæêژΑλרüÞ«Ž½ΑËÚΑ
Þˎ«½μΠΑ (˜ÄŽ˜ΡΑ Þææ˜Ã˜ÄæÞΑ ͊ΡΑ ͍Α Ä”Α ͎Α Ú˜Α Α¢ËÚÃΑ Ë¢Α ÞêÞ«”«˜ÞέÞê××ËÚæέö«õ˜ÚΑæËΑ樘Α¢ÚØÚÞΑ Ä”Α ÄËæΑ ÄΑ
«Äõ˜ÞæØÄæΠΑ(˜ÄŽ˜ΡΑË×æ«ËÄΑλŽμΑ«ÞΑ樘ΑŽËÚژŽæΑÄÞö˜ÚΠ

SOURCE : https://www.downtoearth.org.in/news/agriculture/pri-
vate-and-government-who-is-investing-how-much-on-agriculture--65296
b¨˜ΑÞæꔘÄæΑÞ¨Ë꽔Α˜ΑëĔ¢ê½ΑË¢Α½½Α樘ΑÞטŽæÞΑË¢Α樘Α£Ú«Žê½æêژΑÄ”Α¨˜ÄŽ˜ΑŽËõ˜ÚΑ«æΑ¨Ë½«Þ櫎½½üΠΑ
½ÞËΡΑ樘ΑºÄËö½˜”£˜ΑË¢Α£Ú«Žê½æêژΑ¢ÚËÃΑŽËÄËëŽÞΑÄ”Α#˜Ë£Ú×¨üΑŽÄΑ˜Αêޘ”Α«Äæ˜ÚŽ¨Ä£˜½üΠ

Year 2020 PRICE OF RICE

V͊͋ΠΑr¨«Ž¨ΑË¢Α樘Α¢Ë½½Ëö«Ä£Α¢ŽæËÚÞέ×˽«Ž«˜ÞΑö˜Ú˜Αą˜Žæ«Ä£Α樘Α×Ú«Ž˜ΑË¢ΑÚ«Ž˜Α«ÄΑ+Ĕ«Α«ÄΑ樘ΑژŽ˜ÄæΑׁÞæΧΑ
1. Minimum Support Price
2. Government's trading
3. Government's stockpiling
4. Consumer subsidies
Select the correct answer using the code given below.
(a) 1, 2 and 4 only
(b) 1, 3 and 4 only
(c) 2 and 3 only
(d) 1, 2, 3 and 4

ANSWERS D
131
EXPLANATION
Option 1 is correct:ΑA[TΑژÞê½æÞΑ«ÄΑΑ”«õ˜ÚÞ«ËÄΑË¢ΑÞæˎºÞΑ¢ÚËÃΑ樘ΑËטÄΑÁں˜æΡΑæ¨êÞΡΑ”Ú«õ«Ä£Αê×Α樘Α×Ú«Ž˜Α
¢ËÚΑ樘Αê½æ«Ãæ˜ΑŽËÄÞêØÚÞΠΑ"êÚ樘ÚΡΑA[TΑ×ژõ˜ÄæÞΑ樘Α×Ú«Ž˜ÞΑ¢ÚËÃΑ£Ë«Ä£Α”ËöÄΑλ”«Ú˜Žæ½üμΠΑTÚ«Ž˜Αö«½½ΑÄËæΑ£ËΑ
˜½ËöΑ樘ΑA[TαΑb¨˜Α¢ÚØÚΑŽÄΑޘ½½Α樘Α×Ú˔ꎘΑæËΑ樘Α£Ëõ˜ÚÄØÄæΑæΑ樘ΑA[TΠΑ+æΑö«½½Α½ÞËΑ×ژõ˜ÄæΑ樘Α
×Ú«Ž˜Α ¢ÚËÃΑ £Ë«Ä£Α ê×Α λ«Ä”«Ú˜Žæ½üμαΑ b¨˜Α ×Ú«Ž˜Α ö«½½Α Þ¨ËËæΑ ê×Α ËĽüΑ ö¨˜ÄΑ 樘Α ×Ú˔êŽæ«ËÄΑ «ÞΑ ½˜ÞÞΡΑ êæΑ «¢Α 樘Α
×Ú˔êŽæ«ËÄΑö«½½Α«ÄŽÚ˜Þ˜ΡΑ樘ÄΑ樘Α×Ú«Ž˜ÞΑö«½½ΑÄËæΑ˜ΑæËËΑ¨«£¨ΠΑ
Option 2 is correct:Αb¨Ë꣨Α¢ŽæËÚÞΑ½«º˜ΑŽ½«Ãæ«ŽΑިˎºÞΑ”˜æ˜ÚëĘΑÚ«Ž˜Α×Ú˔êŽæ«ËÄΑÄ”Α×Ú«Ž˜ÞΑ«ÄΑ樘ΑÞ¨ËÚæΑ
ÚêÄΡΑ樘Α¢êæêژΑގ˜ÄÚ«ËΑË¢ΑÚ«Ž˜Α×Ú«Ž˜ÞΑÃêÞæΑ˜ΑÞ˜”ΑËÄΑ½ËÄ£αæ˜ÚÃΑ×Ú˶˜Žæ«ËÄÞΑË¢ΑÚ«Ž˜Α”˜ÃÄ”ΑÄ”ΑÞê××½üΠΑ
bځ”«Ä£Α ژÞê½æÞΑ «ÄΑ ÄΑ «ÄŽÚ˜Þ˜”Α ”˜ÃÄ”Α «ÄΑ樘Α «Äæ˜Úāæ«Ëā½Α Áں˜æÞΡΑö¨«Ž¨Α ą˜ŽæÞΑ樘Α ×Ú«Ž˜Α Ë¢Α Ú«Ž˜Α «ÄΑ
+Ĕ«ΠΑ
H×æ«ËÄΑ ͌Α «ÞΑ ŽËÚژŽæ΢Α ;ËöΑ ÞæˎºΑ ½˜õ˜½ÞΑ ŽËÄÞæځ«ÄΑ 樘Α «½«æüΑæËΑ êą˜ÚΑ 樘Α ×Ú«Ž˜Α ګޘΑ ژÞê½æ«Ä£Α ¢ÚËÃΑ Ë樘ÚΑ
¢ŽæËÚÞΠΑ
H×æ«ËÄΑ ͍Α «ÞΑ ŽËÚژŽæ΢Α [êÞ«”«˜ÞΑ ½Ëö˜ÚΑ樘Α ×Ú«Ž˜ÞΑ¢ËÚΑ樘Α ê½æ«Ãæ˜Α ŽËÄÞêØÚΠΑ (˜ÄŽ˜Α ½½Α樘Α £«õ˜ÄΑ¢ŽæËÚÞΑ
ą˜ŽæέΑ¨õ˜Αą˜Žæ˜”Π
ANALYSIS:
ANALYSIS:

Scope for elimination

ÞΑö˜ΑºÄËöΑ樁æΑö¨˜Ä˜õ˜ÚΑ£Ëõ˜ÚÄØÄæΑ«Äæ˜Úõ˜Ä˜ÞΑ«ÄΑΑ¢Ú˜˜ΑÁں˜æΡΑ樘Α”«ÞŽËõ˜ÚüΑË¢Α×Ú«Ž˜Α«ÞΑą˜Žæ˜”Αλö¨˜æ¨˜ÚΑ
«æΑ«ÞΑ«ÄŽÚ˜Þ˜”ΑËÚΑ”˜ŽÚ˜Þ˜”Α«ÞΑÄËæΑ«Ã×ËÚæÄæΑ¢ËÚΑ樫ÞΑÙê˜Þæ«ËÄμΠΑ(˜ÄŽ˜ΡΑö˜ΑŽÄΑŽËØΑæËΑÄÞö˜ÚΑ«ÄΑ樫ÞΑÙê˜Þæ«ËÄΑ
樁æΑö¨«Ž¨˜õ˜ÚΑË×æ«ËÄΑ«ÞΑ”«Ú˜Žæ½üΑËÚΑ«Ä”«Ú˜Žæ½üΑژ½æ˜”Αö«æ¨Α樘ΑŽæ«ËÄΑË¢Α樘Α#Ëõ˜ÚÄØÄæΡΑ樘Α×Ú«Ž˜ΑË¢ΑÄüΑ£Ë˔Α
λ¨˜Ú˜ΑÚ«Ž˜μΑö«½½Α˜Αą˜Žæ˜”ΠΑ#Ëõ˜ÚÄØÄæΑ«ÞΑ«Äõ˽õ˜”Α«ÄΑA[TΑÄ”ΑÞêÞ«”«˜ÞΠΑ½ÞËΑË×æ«ËÄÞΑ͋ΑÄ”Α͌Α”«Ú˜Žæ½üΑ¨«ÄæÞΑæΑ
樘ΑŽæ«ËÄΑË¢Α樘Α£Ëõ˜ÚÄØÄæΡΑ樘ژ¢ËژΡΑË×æ«ËÄΑλ”μΑ«ÞΑ樘ΑŽËÚژŽæΑÄÞö˜ÚΠ

SOURCE : https://shs.hal.science/halshs-02275376/document
£Ú«Žê½æêژΑ «ÞΑ ÄΑ «Ã×ËÚæÄæΑ æË׫ŽΑ Ä”Α êĔ˜ÚΑ 樘Α æË׫ŽΑ Ë¢Α «Äæ˜Úõ˜Äæ«ËÄΑ Ë¢Α 樘Α £Ëõ˜ÚÄØÄæΑ «ÄΑ
£Ú«Žê½æêژΡΑ樫ÞΑÙê˜Þæ«ËÄΑÃêÞæΑ¨õ˜Α˜˜ÄΑŽËõ˜Ú˜”ΠΑ

Year 2020 KISAN CREDIT CARD

Q13. Under the Kisan Credit Card scheme, short-term credit support is given to farmers for which of
the following purposes?
1. Working capital for maintenance of farm assets
2. Purchase of combine harvesters, tractors and mini trucks.
3. Consumption requirements of farm households
4. Post-harvest expense
5. Construction of family house and setting up of village cold storage facility.
Select the correct answer using the code given below:
(a) 1, 2 and 5 only
(b) 1, 3 and 4 only
(c) 2, 3, 4 and 5 only
(d) 1, 2, 3 4 and 5

ANSWERS B
132
EXPLANATION
9«ÞÄΑژ”«æΑÚ”Α[Ž¨˜Ã˜Αλ9μ΢Α+æΑ«ÞΑΑæüטΑË¢Α½ËÄΑ£«õ˜ÄΑæËΑ¢ÚØÚÞΠΑb¨˜Αގ¨˜Ã˜ΑöÞΑ«ÄæÚ˔ꎘ”Α«ÄΑ͊͒͒͑ΠΑ
+æΑ«ÞΑÄËæΑΑŽÚ˜”«æΑŽÚ”Α¢ËÚΑ¢ÚØÚÞΠΑHence options 2 and 5 are not correct.
+æΑ«ÞΑ½«º˜ΑÄΑËõ˜Ú”ځ¢æΑގ¨˜Ã˜Α¢ËÚΑ¢ÚØÚÞαΑHĘΑŽÄΑËõ˜Ú”ځöΑê×ΑæËΑΑ½«Ã«æΠΑĔΑ«Äæ˜Ú˜ÞæΑ¨ÞΑæËΑ˜Αׁ«”ΑËÄΑ
樘ΑÃËêÄæΑÞËΑËõ˜Ú”ځöÄΠΑ½ÞËΡΑ樘Α½«Ã«æΑË¢ΑËõ˜Ú”ځ¢æΑ«ÞΑĈû˜”ΑËÄΑ樘ΑÞ«ÞΑË¢Α樘Αõ½ê˜ΑË¢Α½Ä”ΡΑæ¨Ë꣨Α樘Α
õ½ê˜ΑË¢ΑŽÚË×ΑÁüΑ½ÞËΑ˜ΑŽËÄÞ«”˜Ú˜”ΠΑ
The Features of This Scheme:
"ÚØÚÞΑʘ”ΑÃËĘüΑ˜¢ËژΑÞËö«Ä£ΠΑ"ËÚΑ˜ûÃ×½˜ΡΑæËΑêüΑޘ˜”ÞΡΑ¢˜Ú櫽«Þ˜ÚÞΑ˜æŽΠ
+ÄΑ樫ÞΑގ¨˜Ã˜ΡΑ樘üΑ¨õ˜ΑæËΑׁüΑŽºΑ樘Α½ËÄΑÃËêÄæέΑËõ˜Ú”ځ¢æΑ¢æ˜ÚΑ樘Α¨Úõ˜ÞæΠΑ
½ÞËΡΑ«Äæ˜Ú˜ÞæΑŽ¨Ú£˜”Α«ÞΑŽËϘÞÞ«Ëā½αΑ͍Ρ͎Ρ͏АαΑ êæΑ«¢Α樘üΑ¢«½ΑæËΑׁüŽºΑ×ËÞæΑ樘Α¨Úõ˜ÞæΡΑ樘ÄΑ樘Α
«Äæ˜Ú˜ÞæΑŽ¨Ú£˜”Α«ÄŽÚ˜Þ˜ÞΑæËΑ͊͌α͍͊АαΑb¨˜ΑÞêÞ«”üΑŽËÃ×ËĘÄæΑ«ÞΡΑæ¨êÞΡΑژÃËõ˜”ΠΑ
b¨˜Αގ¨˜Ã˜Α¨ÞΑ˜˜ÄΑõ˜ÚüΑÞꎎ˜ÞÞ¢ê½αΑAËژΑ樁ÄΑ͉͊ΑŽÚËژΑ9ÞΑ¨õ˜Α½Ú˜”üΑ˜˜ÄΑ«ÞÞꘔΠ
ËÃØڎ«½ΡΑŽËËטځæ«õ˜ΡΑW˜£«Ëā½ΑWêځ½Α ÄºÞαΑ½½ΑæüטÞΑË¢ΑÄºÞΑ«ÞÞÞê˜Α9ÞΑ
½ÞËΡΑ樘Α½ËÄÞΑ£«õ˜ÄΑæËΑ¢ÚØÚÞΑæ¨ÚË꣨Α9ΑÚ˜ΑŽËêÄ昔Α«ÄΑTÚ«ËÚ«æüΑ[˜ŽæËÚΑ;˜Ä”«Ä£ΑαΑ+æΑ«ÞΑΑ؁ÄÞΑË¢Α
£«õ«Ä£Α½ËÄÞΑæËΑ¢ÚØÚÞΠΑ
+æΑ«ÞΑõ˜ÚüΑ¨˜½¢ê½ΑÄ”Α˜Ä˜ĈŽ«½Α¢ËÚΑ¢ÚØÚÞΡΑÞΑ«ÄΑËڔ˜ÚΑæËΑ«ÄŽÚ˜Þ˜Α£Ú«Žê½æêځ½Α×Ú˔êŽæ«ËÄΡΑ樘Α¢ÚØÚÞΑ
ʘ”ΑæËΑêüΑõÚ«ËêÞΑ«Ä×êæÞΑÞꎨΑÞΑޘ˜”ÞΡΑ¢˜Ú櫽«Þ˜ÚÞΑ˜æŽΠΑb¨«ÞΑÄËæΑËĽüΑ½˜”ÞΑæËΑÄΑ«ÄŽÚ˜Þ˜Α«ÄΑ¢ÚØÚÞϐΑ
«ÄŽËØΑêæΑ½ÞËΑ«ÄΑÄΑ«ÄŽÚ˜Þ˜Α«ÄΑ£Ú«Žê½æêځ½Α×Ú˔êŽæ«ËÄΠΑ
H¶˜Žæ«õ˜ÞέTêÚ×ËޘΑ 9«ÞÄΑ ژ”«æΑ Ú”Α [Ž¨˜Ã˜Α «ÃÞΑ æΑ ×ÚËõ«”«Ä£Α ”˜Ùêæ˜Α Ä”Αæ«Ã˜½üΑ ŽÚ˜”«æΑ Þê××ËÚæΑ
¢ÚËÃΑ樘ΑÄº«Ä£ΑÞüÞæ˜ÃΑêĔ˜ÚΑΑÞ«Ä£½˜Αö«Ä”ËöΑæËΑ樘Α¢ÚØÚÞΑ¢ËÚΑ樘«ÚΑŽê½æ«õæ«ËÄΑϞΑË樘ÚΑʘ”ÞΑÞΑ
«Ä”«Žæ˜”Α˜½Ëö΢Α
bËΑؘæΑ樘ΑÞ¨ËÚæΑæ˜ÚÃΑŽÚ˜”«æΑژÙê«Ú˜Ã˜ÄæÞΑ¢ËÚΑŽê½æ«õæ«ËÄΑË¢ΑŽÚË×ÞΑ
TËÞæΑ¨Úõ˜ÞæΑ˜ûטÄޘÞΑ
Produce Marketing loan
ËÄÞêÃ×æ«ËÄΑژÙê«Ú˜Ã˜ÄæÞΑË¢Α¢ÚØÚΑ¨Ëêޘ¨Ë½”Α
rËÚº«Ä£ΑŽ×«æ½Α¢ËÚΑÁ«Äæ˜ÄÄŽ˜ΑË¢Α¢ÚÃΑÞޘæÞΑÄ”ΑŽæ«õ«æ«˜ÞΑ½½«˜”ΑæËΑ£Ú«Žê½æêژΡΑ½«º˜Α”«ÚüΑÄ«Ã½ÞΡΑ
«Ä½Ä”ΑĈÞ¨˜ÚüΑ˜æŽΠΑ
+Äõ˜ÞæØÄæΑ ŽÚ˜”«æΑ ژÙê«Ú˜Ã˜ÄæΑ ¢ËÚΑ £Ú«Žê½æêژΑ Ä”Α ½½«˜”Α Žæ«õ«æ«˜ÞΑ ½«º˜Α ×êÃ×Α ޘæÞΡΑ Þ×ځü˜ÚÞΡΑ ”«ÚüΑ
Ä«Ã½ÞΑ˜æŽΠΑ
(˜ÄŽ˜ΑÞææ˜Ã˜ÄæÞΑ͊ΡΑ͌ΑÄ”Α͍ΑÚ˜ΑŽËÚژŽæΠΑ(˜ÄŽ˜ΑË×æ«ËÄΑλμΑ«ÞΑ樘ΑŽËÚژŽæΑÄÞö˜ÚΠ

ANALYSIS:

Scope for elimination


b¨˜Α õ˜ÚüΑ Þ«ŽΑ êĔ˜ÚÞæÄ”«Ä£Α The scheme takes care of the following:
was enough to answer this bËΑؘæΑ樘ΑÞ¨ËÚæΑæ˜ÚÃΑŽÚ˜”«æΑژÙê«Ú˜Ã˜ÄæÞΑ¢ËÚΑŽê½æ«õæ«ËÄΑË¢ΑŽÚË×ÞΑ
Ùê˜Þæ«ËÄΠΑ 9Α «ÞΑ ÄËæΑ ½«º˜Α Α
TËÞæΑ¨Úõ˜ÞæΑ˜ûטÄޘÞΑ
personal credit card rather a
£Ëõ˜ÚÄØÄæΑ ގ¨˜Ã˜ΡΑ Produce Marketing loan
樘ژ¢ËژΡΑ Þææ˜Ã˜ÄæΑ ͋Α Ä”Α ͎Α ËÄÞêÃ×æ«ËÄΑژÙê«Ú˜Ã˜ÄæÞΑË¢Α¢ÚØÚΑ¨Ëêޘ¨Ë½”Α
Ú˜Α ÄËæΑ ŽËÚژŽæΠΑ HϘΑ üËêΑ
rËÚº«Ä£ΑŽ×«æ½Α¢ËÚΑÁ«Äæ˜ÄÄŽ˜ΑË¢Α¢ÚÃΑÞޘæÞΑÄ”ΑŽæ«õ«æ«˜ÞΑ½½«˜”Α
ژÃËõ˜ΑÞææ˜Ã˜ÄæΑ͋ΑÄ”Α͎ΡΑö˜Α
æËΑ£Ú«Žê½æêژΡΑ½«º˜Α”«ÚüΑÄ«Ã½ÞΡΑ«Ä½Ä”ΑĈÞ¨˜ÚüΑ˜æŽΠΑ
ö«½½Α˜Α½˜¢æΑö«æ¨ΑËĽüΑËĘΑË×æ«ËÄΡΑ
樁æΑ«ÞΡΑË×æ«ËÄΑλμΡΑö¨«Ž¨Α«ÞΑ樘Α +Äõ˜ÞæØÄæΑ ŽÚ˜”«æΑ ژÙê«Ú˜Ã˜ÄæΑ¢ËÚΑ £Ú«Žê½æêژΑ Ä”Α ½½«˜”Α Žæ«õ«æ«˜ÞΑ
ŽËÚژŽæΑÄÞö˜ÚΠΑ ½«º˜Α×êÃ×ΑޘæÞΡΑÞ×ځü˜ÚÞΡΑ”«ÚüΑÄ«Ã½Þ

SOURCE : https://sbi.co.in/web/agri-rural/agriculture-banking/crop-loan/kisan-credit-card
133

Year 2019 AGRICULTURAL COMMODITIES

Q14. Consider the following statements:


1. In the case of all cereals, pulses and oil-seeds, the procurement at Minimum Support Price (MSP) is unlimited in any
State/UT of India.
2. In the case of cereals and pulses, the MSP is fixed in any State/UT at a level to which the market price will never rise.
Which of the statements given above is/are correct?
(a) 1 only
(b) 2 only
(c) Both 1 and 2
(d) Neither 1 nor 2

ANSWERS D

EXPLANATION
Þ˜”Α ËÄΑ 樘Α ژŽËÃØĔæ«ËÄÞΑ Ë¢Α 樘Α ËÃëÞÞ«ËÄΑ ¢ËÚΑ £Ú«Žê½æêځ½Α ËÞæÞΑ Ä”Α TÚ«Ž˜ÞΑ λTμΡΑ 樘Α
˜×ÚæØÄæΑ Ë¢Α £Ú«Žê½æêژΑ Ä”Α ËËטځæ«ËÄΡΑ A«Ä«ÞæÚüΑ H¢Α £Ú«Žê½æêژΑ ĔΑ "ÚØÚÞϖΑ r˜½¢Ú˜ΡΑ ”˜Ž½Ú˜ÞΑ
A«Ä«ÃêÃΑ [ê××ËÚæΑ TÚ«Ž˜ÞΑ λA[TμΑ ¢ËÚΑ ÃËژΑ 樁ÄΑ ͉͋Α ÁĔæ˜”Α ŽÚË×ÞΡΑ Ä”Α "«ÚΑ Ä”Α W˜ÃêĘځæ«õ˜Α TÚ«Ž˜Α
(FRP) for Sugarcane.
˜Ú˜½ÞΑλ͐μΑαΑׁ””üΡΑö¨˜æΡΑÚ½˜üΡΑ¶ËöÚΡΑ¶ÚΡΑÁ«ā˜ΑÄ”Αځ£«Α
Tê½Þ˜ÞΑλ͎μΑαΑ£ÚÃΡΑÚ¨ÚέæêÚΡΑÃËËÄ£ΡΑêځ”ΑÄ”Α½˜Ä櫽Α
H«½Þ˜˜”ÞΑλ͑μΑαΑ£ÚËêĔÄêæΡΑځטޘ˜”έÃêÞæÚ”ΡΑæËÚ«ΡΑÞËü˜ÄΡΑÞêÄĉËö˜ÚΑޘ˜”ΡΑޘށÃêÃΡΑށćËö˜ÚΑޘ˜”Α
Ä”ΑÄ«£˜Úޘ˜”ΡΑ
WöΑŽËææËÄΡΑ
WöΑ¶êæ˜ΡΑË×ځ
b¨˜Α«”˜Α˜¨«Ä”ΑA[TΑ«ÞΑæËΑ£«õ˜Α£êÚÄ昘”Α×Ú«Ž˜ΑÄ”ΑÞÞêژ”ΑÁں˜æΑæËΑ樘Α¢ÚØÚÞΑÄ”Α×ÚË明æΑ樘ÃΑ¢ÚËÃΑ
樘Α ×Ú«Ž˜Α ĉêŽæêæ«ËÄÞΑ Ä”Α Áں˜æΑ «Ãטڢ˜Žæ«ËÄÞΠΑ TΑ ŽËÄÞ«”˜ÚÞΑ õÚ«ËêÞΑ ¢ŽæËÚÞΑ ÞꎨΑ ÞΑ 樘Α ŽËÞæΑ Ë¢Α
Žê½æ«õæ«ËÄΑÄ”Α×Ú˔êŽæ«ËÄΡΑ×Ú˔êŽæ«õ«æüΑË¢ΑŽÚË×ÞΡΑÄ”ΑÁں˜æΑ×Ú«Ž˜ÞΑ¢ËÚΑ樘Α”˜æ˜Úëāæ«ËÄΑË¢ΑA[TÞΠΑA[TΑ
Ĉû˜”Α¢ËÚΑ˜Ž¨ΑŽÚË×Α«ÞΑêÄ«¢ËÚÃΑ¢ËÚΑ樘Α˜Äæ«Ú˜ΑŽËêÄæÚüΠΑ(Ëö˜õ˜ÚΡΑ樘ژΑ«ÞΑÄËΑ«ÄÞæÄŽ˜ΑË¢ΑA[TΑ˜«Ä£ΑĈû˜”Α«ÄΑ
ÄüΑ[ææ˜έfbΑæΑΑ½˜õ˜½ΑæËΑö¨«Ž¨Α樘ΑÁں˜æΑ×Ú«Ž˜Αö«½½ΑĘõ˜ÚΑګޘΠΑHence statement 2 is not correct.
r¨«½˜Α樘Α£Ëõ˜ÚÄØÄæΑÄÄËêϘÞΑA[TΑ¢ËÚΑŽÚË×ÞΡΑ˜ą˜Žæ«õ˜ΑA[TΑ½«Äº˜”Α×ÚˎêژØÄæΑˎŽêÚÞΑÁ«Ä½üΑ¢ËÚΑ
wheat, rice and cotton.Α b¨˜Α ×ÚˎêژØÄæΑ «ÞΑ ½ÞËΑ ½«Ã«æ˜”Α æËΑ Α ¢˜öΑ Þææ˜ÞΠΑ ê˜Α æËΑ ½«Ã«ææ«ËÄÞΑ ËÄΑ 樘Α
×ÚˎêژØÄæΑÞ«”˜ΑλËæ¨ΑŽÚË×Αö«Þ˜ΑÄ”ΑÞææ˜αö«Þ˜μΑÄ”Α˜õ˜ÄΑ¢æ˜ÚΑ¨õ«Ä£ΑÄΑËטÄα˜Ä”˜”Α×ÚˎêژØÄæΑæΑ
A[TΡΑ½½Α¢ÚØÚÞΑ”ËΑÄËæΑژŽ˜«õ˜Α˜Ä˜ĈæÞΑË¢ΑÄΑ«ÄŽÚ˜Þ˜Α«ÄΑA[TΠΑHence statement 1 is not correct
134

ANALYSIS:
ANALYSIS:
A[TΑ ¨ÞΑ ½öüÞΑ Þæü˜”Α «ÄΑ ĘöÞΑ ÞΑ «æΑ «ÞΑ ËĘΑ Ë¢Α 樘Α ÃËÞæΑ ŽËÄæ˜Äæ«ËêÞΑ Áææ˜ÚΑ ¢ËÚΑ Ëæ¨Α #Ëõ˜ÚÄØÄæΑ Ä”Α
"ÚØÚÞΠΑb¨˜ΑA[TΑöÞΑ½ÞËΑ«ÄΑĘöÞΑ”êÚ«Ä£Α樘ΑÁÞÞ«õ˜Α¢ÚÃΑ×ÚËæ˜ÞæÞΑ«ÄΑ+Ĕ«ΠΑ+æΑ¨ÞΑÞæü˜”Α«ÄΑĘöÞΑޫϘΑ
樘Α«ÄŽ˜×æ«ËÄΑË¢ΑA[TΠΑ(˜ÄŽ˜ΡΑ«æΑ«ÞΑΑژŽêÚÚ«Ä£Α樘ØΑ«ÄΑfT[ΣΑÙê˜Þæ«ËÄÞΑژ½æ˜”ΑæËΑA[TΑ¨õ˜Α˜˜ÄΑÞº˜”Α«ÄΑ
͉͋͊͑ΡΑ͉͋͊͒ΡΑ͉͉͋͋ΑÄ”Α͉͋͋͌

Scope for elimination


+¢Α ö˜Α ŽÚ˜¢ê½½üΑ ÄË櫎˜Α 樘Α ĈÚÞæΑ Þææ˜Ã˜ÄæÞΡΑ «æΑ ށüÞΑ ;;Α Ž˜Ú˜½ÞΡΑ ×ê½Þ˜ÞΑ Ä”Α Ë«½Þ˜˜”ÞΑ Ä”Α fB;+A+bΑ
×ÚˎêژØÄæΑö¨«Ž¨ΑÚ˜Α˜ûæژØΑÞææ˜Ã˜ÄæÞΑÄ”ΑÄüΑ˜Äæ«æüΑ樁æΑ«ÞΑ×ÚˎêÚ«Ä£Αλ˜õ˜ÄΑö«æ¨ËêæΑA[TμΑö«½½Α¨õ˜Α
ÞËØΑŽ×Ž«æüΑö¨«Ž¨Α«æΑŽÄΑÄËæΑÚ˜Ž¨˜”ΠΑb¨˜ΑށØΑ«ÞΑæÚê˜Α¢ËÚΑ£Ëõ˜ÚÄØÄæΑ½ÞËΡΑ+æΑŽÄΑÄËæΑ¨õ˜ΑfB;+A+bΑ
×ÚˎêژØÄæΑ Ë¢Α ;;Α Ž˜Ú˜½ÞΡΑ ×ê½Þ˜ÞΑ Ä”Α Ë«½Þ˜˜”ÞΠΑ (˜ÄŽ˜Α Þææ˜Ã˜ÄæΑ ͊Α «ÞΑ öÚËÄ£ΠΑ A[TΑ «ÞΑ 樘Α ëÃêÃêÃΑ
Þê××ËÚæΑ×Ú«Ž˜ΡΑÁں˜æΑ×Ú«Ž˜ΑŽÄΑ˜Α½Ëö˜ÚΑλÄ”Α樁æΑ«ÞΑ樘ΑŽÞ˜Αö¨˜ÄΑ¢ÚØÚΑöË꽔ΑöÄæΑæËΑޘ½½ΑæΑA[TμΑÄ”Α
¨«£¨˜ÚΑ½ÞËΑλö¨˜ÄΑ樘Α¢ÚØÚΑöË꽔ΑöÄæΑæËΑޘ˜ΑæËΑ樘ΑÁں˜æμΑÄ”Α樘ΑށØΑÞΑA[TΑ½ÞËΑλ¢ÚØÚΑö«½½Α¨õ˜Α
樘ΑË×æ«ËÄΑö¨˜Ú˜ΑæËΑޘ½½μΠΑb¨«ÞΑÞææ˜Ã˜ÄæΑ½ÞËΑ¨ÞΑÄΑ˜ûæژØΑŽËĔ«æ«ËÄΑϏr+;;ΑBqWΑW+[ϐΑö¨«Ž¨Α¨ÞΑÄËΑ
Þ˜ΑæËΑ«æΠΑHence both statements are wrong.

SOURCE : This theme was covered in the Economic Survey 2017-18.

Additional Information
135

Year 2020 AGRICULTURAL BANKING

Q15. Consider the following statements:


1. In terms of short-term credit delivery to the agriculture sector, District Central Cooperative Banks (DCCBs) deliver
more credit in comparison to Scheduled Commercial Banks and Regional Rural Banks.
2. One of the most important functions of DCCBs is to provide funds to the Primary Agriculture Credit Societies.
Which of the statements given above is/are correct?
(a) 1 only
(b) 2 only
(c) Both 1 and 2
(d) Neither 1 nor 2

ANSWERS B

EXPLANATION
«ÞæÚ«ŽæΑ ˜Äæځ½Α ËΑ αËטځæ«õ˜Α ÄºÞΑ λ ÞμΑ Ú˜Α Α æüטΑ Ë¢Α Úêځ½Α ŽËΑ αËטځæ«õ˜ÞΡΑ ½ËÄ£Α ö«æ¨Α [ææ˜Α ËΑ
αËטځæ«õ˜Α ÄºÞΑλ[æ ÞμΑÄ”ΑTګÁÚüΑ£Ú«Žê½æêځ½Αژ”«æΑ[ˎ«˜æ«˜ÞΑλT[μΠΑ[¨ËÚæΑαæ˜ÚÃΑŽËΑαΑËטځæ«õ˜ÞΑ
Ú˜ΑÚځģ˜”Α«ÄΑΑæ¨Ú˜˜Αα櫘ÚΑÞæÚêŽæêژΑ«ÄΑÃËÞæΑË¢Α樘ΑÞææ˜ÞΡΑö«æ¨Α[æ ÞΑæΑ樘Α×˜ûΑ½˜õ˜½ΡΑ ÞΑæΑ樘Α
«Äæ˜Úؔ«æ˜Α½˜õ˜½ΑÄ”ΑT[ΑæΑ樘Α£ÚÞÞÚËËæÞΑ½˜õ˜½ΠΑ
½æ¨Ë꣨Α樘Α¢ËŽêÞΑË¢ΑÚêځ½ΑŽËËטځæ«õ˜Α½˜Ä”«Ä£Α«ÞΑ£Ú«Žê½æêژΡΑ樘ΑÞ¨Ú˜Α«ÄΑŽÚ˜”«æΑĉËöΑæËΑ£Ú«Žê½æêژΑË¢Α
Úêځ½ΑŽËËטځæ«õ˜ÞΑ«ÞΑËĽüΑ͊͋Π͊АΡΑÞΑŽËÃׁژ”ΑæËΑ͐͏АΑË¢Α[Ž¨˜”꽘”ΑËÃØڎ«½Α ÄºÞΑλ[ ÞμΡΑÄ”Α͊͊Π͒АΑ
Ë¢ΑW˜£«Ëā½ΑWêځ½Α ÄºÞΠ Hence statement 1 is not correct .
 ÞΑÃˍ«½«Þ˜Α”˜×ËÞ«æÞΑ¢ÚËÃΑ樘Α×ꍽ«ŽΑÄ”Α×ÚËõ«”˜ΑŽÚ˜”«æΑæËΑ樘Α×ꍽ«ŽΑÄ”ΑT[ΠΑHence statement 2
is correct.

ANALYSIS:
ANALYSIS:
b¨«ÞΑöÞΑÄËæΑΑõ˜ÚüΑŽËÃÃËÄΑ樘ØΡΑ«¢Αö˜ΑŽËÃׁژΑ樫ÞΑö«æ¨ΑW˜£«Ëā½ΑWêځ½Α ÄºÞΑËÚΑ樘ΑB WΡΑêæΑÄËæΑ
ËêæΑË¢Α樘ΑÞü½½êÞΠ

SOURCE : https://www.rbi.org.in/scripts/PublicationsView.aspx?Id=19366
136

Additional Information
b¨˜Α[æÚêŽæêژΑË¢ΑËαËטځæ«õ˜ÞΑüΑÞޘæΑ[«ā˜

[Ž¨˜”꽘”Αf([Αλ͎͍μ
͍͐Π͉
fڍÄ
½½ΑËαËטځæ«õ˜Þ ËαËטځæ«õ˜Þ 100
λ͊͒͐Π͐͒͋μ λ͊Ρ͎͍͍μ BËÄα[Ž¨˜”꽘”Αf([Αλ͊Ρ͍͉͒μ
͎͌Π͌
͎͌Π͉  ÞΑλ͌͏͌μ T[Αλ͎͒Ρ͋͌͑μ
100 ͋͌Π͏
[æ [Αλ͌͌μ ͎͒Π͒
͏͍Π͐ 22.0
100
Wêځ½ 1.5
ËαËטځæ«õ˜Þ 2.0
[W [Αλ͊͌μ
λ͊Ρ͎͍͍μ TW [Αλ͏͉͊μ

b¨˜ΑËõ˜ΑĈ£êژΑÞ¨ËöÞΑ樘ΑÞޘæαÞ«ā˜Αλ«ÄΑŽ«ÚŽ½˜ÞμΑÄ”ΑÄê͘ÚΑË¢ΑÚÄŽ¨˜ÞΑλ«ÄΑÚŽº˜æÞμ

Year 2023 SMALL FARMER LARGE FIELD

Q16. Which one of the following best describes the concept of 'Small Farmer Large Field'?
(a) Resettlement of a large number of people, uprooted from their countries due to war, by giving them a large
cultivable land which they cultivate collectively and share the produce
(b) Many marginal farmers in an area organize themselves into groups and synchronize and harmonize selected
agricultural operations
(c) Many marginal farmers in an area together make a contract with a corporate body and surrender their land to
the corporate body for a fixed term for which the corporate body makes a payment of agreed amount to the
farmers
(d) A company extends loans, technical knowledge and material inputs to a number of small farmers in an area so
that they produce the agricultural commodity required by the company for its manufacturing process and commer-
cial production

ANSWERS B

EXPLANATION
[Á½½Α"ÚØÚÞΑ;Ú£˜Α"«˜½”Αλ[";"μΑ«ÞΑΑŽË½½˜Žæ«õ˜ΑŽæ«ËÄΑÃ˔˜½ΑæËΑËõ˜ÚŽËØΑ樘Α”«Þ”õÄ恣˜ÞΑ¢Ž˜”ΑüΑ
ë½½«ËÄÞΑË¢ΑÞÁ½½ΑÄ”ΑÁڣ«Ä½Α¢ÚØÚÞΑ”ê˜ΑæËΑ”«Þ˜ŽËÄËë˜ÞΑË¢Αގ½˜ΑÄ”Α½ŽºΑË¢ΑÚ£«Ä«Ä£Α×Ëö˜ÚΑ«ÄΑ樘Α
Þê××½üΑŽ¨«ÄΠΑb¨«ÞΑÃ˔˜½Α«ÞΑׁÚ櫎«×æËÚüΑÄ”Αĉ˜û«½˜ΑÄ”Α½½ËöÞΑÞÁ½½Α¢ÚØÚÞΑæËΑ˜Ä˜ĈæΑ¢ÚËÃΑŽ¨«˜õ«Ä£Α
˜ŽËÄËë˜ÞΑË¢Αގ½˜ΑüΑËÚ£Ä«ā«Ä£Α樘Ãޘ½õ˜ÞΑ«ÄæËΑ£ÚËê×ÞΑÄ”ΑÞüϨÚËÄ«ā«Ä£ΑÄ”Α¨ÚÃËÄ«ā«Ä£Αޘ½˜Žæ˜”Α
Ëטځæ«ËÄÞΠΑ+æΑöÞΑ׫½Ë昔Α«ÄΑæöËΑõ«½½£˜ÞΑË¢ΑH”«Þ¨ΡΑö«æ¨Α͊͊͋Α¢ÚØÚÞΑλ͎͌Α¢˜Ã½˜ÞΑÄ”Α͐͐ΑÁ½˜ÞμΠΑb¨˜Þ˜Α
¢ÚØÚÞΑ ËÚ£Ä«ā˜”Α 樘Ãޘ½õ˜ÞΑ «ÄæËΑ £ÚËê×ÞΑ Ä”Α ÞüϨÚËÄ«ā˜”Α 樘«ÚΑ Ëטځæ«ËÄÞΑ ÞꎨΑ ÞΑ ÄêÚޘÚüΑ ˜”Α
Áā£˜Ã˜ÄæΡΑæځÄÞ×½Äæ«Ä£ΡΑÄ”Α¨Úõ˜Þæ«Ä£ΑŽË½½˜Žæ«õ˜½üΑæËΑŽ¨«˜õ˜Α˜ŽËÄËë˜ÞΑË¢Αގ½˜ΠΑb¨˜Α[";"Α¢ÚØÚÞΑ
×êڎ¨Þ˜”Α«Ä×êæÞΑλޘ˜”ΑÄ”Α¢˜Ú櫽«ā˜ÚμΑÄ”ΑÞ˽”Αׁ””üΑÞΑΑ£ÚËê×ΑæËΑ«ÄŽÚ˜Þ˜Α樘«ÚΑÚ£«Ä«Ä£Α×Ëö˜ÚΑ«ÄΑ
×Ú«Ž˜ΑĘ£Ë櫁æ«ËÄÞΠΑb¨˜ΑژÞê½æÞΑ¢ÚËÃΑ樫ÞΑ׫½ËæΑÞæê”üΑÞ¨Ëö˜”Α樁æΑ樘ΑׁÚ櫎«×æ«Ä£Α¢ÚØÚÞΑ½ÃËÞæΑ”Ëꍽ˜”Α
樘«ÚΑ ×ÚËĈæÞΠΑ ׁÚæΑ ¢ÚËÃΑ 樘Α ÃËĘæÚüΑ ˜Ä˜ĈæÞΡΑ 樘ޘΑ ¢ÚØÚÞΑ ށõ˜”Α æ«Ã˜Α «ÄΑ ÁÄüΑ ¶Ë«ÄæΑ Žæ«õ«æ«˜ÞΡΑ
«ÄŽ½ê”«Ä£Α«Ä×êæΑλޘ˜”ΑÄ”Α¢˜Ú櫽«ā˜ÚμΑ×êڎ¨Þ˜ΡΑׁ””üΑށ½˜ΡΑÄ”ΑÄêÚޘÚüΑ˜”ΑÁā£˜Ã˜ÄæΠΑH樘ÚΑ«Ã×ËÚæÄæΑ
˜Ä˜ĈæÞΑ Ë¢Α 樘Α ¨ÚÃËÄ«āæ«ËÄΑ Ä”Α ÞüϨÚËÄ«āæ«ËÄΑ Ë¢Α ¢ÚëģΑ Ëטځæ«ËÄÞΑ ö˜Ú˜Α Þˎ«½Α ¨ÚÃËÄüΑ Ä”Α
ÞêÞ恫ā«½«æüΑË¢Α樘Α¢ÚëģΑÞüÞæ˜ÃΠ Hence option (b) is the correct answer.
137

ANALYSIS:

Scope for elimination


½æ¨Ë꣨ΑË×æ«ËÄΑλμΑÄ”ΑË×æ«ËÄΑλŽμΑöË꽔ΑŽæŽ¨Α樘Αææ˜Äæ«ËÄΑË¢Α樘ΑŽÄ”«”æ˜ÞΑλ«¢ΑËĘΑ××½«˜ÞΑ”˜Ùêæ˜Α½Ë£«ŽμΡΑ
«æΑ루æΑ˜ΑÚ«ÞºüΑæËΑޘæ潘Α¢ËÚΑËĘΑË¢Α樘ΑæöËΑË×æ«ËÄÞΠΑ+¢Α樘ΑÙê˜Þæ«ËÄΑ«ÞΑÄËæΑ”«ĆŽê½æΡΑ«æΑ«ÞΑÄËæΑõ˜ÚüΑ˜ÞüΑ˜«æ¨˜ÚΠ

SOURCE : +æΑöÞΑ˜û×½«Ä˜”Α«ÄΑΑBËõ˜Ã˜ÚΑ͉͋͊͑Α+Ĕ«ÄΑû×ژÞÞΑÚ櫎½˜ΠΑT½˜Þ˜ΑĈĔΑ樘Αގژ˜ÄÞ¨ËæΑË¢Α樘Α
article below.

 


PRELIMS MENTORING PROGRAM 2024


2024
15 March Duration: 2 Months

(«£¨½üΑ experienced
Highly ˜ûטګ˜ÄŽ˜”Α and
Ä”Α qualified
Ùꁽ«Ĉ˜”Α team
昁ÃΑ of
Ë¢Α [Ž¨˜”꽘”Α group
Scheduled £ÚËê×Α sessions
ޘÞÞ«ËÄÞΑ for
¢ËÚΑ strategy
Þæځ昣üΑ discussion,
”«ÞŽêÞÞ«ËÄΡΑ
A˜ÄæËÚÞΑ for
Mentors ¢ËÚΑ continuous
ŽËÄæ«ÄêËêÞΑ support
Þê××ËÚæΑ and
Ä”Α ½«õ˜Α×ځŽæ«Ž˜ΡΑÄ”Αט˜ÚΑ«Äæ˜ÚŽæ«ËÄ
live practice, and peer interaction
guidance
[טŽ«½Α emphasis
Special ˜ÃרÞ«ÞΑ on
ËÄΑ subjects
Þꍶ˜ŽæÞΑ and
Ä”Α topics
æË׫ŽÞΑ with
ö«æ¨Α
¨«£¨αގËÚ«Ä£Α×Ëæ˜Ä櫁½
high-scoring potential
Α structured
A ÞæÚêŽæêژ”Α plan
×½ÄΑ of
Ë¢Α revision
ژõ«Þ«ËÄΑ for
¢ËÚΑ GS
#[Α
Tژ½«ÃÞΡΑ[bΡΑÄ”ΑêÚژÄæΑą«ÚÞ
Prelims, CSAT, and Current Affairs
HĘαæËαËĘΑ dedicated
One-to-one ”˜”«Žæ˜”Α sessions
ޘÞÞ«ËÄÞΑ with
ö«æ¨Α mentors
ØÄæËÚÞΑ for
¢ËÚΑ
×ژׁځæ«ËÄΑÁā£˜Ã˜Äæ
preparation management
ą˜Žæ«õ˜Α Utilization
Effective f櫽«āæ«ËÄΑ of
Ë¢Α learning
½˜ÚÄ«Ä£Α resources,
ژÞËêڎ˜ÞΡΑ
«ÄŽ½ê”«Ä£Α PYQs,
including TxVÞΡΑ Quick
V꫎ºΑ Revision
W˜õ«Þ«ËÄΑ Modules
A˔꽘ÞΑ
λVWAÞμΡΑÄ”ΑTbα͌͏͎
(QRMs), and PT-365 ËÄæ«ÄêËêÞΑ performance
Continuous טڢËÚÁϘΑ ÞޘÞÞØÄæΑ
assessment Ä”Α
and
«Ã×ÚËõ˜Ã˜Äæ
improvement

W˜Þ˜ÚŽ¨Α based
Research Þ˜”Α subject-wise
Þꍶ˜Žæαö«Þ˜Α strategy
Þæځ昣üΑ
+Äæ˜ÚŽæ«õ˜ΑޘÞÞ«ËÄΑö«æ¨ΑbË×טÚÞΑÄ”Α
Interactive session with Toppers and Bureaucrats
êژêŽÚæÞΑ
”ËŽêØÄæÞ
documents
¢ËÚΑ×ژׁځæ«ËÄΑ«ÄÞ«£¨æÞΑÄ”ΑÃËæ«õæ«ËÄ
for preparation insights and motivation
138

Year 2023 Niger


ANALYSIS:
Q17. Consider the following statements:
1. The Government of India provides Minimum Support Price for niger (Guizotia abyssinicia) seeds.
2. Niger is cultivated as a Kharif crop.
3. Some tribal people in India use niger seed oil for cooking.
How many of the above statements are correct?
(a) Only one
(b) Only two
(c) All three
(d) None

ANSWERS C

EXPLANATION

HĘΑË¢Α樘Α͍͊Αº¨Ú«¢ΑŽÚË×ÞΑ¢ËÚΑö¨«Ž¨Α樘Α˜ÄæژΑژ½˜Þ˜ÞΑΑëīÃêÃΑÞê××ËÚæΑ×Ú«Ž˜ΑλA[TμΑ˜õ˜ÚüΑü˜ÚΑ«ÞΑÄΑ
êĽ«º˜½üΑ×½ÄæΑŽ½½˜”ΑÄ«£˜ÚΑËÚΑځÃ櫽Αλ#ê«āË櫁ΑüÞÞ«Ä«ŽμΠΑb¨«ÞΑ½˜ÞޘÚΑºÄËöÄΑË«½Þ˜˜”ΑŽËÃÁĔÞΑËĘΑË¢Α樘Α
¨«£¨˜ÞæΑ A[TÞΡΑ Ĉû˜”Α ËÄΑ 樘Α Þ«ÞΑ Ë¢Α 樘Α ŽËÞæΑ Ë¢Α ×Ú˔êŽæ«ËÄΑ Ä”Α Áں˜æΑ ”˜ÃÄ”ΠΑ x˜æΡΑ 樘Α Ú˜Α êĔ˜ÚΑ «æÞΑ
Žê½æ«õæ«ËÄΑ¨ÞΑ˜˜ÄΑÞ昁”«½üΑިګĺ«Ä£ΑËõ˜ÚΑ樘ΑׁÞæΑæ¨Ú˜˜Α”˜Ž”˜ÞΠΑ+ÄΑ͎͊͒͑α͑͏ΡΑÄ«£˜ÚΑöÞΑŽê½æ«õæ˜”ΑËÄΑ
ÚËêĔΑ͉Π͏Αë½½«ËÄΑ¨˜ŽæÚ˜ÞΑ먁μΠ Hence statement 1 and statement 2 is correct.

+ÄΑ͉͉͋͋α͋͊ΡΑ樘ΑŽê½æ«õæ˜”ΑÚ˜ΑÞ¨ÚêĺΑüΑ͉͑ΑטÚΑŽ˜ÄæΑæËΑΑ½«æ潘ΑËõ˜ÚΑ͉Π͊Αë½½«ËÄΑ¨ΡΑ樘Α½Ëö˜ÞæΑÃËÄ£Α樘Α͍͊Α
º¨Ú«¢ΑŽÚË×ÞΑö«æ¨ΑA[TΠΑb¨˜Α×ګÁÚüΑژÞËÄΑÞꎨΑΑÞׁÚޘ½üΑ£ÚËöÄΑŽÚË×Α«ÞΑׁÚæΑË¢Α樘Α˜ûŽ½êÞ«õ˜ΑA[TΑ½«ÞæΑ«ÞΑ
˜ŽêޘΑ Ä«£˜ÚΑ ޘ˜”Α ¨ÞΑæځ”«æ«Ëā½½üΑ ύ˜˜ÄΑ樘Α ½«¢˜½«Ä˜Α Ë¢ΑæÚ«½Α £Ú«Žê½æêژΑ Ä”Α ˜ŽËÄËÃüΑ «ÄΑ ޘõ˜Ú½Α Þææ˜ÞΑ
ŽÚËÞÞΑ+Ĕ«ΠΑ
b¨˜ΑæÚ«½Α×Ë×꽁æ«ËÄΑêޘÞΑÄ«£˜ÚΑޘ˜”ΑË«½Α¢ËÚΑŽË˺«Ä£ΡΑ樘Α×ژÞÞΑŽº˜Α×ËÞæΑË«½α˜ûæځŽæ«ËÄΑÞΑ½«õ˜ÞæˎºΑ¢˜˜”ΡΑÄ”Α
½ÞËΑŽËÄÞêØΑ樘Αޘ˜”ÞΑÞΑΑŽËĔ«Ã˜ÄæΠΑB«£˜ÚΑޘ˜”ΑË«½Α¨ÞΑؔ«Ž«Ä½Α×ÚËטÚ櫘ÞΡΑö¨«Ž¨Α«ÞΑ樘ΑژÞËÄΑ¢ËÚΑ«æÞΑ
ŽËÃØڎ«½Α ”˜ÃÄ”Α üΑ 樘Α ŽËÞØ櫎ÞΡΑ טڢêØګ˜ÞΑ Ä”Α Ë樘ÚΑ ½½«˜”Α «Ä”êÞæÚ«˜ÞΠΑ (˜ÄŽ˜Α Þææ˜Ã˜ÄæΑ ͌Α «ÞΑ
correct.

ANALYSIS:

Scope for elimination


BËΑ ގËטΑ ÞΑ 樘Α Ùê˜Þæ«ËÄΑ «ÞΑ ×êژ½üΑ ¢ŽæꁽΑ Ä”Α 樘Α fT[ϐÞΑ ĘöΑ ׁææ˜ÚÄΑ ¨ÞΑ ˜½«Ã«Äæ˜”Α 樘Α ގËטΑ ¢ËÚΑ
˜½«Ã«Äæ«ËÄΑ××½ü«Ä£Α樘ΑŽ½ÞÞ«ŽΑ˜½«Ã«Äæ«ËÄΑ明¨Ä«Ùê˜Π

BËÚÁ½½üΡΑA[TΑ«ÞΑ½öüÞΑ«ÄΑ樘ΑĘöÞΑÄ”Α¨ÞΑ˜˜ÄΑõ˜ÚüΑ«Ã×ËÚæÄæΑ¢ÚËÃΑ樘Α[Α×Ë«ÄæΑË¢Αõ«˜öΠ
ׁÚæΑ¢ÚËÃΑ樁æΡΑΑÄ«£˜Úޘ˜”ΑöÞΑ«ÄΑ樘ΑĘöÞΑ¢ËÚΑ樘Α”˜Ž½«Ä˜Α«ÄΑÚ˜ΑêĔ˜ÚΑ×Ú˔êŽæ«ËÄΠ

SOURCE : Standard newspaper or magazines (Down To Earth) as this current affairs-derived


https://www.downtoearth.org.in/news/agriculture/illusive-oil-
seed-india-s-niger-seed-cultivation-is-declining-here-is-why-84380
139
140

FOOD PROCESSING: INDUSTRIES


λ͋ΑVê˜Þæ«ËÄÞμ

Year 2022 E-COMMERCE

V͊ΠΑr«æ¨Α ژ¢˜Ú˜ÄŽ˜ΑæËΑ¢Ëژ«£ÄαËöʔΑ ˜αŽËÃØڎ˜ΑĈÚÃÞΡΑ Ëטځæ«Ä£Α «ÄΑ +Ĕ«ΡΑö¨«Ž¨Α Ë¢Α樘Α¢Ë½½Ëö«Ä£Α


Þææ˜Ã˜ÄæÞΑ«ÞέÚ˜ΑŽËÚژŽæΧ
1. They can sell their own goods in addition to offering their platforms as market-places.
2. The degree to which they can own big sellers on their platforms is limited.
Select the correct answer using the code given below:
(a) 1 only
(b) 2 only
(c) Both 1 and 2
(d) Neither 1 nor 2

ANSWERS D

ËÄÞêØÚΑ TÚË明æ«ËÄΑ λαËÃØڎ˜μΑ W꽘ÞΡΑ ͉͉͋͋Α ”˜ĈĘΑ ˜αŽËÃØڎ˜Α ˜Äæ«æüΑ 끽ÞËΑ¢Ëژ«£ÄΑ ËöʔμΑ ÞΑ ÄüΑ
טÚÞËÄΑö¨ËΑËöÄÞΡΑËטځæ˜ÞΑËÚΑÁā£˜ÞΑ”«£«æ½ΑËÚΑ˜½˜ŽæÚËÄ«ŽΑ¢Ž«½«æüΑËÚΑ×½æ¢ËÚÃΑ¢ËÚΑ˜½˜ŽæÚËÄ«ŽΑŽËÃØڎ˜ΡΑ
êæΑ”˘ÞΑÄËæΑ«ÄŽ½ê”˜ΑΑޘ½½˜ÚΑËą˜Ú«Ä£Α¨«ÞΑ£Ë˔ÞΑËÚΑޘÚõ«Ž˜ÞΑ¢ËÚΑށ½˜ΑËÄΑΑÁں˜æ×½Ž˜Α˜αŽËÃØڎ˜Α˜Äæ«æüΠΑ
Hence statement 1 is not correct.
Marketplace based model of e-commerce ؁ÄÞΑ×ÚËõ«”«Ä£ΑÄΑ+bΑ×½æ¢ËÚÃΑüΑÄΑ˜αŽËÃØڎ˜Α˜Äæ«æüΑËÄΑΑ
”«£«æ½ΑϞΑ˜½˜ŽæÚËÄ«ŽΑĘæöËÚºΑæËΑŽæΑÞΑΑ¢Ž«½«ææËÚΑ˜æö˜˜ÄΑêü˜ÚΑÄ”Αޘ½½˜ÚΠ
HÄΑË樘ÚΑ¨Ä”ΡΑInventory based model of e-commerce ؁ÄÞΑÄΑ˜αŽËÃØڎ˜ΑŽæ«õ«æüΑö¨˜Ú˜Α«Äõ˜ÄæËÚüΑË¢Α
£Ë˔ÞΑÄ”ΑޘÚõ«Ž˜ÞΑ«ÞΑËöʔΑüΑ˜αŽËÃØڎ˜Α˜Äæ«æüΑÄ”Α«ÞΑÞ˽”ΑæËΑ樘ΑŽËÄÞêØÚÞΑ”«Ú˜Žæ½üΠΑ(Ëö˜õ˜ÚΡΑ"+Α«ÞΑ
ÄËæΑטÚëæ昔Α«ÄΑ«Äõ˜ÄæËÚüΑÞ˜”ΑÃ˔˜½ΑË¢Α˜αŽËÃØڎ˜Π
ŽŽËڔ«Ä£ΑæËΑ£ê«”˜½«Ä˜ÞΑ¢ËÚΑ"+Αλ"Ëژ«£ÄΑ«Ú˜ŽæΑ+Äõ˜ÞæØÄæμΑËÄΑαËÃØڎ˜ΡΑαŽËÃØڎ˜Α˜Äæ«æüΑ×ÚËõ«”«Ä£Α
ΑÁں˜æ×½Ž˜Αö«½½ΑÄËæΑ˜û˜ÚŽ«Þ˜ΑËöĘÚÞ¨«×ΑËÚΑŽËÄæÚ˽ΑËõ˜ÚΑ樘Α«Äõ˜ÄæËÚüΑ«Π˜ΠΑ£Ë˔ÞΑ×êÚ×ËÚ昔ΑæËΑ˜ΑÞ˽”Π Such
an ownership or control over the inventory will render the business into inventory based model.
+Äõ˜ÄæËÚüΑË¢ΑΑõ˜Ä”ËÚΑö«½½Α˜Α”˜˜Ã˜”ΑæËΑ˜ΑŽËÄæÚ˽½˜”ΑüΑ˜αŽËÃØڎ˜ΑÁں˜æ×½Ž˜Α˜Äæ«æüΑ«¢ΑÃËژΑ樁ÄΑ͎͋АΑ
Ë¢Α ×êڎ¨Þ˜ÞΑ Ë¢Α ÞꎨΑ õ˜Ä”ËÚΑ Ú˜Α ¢ÚËÃΑ 樘Α Áں˜æ×½Ž˜Α ˜Äæ«æüΑ ËÚΑ «æÞΑ £ÚËê×Α ŽËÃׁī˜ÞΠ There are no
guidelines on the limit of the big sellers that such an e-commerce platform have. Hence statement 2 is
not correct.

ANALYSIS:

ځ¢æΑË¢Α×ÚË×Ëޘ”ΑÃ˜Ä”ØÄæÞΑæËΑ樘ΑËÄÞêØÚΑTÚË明æ«ËÄΑλαŽËÃØڎ˜μΑW꽘ÞΡΑ͉͉͋͋Αö˜Ú˜Α«ÄΑ樘ΑĘöÞΠ
141

[ŽËטΑ¢ËÚΑ˜½«Ã«Äæ«ËÄ

Scope for elimination: BËΑގËטΑÞΑ樘ΑÞææ˜Ã˜ÄæÞΑÚ˜Α¢ŽæꁽΑ«ÄΑāæêژΠ

Market-place model:Αb¨˜ΑÁں˜æ×½Ž˜ΑÃ˔˜½Αژ¢˜ÚÞΑæËΑ樘ΑêޫĘÞÞΑÞæځ昣üΑ«ÄΑö¨«Ž¨Α˜αŽËÃØڎ˜Α
Áں˜æ×½Ž˜ÞΑ×ÚËõ«”˜ΑÄêØÚËêÞΑõ˜Ä”ËÚÞΑö«æ¨ΑΑŽ˜Äæځ½«ā˜”Α×½æ¢ËÚÃΑö¨˜Ú˜Α樘üΑÁüΑޘ½½Α樘«ÚΑ«æ˜ÃÞΑ
ö¨«½˜Α˜Ä££«Ä£Αö«æ¨Α×Ëæ˜Ä櫁½Αêü˜ÚÞΠ
Inventory model: b¨˜Α+Äõ˜ÄæËÚüαÞ˜”ΑÃ˔˜½Α«ÞΑËĘΑ«ÄΑö¨«Ž¨Α˜αŽËÃØڎ˜Α×½æ¢ËÚÃÞΑ¨Ë½”Α«Äõ˜ÄæËÚüΑ
¢ÚËÃΑÚÄ”ÞΡΑØڎ¨ÄæÞΡΑÄ”Αޘ½½˜ÚÞΑÄ”Αޘ½½Α«æΑ”«Ú˜Žæ½üΑæËΑŽêÞæËØÚÞΠΑ+Äõ˜ÄæËÚüαÞ˜”ΑÁں˜æ×½Ž˜ÞΑ
Áā£˜Α樘ΑÞæˎº×«½˜ÞΑË¢Α樘«ÚΑõ˜Ä”ËÚÞΡΑÞÞ«Þæ«Ä£Α樘ÃΑ«ÄΑËڔ˜ÚΑ¢ê½Ĉ½Ã˜ÄæΠ

[HfWΑ΢ ¨ææ×Þ΢έݎËÄÞêØځą«ÚÞΠÄ«ŽΠ«ÄέÞ«æ˜Þέ”˜¢ê½æέĈ½˜ÞέА͉͋ŽËÃØڎ˜А͉͋Ú꽘ÞΠה¢
¨ææ×Þ΢έέ”×««æΠ£ËõΠ«ÄέÞ«æ˜Þέ”˜¢ê½æέĈ½˜Þέ×Ä͌ε͉͋͊͏ε͉Πה¢
¨ææ×Þ΢έέ׫Π£ËõΠ«ÄέTژÞÞژ½˜Þ˜Þ¨Ú˜ΠÞ×ûΧTW+Ͻ͎͎͎͉͊͒͑

Year 2023 MSME

V͋ΠΑËÄÞ«”˜ÚΑ樘Α¢Ë½½Ëö«Ä£ΑÞææ˜Ã˜ÄæÞΑö«æ¨Αژ¢˜Ú˜ÄŽ˜ΑæËΑ+Ĕ«΢Α
1. According to the 'Micro Small and Medium enterprises Development (MSMED) Act, 2006, the 'medium enterprises'
are those with investments in plant and machinery between Rs. 15 crore and Rs. 25 crore.
2. All bank loans to the Micro, Small and Medium Enterprises qualify under the Priority sector.
Which of the statements given above is/are correct?
(a) 1 Only
(b) 2 Only
(c) Both 1 and 2
(d) Neither 1 nor 2

ANSWERS D

EXPLANATION
fÄ«ËÄΑA«Ä«ÞæÚüΑË¢ΑA«ŽÚËΡΑ[Á½½ΑÄ”ΑA˜”«êÃΑÄæ˜Ú×ګޘÞΑλAέËΑA[AÞμΑ¨ÞΑ«ÞÞꘔΑ#ā˜ææ˜ΑÄËæ«ĈŽæ«ËÄΑæËΑ
ׁõ˜Α öüΑ ¢ËÚΑ 樘Α «Ã×½˜Ã˜Äææ«ËÄΑ Ë¢Α 樘Α ê×öÚ”Α ژõ«Þ«ËÄΑ «ÄΑ 樘Α ”˜ĈÄ«æ«ËÄΑ Ä”Α ŽÚ«æ˜Ú«Α Ë¢Α A[AÞΑ «ÄΑ 樘Α
ŽËêÄæÚüΠΑb¨˜ΑĘöΑ”˜ĈÄ«æ«ËÄΑÄ”ΑŽÚ«æ˜Ú«ËÄΑö«½½ΑŽËØΑ«ÄæËΑ˜ą˜ŽæΑ¢ÚËÃΑ͊ÞæΑ6ê½üΡΑ͉͉͋͋Π
As per the new criteria, a medium enterprise is where the investment in Plant and Machinery or
Ùê«×ØÄæΑ”˘ÞΑÄËæΑ˜ûŽ˜˜”ΑĈ¢æüΑŽÚËژΑÚêט˜ÞΑÄ”ΑæêÚÄËõ˜ÚΑ”˘ÞΑÄËæΑ˜ûŽ˜˜”ΑæöËΑ¨êĔژ”ΑÄ”ΑĈ¢æüΑ
crore rupees. In terms of Master Direction on ‘Priority Sector Lending (PSL) – Targets and
½ÞÞ«ĈŽæ«ËÄϐΑ ”æ˜”Α [˜×æ˜Ã˜ÚΑ ͍ΡΑ ͉͉͋͋ΡΑ ½½Α ÄºΑ ½ËÄÞΑ æËΑ A[AÞΑ ŽËÄ¢ËÚëģΑ æËΑ 樘Α ŽËĔ«æ«ËÄÞΑ
×ژގګ˜”Α樘ژ«ÄΑÙꁽ«¢üΑ¢ËÚΑŽ½ÞÞ«ĈŽæ«ËÄΑêĔ˜ÚΑ×Ú«ËÚ«æüΑޘŽæËÚΑ½˜Ä”«Ä£Π
ŽŽËڔ«Ä£ΑæËΑ樘ޘΑ”«Ú˜Žæ«ËÄÞΡΑA[AÞΑÞ¨Ë꽔Α˜Α˜Ä££˜”Α«ÄΑ樘ΑÁÄꢁŽæêژΑËÚΑ×Ú˔êŽæ«ËÄΑË¢Α£Ë˔ÞΡΑ«ÄΑ
ÄüΑÁÄĘÚΡΑטÚ恫īģΑæËΑÄüΑ«Ä”êÞæÚüΑÞטŽ«Ĉ˜”Α«ÄΑ樘Α"«ÚÞæΑ[Ž¨˜”꽘ΑæËΑ樘Α+ĔêÞæÚ«˜ÞΑλ˜õ˜½Ë×ØÄæΑÄ”Α
W˜£ê½æ«ËÄμΑŽæΡΑ ͎͊͒͊Α ËÚΑ ˜Ä££˜”Α «ÄΑ ×ÚËõ«”«Ä£Α ËÚΑ ژĔ˜Ú«Ä£Α Ë¢Α ÄüΑ ޘÚõ«Ž˜Α ËÚΑ ޘÚõ«Ž˜ÞΠΑ½½Α ÄºΑ ½ËÄÞΑæËΑ
A[AÞΑŽËÄ¢ËÚëģΑæËΑ樘ΑËõ˜Α£ê«”˜½«Ä˜ÞΑÙꁽ«¢üΑ¢ËÚΑŽ½ÞÞ«ĈŽæ«ËÄΑêĔ˜ÚΑ×Ú«ËÚ«æüΑޘŽæËÚΑ½˜Ä”«Ä£ΠΑHence
both statements are not correct.
142

ANALYSIS:
ANALYSIS:
A[AÞΑö˜Ú˜ΑژŽ½ÞÞ«Ĉ˜”ΑÄ”ΑA[AÞΑÄ”ΑT[;ΑÚ˜ΑË¢æ˜ÄΑޘ˜ÄΑ«ÄΑ樘ΑĘöÞΠ
(ËöΑæËΑŽËõ˜ÚΧΑ[æÄ”Ú”ΑĘöÞׁטÚ
¨ææ×Þ΢έέöööΠ樘¨«Ä”êЎËÃݍêޫĘÞÞέÃÞØÞαÄËæαÞææ«Žαêæα£ÚËö«Ä£α«ÄαÞ«ā˜αÞޘÚæÞαΑĈÄëÄ݁Úæ«α
Ž½˜͏͏͏͏͉͉͒͑ИŽ˜
Αގژ˜ÄÞ¨ËæΑË¢Α樘ΑËõ˜ΑÚ櫎½˜Α«ÄΑö¨«Ž¨ΑA[AÞΑ¨õ˜Α˜˜ÄΑŽæ˜£Ëګޘ”Α«ÞΑÞ¨Ú˜”Α˜½Ëö

½æ¨Ë꣨Α樘ΑޘŽËĔΑÞææ˜Ã˜ÄæΑ½ÞËΑ«ÄŽ½ê”˜ÞΑT[;ΡΑêæΑËõ˜Ú½½Α樘ΑÙê˜Þæ«ËÄΑ«ÞΑژ½æ˜”ΑæËΑA[AÞΑÄ”ΑÄËæΑ
Äº«Ä£Α«ÄΑ£˜Ä˜Ú½Π

[ŽËטΑ¢ËÚΑ˜½«Ã«Äæ«ËÄ
BËΑގËטΑÞΑ樘ΑÙê˜Þæ«ËÄΑ«ÞΑ¢ŽæꁽΠ
b¨˜ΑޘŽËĔΑÞææ˜Ã˜ÄæΑ«ÞΑΑ½Äº˜æΑÞææ˜Ã˜ÄæΑÄ”Α樘ژ¢ËژΑޘ˜Ã«Ä£½üΑöÚËÄ£ΡΑêæΑ£«ÄΑ樘ΑöËڔΑϏ½½ϐΑÁüΑ
˜ΑΑæځ×ΠΑb¨˜ΑW˜Þ˜ÚõΑ ÄºΑË¢Α+Ĕ«ϐÞΑö˜Þ«æ˜ÞΑÄÞö˜ÚÞΑ樘Α”ËêæΑژ½æ˜”ΑæËΑ˜½«£««½«æüΑË¢Α樘ΑA[AÞΑ¢ËÚΑ
TÚ«ËÚ«æüΑ[˜ŽæËÚΑ;˜Ä”«Ä£ΑλT[;μΠΑ

[HfWΑ΢ ¨ææ×Þ΢έέÃÞÃ˜Π£ËõΠ«ÄέºÄËö၍ËêæαÃÞØ
143

½ÞÞ«ĈŽæ«ËÄ Micro Small Medium

Manufacturing +Äõ˜ÞæØÄæΑ«ÄΑT½ÄæΑ +Äõ˜ÞæØÄæΑ«ÄΑT½ÄæΑ +Äõ˜ÞæØÄæΑ«ÄΑT½ÄæΑ


Enterprises and Ä”ΑAŽ¨«Ä˜ÚüΑËÚΑ Ä”ΑAŽ¨«Ä˜ÚüΑËÚΑ Ä”ΑAŽ¨«Ä˜ÚüΑËÚΑ
Enterprises Ùê«×ØÄæ΢ Ùê«×ØÄæ΢ Ùê«×ØÄæ΢

rendering Services BËæΑÃËژΑ樁ÄΑWÞΠ͊Α BËæΑÃËژΑ樁ÄΑWÞΠ͉͊Α BËæΑÃËژΑ樁ÄΑWÞΠ͎͉Α


ŽÚËژΑÄ”ΑÄÄꁽΑ ŽÚËژΑÄ”ΑÄÄꁽΑ ŽÚËژΑÄ”ΑÄÄꁽΑ
bêÚÄËõ˜ÚΑΣΑÄËæΑÃËژΑ bêÚÄËõ˜ÚΑΣΑÄËæΑÃËژΑ bêÚÄËõ˜ÚΑΣΑÄËæΑÃËژΑ
樁ÄΑWÞΠΑ͎ΑŽÚËژ 樁ÄΑWÞΠΑ͎͉ΑŽÚËژ 樁ÄΑWÞΠΑ͎͉͋ΑŽÚËژ

INFRASTRUCTURE
λ͋ΑVê˜Þæ«ËÄÞμ

Year 2017 NIIF

V͊ΠΑ r«æ¨Α ژ¢˜Ú˜ÄŽ˜Α æËΑ ϏBæ«Ëā½Α +Äõ˜ÞæØÄæΑ Ä”Α +ĢځÞæÚêŽæêژΑ "êĔϐΡΑ ö¨«Ž¨Α Ë¢Α 樘Α ¢Ë½½Ëö«Ä£Α
Þææ˜Ã˜ÄæÞΑ«ÞέÚ˜ΑŽËÚژŽæΧΑ
1. It is an organ of NITI Aayog.
2. It has a corpus of Rs. 4,00,000 crore at present.
Select the correct answer using the code given below.
(a) 1 only
(b) 2 only
(c) Both 1 and 2
(d) Neither 1 nor 2

ANSWERS D

EXPLANATION

Bæ«Ëā½Α +Äõ˜ÞæØÄæΑ Ä”Α +ĢځÞæÚêŽæêژΑ "êĔΑ λB++"μΑ «ÞΑ Α ¢êĔΑ ŽÚ˜æ˜”Α üΑ 樘Α #Ëõ˜ÚÄØÄæΑ Ë¢Α +Ĕ«Α ¢ËÚΑ
˜Ä¨ÄŽ«Ä£Α «Ä¢ÚÞæÚêŽæêژΑ ĈāĎ«Ä£Α «ÄΑ 樘Α ŽËêÄæÚüΠΑ +æÞΑ ŽÚ˜æ«ËÄΑ öÞΑ ÄÄËêϘ”Α «ÄΑ 樘Α fÄ«ËÄΑ ꔣ˜æΑ
͉͎͋͊α͊͏ΠΑ#Ëõ˜ÚÄ«Ä£ΑËêĎ«½ΑË¢ΑB++"Α«ÞΑÄΑ”õ«ÞËÚüΑË”üΑαΑŽ¨«Ú˜”ΑüΑ樘Α"«ÄÄŽ˜ΑA«Ä«Þæ˜ÚΠΑ+æΑ«ÞΑÄËæΑÄΑËÚ£ÄΑ
Ë¢ΑB+b+ΑüË£ΠΑHence statement 1 is not correct.
b¨˜Α×ÚË×Ëޘ”ΑŽËÚ×êÞΑË¢ΑB++"Α«ÞΑWÞΠΑ͍͉Ρ͉͉͉ΑÚËژÞΑλÚËêĔΑf[Α͏Α «½½«ËÄμΠΑHence statement 2 is also not
correct.
144

ANALYSIS:

+ĢځÞæÚêŽæêژΑ¨ÞΑ˜˜ÄΑΑõ˜ÚüΑ«Ã×ËÚæÄæΑ¢ËŽêÞΑÚ˜ΑË¢Α樘Α£Ëõ˜ÚÄØÄæΑÄ”Α樘ژ¢ËژΑB++"Α¨”ΑژÁ«Ä˜”Α
ŽËÄæ«ÄêËêÞ½üΑ«ÄΑĘöÞΑ«ÄΑ樘Α͉͋͊͐ΠΑb¨˜ΑæË׫ŽΑ«ÞΑ½ÞËΑ«Ã×ËÚæÄæΑ¢ÚËÃΑ樘ΑטÚÞטŽæ«õ˜ΑË¢ΑŽËõ˜Ú«Ä£Αº˜üΑĈāĎ«½Α
«ÄÞæ«æêæ«ËÄÞΑË¢Α樘ΑŽËêÄæÚüΠΑ

[ŽËטΑ¢ËÚΑ˜½«Ã«Äæ«ËÄ
BËæΑÃꎨΑގËטΑ«ÞΑ樘ژΑÞΑ«æΑ«ÞΑΑ×êژ½üΑ¢ŽæꁽΑÙê˜Þæ«ËÄΡΑêæΑËĘΑŽË꽔Α¨õ˜Αæ¨Ë꣨æΑ¨ËöΑB+b+ΑüË£ΡΑΑ
”˜½«˜Úæ«õ˜ΑÄ”Α×½ÄÄ«Ä£ΑË”üΡΑŽË꽔ΑŽÚ˜æ˜ΑÞꎨΑΑ¢êĔΑæËΑËËÞæΑ«Ä¢ÚÞæÚêŽæêژΠΑb¨êÞΑüΑÞÞêëģΑ樁æΑ
Þææ˜Ã˜ÄæΑ͊Α«ÞΑÄËæΑŽËÚژŽæΡΑËĘΑŽË꽔Α¨õ˜Α£ËæΑÚ«”ΑË¢ΑË×æ«ËÄÞΑλμΑÄ”ΑλŽμΠΑ

[HfWΑ΢ ¨ææ×΢έέöööΠ樘¨«Ä”êЎËÃݍêޫĘÞÞέ+ĔêÞæÚüέ#ËõæΠαÃê½½ÞαА͋А͉͑АΑ͒͑Ëêæα
ŽËÃ˜α½«Äº˜”А͋А͉͑А͒͒α«ÄŽ˜Äæ«õ˜α¢ËÚαB++"Ꭸ«˜¢έÚ櫎½˜͍͊͌͐͑͌͒͒ИŽ˜
¨ææ×΢έ݁Ú樁ט”«Π«Äέ«Ä”˜ûΠר×А͌"æ«æ½˜А͌Bæ«Ëā½ε+Äõ˜Þæα
ØÄæεÄ”ε+ĢځÞæÚêŽæêژε"êĔελB++"μ
¨ææ×΢έέÄ««ĈĔ«ΠŽËÃέ«Ä”˜ûΠ¨æý

””«æ«Ëā½Α+Ä¢ËÚÁæ«ËÄ

+""WBΑ brBΑB+"ΑBΑB++"
NIF: National Investment Fund (NIF)Α öÞΑ ˜Þ恍½«Þ¨˜”Α æËΑ ژŽ˜«õ˜Α ”«Þ«Äõ˜ÞæØÄæΑ
×Úˎ˜˜”ÞΑ Ë¢Α Ž˜Äæځ½Α ×ꍽ«ŽΑ ޘŽæËÚΑ ˜Äæ˜Ú×ګޘÞΑ Ä”ΑæËΑ «Äõ˜ÞæΑ樘Α ށØΑæËΑ £˜Ä˜Úæ˜Α
˜ÚÄ«Ä£ÞΑö«æ¨ËêæΑ”˜×½˜æ«Ä£Α樘ΑŽËÚ×êÞΠΑb¨˜Α˜ÚÄ«Ä£ÞΑË¢Α樘Α"êĔΑö˜Ú˜ΑæËΑ˜Αêޘ”Α¢ËÚΑ
ޘ½˜Žæ˜”Α ˜Äæځ½Α Þˎ«½Α ö˜½¢Ú˜Α [Ž¨˜Ã˜ÞΑ ϞΑ Ž×«æ½Α «Äõ˜ÞæØÄæΑ ژÙê«Ú˜Ã˜ÄæÞΑ Ë¢Α
×ÚËĈ恍½˜ΑÄ”Αژõ«õ½˜ΑT[fÞΠΑb¨«ÞΑ¢êĔΑöÞΑº˜×æΑËêæÞ«”˜Α樘ΑŽËÄÞ˽«”æ˜”Α¢êĔΑË¢Α
+Ĕ«ΠΑb¨˜Α¢êĔΑ˜ŽÃ˜ΑËטځæ«Ëā½Α«ÄΑ͉͉͎͋ΠΑÄæ«Ú˜Α”«Þ«Äõ˜ÞæØÄæΑ×Úˎ˜˜”ÞΑÚ˜ΑæËΑ˜Α
ŽÚ˜”«æ˜”ΑæËΑ樘Α˜û«Þæ«Ä£ΑϏTꍽ«ŽΑŽŽËêÄæϐΑêĔ˜ÚΑ樘Α¨˜”ΑB+"ΑÄ”Α樘üΑöË꽔ΑژÁ«ÄΑ
樘ژΑêÄ櫽Αö«æ¨”ځöÄέ«Äõ˜Þ昔Α¢ËÚΑ樘Α××ÚËõ˜”Α×êÚ×ËޘΠΑb¨˜Α½½ËŽæ«ËÄÞΑËêæΑË¢Α樘Α
B+"Αö«½½Α˜Α”˜Ž«”˜”Α«ÄΑ樘ΑÄÄꁽΑ#Ëõ˜ÚÄØÄæΑ ꔣ˜æΠΑ

NIIF: National Investment and Infrastructure FundΑ«ÞΑ+Ĕ«ϐÞΑĈÚÞæΑ«Ä¢ÚÞæÚêŽæêژΑÞטŽ«ĈŽΑ«Äõ˜ÞæØÄæΑ¢êĔΠΑ


b¨«ÞΑ«ÞΑ”«ą˜Ú˜ÄæΑ¢ÚËÃΑ樘ΑBæ«Ëā½Α+Äõ˜ÞæØÄæΑ"êĔΠΑb¨˜Αˍ¶˜Žæ«õ˜ΑË¢ΑB++"ΑöË꽔Α˜ΑæËΑÁû«Ã«ā˜Α˜ŽËÄËëŽΑ
«ÃׁŽæΑ Á«Ä½üΑæ¨ÚË꣨Α «Ä¢ÚÞæÚêŽæêژΑ ”˜õ˜½Ë×ØÄæΑ «ÄΑ ŽËÃØڎ«½½üΑõ«½˜Α ×Ú˶˜ŽæÞΡΑ Ëæ¨Α £Ú˜˜ÄĈ˜½”Α Ä”Α
ÚËöÄĈ˜½”ΡΑ«ÄŽ½ê”«Ä£ΑÞ恽½˜”Α×Ú˶˜ŽæÞΠΑb¨˜ΑŽËÚ×êÞΑË¢ΑB++"Α«ÞΑWÞΠΑ͍͉Ρ͉͉͉ΑÚËژÞΠΑb¨˜Α£Ëõ˜ÚÄØÄæΑŽÄΑ×ÚËõ«”˜Α
ê×ΑæËΑ͉͉͉͉͋ΑŽÚËژÞΑטÚΑÄÄêÃΑ«ÄæËΑ樘ޘΑ¢êĔÞΠΑ#Ëõ˜ÚÄØÄæϖÞΑŽËÄæÚ«êæ«ËÄέÞ¨Ú˜Α«ÄΑ樘ΑŽËÚ×êÞΑö«½½Α˜Α͍͒АΑ
«ÄΑ ˜Ž¨Α ˜Äæ«æüΑ ޘæΑ ê×Α ÞΑ ÄΑ ½æ˜Úāæ˜Α +Äõ˜ÞæØÄæΑ "êĔΑ λ+"μΑ Ä”Α ö«½½Α Ę«æ¨˜ÚΑ ˜Α «ÄŽÚ˜Þ˜”Α ˜üËĔΑ ÄËÚΑ
½½Ëö˜”ΑæËΑ¢½½Α˜½ËöΡΑ͍͒АΠΑb¨˜Αö¨Ë½˜ΑË¢Α͍͒АΑöË꽔Α˜ΑŽËÄæÚ«ê昔ΑüΑ樘Α#Ëõ˜ÚÄØÄæΑ”«Ú˜Žæ½üΠΑW˜ÞæΑ«ÞΑËטÄΑ
¢ËÚΑŽËÄæÚ«êæ«ËÄΑ¢ÚËÃΑË樘ÚÞΠ
145

Year 2023 PORTS

V͋ΠËÄÞ«”˜ÚΑ樘Α¢Ë½½Ëö«Ä£Αׁ«ÚÞ΢Α
TËÚæΑΑ r˜½½ΑºÄËöÄΑÞΑ
1. Kamarajar Port : First major port in India registered as a company
2. Mundra Port : Largest privately owned port in India
3. Visakhapatnam: Largest container port in India
How many of the above pairs are correctly matched?
(a) Only one pair
(b) Only two pairs
(c) All three pairs
(d) None of the pairs

ANSWERS B

EXPLANATION
Pair 1 is correctly matched: 9ÃÚ¶ÚΑ TËÚæΡΑ ½ËŽæ˜”Α ËÄΑ 樘Α ËÚËÁĔ˜½Α ˁÞæΑ ËêæΑ ͍͋Α ºÃΑ ÄËÚæ¨Α Ë¢Α
¨˜Äā«Α TËÚæΡΑ ¨˜Äā«ΡΑ «æΑ «ÞΑ 樘Α ͊͋æ¨Α Á¶ËÚΑ ×ËÚæΑ Ë¢Α +Ĕ«ΡΑ Ä”Α 樘Α ĈÚÞæΑ ×ËÚæΑ «ÄΑ +Ĕ«Α ö¨«Ž¨Α «ÞΑ Α ×ꍽ«ŽΑ
ŽËÃׁÄüΠΑ b¨˜Α ×ËÚæΑ öÞΑ ”˜Ž½Ú˜”Α ÞΑ Α Á¶ËÚΑ ×ËÚæΑ êĔ˜ÚΑ 樘Α +Ĕ«ÄΑ TËÚæÞΑ ŽæΡΑ ͉͊͒͑Α «ÄΑ AÚŽ¨Α ͊͒͒͒Α Ä”Α
«ÄŽËÚ×Ëځ昔ΑÞΑÄÄËژΑTËÚæΑ;«Ã«æ˜”ΑêĔ˜ÚΑ樘ΑËÃׁī˜ÞΑŽæΡΑ͎͊͒͏Α«ÄΑHŽæˍ˜ÚΑ͊͒͒͒ΠΑb¨˜Α9ÃÚ¶ÚΑTËÚæΑ
«ÞΑ樘ΑËĽüΑŽËÚ×Ëځæ«ā˜”ΑÁ¶ËÚΑ×ËÚæΑÄ”Α«ÞΑژ£«Þæ˜Ú˜”ΑÞΑΑŽËÃׁÄüΠΑ
Pair 2 is correctly matched:ΑTËÚæΑË¢ΑAêĔځΑ«ÞΑ+Ĕ«ϐÞΑ«££˜ÞæΑ×Ú«õæ˜Α×ËÚæΠΑ;ˎæ˜”Α«ÄΑ樘Α9ê推Α”«ÞæÚ«ŽæΑË¢Α
樘ΑÞææ˜ΑË¢Α#궁ځæΡΑAêĔځΑ½«˜ÞΑËÄΑ樘ΑÄËÚæ¨ΑÞ¨ËژÞΑË¢Α樘Α#ê½¢ΑË¢Α9ê推Π
T«ÚΑ ͌Α «ÞΑ ÄËæΑ ŽËÚژŽæ½üΑ Á推˜”΢ΑÞΑ Ë¢Α AüΑ ͉͋͋͌ΡΑ樘Α 6ö¨Ú½½Α B˜¨ÚêΑ TËÚæΑêæ¨ËÚ«æüΑ λ6BTμΑ æΑ Bõ«Α
Aế«Α öÞΑ 樘Α ×ژë˜ÚΑ ŽËÄ恫ĘÚΑ ¨Ä”½«Ä£Α ×ËÚæΑ «ÄΑ +Ĕ«ΡΑ ŽŽËêÄæ«Ä£Α ¢ËÚΑ ÚËêĔΑ ͎͉АΑ Ë¢Α 樘Α æË恽Α
ŽËÄ恫Ęګā˜”ΑŽÚ£ËΑõ˽êØΡΑŽÚËÞÞΑ樘ΑÁ¶ËÚΑ×ËÚæÞΑË¢Α+Ĕ«ΠΑËÃëÞÞ«ËʔΑËÄΑ͋͏Αæ¨ΑAüΑ͊͒͑͒ΡΑ«ÄΑæ¨Ú˜˜Α
”˜Ž”˜ÞΑ Ë¢Α «æÞΑ Ëטځæ«ËÄÞΡΑ 6BTΑ ¨ÞΑ æځÄÞ¢ËÚؔΑ ¢ÚËÃΑ Α ê½ºαΑ ŽÚ£ËΑ æ˜Úëā½Α æËΑ ˜ŽËØΑ 樘Α ×ژë˜ÚΑ
ŽËÄ恫ĘÚΑ×ËÚæΑ«ÄΑ樘ΑŽËêÄæÚüΠΑ6BTΑ«ÞΑŽËÄĘŽæ˜”ΑæËΑËõ˜ÚΑ͉͉͋Α×ËÚæÞΑ«ÄΑ樘ΑöËÚ½”ΑÄ”Α«ÞΑځ嘔Α͋͏æ¨Α«ÄΑ樘Α
½«ÞæΑË¢ΑæË×Α͉͉͊ΑËÄ恫ĘÚΑTËÚæÞΑ£½Ë½½üΠΑÞΑטÚΑ樘ΑژŽ˜ÄæΑĈĔ«Ä£ÞΑ«æΑ«ÞΑ˜½«˜õ˜”Α樁æΑ樁æΑAêĔځΑ×ËÚæΑ«ÞΑ
the biggest container port.

ANALYSIS:
ANALYSIS:
b¨˜Α ×ËÚæÞΑ Ë¢æ˜ÄΑ ×טÚΑ «ÄΑ
樘Α ĘöÞΡΑ ˜ÞטŽ«½½üΑ
AêĔځΑ ×ËÚæΑ ö¨«Ž¨Α «ÞΑ Α
×Ú«õæ˜½üΑËöʔΑ×ËÚæΑË¢Α樘Α
”Ä«Α£ÚËê×Π

[ŽËטΑ¢ËÚΑ˜½«Ã«Äæ«ËÄ
There is no scope for elimination ÞΑÄüΑÄê͘ÚΑË¢Αׁ«ÚÞΑÁüΑ˜ΑŽËÚژŽæ½üΑÁ推˜”Π
Source: ÄΑ Ú櫎½˜Α «ÄΑ b¨˜Α («Ä”êΑ λ×Ú«½ΡΑ ͉͋͋͌μΑ ”«ÞŽêÞޘÞΑ 樘Α ×ÚËĈæΑ Ë¢Α 樘Α 9ÃÚ¶ÚΑ TËÚæΑ ;æ”ΠΑb¨«ÞΑ Ϗ;æ”ϐΑ
means that the port is registered as a company.Α+¢ΑÄΑÞ׫ځÄæΑׁüÞΑ£Ú˜æ˜ÚΑææ˜Äæ«ËÄΑæËΑ¨ËöΑæ˜ÚÃÞΑÚ˜Α
ØÄæ«ËʔΑ«ÄΑ樘ΑÚ櫎½˜ÞΡΑ樘ÄΑ”˜Ĉī昽üΑ樘üΑŽË꽔ΑÄÞö˜ÚΑλ+æΑ«ÞΑÄËæΑ˜ÞüΑæ¨Ë꣨μΠΑT½˜Þ˜ΑĈĔΑ˜½ËöΑ樘Α
ގژ˜ÄÞ¨ËæΑË¢Α樁æΑÚ櫎½˜
146

ENERGY SECTOR
λ͌ΑVê˜Þæ«ËÄÞμ

Year 2019 REGULATORY BOARD

ΑV͊ΠΑËÄÞ«”˜ÚΑ樘Α¢Ë½½Ëö«Ä£ΑÞææ˜Ã˜ÄæÞ΢
1. Petroleum and Natural Gas Regulatory Board (PNGRB) is the first regulatory body set up by the Government of India.
2. One of the tasks of PNGRB is to ensure competitive markets for gas.
3. Appeals against the decisions of PNGRB go before the Appellate Tribunals for Electricity.
Which of the statements given above are correct?
(a) 1 and 2 only
(b) 2 and 3 only
(c) 1 and 3 only
(d) 1, 2 and 3

ANSWERS B

EXPLANATION

Statement 1 is not correct: b¨˜ΑT˜æÚ˽˜êÃΑÄ”ΑBæêځ½Α#ÞΑW˜£ê½æËÚüΑ ˁڔΑλTB#W μΑöÞΑŽËÄÞæ«æê昔Α


êĔ˜ÚΑb¨˜ΑT˜æÚ˽˜êÃΑÄ”ΑBæêځ½Α#ÞΑW˜£ê½æËÚüΑ ˁڔΑŽæΡΑ͉͉͋͏ΠΑ
Statement 2 is correct:Αb¨˜ΑŽæΑ×ÚËõ«”˜Α¢ËÚΑ樘Α˜Þ恍½«Þ¨Ã˜ÄæΑË¢ΑT˜æÚ˽˜êÃΑÄ”ΑBæêځ½Α#ÞΑW˜£ê½æËÚüΑ
ˁڔΑ æËΑ ×ÚË明æΑ 樘Α «Äæ˜Ú˜ÞæÞΑ Ë¢Α ŽËÄÞêØÚÞΑ Ä”Α ˜Äæ«æ«˜ÞΑ ˜Ä££˜”Α «ÄΑ ÞטŽ«Ĉ˜”Α Žæ«õ«æ«˜ÞΑ ژ½æ«Ä£Α æËΑ
טæÚ˽˜êÃΡΑ טæÚ˽˜êÃΑ ×Ú˔êŽæÞΑ Ä”Α āæêځ½Α £ÞΑ Ä”Α æËΑ ×ÚËÃËæ˜Α ŽËÃטæ«æ«õ˜Α Áں˜æÞΑ Ä”Α ¢ËÚΑ Áææ˜ÚÞΑ
ŽËÄĘŽæ˜”Α樘ژö«æ¨ΑËÚΑ«ÄŽ«”˜Ä恽Α樘ژæËΠ
Statement 3 is correct: b¨˜Α×ט½½æ˜ΑbÚ«êā½Α˜Þ恍½«Þ¨˜”ΑêĔ˜ÚΑޘŽæ«ËÄΑ͉͊͊ΑË¢Α樘Α½˜ŽæÚ«Ž«æüΑŽæΡΑ͉͉͋͌Α
λ͌͏ΑË¢Α͉͉͋͌μΑ«ÞΑ樘Α×ט½½æ˜ΑbÚ«êā½Α¢ËÚΑ樘Α×êÚ×ËޘÞΑË¢ΑT˜æÚ˽˜êÃΑÄ”ΑBæêځ½Α#ÞΑW˜£ê½æËÚüΑ ˁڔΑŽæΡΑ
͉͉͋͏Π

ANALYSIS:
b¨˜Α#Ëõ˜ÚÄØÄæΑ¨ÞΑ˜Äõ«Þ£˜”ΑæËΑ”˜õ˜½Ë×Α樘ΑBæ«Ëā½Α#ÞΑ#Ú«”ΠΑ"ËÚΑ樫ÞΑ樘ΑT˜æÚ˽˜êÃΑÄ”ΑBæêځ½Α#ÞΑ
W˜£ê½æËÚüΑ ˁڔΑλTB#W μΑ«ÞΑ樘Αêæ¨ËÚ«æüΑæËΑ£ÚÄæΑêæ¨ËÚ«āæ«ËÄΑæËΑ樘Α˜Äæ«æ«˜ÞΑ¢ËÚΑ樘Α”˜õ˜½Ë×ØÄæΑË¢Α
«æüΑ#ÞΑ«ÞæÚ«êæ«ËÄΑλ#μΑĘæöËÚºΑ«ÄΑ#˜Ë£Ú×¨«Ž½ΑژÞΑλ#ÞμΑÞΑטÚΑTB#W ΑŽæΡΑ͉͉͋͏Π

[ŽËטΑ¢ËÚΑ˜½«Ã«Äæ«ËÄ
b¨˜Α ژ£ê½æËÚüΑ Ë”üΑ «ÞΑ Ë¢Α ژŽ˜ÄæΑ ËÚ«£«ÄΠΑ b¨˜Α æËëŽΑ ĘڣüΑ W˜£ê½æËÚüΑ ˁڔΑ öÞΑ ŽËÄÞæ«æê昔Α ËÄΑ ͎͊Α
BËõ˜Ã˜ÚΑ͊͒͑͌Πb¨˜Ú˜¢ËژΡΑŽ½˜Ú½üΑÞææ˜Ã˜ÄæΑ͊Α«ÞΑÄËæΑŽËÚژŽæΠΑfÞ«Ä£Α樁æΑ½Ë£«ŽΡΑö˜ΑŽÄΑژŽ¨Α樘ΑŽËÚژŽæΑ
ÄÞö˜ÚΠ

[HfWΑ΢ ¨ææ×΢έέöööΠ×Ä£ÚΠ£ËõΠ«Ä݁ËêæαêÞΠ¨æý
147

Year 2019 COAL SECTOR

V͋ΠΑËÄÞ«”˜ÚΑ樘Α¢Ë½½Ëö«Ä£ΑÞææ˜Ã˜ÄæÞ΢
1. Coal sector was nationalized by the Government of India under Indira Gandhi.
2. Now, coal blocks are allocated on lottery basis.
3. Till recently, India imported coal to meet the shortages of domestic supply, but now India is self-sufficient in coal
product.
Which of the statements given above is/are correct?
(a) 1 only
(b) 2 and 3 only
(c) 3 only
(d) 1, 2 and 3

ANSWERS A

EXPLANATION

Statement 1 is correct: ”˜Ùêæ˜ΑŽ×«æ½Α«Äõ˜ÞæØÄæΑæËΑؘæΑ樘ΑêÚ£˜ËÄ«Ä£Α˜Ä˜Ú£üΑʘ”ÞΑË¢Α樘ΑŽËêÄæÚüΑ


öÞΑÄËæΑ¢ËÚ樎ËëģΑ¢ÚËÃΑ樘Α×Ú«õæ˜ΑŽË½ΑëĘΑËöĘÚÞΠΑfÄގ«˜Äæ«ĈŽΑëīģΑ×ځŽæ«Ž˜ÞΑ”Ë×昔ΑüΑÞËØΑ
Ë¢Α 樘ÃΑ Ä”Α ×ËËÚΑ öËÚº«Ä£Α ŽËĔ«æ«ËÄÞΑ Ë¢Α ½ËêÚΑ «ÄΑ ÞËØΑ Ë¢Α 樘Α ×Ú«õæ˜Α ŽË½Α ëĘÞΑ ˜ŽÃ˜Α Áææ˜ÚÞΑ Ë¢Α
ŽËϘÚÄΑ¢ËÚΑ樘Α #Ëõ˜ÚÄØÄæΠΑ HÄΑ ŽŽËêÄæΑ Ë¢Α樘ޘΑ ژÞËÄÞΡΑ樘Α ˜Äæځ½Α #Ëõ˜ÚÄØÄæΑæË˺Α Α ”˜Ž«Þ«ËÄΑæËΑ
āæ«Ëā½«Þ˜Α樘Α×Ú«õæ˜ΑŽË½ΑëĘÞΠΑb¨˜Αāæ«Ëā½«Þæ«ËÄΑöÞΑ”ËĘΑ«ÄΑæöËΑרÞ˜ÞΡΑ樘ΑĈÚÞæΑö«æ¨Α樘ΑŽËº«Ä£Α
ŽË½ΑëĘÞΑ«ÄΑ͊͒͐͊α͐͋ΑÄ”Α樘ÄΑö«æ¨Α樘ΑÄËÄαŽËº«Ä£ΑŽË½ΑëĘÞΑ«ÄΑ͊͒͐͌Π
Statement 2 is not correct:Α ¢æ˜ÚΑ 樘Α [ê×ژØΑ ËêÚæΑ ŽÄŽ˜½½˜”Α 樘Α ŽË½Α ½ËŽºΑ ½½ËŽæ«ËÄÞΑ «ÄΑ ͉͍͋͊ΠΑ bËΑ
Áā£˜Α Ä”Α ژ½½ËŽæ˜Α樘Α ŽÄŽ˜½½˜”Α ½ËŽºÞΑ «ÄΑ ΑæځÄÞׁژÄæΑ Ä”Α ŽŽËêÄ恍½˜Α ÁÄĘÚΡΑ樘Α ˁ½Α A«Ä˜ÞΑ
λ[טŽ«½Α TÚËõ«Þ«ËÄÞμΑ ŽæΡΑ ͉͎͋͊Α öÞΑ ˜ÄŽæ˜”ΠΑ ā½«Ä£Α ×ÚËõ«Þ«ËÄÞΑ ö˜Ú˜Α Á”˜Α «ÄΑ 樘Α Ëõ˜Α ށ«”Α ŽæΑ ¢ËÚΑ
Ϗ½½ËŽæ«ËÄΑË¢ΑŽË½ΑëĘÞΑüΑöüΑË¢ΑêŽæ«ËÄΑÄ”Α½½ËæØÄæΑ¢ËÚΑ樘Αށ½˜ΑË¢ΑŽË½Πϐ
+ÄΑΑژŽ˜ÄæΑÄËæ«ĈŽæ«ËÄΑüΑ樘Α#Ëõ˜ÚÄØÄæΑËÄΑ͉͋æ¨Α"˜ÚêÚüΡΑ樘Α£Ëõ˜ÚÄØÄæΑËטʔΑê×ΑŽËÃØڎ«½ΑŽË½Α
ëīģΑ¢ËÚΑ樘Α×Ú«õæ˜ΑޘŽæËÚΑËÄΑ"˜ÚêÚüΑ͉͋ΡΑÄ”Α××ÚËõ˜”Α樘ΑØæ¨Ë”˽ˣüΑ¢ËÚΑêŽæ«ËÄΑË¢ΑŽË½ΑëĘÞΑέΑ
½ËŽºÞΑ¢ËÚΑށ½˜ΑË¢ΑŽË½Π
b¨˜Ú˜Αö«½½Α˜ΑÄΑϏÞŽ˜Ä”«Ä£Α¢ËÚöÚ”ΑêŽæ«ËÄϐΑααΑΑæöËαÞ恣˜ΑËĽ«Ä˜ΑêŽæ«ËÄΑŽËÃ×Ú«Þ«Ä£Αλ«μΑ明¨Ä«Ž½Α«”ΑÄ”Α
λ««μΑĈāĎ«½Α«”Αö«æ¨Α«Ä«æ«½ΑÄ”ΑĈā½Α×Ú«Ž˜ΑËą˜ÚÞΠΑb¨˜Α«”ΑׁځØæ˜ÚΑö«½½Α˜Α樘Α×Ú«Ž˜ΑËą˜ÚΑ«ÄΑέæËÄĘΑö¨«Ž¨Α
ö«½½Α˜Αׁ«”ΑæËΑ樘Α[ææ˜Α£Ëõ˜ÚÄØÄæΑËÄΑ樘ΑŽæꁽΑ×Ú˔êŽæ«ËÄΑË¢ΑŽË½Π
Statement 3 is not correct:Α+Ĕ«Α«ÞΑÞ櫽½ΑËĘΑË¢Α樘Α½Ú£˜ÞæΑ«Ã×ËÚæ˜ÚΑË¢Αˁ½ΠΑËØÞ櫎ΑŽË½Α¨ÞΑ˜˜ÄΑ½˜Α
æËΑؘæΑËĽüΑ͎͐АΑË¢ΑËêÚΑÄÄꁽΑŽË½Α”˜ÃÄ”ΠΑb¨˜ΑÞ¨ËÚ梁½½Α«ÄΑ½ËŽ½ΑŽË½Αõ«½«½«æüΑ«ÞΑØæΑæ¨ÚË꣨Α«Ã×ËÚæÞΑ
Ë¢ΑÚËêĔΑ͉͉͋ΑÃæΠ

ANALYSIS:
ANALYSIS:
[ŽËטΑ¢ËÚΑ˜½«Ã«Äæ«ËÄ
+Ĕ«Α”ËΑÄËæΑ¨õ˜Α£Ë˔ΑÙꁽ«æüΑŽË½Α¨˜ÄŽ˜Αö˜Α«Ã×ËÚæΑËêÚΑ£Ë˔ΑŽ¨êĺΑË¢ΑËêÚΑژÙê«Ú˜Ã˜ÄæΑÞËΑ͌ڔΑÞææ˜Ã˜ÄæΑ
ŽÄΑ˜Α˜½«Ã«Äæ˜”ΠΑBËöΑö˜ΑŽÄΑŽËØΑæËΑÄÞö˜ÚΑ͊Π

[HfWΑ΢ ŽËÄËëŽÞΑÞêÚõ˜üΑ¨ææ×Þ΢έέöööΠ樘¨«Ä”êЎËÃݍêޫĘÞÞέ+ĔêÞæÚüέö¨üᨁÞαŽË½αëÄα
«Ä£α˜˜ÄαËטʔαê×݁Ú櫎½˜͎͋͋͑͊͏͑͒ИŽ˜
148

ˁ½Α«ÞΑΑËĘΑË¢Α樘Α«Ã×ËÚæÄæΑëĘځ½ÞΑö¨«Ž¨Α«ÞΑÁ«Ä½üΑêޘ”Α«ÄΑ樘Α£˜Ä˜Úæ«ËÄΑË¢Α樘ÚÁ½Α×Ëö˜ÚΑÄ”Α
Þؽæ«Ä£ΑË¢Α«ÚËÄΑËژΠΑ
ˁ½Α ˎŽêÚÞΑ «ÄΑ ÚˎºΑ ޘÙê˜ÄŽ˜ÞΑ Á«Ä½üΑ Ë¢ two geological ages, namely Gondwana and Tertiary
deposits.
ËêæΑ͉͑ΑטÚΑŽ˜ÄæΑË¢Α樘ΑŽË½Α”˜×ËÞ«æÞΑ«ÄΑ+Ĕ«Α«ÞΑË¢Α«æêëÄËêÞΑæüטΑÄ”Α«ÞΑË¢ΑÄËÄαŽËº«Ä£Α£Ú”˜ΠΑ
b¨˜Α ÃËÞæΑ «Ã×ËÚæÄæΑ #ËĔöÄΑ ŽË½Α Ĉ˜½”ÞΑ Ë¢Α +Ĕ«Α Ú˜Α ½ËŽæ˜”Α «ÄΑ Ã˔ÚΑ q½½˜üΠΑ b¨˜üΑ ½«˜Α «ÄΑ
6¨Úº¨Ä”α ˜Ä£½ΑŽË½Α˜½æΑÄ”Α樘Α«Ã×ËÚæÄæΑŽË½ΑĈ˜½”ÞΑ«ÄΑ樫ÞΑژ£«ËÄΑÚ˜ΑWÄ«£Ä¶ΡΑ6¨Ú«ΡΑ ˺ÚËΡΑ
#«Ú«”«¨ΡΑ9ÚÄ×êځΠΑ
6¨Ú«Α«ÞΑ樘Α½Ú£˜ÞæΑŽË½ΑĈ˜½”Α¢Ë½½Ëö˜”ΑüΑWÄ«£Ä¶Π
Tertiary coals occur in Assam, Arunachal Pradesh, Meghalaya and Nagaland.Α+æΑ«ÞΑ˜ûæځŽæ˜”Α¢ÚËÃΑ
ÚÄ£«Ú«ΡΑ¨˜Úځ×êĶ«ΡΑA˜ö½ËÄ£ΑÄ”Α;Ä£Ú«ÄΑλA˜£¨½üμΣΑAºêÃΡΑ6«×êÚΑÄ”ΑBā«ÚΑ«ÄΑê×טÚΑÞށÃΡΑ
BÃŽ¨«ºΑγBÃרêºΑλÚêāŽ¨½ΑTځ”˜Þ¨μΑÄ”Α9½ºËæΑλ6ÃÃêΑÄ”Α9Þ¨Ã«ÚμΠΑ
The brown coal or lignite occur in the coastal areas of Tamil Nadu, Puducherry, Gujarat and
Jammu and Kashmir.

Year 2023 URANIUM

V͌ΠΑËÄÞ«”˜ÚΑ樘Α¢Ë½½Ëö«Ä£ΑÞææ˜Ã˜ÄæÞ΢Α
Statement-I: India, despite having uranium deposits, depends on coal for most of its electricity production.
Statement-II: Uranium, enriched to the extent of at least 60%, is required for the production of electricity. Which one
of the following is correct in respect of the above statements?
(a) Both Statement-I and Statement-II are correct and Statement-II is the correct explanation for Statement-I
(b) Both Statement-I and Statement-II is not the correct explanation for Statement-I
(c) Statement-I is correct but statement-II is incorrect
(d) Statement-I is incorrect but Statement-II is correct

ANSWERS C

EXPLANATION

Statement I is correct: æËëŽΑA«Ä˜Ú½ÞΑ«Ú˜ŽæËځæ˜Α¢ËÚΑû×½Ëځæ«ËÄΑÄ”ΑW˜Þ˜ÚŽ¨ΑλAμΡΑΑŽËÄÞæ«æê˜ÄæΑ


êÄ«æΑË¢Α樘Α˜×ÚæØÄæΑË¢ΑæËëŽΑĘڣüΑλμΡΑ½½ΑËõ˜ÚΑ+Ĕ«Α¨ÞΑ˜Þ恍½«Þ¨˜”Α͌Ρ͑͋Ρ͏͎͐ΑæËÄĘÞΑ«ÄΑÞ«æêΑf͌H͑Α
«ÄΑ ͍͐Α êځīêÃΑ ”˜×ËÞ«æÞΑ «ÄΑ Ĕ¨ÚΑ Tځ”˜Þ¨ΡΑ b˜½Ä£ÄΡΑ 6¨Úº¨Ä”ΡΑ A˜£¨½üΡΑ W¶Þ樁ÄΡΑ 9Úā恺ΡΑ
¨¨ææ«Þ£Ú¨ΡΑfææÚΑTځ”˜Þ¨ΡΑfææÚº¨Ä”ΡΑ(«ÃŽ¨½ΑTځ”˜Þ¨ΑÄ”ΑA¨ÚÞ¨æځΠΑ(Ëö˜õ˜ÚΡΑ«ÄΑ+Ĕ«ΑÁ¶ËÚΑ
×Ú˔êŽæ«ËÄΑË¢Α½˜ŽæÚ«Ž«æüΑ«ÞΑŽ¨«˜õ˜”Αæ¨ÚË꣨ΑŽË½ΑÞ˜”Α樘ÚÁ½Α×Ëö˜ÚΑ×½ÄæΑö¨«Ž¨Α«ÞΑÚËêĔΑ͎͐АΑË¢Α樘Α
æË恽Α×Ëö˜ÚΑ£˜Ä˜Úæ«ËÄΠΑ
Statement II is not correct: fځīêÃΑ ˜ÄÚ«Ž¨˜”Α æËΑ ŽËϘÄæځæ«ËÄÞΑ Ëõ˜Α ͉Π͐АΑ êæΑ ½˜ÞÞΑ 樁ÄΑ ͉͋АΑ
êځīêÃα͎͋͌Α«ÞΑ”˜ĈʔΑÞΑ½ËöΑ˜ÄÚ«Ž¨˜”ΑêځīêÃΑλ;fμΠΑAËÞæΑÄꎽ˜ÚΑژŽæËÚÞΑêޘΑ;fΑ樁æΑ«ÞΑËêæΑ͌α͎АΑ
êځīêÃα͎͋͌ΠΑ fځīêÃΑ ˜ÄÚ«Ž¨˜”ΑæËΑ ÃËژΑ樁ÄΑ ͉͋АΑ êځīêÃα͎͋͌Α «ÞΑ ”˜ĈʔΑ ÞΑ ¨«£¨½üΑ ˜ÄÚ«Ž¨˜”Α êځīêÃΑ
λ(fμΠΑb¨˜Α£½Ë½ΑÄꎽ˜ÚΑöæŽ¨”Ë£Αλ+μΑ¨ÞΑ¢ËêĔΑêځīêÃΑׁÚ櫎½˜ÞΑ˜ÄÚ«Ž¨˜”ΑæËΑ͑͌Π͐АΑ×êÚ«æüΑαΑõ˜ÚüΑ
Ž½ËޘΑæËΑö˜×ËÄÞΑ£Ú”˜ΑαΑæΑ+ځÄϖÞΑêĔ˜Ú£ÚËêĔΑ"ËڔËΑÞ«æ˜ΠΑ+ځÄΑ¨ÞΑ˜˜ÄΑ˜ÄÚ«Ž¨«Ä£ΑêځīêÃΑæËΑê×ΑæËΑ͏͉АΑ
×êÚ«æüΑޫϘΑ×Ú«½Α͉͋͋͊ΑΑ½˜õ˜½ΑæΑö¨«Ž¨ΑÄËÄ×Ú˽«¢˜Úæ«ËÄΑ˜ûטÚæÞΑށüΑb˜¨ÚÄΑ¨ÞΑÄËΑŽ«õ«½«ÄΑêޘΠ
149

ANALYSIS:
ANALYSIS:
W˜Ž˜Äæ½üΡΑ+Α¨ÞΑ¢ËêĔΑ͍͑АΑ˜ÄÚ«Ž¨˜”ΑêځīêÃΑæΑ+ځÄϖÞΑÄꎽ˜ÚΑÞ«æ˜ΑÄ”Α½ÞËΑ
(꣘Α”˜×ËÞ«æÞΑË¢ΑfځīêÃΑö˜Ú˜Α¢ËêĔΑ«ÄΑW¶Þ樁ÄΠ

[ŽËטΑ¢ËÚΑ˜½«Ã«Äæ«ËÄ
Statement II is cannot be correct ÞΑfځīêÃΑÄÚ«Ž¨Ã˜ÄæΑ¨ÞΑ½öüÞΑ˜˜ÄΑ«ÄΑ樘ΑĘöÞΑλ˜ÄæÚ«¢ê£˜ÞμΑÄ”Α
+ځÄϐÞΑÄꎽ˜ÚΑޘŽæËÚΑÄ”Α樘ΑÄÞΑ«Ã×Ëޘ”ΑüΑT͎ΑϹΑ͊ΠΑ+æΑ«ÞΑºÄËöÄΑ樁æΑ樘Α˜ÄÚ«Ž¨Ã˜ÄæΑ½˜õ˜½ΑژÙê«Ú˜”Α¢ËÚΑ
Äꎽ˜ÚΑ ×Ëö˜ÚΑ ژŽæËÚÞΑ «ÞΑ ÄËÚÁ½½üΑ ÚËêĔΑ ͎АΠΑ +¢Α [ææ˜Ã˜ÄæΑ II is wrong then options, A, B and D are
eliminated.

[HfWΑ΢ [æÄ”Ú”ΑĘöÞׁטÚÞ
¨ææ×Þ΢έέ«Ä”«Ä˜û×ژÞÞЎËÃ݁Ú櫎½˜έöËÚ½”έ«ÚÄα˜ÄÚ«Ž¨˜ÞαêځīêÃαæËα͏͉αטÚ᎘ÄæαΑ
×êÚ«æüα¢ËڔËαÞ«æ˜α͑͋͑͋͒͒͌έ
¨ææ×Þ΢έέöööЍêޫĘÞÞαÞæÄ”Ú”ΠŽËÃ݁Ú櫎½˜έ˜ŽËÄËÃüα×˽«Žüέځ¶Þ樁Äα˜Äæ˜ÚÞαêځīêÃαΑ
ëīģα«ÞÞê˜Þα½Ë«αæËαêځīêÃαŽËÚ×Ëځæ«ËÄα͉͊͋͋͏͉͉͋͐͑͌͐ε͊Π¨æý
150

PLANNING
λ͊ΑVê˜Þæ«ËÄÞμ

Year 2019 FIVE YEAR PLANS

V͊ΠΑr«æ¨Αژ¢˜Ú˜ÄŽ˜ΑæËΑ+Ĕ«ϐÞΑ"«õ˜αx˜ÚΑT½ÄÞΡΑö¨«Ž¨ΑË¢Α樘Α¢Ë½½Ëö«Ä£ΑÞææ˜Ã˜ÄæÞΑ«ÞέÚ˜ΑŽËÚژŽæΧ
1. From the Second Five-Year Plan, there was a determined thrust towards the substitution of basic and capital goods
industries.
2. The Fourth Five-Year Plan adopted the objective of correcting the earlier trend of increased concentration of wealth
and economic power.
3. In the Fifth Five-Year Plan, for the first time, the financial sector was included as an integral part of the Plan.
Select the correct answer using the code given below.
(a) 1 and 2 only
(b) 2 only
(c) 3 only
(d) 1, 2 and 3

ANSWERS A

EXPLANATION

Statement 1 is correctΑ«ÄΑ”Ë×æ«Ä£Α樘ΑÞæځ昣üΑË¢Α+Ã×ËÚæΑ[êÞæ«æêæ«ËÄΑ+ĔêÞæÚ«½«āæ«ËÄΑλ+[+μΑ«ÄΑ樘ΑĈ¢æ«˜ÞΠΑ
b¨˜Α Ž¨«˜¢Α ˍ¶˜Žæ«õ˜Α öÞΑ æËΑ ê«½”Α Α ޘ½¢αژ½«ÄæΑ ˜ŽËÄËÃüΠΑ "ÚËÃΑ 樘Α [˜ŽËĔΑ "«õ˜Α x˜ÚΑ T½ÄΡΑ 樘ژΑ öÞΑ Α
”˜æ˜ÚëʔΑæ¨ÚêÞæΑæËöÚ”ÞΑÞêÞæ«æêæ«ËÄΑË¢ΑÞ«ŽΑÄ”ΑŽ×«æ½Α£Ë˔ÞΑ«Ä”êÞæÚ«˜ÞΠΑb¨˜Α+[+ΑÞæځ昣üΑöÞΑÞ˜”Α
ËÄΑ樘ΑÃ˔˜½ΑË¢Α£ÚËöæ¨ΑÞΑ×ÚË×ËêĔ˜”ΑüΑTΑA¨½ËÄˍ«ÞΠ
Statement 2 is correct- b¨˜Α "ËêÚæ¨Α T½ÄΑ ×ÚËõ«”˜”Α Α ʎ˜ÞށÚüΑ ŽËÚژŽæ«õ˜Α æËΑ 樘Α ˜Ú½«˜ÚΑ æژĔΑ ö¨«Ž¨Α
¨˜½×˜”Α ׁÚ櫎꽁ڽüΑ樘Α ÞæÚËÄ£˜ÚΑ ޘŽæ«ËÄÞΑ «ÄΑ £Ú«Žê½æêژΑ ÞΑö˜½½Α ÞΑ «ÄΑ «Ä”êÞæÚüΑæËΑ ˜Ä½˜Α樘ÃΑ ځ׫”½üΑæËΑ
˜Ä½Ú£˜ΑÄ”Α”«õ˜ÚÞ«¢üΑ樘Α×Ú˔êŽæ«ËÄΑÞ˜ΠΑ+ÄΑ樘Α½ËÄ£ΑÚêÄΡΑ樘Α¢ê½½Α×Ëæ˜Ä櫁½ΑË¢Α£ÚËöæ¨ΑŽÄÄËæΑ˜Αژ½«Þ˜”Α
êĽ˜ÞÞΑ樘Α˜Ä˜Ú£«˜ÞΑË¢Α½½ΑËêÚΑטË×½˜ΑÚ˜Α×êæΑæËΑ×ÚËĈ恍½˜ΑêޘΠΑb¨˜Α˜ÃרÞ«ÞΑËÄΑÞ×ژ”«Ä£Α樘Α«ÃטæêÞΑÄ”Α
˜Ä˜ĈæÞΑË¢Α˜ŽËÄËëŽΑ£ÚËöæ¨ΑæËΑ樘Αö˜º˜ÚΑޘŽæ«ËÄÞΑ«ÞΑæ¨êÞΑʎ˜ÞށÚüΑ«ÄΑ樘Α«Äæ˜Ú˜ÞæΑË¢Α˜Ùꁽ«æüΑÞΑö˜½½ΑÞΑ
£ÚËöæ¨ΠΑb¨˜ΑT½ÄΑö«½½ΑÄËöΑÞÞ«ÞæΑ樘Α½˜ÞÞΑ×ÚËÞטÚËêÞΑޘŽæ«ËÄÞΑË¢ΑËêÚΑ¢ÚëģΑ×Ë×꽁æ«ËÄΑæËΑ«Ã×ÚËõ˜Α樘«ÚΑ
×ËÞ«æ«ËÄΑÄ”ΑÁº˜ΑΑü˜æΑ«££˜ÚΑŽËÄæÚ«êæ«ËÄΑæËΑ樘Αāæ«Ëā½Α˜ŽËÄËÃüΠ
Statement 3 is not correct:Αb¨˜ΑĈāĎ«½ΑޘŽæËÚΑ˜ŽËØÞΑÄΑ«Ä昣ځ½ΑׁÚæΑË¢Α樘Α×½ÄΑ«ÄΑ樘Α͒æ¨ΑĈõ˜αü˜ÚΑ
×½ÄΠ

ANALYSIS:
Lorem
5-year ×½ÄÞΑ Ú˜Α £«õ˜ÄΑ «ÄΑ ͊͊æ¨Α Ž½ÞÞΑ BWbΑ ö¨«Ž¨Α «ÞΑ Α Þ«ŽΑ ÞËêڎ˜Α Ä”Α 樫ÞΑ æË׫ŽΑ «ÞΑ ŽÚꎫ½Α «ÄΑ
êĔ˜ÚÞæÄ”«Ä£Α樘Α˜ŽËÄËëŽΑ¨«ÞæËÚüΑË¢Α+Ĕ«Π
151

[ŽËטΑ¢ËÚΑ˜½«Ã«Äæ«ËÄ

æΑ ½˜ÞæΑ 樘Α ͌ڔΑ Þææ˜Ã˜ÄæΑ ŽË꽔Α ¨õ˜Α ˜˜ÄΑ ˜½«Ã«Äæ˜”Α ÞΑ 樘Α ͎æ¨Α "«õ˜Αx˜ÚΑ öÞΑ æËËΑ ˜Ú½üΑ æËΑ ¢ËŽêÞΑ ËÄΑ Α
Ã˔˜ÚÄΑʘ”Π

[HfWΑ΢ ¨ææ×Þ΢έέöööΠīע×ΠËÚ£Π«Äέؔ«έה¢έË˺Þέ 9ε͌͌έ¨×α


æ˜ÚÞέ͊ΠА͉͋+Ã×ËÚæА͉͋[êÞæ«æêæ«ËÄА͉͋[æځ昣üА͉͋H¢А͉͋ŽËÄËëŽА͉͋˜õ˜½Ë×ØÄæΠה¢
¨ææ×΢έέ×½ÄÄ«Ä£ŽËÃëÞÞ«ËÄΠÄ«ŽΠ«Äέ×½ÄÞέ×½Äژ½έĈõ˜üÚέ͍æ¨έ͍××Ú˜Π¨æÃ
¨ææ×Þ΢έݍË˺ÞΠ£ËË£½˜ΠŽËΠ«ÄݍË˺ÞΧ«”Ͻ͐͐BböV 6Ϟ×£ϽTb͊͋͐Ϟ”α
ÙϽΠϹ+æϹöÞϹ樘ϹĈÚÞæϹæ«Ã˜Ϲö¨˜ÄϹ樘ϹĈāĎ«½ϹޘŽæËÚϹöÞϹ«ÄŽ½ê”˜”ϹÞϹÄϹ«Ä昣ځ½ϹׁÚ
æϹË¢Ϲ樘Ϲ×½ÄΠϞ¨½Ͻ˜ÄϞށϽwϞõ˜”Ͻ͉¨f9ö¶͍͎͑͐ÙÞ嫨r#ºÄ9(¢õ͉AöV͏+9¶άõϽË
Ęׁ£˜ϞÙϽΠА͉͋+æА͉͋öÞА͉͋樘А͉͋ĈÚÞæА͉͋æ«Ã˜А͉͋ö¨˜ÄА͉͋樘А͉͋ĈāĎ«½А͉͋ޘŽæË
ÚА͉͋öÞА͉͋«ÄŽ½ê”˜”А͉͋ÞА͉͋ÄА͉͋«Ä昣ځ½Аׁ͉͋ÚæА͉͋Ë¢А͉͋樘А͉͋×½ÄΠϞ¢Ͻ¢½Þ˜
152

1991 Reforms
λ͋ΑVê˜Þæ«ËÄÞμ

Year 2017 LPG 1991

V͊ΠΑr¨«Ž¨ΑË¢Α樘Α¢Ë½½Ëö«Ä£Α¨Þέ¨õ˜ΑˎŽêÚژ”Α«ÄΑ+Ĕ«Α¢æ˜ÚΑ«æÞΑ½«˜Ú½«āæ«ËÄΑË¢Α˜ŽËÄËëŽΑ×˽«Ž«˜ÞΑ«ÄΑ
͊͒͒͊Χ
1. Share of agriculture in GDP increased enormously.
2. Share of India’s exports in world trade increased.
3. FDI inflows increased.
4. India’s foreign exchange reserves increased enormously.
Select the correct answer using the codes given below:
(a) 1 and 4 only
(b) 2, 3 and 4 only
(c) 2 and 3 only
(d) 1, 2, 3 and 4

ANSWERS B

EXPLANATION

b¨˜Α×ËÞæαژ¢ËÚÃΑטګ˔ΑÞ¨ËöÞΑ樘Α£Ú”ꁽΑ”˜Ž½«Ä˜Α«ÄΑ樘Α£Ú«Žê½æêژΑޘŽæËÚϐÞΑŽËÄæÚ«êæ«ËÄΑæËΑ樘Α+Ĕ«ÄΑ
˜ŽËÄËÃüΠΑ +Ĕ«ϐÞΑ æځ”«æ«Ëā½Α ˎŽêׁæ«ËÄΡΑ £Ú«Žê½æêژΑ ÄËöΑ ŽËÄæÚ«êæ˜ÞΑ ËĽüΑ ËêæΑ ͎͊АΑ æËΑ 樘Α #TΡΑ ”ËöÄΑ
¢ÚËÃΑ͋͒Αטڎ˜ÄæΑ«ÄΑ͊͒͒͊ΠΑHence, statement 1 is incorrect.
[¨Ú˜ΑË¢Α+Ĕ«ϖÞΑ˜û×ËÚæÞΑ«ÄŽÚ˜Þ˜”ΑޫϘΑ͊͒͒͊ΠΑHence statement 2 is correct.
˜¢ËژΑ͊͒͒͊ΡΑ¢Ëژ«£ÄΑ«Äõ˜ÞæØÄæΑöÞΑĘ£½«£«½˜ΠΑb¨˜ΑĈÚÞæΑü˜ÚΑË¢Αژ¢ËÚÃΑށöΑΑæË恽Α¢Ëژ«£ÄΑ«Äõ˜ÞæØÄæΑË¢Α
ËĽüΑϮ͍͐Αë½½«ËÄΠΑ(Ëö˜õ˜ÚΡΑ«Äõ˜ÞæØÄæÞΑ¨õ˜ΑÞ昁”«½üΑګޘÄΑޫϘΑ樘ÄΡΑ˜ûŽ˜×æΑ¢ËÚΑˎŽÞ«Ëā½Α½«×ÞΑ˜æö˜˜ÄΑ
͊͒͒͐ΑÄ”Α͉͉͉͋ΑÄ”Α͉͉͋͑ΑÄ”Α͉͋͊͋ΑγΑËö«Ä£ΑæËΑ樘Α£½Ë½Α˜ŽËÄËëŽΑÞ½Ëö”ËöÄΠΑÞΑË¢Α͌͊ΑAÚŽ¨Α͉͋͊͏ΡΑ樘Α
ŽËêÄæÚüΑ¨ÞΑژŽ˜«õ˜”ΑæË恽Α"+ΑË¢ΑϮ͌͐͊Α«½½«ËÄΡΑޫϘΑ͊͒͒͊ΠΑb¨˜Αü˜ÚΑ͉͉͋͑ΑژŽËڔ˜”Α樘Α¨«£¨˜ÞæΑ"+Α«ÄĉËöΑË¢Α
Ϯ͍͌Π͍͉Α«½½«ËÄΠΑb¨˜Α«££˜ÞæΑÞ×êÚæΑ«ÄΑ«ÄĉËöΑöÞΑ˜æö˜˜ÄΑ͉͉͎͋ΑÄ”Α͉͉͋͏ΑγΑ͎͊͐Π͎͍АΠΑÞΑË¢ΑAÚŽ¨Α͉͋͊͏ΡΑ+Ĕ«Α
¨ÞΑææځŽæ˜”ΑϮ͉͊Π͎͎Α«½½«ËÄΑöËÚæ¨ΑË¢Α"+ΠΑHence, statement 3 is correct.
+æΑöÞΑ+Ĕ«ϐÞΑ”«ÞÁ½ΑÞææ˜ΑË¢Α¢ËژûΑژޘÚõ˜ÞΑ樁æΑ¢Ëڎ˜”Α樘Α£Ëõ˜ÚÄØÄæΑæËΑÚ«Ä£Α«ÄΑ˜ŽËÄËëŽΑژ¢ËÚÃÞΠΑBËöΡΑ
͎͋Αü˜ÚÞΑ½æ˜ÚΡΑ¢ËژûΑژޘÚõ˜ÞΑÚ˜ΑæΑΑژŽËڔΑ¨«£¨ΠΑ+ÄΑ͊͒͒͊ΡΑ«æΑÞæË˔ΑæΑ¶êÞæΑϮ͎Π͑Α«½½«ËÄΠΑÞΑË¢Α͍͋Α6êĘΡΑ樘Α
ŽËêÄæÚüϐÞΑ ¢ËژûΑ ژޘÚõ˜ÞΑ Ú˜Α æΑ Ϯ͌͏͉Π͑Α «½½«ËÄΠΑ fÞꁽ½üΡΑ «Ã×ËÚæΑ ŽËõ˜Ú£˜Α Ë¢Α ͐α͑Α ÃËÄæ¨ÞΑ «ÞΑ ŽËÄÞ«”˜Ú˜”Α
ÞêƎ«˜ÄæΠΑb¨˜Α«££˜ÞæΑ¶êÃ×Α«ÄΑژޘÚõ˜ÞΑöÞΑö«æĘÞޘ”Α˜æö˜˜ÄΑ͉͉͋͐ΑÄ”Α͉͉͋͑Αö¨˜ÄΑ樘Αº«ææüΑê½£˜”Α
͎͎АΑæËΑ¨«æΑϮ͉͌͒Π͋Α«½½«ËÄΠΑ(˜ÄŽ˜ΡΑÞææ˜Ã˜ÄæΑ͍Α«ÞΑŽËÚژŽæΠ
153

ANALYSIS:
ANALYSIS:

[ŽËטΑ¢ËÚΑ˜½«Ã«Äæ«ËÄ
+æΑ «ÞΑ ŽËÃÃËÄΑ ºÄËö½˜”£˜Α樁æΑ樘Α ;T#Α ژ¢ËÚÃÞΑ «Ã˜”Α æΑ ˜Þ«Ä£Α Ú꽘ÞΑ Ä”Α ژ£ê½æ«ËÄÞΑ ¢ËÚΑ ×ګÁګ½üΑ樘Α
b˜Ú櫁ÚüΑ[˜ŽæËÚΑÄ”Α樘Α+ĔêÞæÚ«½ΑޘŽæËÚΠΑ£Ú«Žê½æêژΑöÞΑÄËæΑæΑ樘Α¢ËŽêÞΠΑ(˜ÄŽ˜ΑË×æ«ËÄΑ͊ΑŽÄΑ˜Α˜Þ«½üΑ
˜½«Ã«Äæ˜”Π

[HfWΑ΢ ¨ææ×΢έέöööΠĈÚÞæ×ËÞæЎËÃݍêޫĘÞÞέ͎͋αü˜ÚÞαË¢α½«˜Ú½«Þæ«ËÄαα£½«Ã×ޘαË¢α«Ä”«ÞαΑ
£ÚËöæ¨α«Äα͍͊ᎨÚæÞα͋͑͐͐͏͎͍Π¨æý

Year 2020 1991 REFORMS

V͋ΠΑr«æ¨Αژ¢˜Ú˜ÄŽ˜ΑæËΑ樘Α+Ĕ«ÄΑ˜ŽËÄËÃüΑ¢æ˜ÚΑ樘Α͊͒͒͊Α˜ŽËÄËëŽΑ½«˜Ú½«āæ«ËÄΡΑŽËÄÞ«”˜ÚΑ樘Α¢Ë½½Ëöα
«Ä£ΑÞææ˜Ã˜ÄæÞ΢Α
1. Worker productivity (rs. per worker at 2004 -05 prices) increased in urban areas while it decreased in rural areas.
2. The percentage share of rural areas in the workforce steadily increased.
3. In rural areas, the growth in non-farm economy increased.
4. The growth rate in rural employment decreased.
Which of the statements given above is/are correct?
(a) 1 and 2 only
(b) 3 and 4 only
(c) 3 only
(d) 1, 2 and 4 only

ANSWERS B

EXPLANATION

+ÄΑB+b+ΑüË£ϐÞΑׁטÚΑύ¨Ä£«Ä£Α[æÚêŽæêژΑË¢ΑWêځ½ΑŽËÄËÃüΑË¢Α+Ĕ«Α+Ã×½«Žæ«ËÄÞΑ¢ËÚΑÃ×½ËüØÄæΑÄ”Α
#ÚËöæ¨Α͉͋͊͐ώΡΑ¢Ë½½Ëö«Ä£ΑˍޘÚõæ«ËÄÞΑö˜Ú˜ΑÁ”˜΢Α
b¨˜ΑÞ˽êæ˜Α½˜õ˜½ΑË¢Α«ÄŽËØΑטÚΑöËÚº˜ÚΑ«Π˜ΠΑöËÚº˜ÚΑ×Ú˔êŽæ«õ«æüΑ¨ÞΑ«ÄŽÚ˜Þ˜”Α¢ËÚΑËæ¨ΑÚêځ½ΑÄ”ΑêڍÄΑ
Ú˜ÞΠΑ"ËÚΑÚêځ½ΑÚ˜ÞΑ«æΑöÞΑWÞΠΑ͌͐͋͐͌Α«ÄΑ͉͉͍͋Αα͉͎ΑÄ”ΑWÞΠΑ͉͎͎͊͊͐Α«ÄΑ͉͋͊͊Αα͊͋ΡΑö¨«½˜Α¢ËÚΑêڍÄΑÚ˜ÞΑ«æΑöÞΑ
WÞΠΑ͉͍͊͋͊͒Α«ÄΑ͉͉͍͋Αα͉͎ΑÄ”ΑWÞΠΑ͎͎͋͑͋͊Α«ÄΑ͉͋͊͊Αα͊͋ΠΑHence statement 1 is not correct .
ÞΑטÚΑ͉͋͊͊Α˜ÄÞêÞΡΑ͏͑Π͑АΑË¢Α+Ĕ«ϐÞΑ×Ë×꽁æ«ËÄΑÄ”Α͐͋Π͍АΑË¢ΑöËÚº¢Ëڎ˜Αژޫ”˜”Α«ÄΑÚêځ½ΑÚ˜ÞΠΑ(Ëö˜õ˜ÚΡΑ
Þ昁”üΑæځÄÞ«æ«ËÄΑæËΑêڍÄ«āæ«ËÄΑËõ˜ÚΑ樘Αü˜ÚÞΑ¨ÞΑ½˜”ΑæËΑΑ”˜Ž½«Ä˜Α«ÄΑ樘ΑÚêځ½ΑÞ¨Ú˜Α«ÄΑ樘ΑöËÚº¢Ëڎ˜ΡΑ
¢ÚËÃΑ͐͐Π͑АΑ«ÄΑ͊͒͒͌Αα͍͒ΑæËΑ͉͐Π͒АΑ«ÄΑ͉͋͊͊Αα͊͋ΠΑHence statement 2 is incorrect .
ËêæΑæöËΑ樫ڔΑË¢ΑÚêځ½Α«ÄŽËØΑ«ÞΑÄËöΑ£˜Ä˜Úæ˜”Α«ÄΑÄËÄΑα£Ú«Žê½æêځ½ΑŽæ«õ«æ«˜ÞΠΑBËÄΑα¢ÚÃΑ˜ŽËÄËÃüΑ¨ÞΑ
«ÄŽÚ˜Þ˜”Α«ÄΑÚêځ½ΑÚ˜ÞΠΑb¨˜ΑÞ¨Ú˜ΑË¢Α£Ú«Žê½æêژΑ«ÄΑÚêځ½Α˜ŽËÄËÃüΑ¨ÞΑ”˜ŽÚ˜Þ˜”Α¢ÚËÃΑ͎͐АΑ«ÄΑ͊͒͒͌Αα͍͒ΑæËΑ
͌͒АΑ«ÄΑ͉͋͊͊Αα͊͋Π Hence statement 3 is correct .
¢æ˜ÚΑ͉͉͍͋Αα͉͎ΡΑ樘ΑÚêځ½ΑÚ˜ÞΑ¨õ˜Αö«æĘÞޘ”ΑĘ£æ«õ˜Α£ÚËöæ¨Α«ÄΑ˜Ã×½ËüØÄæΑ«ÄÞ׫æ˜ΑË¢Α¨«£¨Α£ÚËöæ¨Α«ÄΑ
Ëêæ×êæΠΑb¨˜Α£ÚËöæ¨Αځæ˜ΑË¢ΑÚêځ½Α˜Ã×½ËüØÄæΑöÞΑ͊Π͍͎АΑ”êÚ«Ä£Α͍͊͒͒Αα͉͉͎͋ΡΑö¨«Ž¨Α¢˜½½ΑæËΑα͉Π͋͑АΑ˜æö˜˜ÄΑ
͉͉͎͋Αα͊͋ΠΑ(˜ÄŽ˜ΑÞææ˜Ã˜ÄæΑ͍Α«ÞΑŽËÚژŽæΑΠ
154

ANALYSIS:
ANALYSIS:
+æΑöÞΑ¢ÚËÃΑ樘ΑׁטÚΑژ½˜Þ˜”ΑüΑB«æ«ΑüË£Α¨Ä£«Ä£Α[æÚêŽæêژΑË¢ΑWêځ½ΑŽËÄËÃüΑË¢Α+Ĕ«Α+Ã×½«Žæ«ËÄÞΑ¢ËÚΑ
Ã×½ËüØÄæΑÄ”Α#ÚËöæ¨Α͉͋͊͐Π

[ŽËטΑ¢ËÚΑ˜½«Ã«Äæ«ËÄ
b¨˜Α ޘŽËĔΑ Þææ˜Ã˜ÄæΑ ŽÄΑ ˜Α ˜Þ«½üΑ ˜½«Ã«Äæ˜”Α ÞΑ ϏÞ昁”üΑ £ÚËöæ¨ϐΑ «ÄΑ ˜ŽËÄËëŽÞΑ «ÞΑ ځژΑ λêĔËêæ˜”½üΑ 樫ÞΑ
ÞÞêÃ×æ«ËÄΑŽÚÚ«˜ÞΑÄΑ˜½˜Ã˜ÄæΑË¢ΑÚ«Þºμ

[HfWΑ΢ Directly from the paper ‘Changing Structure of Rural Economy of India Implications for
Employment and Growth 2017’ by Niti Ayog -Α¨ææ×Þ΢έέöööΠÄ«æ«Π£ËõΠ«ÄέÞ«æ˜Þέ”˜¢ê½æέĈ½˜ÞέΑ
͉͋͋͌α͉͋έ͊͊εWêځ½εŽËÄËÃüε«ÞŽêÞÞ«ËÄεT×˜Úε͉Πה¢
155

POVERTY
λ͊ΑVê˜Þæ«ËÄÞμ

Year 2019 POVERTY LINES

V͊ΠΑ+ÄΑΑ£«õ˜ÄΑü˜ÚΑ«ÄΑ+Ĕ«ΡΑËƎ«½Α×Ëõ˜ÚæüΑ½«Ä˜ÞΑÚ˜Α¨«£¨˜ÚΑ«ÄΑÞËØΑÞææ˜ÞΑ樁ÄΑ«ÄΑË樘ÚÞΑ˜Žêޘ
(a) poverty rates vary from State to State
(b) price levels vary from State to State
(c) Gross State Product varies from State to State
(d) quality of public distribution varies from State to State

ANSWERS B

EXPLANATION

b¨˜ΑT½ÄÄ«Ä£ΑËÃëÞÞ«ËÄΑØæ¨Ë”˽ˣüΑ¢ËÚΑ˜Þæ«Ãæ«Ä£Α×Ëõ˜ÚæüΑæΑāæ«Ëā½ΑÄ”ΑÞææ˜Α½˜õ˜½ΑöÞΑژ£Ú”˜”ΑüΑ
ÞËØΑÞΑ«Ä××ÚË×Ú«æ˜Α«ÄΑ£«õ«Ä£ΑΑژ×ژޘÄææ«õ˜Α׫ŽæêژΑË¢Α樘Α«ÄŽ«”˜ÄŽ˜ΑË¢Α×Ëõ˜ÚæüΑ«ÄΑ樘ΑŽËêÄæÚüΠΑ
b¨˜ΑÁ«ÄΑ×Ë«ÄæÞΑË¢Α樘ΑŽÚ«æ«Ž«ÞÃΑö˜Ú˜΢
樘Α”¶êÞæØÄæΑ×Úˎ˜”êژΣ
樘ΑŽ¨Ë«Ž˜ΑË¢Α”˜ĉæËÚÞΑæËΑژ×ژޘÄæΑ×Ú«Ž˜ΑŽ¨Ä£˜ÞΑ«ÄΑ樘Α×Ëõ˜ÚæüΑ½«Ä˜Σ
樘Α××½«Žæ«ËÄΑË¢Α樘ΑށØΑ×Ëõ˜ÚæüΑ½«Ä˜Α«ÄΑ½½Α樘ΑÞææ˜ÞΡΑö¨«Ž¨Α«Ã×½üΑ樘ΑÞ˜ÄŽ˜ΑË¢Α×Ú«Ž˜Α”«ą˜Ú˜Ä櫁½ÞΑ
ŽÚËÞÞΑ樘ΑÞææ˜ÞΣ
êޘΑË¢ΑΑĈû˜”ΑŽËÄÞêÃ×æ«ËÄΑÞº˜æΑËõ˜ÚΑæ«Ã˜ΣΑÄ”
樘ΑêÄ«¢ËÚÃΑŽËÄÞêÃ×æ«ËÄΑÞº˜æΑ¢ËÚΑ½½Α樘ΑÞææ˜ÞΠ

ANALYSIS:
ANALYSIS:

[ŽËטΑ¢ËÚΑ˜½«Ã«Äæ«ËÄ
#ÚËÞÞΑ [ææ˜Α TÚ˔êŽæΑ õÚ«˜ÞΑ ¢ÚËÃΑ [ææ˜Α æËΑ [ææ˜Α αΑ b¨˜Α #ÚËÞÞΑ [ææ˜Α TÚ˔êŽæΑ λ#[TμΑ «ÞΑ Α ؁ÞêژΑ Ë¢Α 樘Α
˜ŽËÄËëŽΑ Ëêæ×êæΑ Ë¢Α Α Þææ˜ΡΑ Ä”Αö¨«½˜Α «æΑ ÁüΑ ŽËÚژ½æ˜Αö«æ¨Α ×Ëõ˜ÚæüΑ ځæ˜ÞΡΑ «æΑ «ÞΑ ÄËæΑ Α ”«Ú˜ŽæΑ¢ŽæËÚΑ樁æΑ
”˜æ˜ÚëĘÞΑ樘Α ËƎ«½Α ×Ëõ˜ÚæüΑ ½«Ä˜ΠΑb¨˜Ú˜¢ËژΡΑ樫ÞΑ Ë×æ«ËÄΑ «ÞΑ ½˜ÞÞΑ ژ½˜õÄæΠΑ Vꁽ«æüΑ Ë¢Α ×ꍽ«ŽΑ ”«ÞæÚ«êæ«ËÄΑ
õÚ«˜ÞΑ ¢ÚËÃΑ [ææ˜Α æËΑ [ææ˜Α αΑ b¨˜Α Ùꁽ«æüΑ Ë¢Α ×ꍽ«ŽΑ ”«ÞæÚ«êæ«ËÄΑ ÞüÞæ˜ÃÞΑ ŽÄΑ «ÃׁŽæΑ 樘Α õ«½«½«æüΑ Ä”Α
ŽŽ˜ÞÞ««½«æüΑË¢Α¢Ë˔ΑÄ”ΑË樘ÚΑ˜ÞޘÄ櫁½ÞΑêæΑ”˘ÞΑÄËæΑ”«Ú˜Žæ½üΑ”˜æ˜ÚëĘΑ樘ΑËƎ«½Α×Ëõ˜ÚæüΑ½«Ä˜ÞΠΑĔΑ
ÃËژËõ˜ÚΡΑTꍽ«ŽΑ«ÞæÚ«êæ«ËÄΑ«ÞΑΑژÞ×ËÄޘΑæËΑ樘Α˜û«Þæ˜ÄŽ˜ΑË¢Α×Ëõ˜ÚæüΑÄ”ΑÞËΑ«æÞΑÞ˜ÄŽ˜ΑŽÄÄËæΑ˜Αêޘ”Α
ÞΑÄΑ«Ä”«ŽæËÚΑË¢Α×Ëõ˜ÚæüΑ½«Ä˜ΠΑBËöΡΑ½˜æϖÞΑ××½üΑ樘Α˜½«Ã«Äæ«ËÄΑ明¨Ä«Ùê˜΢Α½«Ã«Äæ˜ΑË×æ«ËÄÞΑλŽμΑÄ”Αλ”μΑÞΑ
樘üΑÚ˜Α½˜ÞÞΑ”«Ú˜Žæ½üΑژ½æ˜”ΑæËΑ樘Α”˜æ˜Úëāæ«ËÄΑË¢ΑËƎ«½Α×Ëõ˜ÚæüΑ½«Ä˜ÞΠΑr˜ΑÚ˜Α½˜¢æΑö«æ¨ΑË×æ«ËÄΑλμΑËĽüΠΑ
Hence option (b) is the correct answer.

[HfWΑ΢ ¨ææ×΢έέ×½ÄÄ«Ä£ŽËÃëÞÞ«ËÄΠÄ«ŽΠ«Äέژ×ËÚæÞέ£˜Äژ×έ×Ëõεژ×͉͉͐͐Πה¢
156

+""WBbΑHAA+bb[ΑW;bΑbHΑTHqWbxΑ+BΑ+B+
Working Group (1962)Α"ËÚΑ樘ΑĈÚÞæΑæ«Ã˜Α«ÄΑ͊͒͏͋ΑΑ×Ëõ˜ÚæüΑ½«Ä˜ΑöÞΑÙêÄæ«Ĉ˜”ΑüΑ樫ÞΑ#ÚËê×Α«ÄΑæ˜ÚÃÞΑË¢ΑΑ
ëīÃêÃΑژÙê«Ú˜Ã˜ÄæΑλ¢Ë˔ΑÄ”ΑÄËÄα¢Ë˔μΠ
Dr. Y. K. Alagh (1979): +ÄΑ+Ĕ«ΡΑHƎ«½Α×Ëõ˜ÚæüΑŽËêÄæÞΑ˜£ÄΑ¢ËÚΑ樘ΑĈÚÞæΑæ«Ã˜ΑÞ˜”ΑËÄΑ樘Α××ÚˁŽ¨ΑË¢Α
樫ÞΑbÞºΑ"Ëڎ˜Π
Lakdawala Expert Group (1993): +æΑ öÞΑ ÄΑ «Ã×ËÚæÄæΑ ûטÚæΑ #ÚËê×Α æ¨Ë꣨Α «æΑ ”«”Α ÄËæΑ ژ”˜ĈĘΑ 樘Α
×Ëõ˜ÚæüΑ½«Ä˜ΑÄ”Αژ恫ʔΑ樘ΑöËÚºΑË¢Α樘Α½£¨ΑŽËÃëææ˜Π
b˜Ä”꽺ÚΑûטÚæΑ#ÚËê×Αλ͉͉͋͒μ΢Αb¨«ÞΑŽËÃëæ昘ΑژŽËÃØĔ˜”Αޘõ˜Ú½ΑŽ¨Ä£˜ÞΑæËΑ樘ΑöüΑ×Ëõ˜ÚæüΑ
öÞΑ؁Þêژ”Α+æΑژŽËÃØĔ˜”ΑΑÞ¨«¢æΑöüΑ¢ÚËÃΑÞ«Ä£Α樘Α×Ëõ˜ÚæüΑ½«Ä˜ÞΑ¢ÚËÃΑŽ½ËÚ«˜ΑÄËÚÃÞΑêޘ”Α«ÄΑ
½½Α×Ëõ˜ÚæüΑ˜Þæ«Ãæ«ËÄÞΑޫϘΑ͊͒͐͒ΑÄ”ΑæËöÚ”ÞΑæÚ£˜æΑÄêæÚ«æ«Ëā½ΑËêæŽËØÞΑ«ÄÞ昁”
WÄ£Ú¶ÄΑ ËÃëæ昘Α λ͉͋͊͋μ΢Α Α +æΑ ŽÃ˜Α ¢æ˜ÚΑ 樘Α Α b˜Ä”꽺ÚΑ ËÃëæ昘ΠΑ b¨˜Α b˜Ä”꽺ÚΑ ŽËÃëæ昘ϐÞΑ
ځæ«Ëā½˜ΑÄ”Α××ÚˁŽ¨ΑöÞΑÙê˜Þæ«ËʔΠΑWÄ£Ú¶ÄΑËÃëæ昘ΑöÞΑ½ÞËΑ樘Αʘ”ΑË¢Α樘Α¨ËêÚΑ”ê˜ΑæËΑ
Ž¨Ä£«Ä£Αæ«Ã˜ÞΑÄ”ΑÞ׫ځæ«ËÄÞΑË¢ΑטË×½˜ΑË¢Α+Ĕ«ΡΑWÄ£Ú¶ÄΑËÃëæ昘ΑöÞΑޘæΑê×Α«ÄΑ͉͋͊͋Π
157

SKILL AQUISITION AND HUMAN


DEVELOPMENT
λ͌ΑVê˜Þæ«ËÄÞμ

Year 2017 NATIONAL SKILLS QUALIFICATION FRAMEWORK

V͊ΠΑr«æ¨Αژ¢˜Ú˜ÄŽ˜ΑæËΑϏBæ«Ëā½Α[º«½½ÞΑVꁽ«ĈŽæ«ËÄΑ"ځØöËÚºΑλB[V"μϐΡΑö¨«Ž¨ΑË¢Α樘ΑÞææ˜Ã˜ÄæÞΑ£«õ˜ÄΑ
˜½ËöΑ«ÞέÚ˜ΑŽËÚژŽæΧ
1. Under NSQF, a learner can acquire the certification for competency only through formal learning.
2. An outcome expected from the implementation of NSQF is the mobility between vocational and general education.
Select the correct answer using the code give below:
(a) 1 only
(b) 2 only
(c) Both 1 and 2
(d) Neither 1 nor 2

ANSWERS B

EXPLANATION
b¨˜ΑBæ«Ëā½Α[º«½½ÞΑVꁽ«ĈŽæ«ËÄÞΑ"ځØöËÚºΑλB[V"μΑËÚ£Ä«ā˜ÞΑÙꁽ«ĈŽæ«ËÄÞΑŽŽËڔ«Ä£ΑæËΑΑޘګ˜ÞΑË¢Α½˜õ˜½ÞΑ
Ë¢ΑºÄËö½˜”£˜ΡΑÞº«½½ÞΑÄ”Α×æ«æꔘΠΑb¨˜Þ˜Α½˜õ˜½ÞΑÚ˜Α”˜ĈʔΑ«ÄΑæ˜ÚÃÞΑË¢Α½˜ÚÄ«Ä£ΑËêæŽËØÞΑö¨«Ž¨Α樘Α½˜ÚĘÚΑ
ÃêÞæΑ×ËÞޘÞÞΑژ£Ú”½˜ÞÞΑË¢Αö¨˜æ¨˜ÚΑ樘üΑö˜Ú˜ΑŽÙê«Ú˜”Αæ¨ÚË꣨Α¢ËÚÁ½ΡΑÄËÄα¢ËÚÁ½ΑËÚΑ«Ä¢ËÚÁ½Α½˜ÚÄ«Ä£Π
+æΑ ×ÚËõ«”˜ÞΑ ¢ËÚΑ Ãê½æ«×½˜Α ׁæ¨öüÞΡΑ ¨ËÚ«āËÄ恽Α ÞΑ ö˜½½Α ÞΑ õ˜Ú櫎½ΡΑ Ëæ¨Α ö«æ¨«ÄΑ õˎæ«Ëā½Α ˜”ꎁæ«ËÄΑ Ä”Α
õˎæ«Ëā½Αæځ«Ä«Ä£ΑÄ”ΑÃËÄ£Αõˎæ«Ëā½Α˜”ꎁæ«ËÄΡΑõˎæ«Ëā½Αæځ«Ä«Ä£ΡΑ£˜Ä˜Ú½Α˜”ꎁæ«ËÄΑÄ”Α明¨Ä«Ž½Α
˜”ꎁæ«ËÄΡΑæ¨êÞΑ½«Äº«Ä£ΑËĘΑ½˜õ˜½ΑË¢Α½˜ÚÄ«Ä£ΑæËΑÄË樘ÚΑ¨«£¨˜ÚΑ½˜õ˜½ΠΑHence statement 1 is not correct and
statement 2 is correct.

ANALYSIS:
ANALYSIS:
AÄüΑ ×ÚˣځÃØÞΑ ¨”Α ˜˜ÄΑ ½êϨ˜”Α üΑ 樘Α £Ëõ˜ÚÄØÄæΑ ¢ËÚΑ Þº«½½Α ”˜õ˜½Ë×ØÄæΑ «ÄΑ 樘Α ŽËêÄæÚüΠΑ +ÄΑ 樫ÞΑ
ژ£Ú”ΡΑBV["Α¨”ΑژÁ«Ä˜”Αژ£ê½Ú½üΑ«ÄΑ樘ΑĘöÞΠ

[ŽËטΑ¢ËÚΑ˜½«Ã«Äæ«ËÄ
b¨˜ΑÞææ˜Ã˜ÄæΑ͊ΑêޘÞΑ樘ΑöËڔΑϏËĽüϐΠΑÞΑ×ÚæΑ¢ÚËÃΑ¢ËÚÁ½Αæځ«Ä«Ä£Α樘Α#Ëõ˜ÚÄØÄæΑ¨”Α½ÞËΑ恺˜ÄΑ«ÄæËΑ
ŽËÄÞ«”˜Úæ«ËÄΑ×Ú«ËÚΑ˜ûטګ˜ÄŽ˜Αˍ恫ʔΑæ¨ÚË꣨ΑË樘ÚΑ؁ÄÞΡΑ¨˜ÄŽ˜Α«æΑŽÄΑ˜Αêޘ”Α¢ËÚΑ˜½«Ã«Äæ«ËÄΠΑHĘΑ
ŽÄΑæ¨êÞΑ£˜æΑÚ«”ΑË¢ΑË×æ«ËÄΑλμΑÄ”ΑË×æ«ËÄΑλŽμΑΠ

SOURCE : ¨ææ×΢έέöööΠ樘¨«Ä”êЎËÃέ¢˜æêژÞݘ”ꎁæ«ËÄέ Ú«”£˜α樘αÞº«½½ÞαΑ£×݁Úæ«α


cle14556912.ece
¨ææ×΢έέ׫ΠÄ«ŽΠ«ÄέĘöÞ«æ˜έ×Ú«Äæژ½˜Þ˜ΠÞ×ûΧژ½«”Ͻ͎͊͐͏͌͐
¨ææ×΢έέöööΠÄޔΠ£ËõΠ«ÄέÄÞÙ¢Π¨æý
158

Year 2017 RECOGNITION OF PRIOR LEARNING

V͋ΠΑW˜ŽË£Ä«æ«ËÄΑË¢ΑTÚ«ËÚΑ;˜ÚÄ«Ä£Α[Ž¨˜Ã˜ϐΑ«ÞΑÞËØæ«Ã˜ÞΑØÄæ«ËʔΑ«ÄΑ樘ΑĘöÞΑö«æ¨Αژ¢˜Ú˜ÄŽ˜ΑæË
(a) Certifying the skills acquire by construction workers through traditional channels.
(b) Enrolling the persons in Universities for distance learning programmes.
(c) Reserving some skilled jobs to rural and urban poor in some public sector undertakings.
(d) Certifying the skills acquired by trainees under the National Skill Development Programme.

ANSWERS A

EXPLANATION
b¨˜ΑW˜ŽË£Ä«æ«ËÄΑË¢ΑTÚ«ËÚΑ;˜ÚÄ«Ä£ΑλWT;μΑ«ÞΑÄΑÞޘÞÞØÄæΑË¢Α樘ΑÞº«½½ÞΑŽÙê«Ú˜”ΑüΑ樘Α«Ä”«õ«”ꁽΑæ¨ÚË꣨Α
˜ûטګ˜ÄŽ˜ΡΑˍޘÚõæ«ËÄΑÄ”Αޘ½¢α½˜ÚÄ«Ä£ΠΑb¨˜ΑÞޘÞÞØÄæΡΑö¨«Ž¨Α«ÞΑ¢Ë½½Ëö˜”ΑüΑΑŽ˜Úæ«ĈŽæ«ËÄΡΑ£«õ˜ÞΑÄΑ
˜”£˜ΑæËΑÄΑË樘Úö«Þ˜Α«Ä¢ËÚÁ½ΑöËÚº˜ÚΠΑ+æΑ×ÚËõ«”˜ÞΑ樘Α˜Ã×½Ëü˜˜Αö«æ¨Α樘ΑŽËÄĈ”˜ÄŽ˜ΡΑÞˎ«½ΑژŽË£Ä«æ«ËÄΑ
Ä”Α ˜Ã×Ëö˜ÚØÄæΑ 樁æΑ Ú˜Α ʎ˜ÞށÚüΑ æËΑ Ę£Ë櫁æ˜Α ¨«ÞΑ ¢êæêژΑ ˜Ã×½ËüØÄæΠΑ "˽½Ëö«Ä£Α Þꎎ˜ÞÞ¢ê½Α
ÞޘÞÞØÄæΡΑΑŽÄ”«”æ˜Α«ÞΑ£«õ˜ÄΑΑĈāĎ«½ΑژöÚ”ΡΑ«ÄΑ””«æ«ËÄΑæËΑŽ˜Úæ«ĈŽæ«ËÄΠΑHence option (a) is the
correct answer.

ANALYSIS:
ANALYSIS:
W˜Ž˜Äæ½üΡΑλÚËêĔΑ͉͋͊͐μΑ+;Ϟ"[Α[º«½½ÞΑ˜õ˜½Ë×ØÄæΑËÚ×Ëځæ«ËÄΑ;æ”Αλ+;Ϟ"[Α[º«½½ÞμΑΑ¶Ë«ÄæΑ«Ä«æ«æ«õ˜ΑË¢Α+;Ϟ"[Α
”ꎁæ«ËÄΑ ϞΑ Bæ«Ëā½Α [º«½½ÞΑ ˜õ˜½Ë×ØÄæΑ ËÚ×Ëځæ«ËÄΑ λB[μΑ ½êϨ˜”Α W˜ŽË£Ä«æ«ËÄΑ Ë¢Α TÚ«ËÚΑ ;˜ÚÄ«Ä£Α
λWT;μΑ×ÚˣځÃØΑ«ÄΑ樘Α×ׁژ½ΑޘŽæËÚΑêĔ˜ÚΑTځ”¨ÄΑAÄæÚ«Α9êÞ¨½Αq«ºÞΑx˶ÄΑλTA9qxμΑ͋Π͉Π

SOURCE : ¨ææ×΢έέöööΠ樘¨«Ä”êЎËÃݘ”ꎁæ«ËÄݎÚ˜˜ÚÞέAÞÞ«õ˜α«Äαގ½˜αÄ”αΑ«ÄÄËõα
æ«õ˜α«Äα××ÚˁŽ¨έÚ櫎½˜͊͐͋͑͐͒͊͌ИŽ˜

[ŽËטΑ¢ËÚΑ˜½«Ã«Äæ«ËÄ
PLEASE NOTE:Αb¨«ÞΑÙê˜Þæ«ËÄΑλژ½æ˜”ΑæËΑ[º«½½Α˜õ˜½Ë×ØÄæμΑÄ”Α樘ΑÙê˜Þæ«ËÄΑËõ˜Α끽ÞËΑژ½æ˜”ΑæËΑ[º«½½Α
˜õ˜½Ë×ØÄæμΑÚ˜Α½«Äº˜”ΠΑĔΑËæ¨Α樘ΑÙê˜Þæ«ËÄÞΑ¢˜æêژ”Α«ÄΑ樘ΑށØΑü˜ÚΑλ͉͋͊͐μΠΑΑb¨˜ΑĈÚÞæΑÞææ˜Ã˜ÄæΑ
Ë¢Α 樘Α ×ژõ«ËêÞΑ Ùê˜Þæ«ËÄΑ «ÞΑ Þ˜”Α ËÄΑ 樘Α ށØΑ «”˜Α ÞΑ 樘Α W˜ŽË£Ä«æ«ËÄΑ Ë¢Α TÚ«ËÚΑ ;˜ÚÄ«Ä£ΠΑ +¢Α 樘Α
£Ëõ˜ÚÄØÄæΑ ژŽË£Ä«ā˜ÞΑ ×Ú«ËÚΑ ½˜ÚÄ«Ä£ΡΑ樘ÄΑ ¨ËöΑ ŽÄΑ樘ΑĈÚÞæΑ Þææ˜Ã˜ÄæΑ Ë¢Α樘Α ×ژõ«ËêÞΑ Ùê˜Þæ«ËÄΑ ˜Α
ŽËÚژŽæΥΧ
159

Year 2018 HUMAN CAPITAL DEVELOPMENT

V͌ΠΑ(êÁÄΑŽ×«æ½Α¢ËÚÁæ«ËÄΑÞΑΑŽËϘ×æΑ«ÞΑ˜ææ˜ÚΑ˜û×½«Ä˜”Α«ÄΑæ˜ÚÃÞΑË¢ΑΑ×Úˎ˜ÞÞΑö¨«Ž¨Α˜Ä½˜ÞΑ
1. individuals of a country to accumulate more capital.
2. increasing the knowledge, skill levels and capacities of the people of the country.
3. accumulation of tangible wealth.
4. accumulation of intangible wealth.
Which of the statements given above is/are correct?
(a) 1 and 2 only
(b) 2 only
(c) 2 and 4 only
(d) 1, 3 and 4 only

ANSWERS C

EXPLANATION
(êÁÄΑŽ×«æ½Α¢ËÚÁæ«ËÄΑ«Ä”«Žæ˜ÞΡΑύ樘Α×Úˎ˜ÞÞΑË¢ΑŽÙê«Ú«Ä£ΑÄ”Α«ÄŽÚ˜Þ«Ä£Α樘ΑÄê͘ÚΑË¢ΑטÚÞËÄÞΑö¨ËΑ
¨õ˜Α樘ΑÞº«½½ÞΡΑ˜”ꎁæ«ËÄΑÄ”Α˜ûטګ˜ÄŽ˜Αö¨«Ž¨ΑÚ˜ΑŽÚ«æ«Ž½Α¢ËÚΑ樘Α˜ŽËÄËëŽΑÄ”Α樘Α×˽«æ«Ž½Α”˜õ˜½Ë×ØÄæΑ
Ë¢Α樘ΑŽËêÄæÚüΠΑΑHence option 1 is not the correct answer.
(êÁÄΑŽ×«æ½Α¢ËÚÁæ«ËÄΑ«ÞΑæ¨êÞΑÞÞˎ«æ˜”Αö«æ¨Α«Äõ˜ÞæØÄæΑ«ÄΑÁÄΑÄ”Α¨«ÞΑ”˜õ˜½Ë×ØÄæΑÞΑΑŽÚ˜æ«õ˜ΑÄ”Α
×Ú˔êŽæ«õ˜ΑژÞËêڎ˜ΠώΑHence option 2 is correct and option 3 is not correct.
+ÄæÄ£«½˜Α ö˜½æ¨Α ŽËÄÞ«ÞæÞΑ Ë¢Α ¢ŽæËÚÞΑ ÞꎨΑ ÞΑ 樘Α æÚêÞæΑ ÃËÄ£Α טË×½˜Α «ÄΑ Α Þˎ«˜æüΡΑ ÄΑ ˜ĆŽ«˜ÄæΑ ¶ê”«Ž«½Α
ÞüÞæ˜ÃΡΑ Ž½˜ÚΑ ×ÚËטÚæüΑ Ú«£¨æÞΡΑ ˜ą˜Žæ«õ˜Α £Ëõ˜ÚÄØÄæΡΑ Ä”Α £Ë˔Α ˜”ꎁæ«ËÄΑ ÞüÞæ˜ÃΑ ˜æŽΠΑ (êÁÄΑ Ž×«æ½Α
¢ËÚÁæ«ËÄΑ˜Ä½˜ÞΑŽŽêÃ꽁æ«ËÄΑË¢Α«ÄæÄ£«½˜Αö˜½æ¨ΠΑHence option 4 is correct.

ANALYSIS:
ANALYSIS:

[ŽËטΑ¢ËÚΑ˜½«Ã«Äæ«ËÄ
+æΑ«ÞΑÙê«æ˜Α½Ë£«Ž½Α樁æΑË×æ«ËÄÞΑ͌ΑÄ”Α͍ΑŽÄÄËæΑ˜ΑËæ¨ΑŽËÚژŽæΑæË£˜æ¨˜ÚΠ

SOURCE : +Ĕ«ÄΑŽËÄËëŽΑ˜õ˜½Ë×ØÄæΑλBWbΑ½ÞÞΑ͊͊μ
ûŽ˜Ú×æΑ¢ÚËÃΑBWb 160
161

LABOUR AND EMPLOYMENT


λ͌ΑVê˜Þæ«ËÄÞμ

Year 2017 NATIONAL CAREER SERVICES

V͊ΠΑËÄÞ«”˜ÚΑ樘Α¢Ë½½Ëö«Ä£Α«ÄΑژÞטŽæΑË¢ΑϏBæ«Ëā½ΑÚ˜˜ÚΑ[˜Úõ«Ž˜Þϐ΢
1. National Career Service is an initiative of the Department of Personnel and Training, Government of India.
2. National Career Service has been launched in a Mission Mode to improve the employment opportunities to
uneducated youth of the country.
Which of the above statements is/are correct?
(a) 1 only
(b) 2 only
(c) Both 1 and 2
(d) Neither 1 nor 2

ANSWERS D

EXPLANATION

Bæ«Ëā½ΑÚ˜˜ÚΑ[˜Úõ«Ž˜ΑλB[μΑ×Ú˶˜ŽæΑ«ÞΑÄΑ«Ä«æ«æ«õ˜Α½êϨ˜”ΑüΑ樘ΑA«Ä«ÞæÚüΑË¢Α;ËêÚΑÄ”ΑÃ×½ËüØÄæΑ
λ+Ĕ«μΑ#Ëõ˜ÚÄØÄæΑË¢Α+Ĕ«ΑÞΑΑA«ÞÞ«ËÄΑA˔˜ΑTÚ˶˜ŽæΑ¢ËÚΑ˜Þ恍½«Þ¨«Ä£ΑÙ꫎ºΑÄ”Α˜ĆŽ«˜ÄæΑŽÚ˜˜ÚΑژ½æ˜”Α
ޘÚõ«Ž˜ÞΠ Hence statement 1 is not correct.
+æÞΑ×êÚ×ËޘΑ«ÞΑæËΑ×ÚËõ«”˜ΑΑõÚ«˜æüΑË¢Α˜Ã×½ËüØÄæαژ½æ˜”ΑޘÚõ«Ž˜ÞΡΑÞꎨΑÞΑŽÚ˜˜ÚΑŽËêÄޘ½½«Ä£ΡΑõˎæ«Ëā½Α
£ê«”ÄŽ˜ΡΑÄ”Α«Ä¢ËÚÁæ«ËÄΑËÄΑÞº«½½Α”˜õ˜½Ë×ØÄæΑŽËêÚޘÞΡΑæËΑ¶ËΑޘ˜º˜ÚÞΑÄ”Α˜Ã×½Ëü˜ÚÞΑ«ÄΑ樘ΑŽËêÄæÚüΠΑ+æΑ
«ÃÞΑæËΑ«Ã×ÚËõ˜Α樘Α˜Ã×½ËüØÄæΑË××ËÚæêī櫘ÞΑ¢ËÚΑ½½Α¶ËΑޘ˜º˜ÚÞΡΑÄËæΑ¶êÞæΑ樘ΑêĘ”êŽæ˜”ΑüËêæ¨ΠΑHence
statement 2 is not correct.

ANALYSIS:
ANALYSIS:

[ŽËטΑ¢ËÚΑ˜½«Ã«Äæ«ËÄ
+¢Α ËĘΑ ºÄËöÞΑ ö˜½½Α ö¨æΑ 樘Α ˜ûŽæΑ Ú˽˜Α Ë¢Α ËTbΑ «ÞΑ λfT[Α Þ׫ځÄæÞΑ ʘ”Α æËΑ ºÄËöΑ 樁æμΡΑ 樘ÄΑ ËĘΑ ŽÄΑ
˜½«Ã«Äæ˜ΑË×æ«ËÄÞΑλμΑÄ”ΑλŽμΠΑæΑ樘ΑށØΑæ«Ã˜ΡΑ«ÄΑÞææ˜Ã˜ÄæΑ͋ΡΑϏêĘ”êŽæ˜”ΑüËêæ¨ϐΑ«ÞΑΑ«Α˜ÄË꣨ΑŽ½ê˜Α
λæ¨Ë꣨Α樫ÞΑÞÞêÃ×æ«ËÄΑŽÚÚ«˜ÞΑÞËØΑÃËêÄæΑË¢ΑÚ«ÞºμΠ

SOURCE : ¨ææ×΢έέ׫ΠÄ«ŽΠ«ÄέĘöÞ«æ˜έTÚ«ÄæW˜½˜Þ˜ΠÞ×ûΧژ½«”Ͻ͎͍͍͊͒͑
¨ææ×Þ΢έέöööΠĎÞΠ£ËõΠ«ÄέT£˜Þέ”˜¢ê½æЁÞ×û
¨ææ×΢έέ”£˜æΠÄ«ŽΠ«ÄݎËÄæ˜ÄæέÞæꔘÄæÞέāæ«Ëā½αŽÚ˜˜ÚαޘÚõ«Ž˜Πר×
162

Year 2019 EMPLOYMENT

V͋ΠΑËÄÞ«”˜ÚΑ樘Α¢Ë½½Ëö«Ä£ΑÞææ˜Ã˜ÄæÞ΢
As per the Industrial Employment (Standing Orders) Central (Amendment) Rules, 2018-
1. if rules for fixed-term employment are implemented, it becomes easier for the firms/companies to lay off workers
2. no notice of termination of employment shall be necessary in the case of temporary workman
Which of the statements given above is/are correct?
(a) 1 only
(b) 2 only
(c) Both 1 and 2
(d) Neither 1 nor 2

ANSWERS C

EXPLANATION
Statement 1 is correct:Αb¨˜Α£Ëõ˜ÚÄØÄæΑ¨ÞΑÄËæ«Ĉ˜”ΑĈû˜”Αæ˜ÚÃΑ˜Ã×½ËüØÄæΑ¢ËÚΑ½½ΑޘŽæËÚÞΑæ¨ÚË꣨ΑÄΑ
Ã˜Ä”ØÄæΑæËΑ樘Α+ĔêÞæÚ«½ΑÃ×½ËüØÄæΑλ[æÄ”«Ä£ΑHڔ˜ÚÞμΑ˜Äæځ½ΑW꽘ÞΡΑ͍͊͒͏ΠΑ"«û˜”αæ˜ÚÃΑ˜Ã×½ËüØÄæΑ
¢ËÚΑ½½ΑޘŽæËÚÞΑö«½½ΑÁº˜Α«æΑ˜Þ«˜ÚΑ¢ËÚΑŽËÃׁī˜ÞΑæËΑ¨«Ú˜αÄ”αĈژΑöËÚº˜ÚÞΑ½ËÄ£Αö«æ¨Αژ”ꎫģΑ樘ΑÚ˽˜ΑË¢Α
디½˜Ã˜ÄΠ
Statement 2 is correct:Α BËΑ ÄË櫎˜Α Ë¢Α æ˜Úëāæ«ËÄΑ Ë¢Α ˜Ã×½ËüØÄæΑ Þ¨½½Α ˜Α ʎ˜ÞށÚüΑ «ÄΑ 樘Α ŽÞ˜Α Ë¢Α
æ˜Ã×ËځÚüΑ öËںÁÄΑ ö¨˜æ¨˜ÚΑ ÃËÄ樽üΑ ځ昔ΡΑ ö˜˜º½üΑ ځ昔Α ËÚΑ ׫˜Ž˜Α ځ昔Α Ä”Α ×Úˍæ«ËĘÚÞΑ ËÚΑ ”½«Α
öËںØÄΠ

ANALYSIS:
ANALYSIS:

SOURCE : ¨ææ×Þ΢έέ«Ä”«Ä˜û×ژÞÞЎËÃ݁Ú櫎½˜έêޫĘÞÞݘŽËÄËÃüέ«Ä”êÞæÚ«½α˜Ã×½ËüØÄæαΑÞæÄ”α
«Ä£ΑαËڔ˜ÚÞ᎘Äæځ½αÚ꽘Þα͍͊͒͏αÃ˜Ä”ØÄæα˜Þ˜Þα¨«Ú˜αÄ”αĈژαæËαژα
”ꎘα디½˜ÃÄαÚ˽˜αΑ͎͉͊͌͐͏͎έ
163

Year 2021 CASUAL WORKERS

V͌ΠΑr«æ¨Αژ¢˜Ú˜ÄŽ˜ΑæËΑŽÞꁽΑöËÚº˜ÚÞΑ˜Ã×½Ëü˜”Α«ÄΑ+Ĕ«ΡΑŽËÄÞ«”˜ÚΑ樘Α¢Ë½½Ëö«Ä£ΑÞææ˜Ã˜ÄæÞ΢
1. All casual workers are entitled for Employees Provident Fund coverage.
2. All casual workers are entitled for regular working hours and overtime payment.
3. The government can by a notification specify that an establishment or industry shall pay wages only through its bank
account.
Which of the above statements are correct?
(a) 1 and 2 only
(b) 2 and 3 only
(c) 1 and 3 only
(d) 1, 2 and 3

ANSWERS B

EXPLANATION
T½˜Þ˜ΑÄËæΑæΑĈÚÞæΑ£½ÄŽ˜ΡΑ½½Α樘Αæ¨Ú˜˜ΑÞææ˜Ã˜ÄæÞΑޘ˜ÃΑŽËÚژŽæΡΑêæΑfT[Αژ£Ú”˜”ΑÞææ˜Ã˜ÄæΑ͊ΑÞΑ
incorrect.
(˽”«Ä£Α 樁æΑ ÄΑ ˜Ã×½Ëü˜ÚΑ ŽÄÄËæΑ ”«ą˜Ú˜Ä櫁æ˜Α ˜æö˜˜ÄΑ ŽËÄæځŽæꁽΑ Ä”Α טÚÁĘÄæΑ ˜Ã×½Ëü˜˜ÞΡΑ 樘Α
[ê×ژØΑËêÚæΑ«ÄΑTöÄΑ(ÄÞΑ;«Ã«æ˜”ΑϞΑHÚÞΠΑqÞΑõ«æ«ËÄΑ9ÚÁŽ¨Ú«Α[Ä£¨æÄΑ¨ÞΑÚ꽘”Α樁æΑŽÞꁽΑ
öËÚº˜ÚÞΑ Ú˜Α ½ÞËΑ ˜Äæ«æ½˜”Α æËΑ Þˎ«½Α ޘŽêÚ«æüΑ ˜Ä˜ĈæÞΑ êĔ˜ÚΑ 樘Α Ã×½Ëü˜˜ÞϐΑ TÚËõ«”˜ÄæΑ "êĔÞΑ Ä”Α
A«ÞŽ˜½½Ä˜ËêÞΑTÚËõ«Þ«ËÄÞΑŽæΠΑ êæΡΑ樘ΑŽÚ«æ˜Ú«ËÄΑË¢ΑëīÃêÃΑË¢Α͉͋Α˜Ã×½Ëü˜˜ÞΑ«ÞΑʎ˜ÞށÚüΠΑb¨æΑ«ÞΑö¨æΑ
fT[ΑÃêÞæΑ¨õ˜ΑŽËÄÞ«”˜Ú˜”ΠΑ
˔˜ΑËÄΑ[ˎ«½Α[˜ŽêÚ«æüΡΑ͉͉͋͋ΑαΑb¨˜Α˔˜Αژ׽Ž˜ÞΑīĘΑ½öÞΑژ½æ˜”ΑæËΑÞˎ«½ΑޘŽêÚ«æüΠΑb¨˜Þ˜Α«ÄŽ½ê”˜Α樘Α
Ã×½Ëü˜˜ÞϐΑTÚËõ«”˜ÄæΑ"êĔΑŽæΡΑ͎͊͒͋ΡΑ樘ΑAæ˜ÚÄ«æüΑ ˜Ä˜ĈæΑŽæΡΑ͊͒͏͊ΡΑÄ”Α樘ΑfÄËÚ£Ä«Þ˜”ΑrËÚº˜ÚÞϐΑ[ˎ«½Α
[˜ŽêÚ«æüΑŽæΡΑ͉͉͋͑ΑÃËÄ£ΑË樘ÚÞΠΑb¨˜ΑŽË”˜Α«ÞΑ××½«Ž½˜ΑæËΑÄüΑ˜Þ恍½«Þ¨Ã˜ÄæΑλÞꍶ˜ŽæΑæËΑÞ«ā˜ΑαΑæ¨Ú˜Þ¨Ë½”Α
ÞΑÁüΑ˜ΑÄËæ«Ĉ˜”ΑüΑ樘ΑŽ˜Äæځ½Α£Ëõ˜ÚÄØÄæμΠΑ
[ˎ«½ΑޘŽêÚ«æüΑ¢êĔ΢Αb¨˜ΑŽË”˜ΑÞææ˜ÞΑ樁æΑ樘ΑŽ˜Äæځ½Α£Ëõ˜ÚÄØÄæΑö«½½ΑޘæΑê×ΑÞꎨΑΑ¢êĔΑ¢ËÚΑêÄËÚ£Ä«ā˜”Α
öËÚº˜ÚÞΡΑ£«£ΑöËÚº˜ÚÞΑÄ”Α×½æ¢ËÚÃΑöËÚº˜ÚÞΠΑ"êÚ樘ÚΡΑÞææ˜Α£Ëõ˜ÚÄØÄæÞΑö«½½Α½ÞËΑޘæΑê×ΑÄ”Α”ëīÞæ˜ÚΑ
ޘׁځæ˜ΑÞˎ«½ΑޘŽêÚ«æüΑ¢êĔÞΑ¢ËÚΑêÄËÚ£Ä«ā˜”ΑöËÚº˜ÚÞΠΑ˔˜ΑËÄΑr£˜ÞΡΑ͉͉͋͋ΑΡ[˜Žæ«ËÄΑ͏ΑϞΑ͐ΑÚ˜Αژ½æ˜”ΑæËΑ
Äê͘ÚΑ Ë¢Α ژ£ê½ÚΑ öËÚº«Ä£Α ¨ËêÚÞΑ Ä”Α r˜˜º½üΑ ”üΑ Ë¢Α ژÞæΑ ϞΑ Ëõ˜Úæ«Ã˜Α ×ÚËõ«Þ«ËÄÞΑ ژÞטŽæ«õ˜½üΠ Hence
statements 1 is not correct and statement 2 is correct.
b¨˜Α TüØÄæΑ Ë¢Α r£˜ÞΑ ŽæΡΑ ͊͒͌͏Α ¨ÞΑ ˜˜ÄΑ Ã˜Ä”˜”Α üΑ TüØÄæΑ Ë¢Α r£˜ÞΑ λØĔØÄæμΑ ŽæΡΑ ͉͋͊͐Α
λ˜ą˜Žæ«õ˜Α¢ÚËÃΑ ͋͑Π͊͋Π͉͋͊͏μΑæËΑ ˜Ä½˜Α樘Α ˜Ã×½Ëü˜ÚÞΑæËΑ ׁüΑö£˜ÞΑæËΑ樘«ÚΑ ˜Ã×½Ëü˜˜ÞΑ üΑ λμΑ ŽÞ¨Α ËÚΑ λμΑ
Ž¨˜Ùê˜Α ËÚΑ λŽμΑ ŽÚ˜”«æ«Ä£Α æËΑ 樘«ÚΑ ÄºΑ ŽŽËêÄæΑ b¨˜Α Ã˜Ä”ØÄæΑ «ÄΑ 樘ΑŽæΑ ½ÞËΑ ˜Ä½˜ÞΑ 樘Α ××ÚË×Ú«æ˜Α
#Ëõ˜ÚÄØÄæΑæËΑÞטŽ«¢üΑ樘Α«Ä”êÞæÚ«½ΑËÚΑË樘ÚΑ˜Þ恍½«Þ¨Ã˜ÄæΡΑüΑÄËæ«ĈŽæ«ËÄΑ«ÄΑ樘ΑHƎ«½Α#ā˜ææ˜ΡΑö¨«Ž¨Α
Þ¨½½ΑׁüΑæËΑ˜õ˜ÚüΑטÚÞËÄΑ˜Ã×½Ëü˜”Α«ÄΑÞꎨΑ«Ä”êÞæÚ«½ΑËÚΑË樘ÚΑ˜Þ恍½«Þ¨Ã˜ÄæΡΑ樘Αö£˜ÞΑËĽüΑüΑŽ¨˜Ùê˜ΑËÚΑ
üΑŽÚ˜”«æ«Ä£Α«ÄΑ¨«ÞΑÄºΑŽŽËêÄæΠ Hence statement 3 is correct.

ANALYSIS:
ANALYSIS:

[ŽËטΑ¢ËÚΑ˜½«Ã«Äæ«ËÄ
Αb¨˜ΑÞææ˜Ã˜ÄæÞΑÚ˜Α¢ŽæꁽΑ«ÄΑāæêژΑÄ”Α¨˜ÄŽ˜Α樘ژΑ«ÞΑÄËΑގËטΑ¢ËÚΑ˜½«Ã«Äæ«ËÄΠ

SOURCE : ¨ææ×Þ΢έέöööИ×ĈĔ«Π£ËõΠ«ÄέÞ«æ˜ε”ËŽÞέT"ÞέËöĽˁ”ÞεT"ÞέT"Žæ͎͊͒͋Πה¢
164

Year 2022 UNEMPLOYMENT

V͍ΠΑ+ÄΑ+Ĕ«ΡΑö¨«Ž¨ΑËĘΑË¢Α樘Α¢Ë½½Ëö«Ä£ΑŽËÃ׫½˜ÞΑ«Ä¢ËÚÁæ«ËÄΑËÄΑ«Ä”êÞæÚ«½Α”«Þ×êæ˜ÞΡΑŽ½ËÞêژÞΡΑژæژϨα
ØÄæÞΑÄ”Α½üαËąÞΑ«ÄΑ¢ŽæËÚ«˜ÞΑ˜Ã×½Ëü«Ä£ΑöËÚº˜ÚÞΧ
(a) Central Statistics Office
(b) Department for Promotion of Industry and Internal Trade
(c) Labour Bureau
(d) National Technical Manpower Information System

ANSWERS C

EXPLANATION
;ËêÚΑ êژêΑ ¨ÞΑ ˜˜ÄΑ ˜Ä££˜”Α «ÄΑ ŽË½½˜Žæ«ËÄΡΑ ŽËÃ׫½æ«ËÄΡΑ Ä½üÞ«ÞΑ Ä”Α ”«Þޘëāæ«ËÄΑ Ë¢Α ½ËêÚΑ
Þææ«Þ櫎ÞΑæΑ½½Α+Ĕ«ΑÄ”Α[ææ˜Α½˜õ˜½ΑËÄΑ”«ą˜Ú˜ÄæΑ¢Ž˜æÞΑË¢Α½ËêÚΑޫϘΑ«æÞΑ«ÄŽ˜×æ«ËÄΑ«ÄΑ樘Αü˜ÚΑ͉͊͒͋ΠΑ+æΑ
ژ½˜Þ˜ÞΑ ÄÄꁽΑ ×ꍽ«Žæ«ËÄΑ æ«æ½˜”ΡΑ Ϗ+ĔêÞæÚ«½Α «Þ×êæ˜ÞΡΑ ½ËÞêژÞΡΑ W˜æژĎ¨Ã˜ÄæÞΑ Ä”Α ;üαHąÞΑ «ÄΑ
«Ä”êÞæÚ«˜ÞΑ«ÄΑ+Ĕ«ϐΠΑ+æΑ×ÚËõ«”˜ÞΑÞææ«Þ櫎ÞΑËÄΑöËÚºαÞæË×ׁ£˜ÞΡΑŽ½ËÞêژÞΡΑژæژĎ¨Ã˜ÄæÞΑÄ”Α½ËŽºËêæÞΑö¨«Ž¨Α
«ÞΑêޘ¢ê½Α«ÄΑÁº«Ä£Α×˽«Ž«˜ÞΑ¢ËÚΑÁ«Ä恫īģΑŽËڔ«½ΑÄ”Α¨ÚÃËÄ«ËêÞΑژ½æ«ËÄÞΑ˜æö˜˜ÄΑ樘ΑÁā£˜Ã˜ÄæΑ
Ä”Α樘ΑöËÚº˜ÚÞΠΑb¨«ÞΑژ×ËÚæΑ«ÞΑ½ÞËΑ«Ã˜”ΑæΑؘæ«Ä£Α樘Α”˜ÃÄ”Α¢ËÚΑ¨«ÞæËÚ«Ž½Α”æΑÄ”Α«Ä¢ËÚÁæ«ËÄΑËÄΑ
«Ä”êÞæÚ«½Α ŽËÄĉ«ŽæÞΑ Ä”Α 樘«ÚΑ ŽêޘÞΑ üΑ ×½ÄĘÚÞΡΑ ×˽«ŽüÁº˜ÚÞΑ Ä”Α ”˜Ž«Þ«ËÄαÁº˜ÚÞΑ «ÄΑ «Ä”êÞæÚüΑ Ä”Α
£Ëõ˜ÚÄØÄæΠΑHence option (c) is the correct answer.

ANALYSIS:
ANALYSIS:

[ŽËטΑ¢ËÚΑ˜½«Ã«Äæ«ËÄ
B

SOURCE : ¨ææ×Þ΢έέ½ËêÚΠ£ËõΠ«ÄέÞ«æ˜Þέ”˜¢ê½æέĈ½˜ÞέÄÄꁽεW˜×ËÚæε͉͉͋͋͋͊εÄ£½«Þ¨Πה¢

¨ææ×΢έέöööΠ½ËêڍêژêĘöΠ£ËõΠ«ÄέfޘÚËÄæ˜Äæέ+εW˜α
õ«˜öε͉͍͋͊Πה¢Χ×Úε«”Ͻ"¢W͏ÃÄ͍wfqА͌

¨ææ×Þ΢έݘŽËÄËëŽæ«Ã˜ÞΠ«Ä”«æ«Ã˜ÞЎËÃέĘöÞݘŽËÄËÃüέ«Ä”«Žα
æËÚÞέ͎͎͌͐αöËÚº˜ÚÞαą˜Žæ˜”αüὁüαËąÞ၎ÚËÞÞα«Ä”«α˜æö˜˜ÄᶁÄêÚüαÄ”α¶êÄ˜α½
ËêÚαëīÞæÚüᔁæέÚ櫎½˜Þ¨Ëöέ͐͑͋͌͋͏͑͊ЎÃÞΧ¢ÚËÃϽÔÚ
165

SOCIAL SECURITY
λ͌ΑVê˜Þæ«ËÄÞμ

Year 2017 NATIONAL PENSION SYSTEM

V͊ΠΑr¨ËΑÃËÄ£Α樘Α¢Ë½½Ëö«Ä£ΑŽÄΑ¶Ë«ÄΑ樘ΑBæ«Ëā½ΑT˜ÄÞ«ËÄΑ[üÞæ˜ÃΑλBT[μΧ
(a) Resident Indian citizens only
(b) Persons of age from 21 to 55 only
(c) All State Government employees joining the services after the state of notification by the respective State
Governments
(d) All Central Government employees including those of Armed Forces joining the services on or after 1st April, 2004

ANSWERS C

EXPLANATION
NPS (National Pension System) is a defined contribution based Pension Scheme launched by Government of India.
½«£««½«æü΢
NPS is applicable to all new employees of Central Government service (except Armed Forces) and Central
Autonomous Bodies joining Government service on or after 1st January 2004.
NPS is applicable to all the employees of State Governments, State Autonomous Bodies joining services after the
date of notification by the respective State Governments.
All citizens of India between the age of 18 and 60 years as on the date of submission of his / her application to Point
of Presence (POP) / Point of Presence-Service Provider (POP-SP) can join NPS. Recently, NRIs were also allowed to
open National Pension Scheme (NPS) accounts online.
Hence option (c) is the correct answer.

ANALYSIS:
ANALYSIS:
b¨˜Αގ¨˜Ã˜Α«ÞΑËĘΑË¢Α樘ΑÃËÞæΑ«Ã×ËÚæÄæΑގ¨˜Ã˜ÞΑ«ÄΑT˜ÄÞ«ËÄΑ[˜ŽæËÚΑÄ”Αæ¨êÞΑ¨Ë½”ÞΑ«Ã×ËÚæÄŽ˜Α¢ÚËÃΑ
樫ÞΑטÚÞטŽæ«õ˜ΑÞΑö˜½

[ŽËטΑ¢ËÚΑ˜½«Ã«Äæ«ËÄ
NA

SOURCE : ¨ææ×Þ΢έέ«Ä”«Π£ËõΠ«ÄέÞ×Ë潫£¨æέāæ«Ëā½αטÄÞ«ËÄαÞüÞæ˜Ãαژæ«Ú˜Ã˜Äæα×½Äၽ½Α
¨ææ×΢έέöööΠ樘¨«Ä”êЎËÃݍêޫĘÞÞέŽËÄËÃüέBW+ÞᎁÄαÄËöα××½üαËĽ«Ä˜αΑ¢ËÚαBα
æ«Ëā½αT˜ÄÞ«ËÄα[Ž¨˜Ã˜έÚ櫎½˜͍͍͊͋͑͊͋͒ИŽ˜Α
166

Year 2019 NFSA, 2013

V͋ΠΑr«æ¨Αژ¢˜Ú˜ÄŽ˜ΑæËΑ樘Α×ÚËõ«Þ«ËÄÞΑÁ”˜ΑêĔ˜ÚΑ樘ΑBæ«Ëā½Α"Ë˔Α[˜ŽêÚ«æüΑŽæΡΑ͉͋͊͌ΡΑŽËÄÞ«”˜ÚΑ樘Α
¢Ë½½Ëö«Ä£ΑÞææ˜Ã˜ÄæÞ΢
1. The families coming under the category of 'Below Poverty Line (BPL)' only are eligible to receive subsidies food
grains.
2. The eldest woman in a household, of age 18 years or above, shall be the head of the household for the purpose of
issuance of a ration card.
3. Pregnant women and lactating mothers are entitled to a 'take-home ration' of 1600 calories per day during
pregnancy and for six months thereafter.
Which of the statements given above is/are correct?
(a) 1 and 2 only
(b) 2 only
(c) 1 and 3 only
(d) 3 only

ANSWERS B

EXPLANATION

Statement 1 is not correct: The Act provides for coverage of upto 75% of the rural population and upto 50% of the
urban population for receiving subsidized foodgrains under Targeted Public Distribution System (TPDS), thus covering
about two-thirds of the population.
Statement 2 is correct: It provides that eldest woman of the household of age 18 years or above is to be the head
of the household for the purpose of issuing of ration cards.
Statement 3 is not correct: The official Act does not mention anything about the calories intake. Besides meal to
pregnant women and lactating mothers during pregnancy and six months after the child birth, such women will also
be entitled to receive maternity benefit of not less than Rs. 6,000.

ANALYSIS:
ANALYSIS:

One of the most important social security schemes along with MGNREGA.It can be asked any year (in a similar or
different form)

[ŽËטΑ¢ËÚΑ˜½«Ã«Äæ«ËÄ
In the first statement, the word ‘only’ can be used as a clue to reach the incorrectness of the same. As in South Asia in
general and India in particular, many people live on a slippery slope (those whore not under the poverty line but can
fall below the same; food security is a challenge for them as well) Hence statement 1 can be eliminated. One will be
left with two options (further logical elimination is not possible, as some amount of factual knowledge is required).

SOURCE : ¨ææ×Þ΢έέõ«ºÞט”«Π«ÄέĘöÞέÞææ˜αځĺ«Ä£α«Ä”˜ûα¢ËÚαÄ¢Þα½êϨ˜”ά΢Ѕ΢æ˜ûæϽb¨˜Α
А͉͋Bæ«Ëā½А͉͋"Ë˔А͉͋[˜ŽêÚ«æüА͉͋ŽæΡêĔ˜ÚА͉͋bÚ£˜æ˜”А͉͋Têα
½«ŽА͉͋«ÞæÚ«êæ«ËÄА͉͋[üÞæ˜ÃΠ
167

Year 2023 SELF-HELP GROUPS

V͌ΠΑËÄÞ«”˜ÚΑ樘Α¢Ë½½Ëö«Ä£ΑÞææ˜Ã˜ÄæÞΑ΢Α
1. The Self-Help Group (SHG) programme was originally initiated by the State Bank of India by providing microcredit to
the financially deprived.
2. In an SHG, all members of a group take responsibility for a loan that an individual member takes.
3. The Regional Rural Banks and Scheduled Commercial banks support SHGs.
How many of the above statements are correct?
(a) Only one
(b) Only two
(c) All three
(d) None

ANSWERS B

EXPLANATION

b¨˜ΑËÚ«£«ÄΑË¢Αޘ½¢α¨˜½×Α£ÚËê×ΑŽÄΑ˜ΑæځŽ˜”Α¢ÚËÃΑ#ځؘÄΑ ÄºΑË¢Α Ä£½”˜Þ¨ΡΑö¨«Ž¨ΑöÞΑ¢ËêĔ˜”ΑüΑ


A˨Ã˜”ΑxêÄêÞΠΑ[#(ÞΑö˜Ú˜ΑÞæÚ昔ΑÄ”Α¢ËÚؔΑ«ÄΑ͎͊͒͐ΠΑ+ÄΑ+Ĕ«ΑB WΑ«Ä«æ«æ˜”Α[(#[Α«ÄΑ͊͒͑͏α͊͒͑͐ΠΑ
b¨˜ΑÞ˜ÄŽ˜ΑË¢Α«ÄÞæ«æêæ«Ëā½ΑŽÚ˜”«æÞΑõ«½½˜Α«ÄΑ樘ΑÚêځ½ΑÚ˜Α¨ÞΑ½˜”ΑæËΑ樘Α˜Þ恍½«Þ¨Ã˜ÄæΑË¢Α[(#ÞΠΑb¨˜Α
ŽËϘ×æΑË¢Αޘ½¢Α¨˜½×Α£ÚËê×ÞΑ¨ÞΑ˜˜ÄΑ˜õ˽õ˜”ΑæËΑËÚ£Ä«ā˜Α樘ΑÚêځ½Α×ËËÚΑæËΑؘæΑ樘«ÚΑ×Ú˔êŽæ«õ˜ΑÄ”Α
ŽËÄÞêÃ×æ«ËÄΑʘ”ÞΑËêæΑË¢Α樘«ÚΑށõ«Ä£ΠΑ
b¨Ë꣨Α樘ΑŽÞ¨ΑŽÚ˜”«æΑ¢Ž«½«æüΑ«ÞΑæËΑ˜ΑށĎæ«ËʔΑüΑ樘ΑÄºΑæËΑ[(#ΡΑ樘ΑށĎæ«ËÄΑË¢ΑŽÚ˜”«æΑüΑ[(#ΑæËΑ«æÞΑ
«Ä”«õ«”ꁽΑ Ø͘ÚÞΑ ö«½½Α ˜Α £ê«”˜”Α ÞΑ טÚΑ 樘«ÚΑ æ˜ÚÃÞΑ Ä”Α ”êځæ«ËÄΑ ÞΑ ”˜Ž«”˜”Α üΑ 樘Α £ÚËê×ÞΠΑ b¨«ÞΑ «ÞΑ
˜ûטŽæ˜”ΑæËΑژÁ«ÄΑêϨÄ£˜”ΑÄ”Αæ¨êÞΑ˜ÄÞêژΑŽËÄæ«ÄêÄŽ˜ΑË¢ΑĈāĎ«½Α”«ÞŽ«×½«Ä˜ΑæΑ樘ΑØ͘ÚΑ½˜õ˜½ΠΑ
Α¢˜öΑ Ø͘ÚÞΑ Ë¢Α ÄΑ [(#Α ÁüΑ £Ú”êæ˜Α¢Þæ˜ÚΑæËΑ ÞæÚæΑ ËÚΑ ˜ûׁĔΑ ˜ŽËÄËëŽΑ Žæ«õ«æ«˜ÞΑ ژÙê«Ú«Ä£Α ÃꎨΑ
¨«£¨˜ÚΑ½˜õ˜½ÞΑË¢Α½ËÄÞΑ樁ÄΑژÙê«Ú˜”ΑüΑË樘ÚΑ[(#ΑØ͘ÚÞΠΑ+ÄΑÞꎨΑŽÞ˜ÞΡΑ½½ΑË樘ÚΑØ͘ÚÞΑÁüΑÄËæΑ½«º˜Α
æËΑ ÞæÄ”Α ÃêæꁽΑ £êÚÄ昘Α¢ËÚΑ Α¢˜öΑ ½Ú£˜Α Þ«ā˜”Α ½ËÄÞΠΑ +ÄΑ ÞꎨΑ ŽÞ˜ÞΡΑ Α ÞÁ½½˜ÚΑ ύ6Ë«ÄæΑ ;««½«æüΑ #ÚËê×Α
(JLG)” ¢ÚËÃΑØ͘ÚÞΑË¢ΑÄΑ[(#ΑÁüΑ˜ΑŽÚ˜æ˜”ΠΑb¨˜ΑØ͘ÚÞΑË¢Α6;#Αö«½½ΑŽËÄæ«Äê˜ΑæËΑژÁ«ÄΑØ͘ÚÞΑË¢Α
樘Α[(#ÞΑÄ”ΑŽËÄæ«Äê˜ΑæËΑׁÚ櫎«×æ˜Α«ÄΑ樘ΑŽæ«õ«æ«˜ÞΑË¢Α[(#ÞΑÚ˜Α˜Ú½«˜ÚΠΑ ÄºÞΑÁüΑ˜ÄŽËêځ£˜ΑŽÚ˜æ«ËÄΑ
Ë¢ΑÞꎨΑ˜Äæ˜Ú×ګޘΑέΑ½«õ˜½«¨Ë˔ΑÞ˜”Α6;#ÞΑÞΑΑޘׁځæ˜Α˜Äæ«æüΠΑWW ÞΑÚ˜Αº˜üΑĈāĎ«Ä£Α«ÄÞæ«æêæ«ËÄΑæΑ樘Α
Úêځ½Α½˜õ˜½ΡΑö¨«Ž¨Α恺˜ÞΑ樘ΑژÞ×ËÄÞ««½«æüΑË¢Α¢ê½Ĉ½½«Ä£Α樘ΑŽÚ˜”«æΑژÙê«Ú˜Ã˜ÄæÞΑË¢Α[˜½¢Α(˜½×Α#ÚËê×ÞΑλ[(#ÞμΠΑ
b¨˜ΑWW ÞΑ½ÞËΑ×½üΑΑõ˜ÚüΑŽÚꎫ½ΑÚ˽˜Α«ÄΑ[˜½¢Α(˜½×Α#ÚËê×ÞΑλ[(#ÞμΑαΑ ÄºΑ½«Äº£˜ΠΑ ˜Þ«”˜ÞΑĈāĎ«Ä£Α[(#ÞΡΑ
樘ΑWW ÞΑ½ÞËΑ×½üΑΑõ«æ½ΑÚ˽˜ΑË¢ΑޘÚõ«Ä£ΑÞΑ[;"Α(;TΑTWHAHb+HB;Α+B[b+bfb+HBΑλ[(T+ÞμΠΑW˜ŽË£Ä«ā«Ä£Α
樘Α«Ã×ËÚæÄŽ˜ΑË¢Α[(#Α ÄºΑ½«Äº£˜ΡΑ#Ëõ˜ÚÄØÄæΑ¨ÞΑژÙê˜Þ昔Α½½Αގ¨˜”꽘”ΑŽËÃØڎ«½ΑÄºÞΑæËΑ¢Ë½½ËöΑ
樘Α˜ûÃ×½˜ΑޘæΑüΑÞËØΑ×ꍽ«ŽΑޘŽæËÚΑÄºÞΑÄ”ΑؘæΑ樘Α˜Äæ«Ú˜ΑŽÚ˜”«æΑژÙê«Ú˜Ã˜ÄæÞΑË¢Α[(#ΑØ͘ÚÞΠΑ
;«Äº«Ä£ΑË¢Α[(#ÞΑö«æ¨ΑÄºÞΑ¨ÞΑæ¨êÞΑ˜˜ÄΑ˜ÃרÞ«ā˜”Α«ÄΑ樘ΑAËĘæÚüΑT˽«ŽüΑ[ææ˜Ã˜ÄæÞΑË¢ΑW˜Þ˜Úõ˜Α ÄºΑ
Ë¢Α+Ĕ«ΑÄ”ΑfÄ«ËÄΑ ꔣ˜æΑÄÄËêĎ˜Ã˜ÄæÞΑ¢ÚËÃΑæ«Ã˜ΑæËΑæ«Ã˜ΑÄ”ΑõÚ«ËêÞΑ£ê«”˜½«Ä˜ÞΑ¨õ˜Α˜˜ÄΑ«ÞÞꘔΑæËΑ
ÄºÞΑ«ÄΑ樫ÞΑژ£Ú”Π
(˜ÄŽ˜ΑË×æ«ËÄΑλμΑ«ÞΑ樘ΑŽËÚژŽæΑÄÞö˜ÚΠ
ANALYSIS:

ANALYSIS:

[(#Α«ÞΑÄΑ«Ã×ËÚæÄæΑæË׫ŽΑ˜ÞטŽ«½½üΑæΑΑæ«Ã˜Αö¨˜ÄΑĈāĎ«½Α«ÄŽ½êÞ«ËÄΑ«ÞΑ˜«Ä£Αõ«£ËÚËêÞ½üΑ×ÚËÃË昔ΑÄ”Α
恺«Ä£Α×½Ž˜ΠΑ½ÞËΡΑ樘ژΑö˜Ú˜Αژ×ËÚæÞΑ«ÄΑb¨˜Α(«Ä”êΑËêæΑ樘Α«ÄŽÚ˜Þ«Ä£ΑÄê͘ÚΑË¢Α[(#ÞΑλ͊Π͋ΑŽÚËژÞΑ«ÄΑ
Äê͘ÚμΑÄ”Α樘ΑæÚ£˜æΑޘæΑüΑ樘Α£Ëõ˜ÚÄØÄæΑæËΑŽ¨«˜õ˜Α͉͊ΑŽÚËژΑöËØÄΑׁÚ櫎«×ÄæÞΑ«ÄΑ[(#ÞΠ
168

SOURCE : BWbΑλ+Ĕ«ÄΑŽËÄËëŽΑ˜õ˜½Ë×ØÄæμΑϹΑ[æÄ”Ú”ΑB˜öÞׁטÚ
;«ÄºÞΑË¢Α樘ΑÚ櫎½˜Þ΢Α¨ææ×Þ΢έέöööΠ樘¨«Ä”êЎËÃݍêޫĘÞÞέŽËÄËÃüέα
ޘ½¢α¨˜½×α£ÚËê×ÞᎁÄᨘ½×α«Äαö«”˜Ä«Ä£αöËØÄÞὁËêÚα¢Ëڎ˜αׁÚ櫎«×æ«ËÄᘎËÄËë
ŽαÞêÚõ˜ü݁Ú櫎½˜͏͏͍͎͍͒͋͐ИŽ˜
¨ææ×Þ΢έέöööΠ樘¨«Ä”êЎËÃέĘöÞέāæ«ËÄα
½έ£ËõæᎽËޘαæË၎¨«˜õ˜αæÚ£˜æαË¢α͉͊αŽÚËژαöËØÄαØ͘ÚÞα«Äαޘ½¢α¨˜½×α£ÚËê×ÞαÞ
üÞαëīÞæ˜Ú݁Ú櫎½˜͏͏͎͊͏͎͑͐ИŽ˜
+ÄΑ樘ΑBWbΑË˺Α+Ĕ«ÄΑŽËÄËëŽΑ˜õ˜½Ë×ØÄæΡΑ[(#ÞΑ«ÞΑö«”˜½üΑŽËõ˜Ú˜”
ûŽ˜Ú×æΑ¢ÚËÃΑBWb
169

QUALITY AND STANDARDIZATION


λ͋ΑVê˜Þæ«ËÄÞμ

Year 2017 STANDARDS

V͊ΠΑËÄÞ«”˜ÚΑ樘Α¢Ë½½Ëö«Ä£ΑÞææ˜Ã˜ÄæÞ΢
1. The Standard Mark of Bureau of Indian Standards (BIS) is mandatory for automotive tyres and tubes.
2. AGMARK is a quality Certification Mark issues by the Food and Agriculture Organisation (FAO).
Which of the statements given above is/are correct?
(a) 1 only
(b) 2 only
(c) Both 1 and 2
(d) Neither 1 nor 2

ANSWERS A

EXPLANATION
+ÄΑ͉͉͋͊ΡΑΑŽ˜Úæ«ĈŽæ«ËÄΑ¢ÚËÃΑ樘Α êژêΑË¢Α+Ĕ«ÄΑ[æÄ”Ú”ÞΑλ +[μΑ¨ÞΑ˜˜ÄΑÁ”˜ΑÁĔæËÚüΑ¢ËÚΑ½½ΑæüטÞΑ
Ë¢ΑêæËÃËæ«õ˜ΑæüژÞΑÄ”ΑæꍘÞΠΑ(˜ÄŽ˜ΡΑ[ææ˜Ã˜ÄæΑ͊Α«ÞΑŽËÚژŽæΠΑ
#AW9Α«ÞΑΑŽ˜Úæ«ĈŽæ«ËÄΑÁںΑ˜Ã×½Ëü˜”ΑËÄΑ£Ú«Žê½æêځ½Α×Ú˔êŽæÞΑ«ÄΑ+Ĕ«ΡΑÞÞêÚ«Ä£Α樁æΑ樘üΑŽËÄ¢ËÚÃΑæËΑ
ΑޘæΑË¢ΑÞæÄ”Ú”ÞΑ××ÚËõ˜”ΑüΑ樘Α«Ú˜ŽæËځæ˜ΑË¢ΑAÚº˜æ«Ä£ΑÄ”Α+ÄÞטŽæ«ËÄΡΑÄΑ£˜ÄŽüΑË¢Α樘Α#Ëõ˜ÚÄØÄæΑ
Ë¢Α+Ĕ«ΠΑb¨˜Α#AW9Α«ÞΑ½˜£½½üΑ˜Ä¢Ëڎ˜”Α«ÄΑ+Ĕ«ΑüΑ樘Α£Ú«Žê½æêځ½ΑTÚ˔ꎘΑλ#ځ”«Ä£ΑÄ”ΑAÚº«Ä£μΑŽæΑË¢Α
͊͒͌͐ΑλÄ”ΑÃ˜Ä”˜”Α«ÄΑ͊͒͑͏μΠΑ(˜ÄŽ˜ΡΑÞææ˜Ã˜ÄæΑ͋Α«ÞΑÄËæΑŽËÚژŽæΠΑ
170

ANALYSIS:
ANALYSIS:
SOURCE : ¨ææ×΢έέ«Ä”«Ä˜û×ژÞÞЎËÃ݁Ú櫎½˜έ«Ä”«έā«ºαõ«ÚêÞၽ½αüËêαʘ”αæËαºÄËöα͍͏͐͏͎͍͑έΑ
¨ææ×Þ΢έέöööΠĨ×Π£ËõΠ«Äέ}«ºαõ«ÚêÞε×£Α
¨ææ×΢έέöööΠö¨ËΠ«Äæέؔ«Ž˜ÄæÚ˜έ¢ŽæÞ¨˜˜æÞέā«ºέ˜Äέ

Year 2017 QUALITY COUNCIL OF INDIA

V͋ΠΑr«æ¨Αژ¢˜Ú˜ÄŽ˜ΑæËΑ樘ΑϏVꁽ«æüΑËêĎ«½ΑË¢Α+Ĕ«ΑλV+μϐΡΑŽËÄÞ«”˜ÚΑ樘Α¢Ë½½Ëö«Ä£ΑÞææ˜Ã˜ÄæÞ΢
1. QCI was set up jointly by the Government of India and the Indian Industry.
2. Chairman of QCI is appointed by the Prime Minister on the recommendations of the industry to the Government.
Which of the above statements is/are correct?
(a) 1 only
(b) 2 only
(c) Both 1 and 2
(d) Neither 1 nor 2

ANSWERS C

EXPLANATION

Vꁽ«æüΑ ËêĎ«½Α Ë¢Α +Ĕ«Α λV+μΑ öÞΑ ޘæΑ ê×Α ¶Ë«Äæ½üΑ üΑ 樘Α #Ëõ˜ÚÄØÄæΑ Ë¢Α +Ĕ«Α Ä”Α 樘Α +Ĕ«ÄΑ +ĔêÞæÚüΑ
ژ×ژޘÄ昔Α üΑ 樘Α æ¨Ú˜˜Α ×ژë˜ÚΑ «Ä”êÞæÚüΑ ÞÞˎ«æ«ËÄÞΑ «Π˜ΠΑ ÞÞˎ«æ˜”Α ¨Ã˜ÚÞΑ Ë¢Α ËÃØڎ˜Α Ä”Α
+ĔêÞæÚüΑË¢Α+Ĕ«Αλ[[H(AμΡΑËÄ¢˜”˜Úæ«ËÄΑË¢Α+Ĕ«ÄΑ+ĔêÞæÚüΑλ++μΑÄ”Α"˜”˜Úæ«ËÄΑË¢Α+Ĕ«ÄΑ¨Ã˜ÚÞΑ
Ë¢ΑËÃØڎ˜ΑÄ”Α+ĔêÞæÚüΑλ"++μΡΑæËΑ˜Þ恍½«Þ¨ΑÄ”ΑËטځæ˜Αāæ«Ëā½ΑŽŽÚ˜”«ææ«ËÄΑÞæÚêŽæêژΑÄ”Α×ÚËÃËæ˜Α
Ùꁽ«æüΑæ¨ÚË꣨ΑBæ«Ëā½ΑVꁽ«æüΑÃׁ«£ÄΠΑ(˜ÄŽ˜ΡΑÞææ˜Ã˜ÄæΑ͊Α«ÞΑŽËÚژŽæΠΑ
¨«ÚÁÄΑ Ë¢Α V+Α «ÞΑ ××Ë«Ä昔Α üΑ 樘Α TګØΑ A«Ä«Þæ˜ÚΑ ËÄΑ ژŽËÃØĔæ«ËÄΑ Ë¢Α 樘Α «Ä”êÞæÚüΑ æËΑ 樘Α
£Ëõ˜ÚÄØÄæΠΑb¨˜Α˜×ÚæØÄæΑË¢Α+ĔêÞæÚ«½ΑT˽«ŽüΑϞΑTÚËÃËæ«ËÄΡΑA«Ä«ÞæÚüΑË¢ΑËÃØڎ˜ΑϞΑ+ĔêÞæÚüΡΑ«ÞΑ樘Α
Ä˔½ΑëīÞæÚüΑ¢ËÚΑV+ΠΑ(˜ÄŽ˜ΡΑÞææ˜Ã˜ÄæΑ͋Α«ÞΑŽËÚژŽæΠ

ANALYSIS:
ANALYSIS:

SOURCE : ¨ææ×΢έέöööΠَ«ÄΠËÚ£έËêæΠר×
171

DIGITIZATION
λ͋ΑVê˜Þæ«ËÄÞμ

Year 2017 HACKATHON

V͊ΠΑr¨«Ž¨ΑË¢Α樘Α¢Ë½½Ëö«Ä£ΑÞææ˜Ã˜ÄæÞΑ«ÞέÚ˜ΑŽËÚژŽæΑژ£Ú”«Ä£Α[ÁÚæΑ+Ĕ«Α(Žºæ¨ËÄΑ͉͋͊͐Χ
1. It is a centrally sponsored scheme for developing every city of our country into Smart Cities in a decade.
2. It is an initiative to identify new digital technology innovations for solving the many problems faced by our country.
3. It is a programme aimed at making all the financial transactions in our country completely digital in a decade.
Select the correct answer using the code given below:
(a) 1 and 3 only
(b) 2 only
(c) 3 only
(d) 2 and 3 only

ANSWERS B

EXPLANATION
[ÁÚæΑ+Ĕ«Α(Žºæ¨ËÄΑ͉͋͊͐ΑöÞΑΑ͌͏Α¨ÚÞΑÄËÄαÞæË×Α”«£«æ½Α×Ú˔êŽæΑ”˜õ˜½Ë×ØÄæΑŽËÃטæ«æ«ËÄΑ”êÚ«Ä£Αö¨«Ž¨Α
昁ÃÞΑË¢Αæ¨ËêށĔÞΑË¢Α明¨Ä˽ˣüΑÞæꔘÄæÞΑê«½æΑ«ÄÄËõæ«õ˜Α”«£«æ½ΑÞ˽êæ«ËÄÞΑ¢ËÚΑ樘Α×Úˍ½˜ÃÞΑ×ËÞ昔ΑüΑ
͋͒Α”«ą˜Ú˜ÄæΑŽ˜Äæځ½Α£ËõæΠΑëīÞæÚ«˜ÞέΑ”˜×ÚæØÄæÞΑ˜Π£ΠΑA«Ä«ÞæÚüΑË¢ΑW«½öüÞΡΑûæ˜Úā½Αą«ÚÞΡΑA«Ä«ÞæÚüΑË¢Α
˜¢˜ÄޘΡΑ+[WHΡΑA«Ä«ÞæÚüΑË¢ΑbËêÚ«ÞÃΡΑ˜×æΠΑË¢ΑæËëŽΑĘڣüΡΑ˜æŽΠΑ(˜ÄŽ˜ΡΑ[ææ˜Ã˜ÄæΑ͊ΑÄ”Α͌ΑÚ˜Α«ÄŽËÚژŽæΠΑ
[ÁÚæΑ+Ĕ«Α(Žºæ¨ËÄΑ͉͋͊͐ΡΑ¨ÞΑ˜˜ÄΑ½êϨ˜”ΑæËΑê«½”Α«£«æ½Α+Ĕ«ΑÄ”ΑæËΑ˜Ä££˜Α樘ΑüËêæ¨Α”«Ú˜Žæ½üΑö«æ¨Α
āæ«ËÄΑ ê«½”«Ä£ΠΑ "ËÚΑ 樘Α ĈÚÞæΑ æ«Ã˜ΡΑ £ËõæΠΑ ”˜×ÚæØÄæÞΑ ö˜Ú˜Α ”«Ú˜Žæ½üΑ ˜Ä££«Ä£Α ö«æ¨Α ÞæꔘÄæÞΑ Ä”Α
Ž¨½½˜Ä£«Ä£Α 樘ÃΑ æËΑ ê«½”Α ”«£«æ½Α Þ˽êæ«ËÄÞΑ æËΑ «Ã×ÚËõ˜Α 樘«ÚΑ ˜ĆŽ«˜ÄŽüΡΑ ×½ê£Α ژõ˜Äê˜Α ½˜º£˜ÞΑ Ä”Α
ŽËÚÚê×æ«ËÄΠΑ(˜ÄŽ˜ΡΑ[ææ˜Ã˜ÄæΑ͋Α«ÞΑŽËÚژŽæΠΑ

ANALYSIS:
ANALYSIS:
[ÁÚæΑ+Ĕ«Α(Žºæ¨ËÄΑæË˺Α×½Ž˜Α«ÄΑ͉͋͊͐ΠΑ

[ŽËטΑ¢ËÚΑ˜½«Ã«Äæ«ËÄ
b¨˜Α æÚ£˜æÞΑ Þææ˜”Α «ÄΑ [ææ˜Ã˜ÄæΑ ͊Α Ä”Α ͌Α «Π˜ΠΑ Ϗ”˜õ˜½Ë׫ģΑ ˜õ˜ÚüΑ Ž«æüΑ «ÄæËΑ ÞÁÚæΑ Ž«æüΑ «ÄΑ Α ”˜Ž”˜ϐΑ Ä”Α
ϏÁº«Ä£Α½½ΑĈāĎ«½ΑæځÄށŽæ«ËÄÞΑ”«£«æ½Α«ÄΑΑ”˜Ž”˜ϐΑÚ˜Αõ˜ÚüΑÃ«æ«ËêÞΠΑ[ꎨΑÃ«æ«ËêÞΑæÚ£˜æÞΑŽÄÄËæΑ
˜Α˜ûטŽæ˜”Α¢ÚËÃΑΑ(Žºæ¨ËÄΑ˜õ˜ÄæΑλö¨«Ž¨ΑÚ˜Α£˜Ä˜Ú½½üΑ«”˜ΑÄ”Α×ÚËæËæüטΑê«½”«Ä£Α˜û˜ÚŽ«Þ˜ÞμΠΑb¨êÞΡΑ
Þææ˜Ã˜ÄæΑ͊ΑÄ”Α͌ΑÃêÞæΑ˜ΑöÚËÄ£ΠΑ;˜õ«Ä£ΑËĽüΑË×æ«ËÄΑλμΠΑ

SOURCE : ¨ææ×΢έέ׫ΠÄ«ŽΠ«ÄέĘöÞ«æ˜έTÚ«ÄæW˜½˜Þ˜ΠÞ×ûΧژ½«”Ͻ͎͎͊͒͑͋
172

Year 2020 CYBER INSURANCE

V͋ΠΑ+ÄΑ+Ĕ«ΡΑêĔ˜ÚΑŽü˜ÚΑ«ÄÞêځϘΑ¢ËÚΑ«Ä”«õ«”ꁽÞΡΑö¨«Ž¨ΑË¢Α樘Α¢Ë½½Ëö«Ä£Α˜Ä˜ĈæÞΑÚ˜Α£˜Ä˜Ú½½üΑ
ŽËõ˜Ú˜”ΡΑ«ÄΑ””«æ«ËÄΑæËΑׁüØÄæΑ¢ËÚΑ樘Α¢êĔÞΑÄ”ΑË樘ÚΑ˜Ä˜ĈæÞΧ
1. Cost of restoration of the computer system in case of malware disrupting access to one's computer
2. Cost of a new computer if some miscreant wilfully damages it, if proved so
3. Cost of hiring a specialized consultant to minimize the loss in case of cyber extortion.
4. Cost of defence in the Court of Law if any third party files a suit
Select the correct answer using the code given below:
(a) 1, 2 and 4 only
(b) 1, 3 and 4 only
(c) 2 and 3 only
(d) 1, 2, 3 and 4

ANSWERS B

EXPLANATION
ü˜ÚΑ+ÄÞêځϘΑ«ÞΑ”˜Þ«£Ä˜”ΑæËΑ£êÚ”ΑêޫĘÞޘÞΑ¢ÚËÃΑ樘Α×Ëæ˜Ä櫁½Α˜ą˜ŽæÞΑË¢ΑŽü˜Úαæ恎ºÞΠΑ+æΑ¨˜½×ÞΑ
an
ËÚ£Ä«Þæ«ËÄΑë櫣æ˜ΑÚ«ÞºΑ˜û×ËÞêژΑüΑˢޘææ«Ä£ΑŽËÞæÞΡΑ¢æ˜ÚΑΑŽü˜Úαæ恎ºέÚ˜Ž¨Α¨ÞΑ¨×טʔΠΑbË
Þ«Ã×½«¢üΡΑŽü˜ÚΑ+ÄÞêځϘΑ«ÞΑ”˜Þ«£Ä˜”ΑæËΑŽËõ˜ÚΑ樘Α¢˜˜ÞΡΑ˜ûטÄޘÞΑÄ”Α½˜£½ΑŽËÞæÞΑÞÞˎ«æ˜”Αö«æ¨ΑŽü˜Ú
Ú˜Ž¨˜ÞΑ樁æΑˎŽêÚΑ¢æ˜ÚΑÄΑËÚ£Ä«Þæ«ËÄΑ¨ÞΑ˜˜ÄΑ¨Žº˜”ΑËÚΑ¢ÚËÃΑ樘¢æΑËÚΑ½ËÞÞΑË¢ΑŽ½«˜ÄæݘÃ×½Ëü˜˜
«Ä¢ËÚÁæ«ËÄΠΑ+ÄΑ+Ĕ«ΡΑŽü˜ÚΑ«ÄÞêځϘΑŽËõ˜ÚÞΑλ£˜Ä˜Ú½½üμΑ樘Α¢Ë½½Ëö«Ä£΢
+”˜Äæ«æüΑ樘¢æ
ü˜Úαê½½ü«Ä£ΑÄ”ΑŽü˜ÚαÞ恽º«Ä£
ü˜ÚΑ˜ûæËÚæ«ËÄ
A½öÚ˜Α«ÄæÚêÞ«ËÄ
"«ÄÄŽ«½Α½ËÞÞΑ”ê˜ΑæËΑêāêæ¨ËÚ«ā˜”ΑÄ”Α¢Úê”꽘ÄæΑêޘΑË¢ΑÄºΑŽŽËêÄæΡΑŽÚ˜”«æΑŽÚ”ΑÄ”ΑÃˍ«½˜Αö½½˜æÞ
;˜£½Α˜ûטÄޘÞΑÚ«Þ«Ä£ΑËêæΑË¢ΑÄüΑŽËõ˜Ú˜”ΑÚ«Þº
[ˎ«½ΑA˜”«ΑËõ˜Ú
T¨«Þ¨«Ä£ΑËõ˜Ú
αÁ«½Α[×ËËĈÄ£
A˜”«Α;««½«æüΑ½«ÃÞΑËõ˜Ú
ü˜ÚΑûæËÚæ«ËÄΑËõ˜Ú
TÚ«õŽüΑ ژŽ¨Α Ä”Α æΑ ژŽ¨Α üΑ b¨«Ú”Α TÚæüΠΑ (˜ÄŽ˜Α ËĽüΑ Ë×æ«ËÄÞΑ ͊ΡΑ ͌Α Ä”Α ͍Α Ú˜Α ŽËÚژŽæΠΑ (˜ÄŽ˜Α
Ë×æ«ËÄΑλμΑ«ÞΑ樘ΑŽËÚژŽæΑÄÞö˜ÚΠ

ANALYSIS:
ANALYSIS:

[ŽËטΑ¢ËÚΑ˜½«Ã«Äæ«ËÄ
HĽüΑË×æ«ËÄΑ͋Α«ÞΑژ½æ˜”ΑæËΑ樘ΑרüÞ«Ž½Α”Ã£˜ΑË¢ΑΑŽËÃ×êæ˜ÚΑÄ”Α«æΑ”˘ÞΑÄËæΑ«ÄŽ½ê”˜ΑΑϏŽü˜ÚΑÄ£½˜ϐΠΑ
(˜ÄŽ˜Α«æΑŽÄΑ˜Αêޘ”Α¢ËÚΑ˜½«Ã«Äæ«ËÄΑË¢Α樘ΑË樘ÚΑæ¨Ú˜˜ΑË×æ«ËÄÞΠ
173

SOURCE : ¨ææ×Þ΢έέöööΠ樘¨«Ä”êêޫĘÞÞ½«Ä˜ΠŽËÃέ×ËÚæ¢Ë½«ËέטÚÞËā½αĈα
āĎ˜έŽü˜Úα«ÄÞêځϘαö¨æαÚ˜α樘αË×æ«ËÄÞ݁Ú櫎½˜͌͊͌͑͏͎͉͊ИŽ˜

x WΑ+B[fWBΑHqW[
ËÚ£Ä«Þæ«ËÄΑë櫣æ˜ΑÚ«ÞºΑ˜û×ËÞêژΑüΑˢޘææ«Ä£ΑŽËÞæÞΡΑ¢æ˜ÚΑΑŽü˜Úαæ恎ºέÚ˜Ž¨Α¨ÞΑ¨×טʔΠΑbË
Þ«Ã×½«¢üΡΑŽü˜ÚΑ+ÄÞêځϘΑ«ÞΑ”˜Þ«£Ä˜”ΑæËΑŽËõ˜ÚΑ樘Α¢˜˜ÞΡΑ˜ûטÄޘÞΑÄ”Α½˜£½ΑŽËÞæÞΑÞÞˎ«æ˜”Αö«æ¨ΑŽü˜Ú
Ú˜Ž¨˜ÞΑ樁æΑˎŽêÚΑ¢æ˜ÚΑÄΑËÚ£Ä«Þæ«ËÄΑ¨ÞΑ˜˜ÄΑ¨Žº˜”ΑËÚΑ¢ÚËÃΑ樘¢æΑËÚΑ½ËÞÞΑË¢ΑŽ½«˜ÄæݘÃ×½Ëü˜˜
«Ä¢ËÚÁæ«ËÄΠΑ+ÄΑ+Ĕ«ΡΑŽü˜ÚΑ«ÄÞêځϘΑŽËõ˜ÚÞΑλ£˜Ä˜Ú½½üμΑ樘Α¢Ë½½Ëö«Ä£΢
+”˜Äæ«æüΑ樘¢æ
ü˜Úαê½½ü«Ä£ΑÄ”ΑŽü˜ÚαÞ恽º«Ä£
ü˜ÚΑ˜ûæËÚæ«ËÄ
A½öÚ˜Α«ÄæÚêÞ«ËÄ
"«ÄÄŽ«½Α½ËÞÞΑ”ê˜ΑæËΑêāêæ¨ËÚ«ā˜”ΑÄ”Α¢Úê”꽘ÄæΑêޘΑË¢ΑÄºΑŽŽËêÄæΡΑŽÚ˜”«æΑŽÚ”ΑÄ”ΑÃˍ«½˜Αö½½˜æÞ
;˜£½Α˜ûטÄޘÞΑÚ«Þ«Ä£ΑËêæΑË¢ΑÄüΑŽËõ˜Ú˜”ΑÚ«Þº
[ˎ«½ΑA˜”«ΑËõ˜Ú
T¨«Þ¨«Ä£ΑËõ˜Ú
αÁ«½Α[×ËËĈÄ£
A˜”«Α;««½«æüΑ½«ÃÞΑËõ˜Ú
ü˜ÚΑûæËÚæ«ËÄΑËõ˜Ú
TÚ«õŽüΑ ژŽ¨ΑÄ”ΑæΑ ژŽ¨ΑüΑb¨«Ú”ΑTÚæü
174

MICROECONOMICS
OPPORTUNITY COST
(1 Questions)

Year 2018

Q1. If a commodity is provided free to the public by the Government, then


(a) the opportunity cost is zero.
(b) the opportunity cost is ignored.
(c) the opportunity costs is transferred from the consumers of the product to the tax-paying public.
(d) the opportunity cost is transferred from the consumers of the product to the Government.

ANSWERS C

EXPLANATION

H××ËÚæêÄ«æüΑŽËÞæΑ«ÞΑ樘ΑŽËÞæΑË¢ΑŽ¨ËËÞ«Ä£ΑËĘΑ½æ˜Úāæ«õ˜ΑËõ˜ÚΑÄË樘ÚΑÄ”ΑëÞÞ«Ä£Α樘Α˜Ä˜ĈæΑËą˜Ú˜”ΑüΑ
樘Α¢ËÚ£ËĘΑË××ËÚæêÄ«æüΡΑ«Äõ˜Þæ«Ä£ΑËÚΑË樘Úö«Þ˜ΠΑH××ËÚæêÄ«æüΑŽËÞæΑژ¢˜ÚÞΑæËΑΑ˜Ä˜ĈæΑ樁æΑΑטÚÞËÄΑŽË꽔Α
¨õ˜Α ژŽ˜«õ˜”ΡΑ êæΑ £õ˜Α ê×ΡΑ æËΑ 恺˜Α ÄË樘ÚΑ ŽËêÚޘΑ Ë¢Α Žæ«ËÄΠΑ [ææ˜”Α ”«ą˜Ú˜Äæ½üΡΑ ÄΑ Ë××ËÚæêÄ«æüΑ ŽËÞæΑ
represents an alternative given up when a decision is made.

ANALYSIS:
ANALYSIS:

Scope for elimination


Scope for Elimination: ½æ¨Ë꣨Αޘ˜Ã«Ä£½üΑ˜ÞüΡΑ樘ΑÙê˜Þæ«ËÄΑ«ÞΑÞ½«£¨æ½üΑ”«ĆŽê½æΑæËΑÄÞö˜ÚΑ«ÄΑËĘΑ£ËΠΑ
Elimination technique can be helpful. The good is provided for free by the government, but it is not a free
good (like air or sunlight). The commodity in the context can be any kind of physical infrastructure
road, bridges or government hospitals or free vaccines. That means its cost is borne by the
government. The money spent on the commodity in the context could have been used by the
government for something else. Hence option (a) is not correct. At the same time, the opportunity cost
is not ignored. Hence option (b) is not correct. Options (c) and (d) are close, but as the government gets
money from the taxpayers, hence ultimately the opportunity cost is transferred to the tax-paying public.
In short, the commodity that the consumers are getting for free has a cost which they do not have to
bear e.g. road or other infrastructure.
For such questions, NCERT basic books can be referred to get an exact idea of the concepts, but
ultimately application type questions cannot be covered. The only technique is deeper analysis and
option elimination.

SOURCE : Microeconomics (NCERT Class 12)


175

LAW OF DEMAND AND SUPPLY


(1 Questions)

Year 2021 CAPITAL FORMATION

Q1. Consider the following statements:


Other things remaining unchanged, market demand for a good might increase if
1. price of its substitute increases
2. price of its complement increases
3. the good is an inferior good and income of the consumers increases
4. its price falls
Which of the above statements are correct?
(a) 1 and 4 only
(b) 2, 3 and 4
(c) 1, 3 and 4
(d) 1, 2 and 3

ANSWERS A

EXPLANATION
Law of Demand- Law of Demand states that other things being equal, there is a negative relation between
demand for a commodity and its price. In other words, when price of the commodity increases, demand for
it falls and when price of the commodity decreases, demand for it rises, other factors remaining the same.
Hence option 4 is correct.

The quantity of a good that the consumer demands can increase or decrease with the rise in income
depending on the nature of the good. For most goods, the quantity that a consumer chooses, increases as
the consumer’s income increases and decreases as the consumer’s income decreases. Such goods are
called Normal Goods. Thus, a consumer’s demand for a normal good moves in the same direction as the
income of the consumer. However, there are some goods the demands for which move in the opposite
direction of the income of the consumer. Such goods are called Inferior Goods. As the income of the
consumer increases, the demand for an inferior good falls, and as the income decreases, the demand for
an inferior good rises. Examples of inferior goods include low quality food items like coarse cereals. Hence
option 3 is not correct.

The quantity of a good that the consumer chooses can increase or decrease with the rise in the price of a
related good depending on whether the two goods are substitutes or complementary to each other. Goods
which are consumed together are called complementary goods. Examples of goods which are complement
to each other include tea and sugar, shoes and socks, pen and ink, etc. Since tea and sugar are used
together, an increase in the price of sugar is likely to decrease the demand for tea and a decrease in the
price of sugar is likely to increase the demand for tea. Similar is the case with other complements. With the
increase in price of complemetn, demand reduces. Hence option 2 is not correct.
176

+ÄΑ ŽËÄæځÞæΑ æËΑ ŽËÃ×½˜Ã˜ÄæÞΡΑ £Ë˔ÞΑ ½«º˜Α 昁Α Ä”Α ŽËą˜˜Α Ú˜Α ÄËæΑ ŽËÄÞêؔΑ æË£˜æ¨˜ÚΠΑ +ÄΑ ¢ŽæΡΑ 樘üΑ Ú˜Α
ÞêÞæ«æêæ˜ÞΑ ¢ËÚΑ ˜Ž¨Α Ë樘ÚΠΑ [«ÄŽ˜Α 昁Α «ÞΑ Α ÞêÞæ«æêæ˜Α ¢ËÚΑ ŽËą˜˜ΡΑ «¢Α 樘Α ×Ú«Ž˜Α Ë¢Α ŽËą˜˜Α «ÄŽÚ˜Þ˜ÞΡΑ 樘Α
consumers can shift to tea, and hence, the consumption of tea is likely to go up. On the other hand, if the
×Ú«Ž˜ΑË¢ΑŽËą˜˜Α”˜ŽÚ˜Þ˜ÞΡΑ樘ΑŽËÄÞêÃ×æ«ËÄΑË¢Α昁Α«ÞΑ½«º˜½üΑæËΑ£ËΑ”ËöÄΠΑb¨˜Α”˜ÃÄ”Α¢ËÚΑΑ£Ë˔ΑêÞꁽ½üΑÃËõ˜ÞΑ
in the direction of the price of its substitutes. Hence option 1 is correct.

ANALYSIS:
ANALYSIS:

Conceptual question to check the basic understanding of terms in Economics.

Scope for elimination


Scope for Elimination: There is no direct scope of elimination. However, it was based on the simple law
of demand and supply, hence a candidate with a little bit of understanding could have answered it.

SOURCE : Microeconomics (NCERT Class 12)


177

2 QUESTIONS DROPPED
BY THE UPSC

Year 2020 INTERNATIONAL TRADE

Q1. With reference to the international trade of India at present, which of the following statements
is/are correct?
1. India's merchandise exports are less than its merchandise imports.
2. India's imports of iron and steel, chemicals, fertilisers and machinery have decreased in recent years.
3. India's exports of services are more than its imports of services.
͍ΠΑ+Ĕ«ΑÞêą˜ÚÞΑ¢ÚËÃΑÄΑËõ˜Ú½½Αæځ”˜έŽêÚژÄæΑŽŽËêÄæΑ”˜ĈŽ«æΠ
Select the correct answer using the code given below:
(a) 1 and 2 only
(b) 2 and 4 only
(c) 3 only
(d) 1, 3 and 4 only

Year 2022 CAPITAL FLIGHT

Q2. Consider the following statements:


͊ΠΑb«£¨æΑÃËĘæÚüΑ×˽«ŽüΑË¢Αf[Α"˜”˜Ú½ΑW˜Þ˜Úõ˜ΑŽË꽔Α½˜”ΑæËΑŽ×«æ½Αĉ«£¨æΠ
͋ΠΑ ×«æ½Αĉ«£¨æΑ ÁüΑ «ÄŽÚ˜Þ˜Α樘Α «Äæ˜Ú˜ÞæΑ ŽËÞæΑ Ë¢ΑĈÚÃÞΑö«æ¨Α ˜û«Þæ«Ä£Α ûæ˜Úā½Α ËÃØڎ«½Α ËÚÚËö«Ä£ÞΑ
λ ÞμΠ
͌ΠΑ˜õ½êæ«ËÄΑË¢Α”ËØÞ櫎ΑŽêÚژĎüΑ”˜ŽÚ˜Þ˜ÞΑ樘ΑŽêÚژĎüΑÚ«ÞºΑÞÞˎ«æ˜”Αö«æ¨Α ÞΠ
Which of the statements given above are correct?
(a) 1 and 2 only
(b) 2 and 3 only
(c) 1 and 3 only
(d) 1, 2 and 3
179

GUIDANCE FOR UPSC CSE PREPARATION


Looking for Guidance?
Unable to find a
good option?

Preparing for
UPSC?

DON’T WORRY! TALK TO OUR EXPERTS AND


GET GUIDANCE REGARDING:

TIME MANAGEMENT &


MAKING STUDY PLANS
NEWSPAPER
READING
CURRENT AFFAIRS
PREPARATION
DECIDING YOUR
OPTIONALS
IMPORTANCE
OF NOTES
STRATEGY FOR
ANSWER WRITING

+91 8468022022 SCAN


TO BOOK A SESSION: +91 9019066066 QR CODE
39 in Top 50
Selections
in CSE 2022

8 in Top 10
Selections
in CSE 2021

SHUBHAM KUMAR
CIVIL SERVICES
EXAMINATION 2020

HEAD OFFICE MUKHERJEE NAGAR CENTER GTB NAGAR CENTER FOR DETAILED ENQUIRY
Apsara Arcade, 1/8-B 1 Floor, Plot No. 857, Ground Floor,
st
Classroom & Enquiry Office, Please Call:
Near Gate-6 Karol Bagh Mukherjee Nagar, Opposite Punjab above Gate No. 2, GTB Nagar +91 8468022022,
DELHI Metro Station & Sindh Bank, Mukherjee Nagar Metro Building, Delhi - 110009 +91 9019066066

enquiry@visionias.in /c/VisionlASdelhi /visionias.upsc /vision _ias VisionIAS_UPSC

AHMEDABAD BENGALURU BHOPAL CHANDIGARH DELHI GUWAHATI HYDERABAD JAIPUR JODHPUR LUCKNOW PRAYAGRAJ PUNE RANCHI

You might also like